You are on page 1of 825

A-PDF Merger DEMO : Purchase from www.A-PDF.

com to remove the watermark

2008 ACJC H1 P1 and P2


Q1
D

Q2
A

Q3
D

Q4
D

Q5
A

Q6
B

Q7
D

Q8
A

Q9
D

Q10
D

Q11
C

Q12
A

Q13
D

Q14 Q15
B
D

Q16
C

Q17
B

Q18
A

Q19
A

Q20
B

Q21
B

Q22
A

Q23
D

Q24
C

Q25
A

Q26
C

Q27
B

Q28
C

Q29
C

1(a)

the tesla measures magnetic flux density which is defined as B =

F
IL

therefore the tesla in SI units is kg s-2 A-1

(b)
(i)

% uncertainty in E =

F A e L
E
(100) = (
+
+ +
) (100)
E
F
A
e
L
=(

5 0.09 x10 -6 0.5 0.01


+
+
) (100)
+
100 1.04 x 10 -6 5.0 2.00

= 24.15 % = 24.2 %
(ignore the ; add in for students if missed out and annotate with "BOD" benefit of
doubt)

(ii)

E
(100) = 24.15 %
E

E = 0.2415 (3.846 x 1010) Pa


= 0.9289 x 1010 Pa
= 0.9 x 1010 Pa ( to 1 sig. fig )
he should expressed value of E as E = (3.8 0.9) 1010 Pa

Q30
D

2
2(a)

(i) The e.m.f. of 2.0 V for a source means that 2 J of work done in

transforming non-electrical energy into electrical energy per


coulomb charge passes through it.
Or

It can also be defined as the 2 J of energy transferred by a source


in driving one coulomb charge round a complete circuit
(ii) P.d. of 1.5 V across a resistor means that 1.5 J of electrical energy
changed to some other forms of energy when one coulomb
charge passes from through the resistor.
or defined as the rate of transformation of electrical energy to other forms of
energy is 1.5 W for every ampere of current passing through the resistor
(b)

(i) emf = I (R1 + R2)


= (200 x 10-6) x(15 + 10) x 103
= 5.0 V
(ii)

A
15 k

10 k

Rv

emf = I [Rv R1/(Rv+ R1) + R2]


correct expression of R for 15 k // with voltmeter Rv
sub 5.0 V = (250 x 10-6) [15 Rv /(Rv+ 15) + 10]x 103
solving Rv = 30 k

3(a)
(b)

(c)
(d)

The 2 waves have a constant phase difference between each other


When the 2 waves arrive at the detector in phase, they will interfere
constructively, hence maximum intensity
When the 2 waves arrive at the detector out of phase by , they will
interfere destructively, hence minimum intensity
Distance L1 to D = 30.41 m
Distance L2 to D = 33.54 m
Path Difference = 3.13 m
The longest wavelength is when Path Difference = 1 x wavelength
Lowest Frequency = 330/3.13 = 105.4 Hz

4(a)

A wave can behave with properties of a particle and a particle, the properties
of a wave.
But we can never observe them simultaneously.

(b)(i)

E = mv2
Hence p2 = 2mE
Hence de Broglie's wavelength, = h/p
h
Therefore =
2mE

(ii)

1.17 x 10-15 m

(iii)

Change in energy = 7.5 eV 3.2 eV = 4.3 eV


E = hc/
p = h/
Hence E = pc
p = 6.88 x 10-19 /3.0 x 108
= 2.29 x 10-27 kg m-1

5a

With friction: COE, gain in KE + WD by friction = loss in GPE,


X at lowest pt max PE loss;
X is nearest to point of release compared to Y & Z lowest WD by friction
max KE gain Max KE (initial KE = 0)
From point of release to Z, gain in KE = loss in GPE WD by friction
Bead just missed reaching loss in GPE WD by friction < 0
Energy lost to friction > loss in GPE = (50.0 103)(9.81)(0.82) = 0.40221 J
energy lost to friction = 0.403 J (to 3 sf)
Energy lost to friction = WD by frictional force F = F (total distance travelled on the track)
(1.26 102 ) (total distance travelled) > 0.40221
total distance travelled > 31.92 m
total distance travelled on the curved track just before reaching Z = 32.0 m
Track and Toy B lubricated, hence average frictional force along track is 2.1 103 N hence
difference in energy loss between lubricated and normal condition
= [(1.26 102 ) (2.1 103 )] (total distance travelled)
= [0.0105] (31.92)
= 0.33516 J
Since frictional force is less when system is lubricated, this energy difference is
becomes the KE of Toy B.
Hence m vB2 = 0.33516
speed of Toy B at Z is vB = 3.66 m s-1

b(i)

(ii)

c(i)

(ii)

Since Toy Z and Toy B are identical, a head-on elastic collision will transfer all of Toy Bs kinetic
energy to Toy Z at point Z.
Since surface is horizontal, initial speed (horizontal) of Toy Z = 3.66 m s1
Consider y-dirn, 0.40 = 0 + (9.81) t2
hence t = 0.2856 s
horizontal distance, d = (3.66)(0.2856)
= 1.045 m
]

6.

(a)
(b)

i.

ii.

Resultant force in all directions is zero.


Resultant torque about all/any axes is zero.
When a single girder is lifted at constant velocity
Tension in AB = Weight of girder
= 500 (9.81)
= 4910 N (3 sig fig)
Considering forces acting on point B
T
TAB

70o
T

Using the sine rule:

OR by resolving forces vertically,

T
T
= AB o
o
2T sin 35o = 500 x 9.81
sin 55
sin 70
T = 4280 N (3 sig fig)
iii.

iv.
v.

vi

vii

viii

Considering the system of the Cab, Jib and Cable AC and taking moments
about H
Since crane would still be in equilibrium,
Clockwise Moments = Anti-Clockwise Moments
TAB (9.0) + 2500(9.81)(4.5) = 20000(9.81)(2.0)
TAB = 31.3 kN (3 sig fig)
Maximum no of girders =

31.3 103
= 6.4 = 6 (must round down)
( 9.81)( 500 )

1) Weight of Jib at E
2) Tension at A acting along AB
3) Tension at A acting along AC
4) Reaction at D (with components acting up and to the right)
Subtract 1 mark for each missing / incorrectly drawn force

9.0
o
= 34.70
13
9.0 + 3.5
o
Angle CAB = tan 1
= 54.25
9.0

Angle DAB = tan 1

Hence angle that AC makes with the jib is 19.55o (shown)


When the crane supports 5 girders, it is in equilibrium and
TAB = weight of 5 girders
Considering the free body of the Jib and taking moments about D,
Clockwise Moments = Anti-Clockwise Moments
5(500)(9.81)(9.0) + 2500(9.81)(4.5) = TAC (sin 19.55o)( 92 + 132 )
TAC = 62.6 kN
When the speed in raising the load is increasing, there must be a resultant
upward force on the girder. As such, the magnitude of tension in AB would
need to be greater than the weight of the girder(s) lifted.
As such, there would be a greater turning moment about point H, making
the crane less stable

7a
b

Wavelength distance betw 2 points in phase


Frequency no. of complete waves passing through a fixed point per unit time.
Distance traveled by a wave in time T is one wavelength
By definition speed = distance traveled over time
Hence v = /T = f
Marks awarded for each correct diagram
TW - The vibration of particles in the wave is perpendicular to the direction of travel
of the wave
LW - The vibration of particles in the wave is parallel to the direction of travel of the
wave

No net transfer of energy


Amplitude of vibration varies with position
Every particle within one wavelength varies in phase from each other

e(i)
(ii)
(iii)

Graph correct
All 5 positions of nodes labeled correctly
When current flows from fixed point to pulley, magnetic force acts upwards.
When current flows from pulley to fixed points, magnetic force acts downwards
As the current changes direction and magnitude periodically, the direction of
magnetic force changes direction periodically,
Hence wire vibrates
As the wire vibrates, a wave is formed. The initial wave and the reflected wave from
the pulley (or fixed point) will meet.

(iv)

Wavelength of the traveling waves depends on the speed which is fixed and the frequency
of the source.
SW occurs when the wavelength of the wave formed =

l
. (n is the number of loops.)
n

8 (a) (i) A photon is a quantum/discrete packet of electromagnetic radiation energy.


(ii) For light of different intensity but same frequency, the energy of the photon remains
the same.
Photoelectric emission is the result of direct interaction between each photon and
an electron and the entire energy hf of the photon is absorbed by the electron
instantaneously and the photon ceases to exist. Hence, instantaneous emission of
electrons occurs.
(iii) 1

2
mvmax = hf

Accept Emax = hf only if Emax is correctly defined to be the maximum kinetic


energy of emitted photoelectron.
Also accept hf0 in place of , provided f0 is correctly defined to be the threshold
frequency, and the entire term hf0 is referred to as the work function.
By Einsteins photoelectric equation, the maximum kinetic energy of emitted
photoelectrons depends on the frequency of the incident light. Hence increasing
the frequency of incident light will increase the maximum kinetic energy of the emitted
photoelectrons.
(b) (i) The work function is the minimum energy required to remove an electron from the
surface of the metal,
hence it is 3.616 x 10-19 J (no explanation, no credit)
(ii) 1
hc
2

mvmax =

3.62 1020 =

h (3.0 108 )
3.616 1019
7
(5.0 10 )

h = 6.63 1034 J s
(c) (i) 1
hc
2
mvmax =

2
1 2
(6.63 1034 )(3.0 108 )
mvmax =
3.616 1019
7
2
(4.4 10 )

= 9.045 1020 J
eVs = 9.045 1020

Vs = 0.565 V
(ii) 1
m(vmax sin 35) 2 = eVs
2
1 2
mvmax sin 2 35 = eVs
2
(9.045 1020 ) sin 2 35 = (1.6 1019 )Vs

Vs = 0.186 V

(d) (i)

(ii)

Candidates
answer.

(iii) Increasing the light intensity simply increases the number of photons falling on the
metal.
However, the incident photons still impart the same amount of energy hf to every
electron because the frequency f of light is kept constant. This means that the
maximum kinetic energy of the emitted photoelectrons will remain unchanged,
and hence the stopping voltage also remains unchanged with increasing light
intensity.
(e) This is to prevent the incident photons from being absorbed by surrounding air
molecules, and hence decreasing the intensity of the incident light.
OR
This is to prevent the emitted photoelectrons from colliding with the surrounding
air molecules, thus losing some of their kinetic energy, and introducing errors into
the measurement of their kinetic energy.
OR
The target metal used is usually very reactive, e.g. caesium, sodium or potassium,
hence the experiment is conducted in a vacuum to prevent oxidation of the target
metal.

Anglo-Chinese Junior College


Physics Preliminary Examination
Higher 1

CANDIDATE
NAME

CLASS

CENTRE
NUMBER

INDEX
NUMBER

PHYSICS

8866/02

Paper 2 Structured questions

21 Aug 2008
2 hours

Candidates answer on the Question Paper


No additional Materials are required
READ THESE INSTRUCTIONS FIRST
Write your Name and Index number in the spaces provided at the boxes above and on all
the work you hand in.
Write in dark blue or black pen.
You may use a soft pencil for any diagrams, graphs or rough working.
Do not use staples, paper clips, highlighters, glue or correction fluid.
Section A
Answer all questions.
Section B
Answer any two question.

For examiners use only

At the end of the examination, fasten all your work securely together.
The number of marks is given in brackets [ ] at the end of each
question or part question.

Section A
1

/ 6

/ 8

/ 8

/ 6

/ 12

Section B
6
7

/ 20

/ 20

Total

This paper consists of 16 printed pages

/ 20

/ 80

DATA AND FORMULAE


Data
speed of light in free space,

3.00 108 m s1

elementary charge,

1.60 1019 C

the Planck constant,

6.63 1034 J s

unified atomic mass constant,

1.66 1027 kg

rest mass of electron,

me

9.11 1031 kg

rest mass of proton,

mp

1.67 1027 kg

9.81 m s2

ut +

v2

u 2 + 2as

p V

hydrostatic pressure,

gh

resistors in series,

R1 + R2 +

1/R

1/R1 + 1/R2 +

acceleration of free fall,

Formulae
uniformly accelerated motion,

work done on/by a gas,

resistors in parallel,

AC/8866/02/08

1
2

at 2

3
For
Examiners
Use

Answer all the questions in the spaces provided.

1 (a)

Express the tesla in terms of the SI base units.

[2]

(b)

The Young's Modulus E of a material of a wire is given by


E=

FL
Ae

where F = tension exerted on the wire,


L = original length of the wire,
A = cross-sectional area of the wire and
e = extension of the wire.
In determining the Young's Modulus E of a material, a student performs
an experiment and makes measurements and records the readings as follows:
F = (100 5) N
L = (2.00 0.01) m
A = (1.04 0.09) x 10-6 m2
e = (5.0 0.5) mm
(i) Using the given formula, calculate the percentage uncertainty in E.

Percentage uncertainty in E = ________

(ii)

[2]

The student expressed E as 3.846 x 1010 Pa. Although his arithmetic and the
units of E are correct, this statement is unsatisfactory. Expressed his results
with its associated uncertainty.

E = __________________ Pa

AC/8866/02/08

[2]

[Turn over

For
Examiners
Use

Explain what is meant by


(i) a battery has an electro-motive force (e.m.f.) of 2.0 V?

[1]
(ii) the potential difference (p.d.) across a resistor is 1.5 V.

[1]

(b)

A cell of negligible internal resistance is connected in series with a microammeter of


negligible resistance and two resistors whose resistances are 10 k and 15 k. The
current is 200 A.
(i)

calculate the e.m.f. of the cell

e.m.f. = _________V [2]


(ii) When a voltmeter is connected in parallel with the 15- k resistor, the current in
the microammeter increases to 250 A.
1. Sketch a diagram of the modified circuit and

2. Calculate the resistance of the voltmeter.

voltmeter resistance = ________ [4]


AC/8866/02/08

3 (a)

For
Examiners
Use

What is meant by two sound sources are coherent?


[1]

(b)

Two loudspeakers L1 and L2 are driven from a common oscillator so that the wave
emitted from L1 and L2 are in phase. The loudspeakers are arranged as shown
below. A detector D is used to detect the sound reaching different points along line
AB, which is 30.0 m away from the loudspeakers.

L1
5.0 m
Detector,
D

15.0 m
L2

B
30.0 m

(i)

When the detector D is moved from A to B, it is observed that the detected signal
passes through a series of maximum and minimum sound intensity. Explain why this
happens?

[2]
(ii)

Calculate the path difference of the 2 waves arriving at the detector in the position
shown.

[3]
(iii)

Hence, calculate the lowest frequency of the sound that can cause the detector in the
position shown to detect a maximum intensity.
(Take speed of sound to be 330 m s1)

[2]
AC/8866/02/08

[Turn over

(a)

For
Examiners
Use

Explain what is meant by the term wave-particle duality.

[1]
(b)

(i)

Show that the de Broglie wavelength of a particle of mass m and kinetic


energy E is given by the expression

h
2mE

[2]
(ii)

Calculate the de Broglie wavelength of an electron of kinetic energy


1.0 x 10-18 J.

De Broglie wavelength =
(c)

[1]

An electron in an atom initially has an energy 7.5 eV above the ground state
energy. It makes a transition to a state with an energy of 3.2 eV above the ground
state energy and emits a photon in the process. Determine the momentum of the
photon emitted.

Momentum =

AC/8866/02/08

kg m s-1

[2]

At an amusement park, a child gently releases a toy roller coaster, Toy B, at the top of
a curved track as shown in Fig. 5 below.

Toy roller
coaster,
Toy B

Point of
release

Fig. 5
Toy roller
coaster,
Toy Z

H
Z

0.40 m
distance, d

Y
X

During the journey on the track, the toy roller coaster, Toy B, is subjected to the
frictional force due to the toys motion on the track. The magnitude of this frictional
force depends on several factors such as the type of material that the track is made
from, the material and manner in which the toy as well as the track are made, as well
as the speed of the toy on the track.
The frictional force between the toy and the curved track may be assumed to have an
average value of 1.26 102 N throughout the entire track under normal condition
and 2.1 103 N when lubricated.
Mass of Toy B = 50.0 g
Distance, H of point Z below point of release of Toy B = 0.82 m.
(a) Explain at which of the labelled point (X, Y or Z) of the track would Toy B has the
maximum kinetic energy.

[2]
(b) (i)

Point Z is below the point of release of Toy B. If Toy B moves along the track
under normal condition, and just missed reaching point Z, find the energy lost to
friction.

(ii) Hence find the total distance travelled by Toy B on the curved track from the point
of release to just before point Z.

Distance = __________ [6]

AC/8866/02/08

[Turn over

For
Examiners
Use

(iii)

(c)

If the track is lubricated, show that the speed of Toy B at point Z is


3.66 m s1.

If the track is lubricated, Toy B will make a head-on elastic collision with the
identical stationary toy roller coaster, Toy Z, at point Z. Determine the horizontal
distance d, that Toy Z will hit the horizontal surface, which is at 0.40 m below
point Z.
(Note that the diagram shows the position of Toy Z after collision and just before
leaving the track.)

[4]

AC/8866/02/08

For
Examiners
Use

For
Examiners
Use

Section B
Answer TWO questions from this section

(a) State the conditions for a rigid body to be in equilibrium.

[2]
(b) The figure below shows a crane being used to lift a load of girders, each of which
has a mass of 500 kg. The jib of the crane has a mass of 2500 kg and the cab
has a mass of 20 000 kg. The centre of mass of the jib and the cab are at their
midpoints, E and F respectively. The hook B, and the cable may be assumed to
have negligible mass. Cables AC and AB are two separate cables. G and H are
the left and right edges of the base of the cab respectively. D is the point at which
the jib is attached to the cab. Point A is at a height of 9.0m from C and 13.0m from
D. Point C is at a distance of 0.5m from the left edge of the cab.

(i)

Determine the tension in the cable AB when a single girder is lifted at constant
velocity.

Tension = ____________ N [1]


(ii) Hence or otherwise determine the tension in cable PBR.

Tension = ____________ N [2]

AC/8866/02/08

[Turn over

For
Examiners
Use

10

(iii) Determine the tension in cable AB which will just topple the crane.

Tension = ___________ N [4]


(iv) Hence determine the maximum number of girders which can be lifted without
the crane toppling over.

Number = _________ [1]


(v) Sketch a labelled free body diagram of the jib showing on it all the external
forces that are acting on the jib.

[3]
(vi) Show that the angle that cable AC makes with the jib is 19.55.

[2]
(vii) Determine the force exerted by the cable AC on the cab at C when the crane
supports 5 girders noting that the jib is in equilibrium

Force = _______ N [3]


(viii) Explain why the crane will be less stable when the speed at which the load is
being raised is increasing.

[2]

AC/8866/02/08

11

(a)

For
Examiners
Use

State the meaning of wavelength and frequency as applied to wave motion.

[2]
(b)

Deduce, from the definition of speed, the equation for the speed of a wave in
terms of its wavelength and frequency.

[3]

(c)

Illustrate with the help of diagrams to explain the meaning of the terms
longitudinal and transverse when applied to a wave.

[4]

AC/8866/02/08

[Turn over

12

(d)

Distinguish between a progressive wave and a stationary wave by completing the


table below

Progressive waves
Energy of transfer

energy is transported at a
speed given by the
product of its frequency
and wavelength.

Amplitude of vibration

Stationary waves

the amplitude of vibration


is the same at all points

Phase

All particles between


consecutive nodes have
the same phase

[3]

AC/8866/02/08

For
Examiners
Use

13

(e)

A horizontal steel wire is fixed at one end and is kept under tension by means of
weights suspended over a pulley, as shown in Fig 7 .The tension affects the
speed with which waves may travel along the wire.

Fig 7
A low voltage alternating supply is connected to the wire between the fixed end
and the pulley. The frequency of the alternating supply can be varied. Magnets
are placed near to the center of the horizontal section of the wire in order to
produce a magnetic field at right angles to the wire.

The tension of the wire is kept fixed as the frequency of the alternating supply is
varied. As the frequency of the supply is varied and at certain frequencies,
stationary waves are produced on the wire.
(i)

By drawing on Fig 7 above, show how the amplitudes of the points of the
stationary wave on the wire will vary along the wire from the fixed point to
the pulley when the frequency is such that the distance between the fixed
end and the pulley corresponds to two wavelengths of the wave on the
string.
[1]

(ii)

On your diagram, label the position of the nodes on the wire.

(iii)

[1]

Explain why the wire vibrates.

[3]
(iv)

Explain how the standing wave is being formed in this application and why it
only occurs at particular frequencies.

[3]
AC/8866/02/08

[Turn over

For
Examiners
Use

14

8 (a)

(i)

In 1887, Heinrich Hertz observed that when a metallic surface is exposed to


monochromatic light, electrons may be emitted. He published these observations in
the journal Annalen der Physik and it eventually came to be known as the
photoelectric effect.
Explain briefly what is meant by a photon.
[1]

(ii) Explain why, for a particular metal and for incident light of suitable frequency,
emission of photoelectrons begins almost instantaneously even if the light has low
intensity.

[2]

(iii) By stating Einsteins photoelectric equation or otherwise, explain why increasing the
frequency of incident light increases the maximum kinetic energy of the emitted
photoelectrons.

[2]

(b)

A clean surface of potassium in a vacuum is irradiated with light of photon energy


3.616 x 1019 J, electrons are found just to emerge, but when light of wavelength
5.000 x 10-7 m is incident, electrons emerge each with a maximum kinetic energy
3.620 x 10-20 J.
(i)

State and explain the value of potassiums work function. Express your answer
in joules and to 4 significant figures.

[2]

AC/8866/02/08

For
Examiners
Use

15

For
Examiners
Use

(ii) Hence estimate the value for Plancks constant.

[2]
(c)

Assume that the same surface of potassium is used but the wavelength of the
incident light is changed to 4.4 107 m . Note that the cathode and anode used here
are plane plates.

Determine the voltage that should be applied between the cathode(emitter) and
anode(collector), assumed to be parallel plates, in order to prevent photoelectrons of
maximum kinetic energy from reaching the anode, if electrons are emitted
(i)

normal to the cathode.

[2]
(ii) at an angle of 35 to the surface of the cathode.

[3]

AC/8866/02/08

[Turn over

16

(d)

(i)

Sketch a graph to show the photocurrent-potential (I-V) characteristic


obtained from a photoelectric effect experiment. Indicate the stopping
voltage on your graph.

[2]
(ii)

Sketch, on the same graph, the effect of increasing the intensity of the
incident light.
[1]

(iii)

Explain why the stopping voltage is independent of the intensity of the


incident radiation.

[2]

(e)

Give one reason for why a photoelectric experiment is usually conducted in an


evacuated vacuum tube.

[1]

AC/8866/02/08

For
Examiners
Use

PDG
Candidate Name

ANDERSON JUNIOR COLLEGE

8866/01

PHYSICS
Higher 1
Paper 1 Multiple Choice

1 hour

Tuesday 16 September 2008


Additional Materials:

Multiple Choice Answer Sheet


Soft clean eraser
Soft pencil (type B or HB recommended)

READ THESE INSTRUCTIONS FIRST


Write in soft pencil
Do not use staples, paper clips, highlighters, glue or correction fluid.
Write your name, PDG on the Answer Sheet in the spaces provided. Shade the last 5 digits of
your NRIC/FIN without the letter.
There are thirty questions on this paper. Answer all questions. For each question there are four
possible answers A, B, C and D.
Choose the one you consider correct and record your choice in soft pencil on the separate
Answer Sheet.
Each correct answer will score one mark. A mark will not be deducted for a wrong answer.
Any rough working should be done in the question paper.

This question paper consists of 10 printed pages and 0 blank page.


8866/01/AJC2008

[Turn over

2
Data
speed of light in free space

c = 3.00 x 108 m s-1

elementary charge

e = 1.60 x 10-19 C

the Planck constant

h = 6.63 x 10-34Js

unified atomic mass constant

u = 1.66 x 10-27 kg

rest mass of electron

me = 9.11 x 10-31 kg

rest mass of proton

mp = 1.67 x 10-27 kg

acceleration of free fall

g = 9.81 m s-2

Formulae

uniformly accelerated motion

s = u t + 1 at
2

v2 = u2 + 2as
work done on/by a gas

W = p V

hydrostatic pressure

p = gh

electric potential

V =

Q
4 0 r

resistors in series

R = R1 + R2 +

resistors in parallel

1/R = 1/R1 + 1/R2 +

8866/01/AJC2008

[Turn over

3
1

A student takes 6 readings of the diameter of a rod: 5.9 mm, 6.0 mm, 6.1 mm, 5.8 mm,
6.0 mm, 6.2 mm.
The actual diameter of the rod is actually 5 mm. Which of the following best describe the
errors in the readings?
A
B
C
D

random error
low
low
high
high

systematic error
high
low
high
low

The capacitance C of a parallel plate capacitor is defined as the charge stored per unit
potential difference between its plates.

C=

Q
V

What is the unit of capacitance, expressed in SI base units?


A CV-1
3

C2kg-1m-2s-2

C A2kg-1m-2

D A2kg-1m-2s4

A student carries out an experiment to determine the gravitational acceleration, using a


pendulum, and obtains a value 9.81 ms-2. The uncertainties in the measurements are
shown.
Uncertainty in length L of pendulum = 0.7 %
Uncertainty in period T of pendulum = 0.1 %
The equation for the period T of pendulum is T = 2

L
g

How should the answer for gravitational acceleration be stated?


A
B
C
D

(9.8 0.1) m s-2


(9.81 0.09) m s-2
(10 8) m s-2
(10 9) m s-2

Which of the following statements is correct?


A
B
C
D

When an object is in motion, its velocity and acceleration are always in the same
direction.
When an object is thrown upwards, its acceleration at the highest point is zero.
When velocity of an object is zero, its acceleration can be non-zero.
When acceleration of an object is zero, its velocity is zero.

8866/01/AJC2008

[Turn over

4
5

The figure below shows the displacement-time graph of a moving object from a point P.
Displacement /m

Time /s
Which of the following statements correctly describes the motion for the object?
A
B
C
D
6

A projectile is fired with an initial speed of 51.2 m s-1 at angle 44.5 above the horizontal on
a long flat firing range. Point A is the maximum height attained by the projectile. What is the
vertical component of the speed and location of the projectile 1.50 s after firing?
A
B
C
D

It is speeding up towards P.
It is slowing down towards P.
It is speeding up away from P.
It is slowing down away from P.

Speed/m s-1
21.2
21.2
35.6
35.6

Location
Before point A
After point A
Before point A
After point A

The 100 kg mass, which rests on a frictionless table, is attached to a 1 g mass through an
inextensible string that goes round a smooth pulley. What is the acceleration of the 100 kg
mass?

100 kg

1g

A
B
C
D
8

9.81 10-5 m s-2


9.81 10-2 m s-2
9.81 m s-2
0 m s-2

A trolley moving without friction on an air track collides with an identical stationary trolley
and they move off sticking together.
Which of the following about the conservation of energy and momentum is correct?
A
B
C
D

momentum
conserved
conserved
not conserved
not conserved

kinetic energy
conserved
not conserved
conserved
not conserved

8866/01/AJC2008

[Turn over

5
9

Which of the following pairs of forces is an action-reaction pair?


A
B
C
D

The force a ladder leaning on a smooth wall exerts on the wall and the normal reaction
force from the wall.
The force a ladder leaning on a smooth wall exerts on the rough floor and the normal
reaction from the floor.
Weight of a parachutist and the pull of the parachute on him when he is moving with
terminal velocity.
Weight of a floating object and the upthrust acting on it.

10 Two identical bar magnets are stored in a light plastic frictionless cylinder of negligible mass.
When the magnets are arranged as shown in the figure below, the balance reads W.
S

Plastic cylinder

N
N
S

Top-pan balance
If the mass of each magnet is M, which of the following is correct?
A
B
C
D

W = Mg
W = 2Mg
Mg < W < 2Mg
W > 2Mg

11 The diagram shows a uniform inclined beam with one end hinged to a wall A and the other
end is supported by a wire at B. The weight of the beam is W and the tension in the wire is
T. The angle EAC is 90.
E

T
D
C

B
W
What is the direction of the force exerted by the wall on the beam?
A

AB

AC

AD

AE

12 A spring obeying Hookes Law has an unstretched length of 50 mm and a spring constant of
200 N m-1. What is the tension in the spring when its length is 65 mm?
A

3000 N

300 N

8866/01/AJC2008

30 N

3N

[Turn over

6
13 A lightweight object and a very heavy object are moving with equal speeds along a level
frictionless surface. Both objects subsequently slide up the same frictionless incline.
frictionless
incline
frictionless level surface
Which object rises to a greater height?
A
B
C
D

The heavy object, because it has greater kinetic energy.


The lightweight object, because it weighs less.
They both slide to the same height.
Cannot be determined from the information given.

14 A block of mass m is pushed against a spring of spring constant k. The spring is


compressed by a distance d, the block is then released. It is launched by the spring along a
horizontal frictionless surface with a final speed v. A second block, this one having mass
4m is pushed against the same spring and released, gaining a final speed 2v. By what
distance was the spring compressed in the second case?
A

4d

16d

25d

15 To accelerate a car at a constant acceleration, the cars engine must


A
B
C
D

develop ever-increasing power.


develop ever-decreasing power.
maintain a constant power output.
maintain a constant turning speed.

16 A warning siren on top of a tall pole is taken to be a point source and radiates sound waves
uniformly in all directions. At a distance 4d, the amplitude of the wave is A.
What is the distance from the siren at the point where the amplitude of the wave is 2A?
A

8d

4d

2d

17 A beam of initially unpolarised light passes through three polariods P1, P2 and P3. The
polarising axis of each polaroid is shown by an arrow. Polariods P1 and P2 are fixed, with
their polarising axes at 45 to one another.

What can be the angle of the polarising axis of polaroid P3 so that the intensity of the
emergent light is minimum?
A

45

8866/01/AJC2008

90

135

[Turn over

7
18 A tall vertical cylinder is filled with water and a tuning fork vibrating at 400 Hz is held over its
open top. The water is slowly run out and the first resonance of the air column is heard
when the water level is 0.21 m below the top of the cylinder. Neglecting end corrections,
how far from the top will the water level be when the next resonance is heard?
A

0.32 m

0.42 m

0.63 m

0.84 m

19 If two waves of the same frequency are superposed in phase, the total intensity is
proportional to
A
B
C
D

the sum of the intensities of the waves.


the square of the mean value of the two amplitudes.
the square of the difference of the two amplitudes.
the square of the sum of the two amplitudes.

20 A and B shown in the figure below are two coherent sound sources which are in phase.
Point X shows permanent zero displacement.
A

270 mm

90 mm

From the options given below, choose the sound wave with the minimum wavelength that
can satisfy this condition.
A

180 mm

90 mm

60 mm

45 mm

21 The graphs show the variation with potential difference V of current I for three circuit
elements.

The three circuit elements are a tungsten filament lamp, a metal wire at constant
temperature and a semiconductor diode.
Which of the following correctly identifies these graphs?
A
B
C
D

Tungsten filament lamp


X
Y
X
Z

metal wire
Y
Z
Z
Y

8866/01/AJC2008

semiconductor diode
Z
X
Y
X

[Turn over

8
22 A particular wire in a circuit has a resistance of 5.0 . What is the resistance of another wire
made of the same material, has the same cross-sectional area and which is three times as
long as the first wire?
A

15

45

125

23 In the circuit below, resistors X and Y, of resistances R and 2R respectively, are connected
to a 6 V battery of negligible internal resistance. When a voltmeter is connected across Y, it
gives a reading of 3 V.

What is the reading of the voltmeter when it is connected across X?


A
B
C
D

zero
Between zero and 2 V
2V
Between 2 V and 6 V

24 What is the equivalent resistance between the points L and M in the circuit below if each of
the resistors has a resistance of 2 ?

N
A

8866/01/AJC2008

[Turn over

9
25 The diagram below shows a wire conductor, RS, positioned perpendicular to an uniform
magnetic field directed into the paper.

What is the direction in which the wire could be moved at a constant speed to produce the
maximum potential difference across its ends, R and S?
A
B
C
D

perpendicular to both the length of the wire and the magnetic field
perpendicular to only the length of the wire
perpendicular to only the magnetic field
in the direction 45 to the length of the wire

26 A plotting compass is placed next to a vertical wire PQ. When there is no current in the wire,
the compass on the horizontal plane ABCD points North as shown in the diagram. Which
diagram shows a possible direction for the compass to point when a current passes from Q
to P?
P

D
Q

27 The figure below shows a cross-section through a solenoid.


current into plane of paper

current carrying conductor

current out of plane of paper


What happens to the current carrying conductor when it is placed along the center axis of
the solenoid?
A
B
C
D

The conductor moves upward.


The conductor moves downward.
The conductor moves in the direction out of the plane of paper.
The conductor remains stationary.

8866/01/AJC2008

[Turn over

10
28 Let 0 be the de Broglie wavelength of an electron accelerated from rest through a potential
difference of 10 V and let 1 be that of an electron accelerated from rest through a potential
difference of 1000 V. The value of the ratio 0/ 1 is
A

1/10

10

100

29 The graph below shows the relationship between the energy E of a photon of
electromagnetic radiation and its wavelength . Points P and Q marked on the graph
represent two types of electromagnetic radiation.
E

Which of the following pairs of electromagnetic radiation correctly identifies P and Q?


A
B
C
D

P
ultraviolet
Infra-red
microwave
microwave

Q
Radio wave
X-ray
Gamma ray
ultraviolet

30 Which of the following statements is true when photoelectric effect occurs?


A
B
C
D

The maximum speed with which electrons are emitted is proportional to the intensity of
the incident light.
The number of electrons emitted per second is proportional to the intensity of the
incident light.
The maximum energy of the emitted electrons increases with the wavelength of the
incident light.
The wavelength of the incident light must be greater than a certain threshold value.

8866/01/AJC2008

[Turn over

PDG

Candidate Name

ANDERSON JUNIOR COLLEGE

PHYSICS
Higher 1

8866/02

Paper 2

2 hours

Tuesday 16 September 2008


Candidates answer on the Question Paper.
No Additional Materials are required.
READ THESE INSTRUCTIONS FIRST
Write your name and PDG on the spaces provided above.
Write in dark blue or black pen.
You may use a soft pencil for any diagrams, graphs or rough working.
Do not use staples, paper clips, highlighters, glue or correction fluid.
Section A
Answer all questions.

For Examiners Use


Section A (40 marks)

Section B
Answer any two questions. Write down the
numbers of the two questions attempted
in the boxes.
You are advised to spend about one hour on
each section.
At the end of the examination, fasten all your
work securely together.
The number of marks is given in brackets [ ] at
the end of each question or part question.

1
2
3
4
5
Section B (40 marks)

Deduction
Total (80 marks)
This question paper consists of 20 printed pages and 0 blank page.
8866/02/AJC2008

[Turn over

2
Data
speed of light in free space

c = 3.00 x 108 m s-1

elementary charge

e = 1.60 x 10-19 C

the Planck constant

h = 6.63 x 10-34Js

unified atomic mass constant

u = 1.66 x 10-27 kg

rest mass of electron

me = 9.11 x 10-31 kg

rest mass of proton

mp = 1.67 x 10-27 kg

acceleration of free fall

g = 9.81 m s-2

Formulae
uniformly accelerated motion

s = u t + 1 at
2

v2 = u2 + 2as
work done on/by a gas

W = p V

hydrostatic pressure

p = gh

electric potential

V =

Q
4 0 r

resistors in series

R = R1 + R2 +

resistors in parallel

1/R = 1/R1 + 1/R2 +

8866/02/AJC2008

3
Section A
Answer all the questions in the spaces provided.
1 (a)

Explain what is meant by the moment of a force.


..
.....[2]

(b)

A cubical box ABCD shown in Fig. 1.1 is filled with sand and weighs 890 N. In order to roll
the box, a horizontal force F can be applied on one of the upper edges.
F

A
Fig. 1.1

(i)

Calculate the force F required to just roll the box without slipping.

F = .. N [2]
(ii)

State the minimum friction between the box and the floor in (i).
.....[1]

(iii)

There exists a more efficient way to roll the box. Determine the magnitude of the
smallest possible force that would have to be applied directly to the box to roll it.

minimum force = .. N [2]


2 (a)

What is meant by the term superposition when applied to waves?


..............................
.....[1]
8866/02/AJC2008
[Turn over

4
(b)

Two loudspeakers L1 and L2 driven from a common oscillator are arranged as shown in
Fig. 2.1. A detector is placed at D. It is found that, as the frequency of the oscillator is
gradually changed from 200 Hz to 1000 Hz, the detected signal passes through a series of
maxima and minima.
40 m
L1
D
9m

L2
Fig. 2.1
(i)

Explain how the maxima and minima can be observed at the same point D.
.........
.........
.........
.........
..[2]

(ii)

Calculate the frequency at which the first minimum is observed. Assume the speed of
sound is 340 m s-1.

frequency = .. [2]
(iii)

If the frequency of the oscillator is fixed at 510 Hz, how far from D must the detector be
placed in order to observe the first order maximum? (Please note that the zero order
maximum is observed at the central point O.)

distance from D = .. m [3]


8866/02/AJC2008

5
3 (a)

Define magnetic flux density and the tesla.


..
..
..
..
..
.....[2]

(b)

A current I passes through a pivoted rectangular wire frame, P, which is initially balanced. A
magnet is then placed near one of the edges of the wire frame, with its magnetic field acting
perpendicular to the edge of the wire frame and this causes the wire frame to tilt. A rider of
mass m is then placed at a point along a section of the wire frame until the frame regains its
balance as shown in Fig. 3.1 below.

I
pivots

rider

MAGNET

y
Fig. 3.1

(i)

Indicate, on Fig 3.1, the direction of the force due to the magnet, on the edge of the
frame nearest to the magnet. Label this force F.
[1]

(ii)

Deduce an expression for the magnetic field strength of the magnet, in terms of x, y, z,
m, I and g, where g is the acceleration of free fall.
[2]

(iii)

State and explain what happens when the polarity of the magnet is reversed.
..
..
..
..[2]
8866/02/AJC2008
[Turn over

6
4 Fig. 4.1 shows some of the energy levels for an atom of hydrogen.
n=
n=5
n=4

0
- 0.545 eV
- 0.849 eV

n=3

- 1.51 eV

n=2

- 3.41 eV

n=1

- 13.60 eV
Fig. 4.1

(a)

If the outer electron of the atom is in the ground state, how much energy must be required to
remove this electron from the atom?

energy = .. J [1]
(b)

(i)

Suppose an electron of energy 11.0 eV collides with the atom. Explain the possible
result of the interaction if the electron of the atom is in the ground state.
....................
..[1]

(ii)

If the electron of energy 11.0 eV is replaced by a photon of energy 11.0 eV, what will be
the result of the interaction?
....................
..[1]

(c)

If an electron returns from the -0.849 eV level to the ground state, what is the wavelength of
the photon emitted?

wavelength = .. m [1]
8866/02/AJC2008

7
(d)

State the region of the electromagnetic spectrum in which this radiation lies.
.....[1]

(e)

On Fig.4.1, draw arrows to show


(i)

another transition which results in the emission of radiation of wavelength shorter than
the radiation in (c) (label this transition P)
[1]

(ii)

a transition which results from absorption of radiation of wavelength longer than the
radiation in (c) (label this transition Q)
[1]

5 A mass M is moving at 5.00 m s-1 along a horizontal frictionless guide which bends into a vertical
circle of radius r, as illustrated in Fig. 7.1.
C
B
r
M

A
Fig. 7.1

Fig. 7.2 and Fig 7.3 show the velocity-time graphs for the vertical and horizontal components
respectively of the velocity along the section ABC of the curve.

Fig. 7.2
8866/02/AJC2008
[Turn over

Fig. 7.3
(a)

With the aid of Fig. 7.2, find an appropriate value for the height of the vertical circle. Hence,
find the value for the radius of the vertical circle, r.

height of vertical circle = .. m [2]


radius of vertical circle, r = .. m [1]

8866/02/AJC2008

9
(b)

(i)

From Fig. 7.2 and Fig. 7.3, find the vertical and horizontal components of the
acceleration of the mass M at B, 200 ms after it passes the point A.

vertical component of the acceleration = .. ms-2 [1]


horizontal component of the acceleration = .. ms-2 [2]
(ii)

Hence, find the magnitude and the direction of the resultant acceleration of the mass M
at B.

magnitude of acceleration = .. m s-2 [1]


direction of acceleration = [2]
(c)

Without detailed mathematical calculation, deduce the total area between the curve and the
time axis of Fig. 7.3. Explain your answer.
.....
.....
.....
.........
.....
.....
.[2]

8866/02/AJC2008
[Turn over

10
Section B
Answer two questions from this section.
6 (a)

The floor of an elevator, mass 1800 kg, is at a distance 3.70 m above a spring as shown in
Fig. 6.1. It is at rest on the first floor when its cable snaps. A safety device clamps the elevator
against guide rails so that a constant frictional force of 4.40 kN opposes the elevators motion.
cable
elevator
guide rails
3.7 m
spring
Fig 6.1
(i)

Show that the acceleration of the elevator is 7.37 m s-2.

acceleration = .. m s-2 [1]


(ii)

Calculate the speed of the elevator when it hits the spring.

speed = .. m s-1 [2]


(iii)

Calculate the average power developed during this process before it hits the spring.

average power = .. kW [2]


8866/02/AJC2008

11
(iv)

The spring is compressed by 0.9 m when the elevator comes to a rest. Using the
principle of conservation of energy, determine the force constant of the spring. Assume
mass of spring is negligible.

force constant = .. N m-1 [2]


(v)

The force constant found in (iv) is known to be accurate to 8%. How should the result
be presented?

force constant = .. N m-1 [1]


(b)

Consider a woman of mass 60 kg, standing on a scale that reads in kilogram, in the elevator
when it is on the first floor as shown in Fig. 6.2. State the reading on the scale after the cable
snaps if the safety device fails to work. Explain your answer.

cable

elevator
scale
Fig. 6.2
.....
.....
.....
........[3]
8866/02/AJC2008
[Turn over

12
(c)

(i)

State the principle of conservation of momentum.


......
.........[1]

(ii)

A 0.25 kg skeet is fired at an angle of 30 to the horizontal with a speed of 25 m s-1 as


shown in Fig. 6.3. When it reaches the maximum height of 7.96 m, it is hit from below
by a 15 g pellet traveling vertically upward at a speed of 200 m s-1. The pellet is
embedded in the skeet and they move together along path A.

25 m s-1
30

7.96 m

200 m s-1

Path A

pellet

skeet

Fig. 6.3
1.

Determine the velocity of the skeet at the maximum height of 7.96 m.

velocity = ... m [1]


direction of velocity = ... [1]
2.

Upon collision, skeet and pellet move off together with a velocity v at an angle to
the horizontal. By applying (c)(i) to the skeet-pellet system in the vertical direction,
show that v sin = 11.321.
[2]

3.

Determine the time of flight for the skeet-pellet system to complete path A.

time of flight = .. s [2]


8866/02/AJC2008

13
4.

Suggest with a reason, whether the momentum of the skeet-pellet system is


conserved in the presence of air resistance.
.........
.........
.........[2]

7 (a)

Define wavelength and frequency. Deduce a relation between these quantities and the speed
of propagation of a wave.
.....
..
.....
.....
.....
.....
.....
........[3]

(b)

Ocean waves, which travel in the open sea at 5 m s-1, arrive at a beach once every 4 s. What
is the distance between wave crests in open sea?

distance = .. m [1]
(c)

In order to investigate stationary waves on a stretched string, a student set up the apparatus
illustrated in the diagram Fig. 7.1.
string

vibrator

pulley

weights

Fig. 7.1

8866/02/AJC2008
[Turn over

14
(i)

Sketch two stationary patterns which could be formed on the string.

[2]

(ii)

Mark the nodes in each sketch.

[1]

(iii)

What is meant by a node?


......
......
.....[1]

(iv)

Briefly explain why a stationary wave is observed on the string only at particular
frequencies of the vibrator.
......
......
......
......
.....[2]
8866/02/AJC2008

15
(v)

If in one of your sketches the distance between nodes is 0.55 m when the frequency of
the vibrator is 40 Hz, what is the speed of the waves on the string?

speed = .. m s-1 [1]


(vi)

It was found that for a given length of the string under the same tension, the frequency
of the vibrator at which a stationary wave is formed is directly proportional to the
number of antinodes formed on the string. Using the apparatus shown in the figure,
how would you show that the above statement is true?
......
......
......
......
.....[2]

(d)

(i)

State what is meant by the photoelectric effect.


......
......
.....[1]

(ii)

Give an expression for E, the energy of a photon, in terms of f, its frequency, and h, the
Planck constant.
.....[1]

(iii)

The work function of a freshly cleaned zinc surface is 3.6 eV. If it is illuminated with
ultraviolet radiation of wavelength 253 nm,
1.

what will be the maximum kinetic energy of the emitted electrons?

maximum kinetic energy = .. eV [1]


8866/02/AJC2008
[Turn over

16
2.

what potential difference would be needed to stop the electrons being emitted?

potential difference = .. V [1]


3.

what would be the threshold frequency for the metal surface?

threshold frequency = .. Hz [1]


(e)

Light of wavelength 546 nm falls on a potassium surface of area 7.5 cm2 in an evacuated
enclosure. The intensity at the surface is 60 mW m-2 and it may be assumed that 1 % of the
photons emit electrons from the surface.
(i)

Calculate the number of photons arriving at the surface per unit time.

number of photons per second = .. [1]


(ii)

Hence calculate the photoelectric current produced.

photoelectric current = .. A [1]


8866/02/AJC2008

17
8 (a)

An I-V characteristic for a thermistor is shown in Fig. 8.1

Fig. 8.1
(i)

Draw a diagram of a circuit which you could use to take measurements to obtain
this characteristic.
[2]

8866/02/AJC2008
[Turn over

18
(ii)

The thermistor, the characteristic of which is given in Fig. 8.1, is used in the circuit
shown in Fig. 8.2.

Fig. 8.2
It is found that there is a current of 95 mA from the supply. Calculate
1.

the current through the 150 resistor,

current = .. A [1]
2.

the resistance of the thermistor,

resistance = .. [2]
(iii)

When the temperature of the thermistor in the circuit in Fig. 8.2 is kept at 0 0C by
immersing it in a mixture of ice and water, it is found that the current from the
supply becomes 75 mA. The resulting values of V and I for the thermistor no longer
lie on the graph of Fig. 8.1. Suggest why the point corresponding to these values is
not on the characteristic.

...

...[2]
8866/02/AJC2008

19
(i)

Write down an expression for F, the force on a long, straight conductor of length l
carrying a current I at an angle to a uniform magnetic field of flux density B.
[1]

(ii)

Draw a clear diagram to illustrate the direction of the force, relative to the direction
of the current and magnetic field, on the conductor in b(i). Label the force F.
[2]

(iii)

(b)

State two methods of increasing the force on a current carrying conductor, that is
within a magnetic field.
...

...

...[2]

(c)

Two long straight parallel wires are separated by a distance d. Each carries a current I in
the same direction.
(i)

Explain, with the aid of sketches, the forces which exist between the two wires.
Indicate the directions of the forces.
[3]

8866/02/AJC2008
[Turn over

20
(ii)

B, the magnetic flux density due to a long straight wire, is given by the expression
B = 0I / 2d
Derive an expression for the force per unit length between the two wires.

(d)

[2]

One particular overhead powerline consists of two parallel cables with a separation of
6.0 m. The current in the cable is 200 A.
(i)

Calculate the force per unit length on each cable.

force per unit length = .. N m-1 [1]


(ii)

Hence explain why it is not possible, by looking at the cables, to detect the instant
at which the current is switched on.
...

...

...[2]

8866/02/AJC2008

21

8866/02/AJC2008

SolutiontoH1PhysicsPrelimPaper1
1

10

11

12

13

14

15

16

17

18

19

20

21

22

23

24

25

26

27

28

29

30

A
Ave reading = 6.0mm (far from actual value of 5.0mm) high systematic error.
Range of scatter = 0.2mm (small scatter) low random error.

Q
V
Q2
=
W
A2 s 2
[C ] =
kgm 2 s 2
C=

= A 2 kg 1m 2 s 4
3

T = 2

L
g

4 2 L
T2
Pg = PL + 2 PT
g=

= 0 .7 + 2 0 .1
= 0 .9 %
g = 0.9% 9.81
= 0.09ms 2
4

A
:

vy = uy + at
= -51.2 sin 44.5 + 9.81(1.5)
= -21.2 ms-1

vy is acting upwards, hence location of projectile must be before point A.


7

A
Taking the two masses as one system, there is only one force acting on the system to
cause motion, namely the weight of the 1 gram mass.

0.001 9.81 = 100.001 a


a = 9.81 10 5 ms 2
8

B
Without external net force, momentum is conserved. Any motion resulting from collision in
which the colliding bodies stick together is inelastic.

A
F

10 B
For top magnet: F = Mg
For bottom magnet: R = Mg + F = 2Mg
The balance reads R = W = 2Mg

M
R
Mg + F
R

11 C
For 3 forces, all forces must pass through a common point for the system to stay in
equilibrium.
12 D
F = kx
= 200 [(65-50)x10-3]
=3N

13 C
mv2 = mgh the value of h is the same since v remains unchanged and h is indept of m

14 B

1 2 1 2
kx = mv
2
2
2
d
mv 2
=
d ' 2 (4m)(2v) 2
d ' = 4d

15 A
P = Fv

F const a const v increases linearly


P increases linearly

16 C
Find the relationship between the amplitude of the wave and the distance from the source.

I A2

1
r2

Therefore A

1
. The amplitude is inversely proportional to the distance. Hence, when the
r

amplitude is twice (A to 2A), the distance is halved (4d to 2d).


17 D
The light wave emerging from P2 has oscillation along the 45 polarising axis. This light
wave is blocked only when it passes through a polariod of polarising axis perpendicular to
its oscillation. Hence the polarising axis of P3 is 45 + 90 = 135
18 C
first resonance : 0.21 = (1/4) wavelength
Next resonance : (3/4) wavelength = (3/4)( 4 x 0.21) = 0.63 m

19 D
intensity is proportional to square of amplitude

20 C
path difference = AX BX = 90 mm = (n + )
n = 1, = 60 mm, n = 2 , = 36 mm (not in the option)
Hence, C is the ans.
(note : n=0, = 180 mm , not minimum)
21 D
22 B
Using R = l / A
Rl
Hence R = 3 x 5 = 15
23 B
p.d across X = (R / 3R) x 6 = 2 V
Voltmeter reading is between 0 V and 2 V since it records p.d lower than the actual value
as it records 3 V rather than (2R / 3R) x 6 = 4 V .
24 C
2 resistor between K and N does not contribute to overall resistance.
Hence overall resistance = [1/(2 + 2) + 1/(2 + 2)]-1
=2
25 A
E = Blv when the wire is moving perpendicular to both the length of the wire and the
magnetic field


26 D
P
N
BEarth

Bcurrent
Bresultant

Q
The direction of the magnetic field caused by the current is anti-clockwise. Using vector
addition, the arrow would most likely be horizontal.
27 D
The direction of the current is parallel to the direction of the B field.
28 C
= h/p

KE = p2/m = qV => p = (2qmV)

=> = h/(2qmV)

0/ 1 = (2qmV1) /(2qmV0) = (1000/10)= 10


29 A
P has higher energy , lower than that of Q
In the options, only A satisfies the conditions.
30 B
intensity = nhf/t per unit area
n is the no. of photons.

n/t intensity

n/t affects ne/t

1
2008 H1 Prelim Paper 2 Suggestion Solutions
Section A
1(a) It is the product of a force and the perpendicular distance from the line of action of the
force to a pivot
(bi)

Let x be the side of a box.


Take moment about an axis through A:
Fx
=
W(x)
F
=
890/2 = 445 N

(ii)

445 N

(iii)

To overcome the same anti-clockwise moment by the weight, the minimum possible
force must act perpendicular to the distance furthest away from pivot A.
Fmin y =
W(x)
Fmin 2 x
Fmin

W(x)

445/ 2 = 315 N

2 a) It means that when two or more waves of the same type superpose or meet at a point in
space, the resultant amplitude at the point is the vector sum of all the individual wave
amplitudes at that point.
b) (i) Sound waves from the two sources will meet or superpose at the point D. If they meet
with constructive interference, a loud sound (maxima) will be detected. This will occur
when the path difference of the two sound waves at D is an integral multiple of the
wavelength.
If they meet with destructive interference, a soft or no sound (minima) will be detected.
This will occur when the path difference is (n + ) times of the wavelength where n is
any integer.
For the arrangement above, the path difference is fixed at the point D. When the
frequency of the sound is changed, the wavelength will change accordingly. At certain
values of frequency, ( hence its wavelength), when it fulfills the conditions for maxima
or minima as stated earlier, maxima and minima will be observed at the point D.
(ii)

L2D = (92 +402) = 41 m


Path difference = L2D L1D = 41 40 = 1 m
For first minimum, path difference = = 1
From v = f
= v/f = 340/f
Hence, (340/f) = 1
f = (340) = 170 Hz

(iii)

f = 510 Hz, = 340/510 = 0.667 m


using two-source interference equation, = ax/D,
maxima separation x = 0.667 x 40/9 = 2.963 m
zero order maxima is 9/2 =4.5 m below D.
first order maxima is 4.5 2.963 = 1.537 = 1.54 m below point D

8866/02/AJC2008

2
3 a) Magnetic flux density at a point is the force per unit length of conductor per unit current
carried, placed at that point at right angles to the field.
The tesla is newton per metre per ampere. (N m-1 A-1)
bi)

I
F(1m)
I

pivots

rider

MAGNET

ii) Clockwise moments =


Anti-clockwise moments
Fy
=
mgx
BIzy
=
mgx
B = mgx / Izy
iii) The wire frame tilts in the opposite direction OR the wire frame is no longer balanced.
The force on the edge of the frame nearest to the magnet is in the opposite direction.
4

(a) Energy required = 0 (-13.60) = 13.60 eV = 13.60x1.60x10-19 = 2.18x10-18J


(b)(i) The electron of the atom will absorb 10.19 eV of energy and be excited to the n=2
level. The incident electron will have 0.81 eV of energy left.
(ii) no change because the energy of the photon does not match the energy of any
transition (from the ground state)
(c) Energy released = [-0.849-(-13.60)] = 12.75 eV
= 12.75x1.60x10-19=2.04x10-18 = hc/
= 6.63x 10-34 x 3.00x108/2.04x10-18= 9.75 x 10-8 m
(d) Ultraviolet region
(e)

n=
n=4

0
- 0.545 eV
- 0.849 eV

n=3

- 1.51 eV

n=5

n=2

- 3.41 eV

n=1

8866/02/AJC2008

- 13.60 eV

3
5 (a)

From Fig. 7.2,


area under the graph = vertical displacement traveled by mass from A to C
= height of circle
= 32 (0.5 5010-3)
= 0.800 m (3sf)
height of circle = diameter of circle
r=

0.800
= 0.400 m (3sf)
2

Check:
Using COE: Total Energy at C = Total Energy at A

1
1
2
2
mvc + mgh = mv A
2
2
1 2
1 2
vc + gh = v A
2
2
2
2
v A vc
52 32
h=
=
= 0.81549 m
2g
2 ( 9.81)
r =

h
= 0.408 m (3sf)
2

(b)

Fig. 7.3

Fig. 7.2

ay =

3.5 0.4
= 23.8 ms -2 (3sf)
170 300 ) 103
(

ax =

8866/02/AJC2008

0 ( 4 )
= 19.5 ms -2 (3sf)
3
(100 305 ) 10

4
(ii)

ay

aresultant

ax

( 23.8 ) + (19.5)

Resultant acceleration =

= 30.8 ms -2 (3sf)

19.5
= 39.3 (3sf)
23.8

= tan 1

The direction of the acceleration is 39.3 south of west.


(c)
The total area between the curve and the time axis is zero.
This is because the net horizontal displacement traveled by the mass M from A to C is
zero.

Section B
6(ai)
Fnet = mg - f
(1800)a = (1800)(9.81) 4400
a = 7.366 ms-2 = 7.37 m s-2
v2 = u2 + 2as
= 0 + 2(7.37)(3.7)
v = 7.3850 = 7.38 m s-1

(ii)

(iii)

<P>

(iv)

Total initial energy of elevator = Total final energy of spring


mgx + mv2 = kx2 + fx
(1800)(9.81)(0.9) + 0.5(1800)(7.3850)2 = 0.5(0.9)2k + 4400(0.9)
k = 1.507 x 105 = 1.51 x 105 N m-1

(v)

k = 0.08k = 0.08(1.507 x 105) = 0.1 x 105


k = (1.5 0.1) x 105 N m-1

(b)

0 kg.
When cable breaks, woman & scale fall with the same acceleration, g = 9.81 m s-2. The
reaction force between them is zero. Hence scale reads zero.

= Fnet <v>
= (1800)(7.37)()(7.3850) / 1000
= 48.985 = 49.0 kW

8866/02/AJC2008

5
(ci)

The total momentum of a system is constant provided no external resultant force acts on
it.

(ii)
1.

v = 25 cos30 = 21.7 m s-1


direction = horizontal to the right

2.

before collision

after collision
v sin

25 cos 30

200

v cos

By COM in vertical direction: total initial p = total final p


:
(mu)pellet + (mu)skeet = (mv)pellet + (mv)skeet
0.015(200) + 0 = (0.25 + 0.015) v sin
v sin = 11.321
3.

s = ut + at2
7.96 = (-v sin )t + (9.81) t2
t = 2.87s

4.

No, because the drag force due to air resistance acts as an external force acting on the
system. This changes the momentum of the system.

7 a)

Wavelength is the distance between two consecutive points in a wave that are at the
same stage of oscillation.
Frequency f is the number of complete oscillations per unit time made by a point in a
wave.
Time taken by a wave to travel a distance of one wavelength is the period T = 1/f
Speed = distance / time => v = /T = /(1/f) = f

b)

v=f

c)

5 = (1/4)

(i)
(any two)

= 5x4 = 20 m

8866/02/AJC2008

6
(ii) Mark the nodes in each sketch.
(iii) The node N means a point where it has permanent zero displacement, no
oscillation.
(iv) Only certain stationary waveforms are possible to be formed on the string with the
two ends as nodes. That is, the string will oscillate with certain values of the
frequency. In order to observe the stationary waveforms, the frequency of the
vibrator must be equal to the certain values of the frequency of the string. This is
called resonance effect. Hence stationary wave is observed at particular
frequencies of the vibrator.
(v) v = f = 40x 0.55x2 = 44 ms-1
(vi) Adjust the frequency of the vibrator until a stationary waveform is observed. Note
the frequency f and count the number of antinodes N formed points where
maximum amplitude is observed.
Vary the frequency of the vibrator until another waveform is observed and note the
value of f and N
Repeat the experiment 6 times and plot a graph of f against N. the graph obtained
should be a straight line passing through origin if the statement is true.
d)

(i) It refers to the phenomenon that when some metal surfaces are illuminated by light
or electromagnetic radiation of certain frequencies, electrons are emitted from the
surfaces.
(ii) E = hf
(iii)
1. using hf = + KE
hf = hc/ = 6.63x 10-34 x 3.00x108/(256x10-9x1.60x10-19) = 4.914 eV
4.914 = 3.6 + KE => KE = 4.914 3.6 = 1.314 = 1.31 eV
2. KE = qV stopping potential V = 1.31 V
3. = hf0

e)

(i)

f0 = /h = 3.6x1.60x10-19/6.63x10-34 = 8.69x1014 Hz

nhf/t = intensity x area

where f = c/

n/t = 60x10-3 x7.5 x 10-4/ [(6.63x10-34x 3.00x108)/546 x 10-9]


= 1.235 x 1014 = 1.24 x 1014
(ii)

current = (n/t)x1%xe = 1.235 x 1014 x 10-2x 1.60 x 10-19


= 1.976 x 10-7 A = 2.0 x 10-7 A

8866/02/AJC2008

7
8.(ai)

mA

(ii)

1. p.d across the 150 resistor = 9V. Hence current through,


I = V/R
= 9 / 150 = 60 m A
2. Current through the thermistor = 95 60 = 35 mA
From the graph, p.d across the thermistor = 5.6 V
Rthermistor = V / I = 5.6 / 35 x 10-3 = 160

(iii)

Fig 4.1 shows characteristic values of the thermistor where the resistance decreases as
the temperature increases and this can be seen from the graph, as the p.d across
thermistor increases, the temperature increases and resistance decreases.
When placed in ice, its temperature is kept constant and its resistance ios maintained at
a certain value. Therefore values recorded will not fall on the characteristic.

bi)

F = BIl sin

ii)

F = BIl sin
I

iii)

1m correct indication of force


1m correct indication of , I and l

- Increase magnetic flux density


- Increase current in conductor
- Increase length of conductor.

8866/02/AJC2008

8
B

ci)

d
Consider 2 conductors X and Y each carrying a current I directed out of the plane of the
paper.
Current in X causes a magnetic field at Y. Direction of magnetic field at Y is vertically
upwards. According to FLHR, Y experiences a force directed towards X.
Similarly,
Current in Y causes a magnetic field at X. Direction of magnetic field at Y is vertically
downwards. According to FLHR, X experiences a force directed towards Y.
Hence there is a force of attraction between them.
ii)

The field of one wire is perpendicular to the other, hence, = 900.


F = BIL ; Force per unit length = F / L = B x I = B = 0I / 2d x I
= 0I2 / 2d

di)

F = 0I2 / 2d

ii)

This force is very small, compared to the weight of the cables. Hence the movement is
not visible by just looking at cables.

= (4 x 10-7 x 2002) / 2 (6) = 1.33 x 10-3 Nm-1

8866/02/AJC2008

Name: ..

HT group: ...

CATHOLIC JUNIOR COLLEGE


JC2 PRELIMINARY EXAMINATION 2008
PHYSICS
Higher 1

8866/ 1

PAPER 1 Multiple Choice


Friday

12 September 2008
1 hour

Additional materials: MCQ answer sheet


Soft clean eraser
Soft pencil (type B or HB is recommended)

READ THESE INSTRUCTIONS FIRST


Write in soft pencil
Do not use staples, paper clips, highlighters, glue or correction fluid
Write your name and tutorial group on the Answer Sheet in the spaces provided unless this
has been done for you.
There are thirty questions in this paper. Answer all questions. For each question there are
four possible answers A, B, C and D.
Choose the one you consider correct and record your choice in soft pencil on the separate
sheet.
Read the instructions on the Answer Sheet very carefully.
Each correct answer will score one mark. A mark will not be deducted for a wrong answer.
Any rough working should be done in this booklet.

This question paper consists of 13 printed pages.

PHYSICS DATA:
speed of light in free space,

= 3.00 x 108 m s-1

elementary charge,

= 1.60 x 10-19 C

the Planck constant,

= 6.63 x 10-34 J s

unified atomic mass constant,

= 1.66 x 10-27 kg

rest mass of electron,

me

= 9.11 x 10-31 kg

rest mass of proton,

mP

= 1.67 x 10-27 kg

acceleration of free fall,

= 9.81 m s-2

PHYSICS FORMULAE:
s =

u t + a t2

v2 =

u2 + 2 a s

work done on / by a gas,

W =

p V

Hydrostatic pressure

P =

gh

electric potential,

V =

Q
4E o r

resistors in series,

R =

R1 + R2 + ...

uniformly accelerated motion,

resistors in parallel,

1/R =

1/R1 + 1/R2 + ...

The power generated by an ideal wind turbine is given by

P=

1
k(v b) 3
2

where k is a constant of the turbine,

is density of the fluid passing through the turbine,


v is the velocity of the fluid.

The possible units of k and b are:


Units of k
A

no units

m s-1

m2

m s-1

no units

m3

D
2

Units of b

m2

m3

Systematic and random errors may be compared by contrasting the following pairs of
properties:
P1: error can possibly be eliminated
P2: error cannot possibly be eliminated
Q1: error is of constant sign and magnitude
Q2: error is of varying sign and magnitude
R1: error will be reduced by averaging repeated measurements
R2: error will not be reduced by averaging repeated measurements
Which properties apply to systematic errors?
P1, Q1, R2

A
3

P1, Q2, R2

P2, Q2, R1

P2, Q1, R1

The true value of a quantity x is x0. In an experiment, the quantity is measured many times and
the number N of readings, giving a value x, is plotted against x.
Which of the following graphs best shows measurements that are precise but not accurate?
A

x0

x0

x0

x0

A fighter-bomber is traveling in level flight at a speed of 55 m s-1. It is on a bombing run to its


target, a tank moving at 30 m s-1 head on. Given its altitude is 400 m from the ground level,
when should the pilot release the bomb if he wants to score a direct hit? Assume negligible air
resistance.

A
B

When the plane is 450 m away from the tank.

When the plane is 770 m away from the tank.

D
5

When the plane is 230 m away from the tank.

When the plane is 1100 m away from the tank.

An object, thrown vertically upwards, rises to a certain height and then falls back to its starting
point. Assuming that air resistance cannot be neglected, which of the following statements is
correct?
A

When the object is rising, the magnitude of its deceleration is greater than the
acceleration due to gravity.

The time of flight for the upward motion is greater than the time of flight for the downward
motion.

The speed of the object when it is thrown up is equal its speed when it returns to the
starting point.

The acceleration of the object at the highest point is zero.

A disc is sliding across a horizontal, frictionless icy surface when it collides inelastically with a
wall at right angles to its path, and then rebounds along its original path. Which of the following
graphs shows the variation with time t of the momentum p of the disc?
A

A helicopter which has blades of diameter 5.0 m is hovering above the ground at a particular
instance. Its blades are rotating in such a way that they are pushing air downwards at a speed
of 18 m s-1. The density of the surrounding air can be taken as 1.02 kg m-3. The upward force
acting on the blades is
A

360 N

1400 N

6500 N

26000 N

An object of mass 20 kg, initially at rest moves along a straight line on a smooth horizontal
surface. A force F acts on the object in its direction of motion. In the figure below, a graph of F
against time t is shown
F/N

20

t/s

What is the velocity of the object at t = 6 s?


A

1.5 m s-1

3.0 m s-1

4.5 m s-1

7.0 m s-1

Figure 9.1 shows the directions and lines of action of two forces applied to a circular disc.
Which one of the arrows in Figure 9.2 best represents the line of action of the third force that
will keep the disc in equilibrium?
D
3N
C
4N

4N

B
Figure 9.1
10

Figure 9.2

A rod with negligible weight is suspended from three identical springs. With a weight W
hanging from the middle of the rod, the extension of each spring is x. The middle spring is then
removed and the weight increased to 3W.
What is the new extension of each spring?

W
A

5x
2

7x
2

9x
2

11x
2

11

The weight indicated on a balance is X when a beaker of water is placed on it. A solid object
has weight Y in air and displaces weight Z of water when completely immersed. The given
diagram shows the object suspended from a spring balance and completely immersed in the
beaker of water.
What are the readings on the spring balance and the weighing machine in the given
arrangement?
Spring
balance

Weighing
machine

Weight balance
X

Y+Z

X+Y-Z

Y+Z

X+Y

D
12

Spring balance
YZ

Y-Z

X+Z

A ball, thrown vertically upwards, rises to a height h and then falls to its starting point. Air
resistance may be taken as negligible. Which graph best shows the variation of kinetic energy
Ek of the ball with the distance s travelled?
A

Ek

Ek

Ek

Ek

13

The diagram shows an arrangement which consists of a wheel of circumference 0.40 m driven
by the electric motor of overall efficiency 80 %. A rope is passed over the wheel. One end of
the rope is attached to a spring balance and the other end supports a load of 100 N. When the
wheel is turning at a rate of 50 rev s-1, the balance reading is 25 N.
50 rev s-1

Spring balance
reading = 25 N

load = 100 N

What is the electrical power supplied to the electric motor?


1.50 kW

A
14

2I

2.34 kW

2.93 kW

3I

4I

The principle of superposition applies only if


A

the waves travel with the same speed

the sources of the waves are coherent

the waves have the same frequency

D
16

Two coherent waves each of intensity I meet in phase at a point K. What is the resultant
intensity at K?
A

15

1.88 kW

the waves are of the same kind

S1 and S2 are two identical, small loud-speakers 0.50 m apart and connected to the same
audio frequency generator. They vibrate in phase producing sound waves of wavelength 0.40
m.
S1

P
S2
A microphone detects a minimum in the pattern of superposition at the point P. If P is 12.00 m
from S2 and PS1 > PS2 then the least possible distance of P from S1 is
A

12.15 m

12.20 m

12.40 m

12.60 m

17

Air filled pipes of equal length are represented in figure below.

fa

fb

fc

In what ratio are the frequencies of the fundamental vibrations of the pipes?
A
18

1: 2: 1

1: 2: 3

2: 1: 2

3: 2: 1

The graph below shows the variation of the current I through a lamp with potential difference V
across it.

Which of the following graphs shows best represents the variation of power P dissipated in the
same lamp, with I2?
A
B
C
D

19

A 10 resistor is connected across three identical cells each having an e.m.f. of 1.5 V and
internal resistance of 3 .

10
What is the current flowing through the resistor?
A

0.079 A

0.12 A

0.14 A

1.7 A

20

Four identical lamps, each carrying a label 1.5 V, 0.5 A, are connected to a cell of 12 V and
zero internal resistance as shown in the circuit below. The resistor R is added to the circuit so
that the lamps are working at normal brightness.

What is the value of R?


2.3
A
B
21

3.3

5.3

6.3

In the circuit shown, a potential difference of 8 V is applied across XY. X is at a higher


potential relative to Y.
A

What is the potential at A if B is earthed?


A
22

-1V

+1V

-5V

+5V

An electron is moving along the axis of a solenoid carrying a current. Which of the following is
a correct statement about the electromagnetic force acting on the electron?
A

The force acts radially inwards.

The force acts radially outwards.

The force acts in the direction of motion.

No force acts.

10

23

A small plastic sphere carrying a negative charge is maintained at a constant height by the
action of a downward vertical electric field.
A uniform magnetic field is applied in the same direction as the electric field.
What does the sphere do?
A
B

Move in a horizontal circle

Move upwards in a spiral path

D
24

Move downwards in a spiral path

Remain stationary

An electron enters the space between two parallel charged plates with an initial velocity u.

While in the electric field its direction changes by 30 and it emerges with a velocity v.
What is the relation between v and u?
A

25

v=

u
cos30 o

v = u cos30 o

v=

u
sin30 o

v = u sin30 o

The figure below shows four long, straight current-carrying wires P, Q, R and S. which are
perpendicular to the plane of the paper. They pass through the corners of a square. Point O is
the point of intersection of the diagonals of the square. The currents in all four wires have the
same magnitude. The currents in wires P, Q and R flow into the plane of the paper while that
in S flows out of the plane of the paper. Which arrow shows the direction of the resultant
magnetic field at O?
Q
P
A
B
O

C
D
S

11

26

A straight, horizontal, current-carrying wire lies at right angles to a horizontal magnetic field.
The field exerts a vertical force of 8.0 mN on the wire.
The wire is rotated, in its horizontal plane, through 30 as shown. The flux density of the
magnetic field is halved.

What is the vertical force on the wire?


A

27

2.0 mN

3.5 mN

4.6 mN

8.0 mN

The work function of a metal may be defined as


A

the minimum wavelength of the incident electromagnetic radiation required to cause


photoelectric emission.

the minimum energy of photons incident on a surface required to cause photoelectric


emission

D
28

the minimum frequency of the incident electromagnetic radiation required to cause


photoelectric emission

the minimum energy required to take an electron from the interior to the surface to cause
photoelectric emission.
A simple model of the energy levels in an atom has only three levels, X, Y, and Z. A transition
from level X to level Z produces radiation of wavelength 400 nm; a transition from level Y to
level Z produces a radiation of wavelength 700 nm. Which of the following deductions made
by the student about the energy level scheme is correct?

A
B

The wavelength of radiation absorbed in a transition from levels X to Y is 300 nm

Level Y has a greater energy than levels X and Z

D
29

The wavelength of radiation emitted in a transition from level X to Y is 300 nm

Levels X has a greater energy than levels Y or Z

White light from a tungsten filament lamp is passed through sodium vapour and viewed through
a spectrometer. Which of the following best describes the spectrum which would be seen?
12

A
B

coloured lines on a white background

dark lines on a coloured background

D
30

coloured lines on a black background

dark lines on a white background


The graph shows the variation with frequency f of the maximum kinetic energy Ek of
photoelectrons emitted from a metal surface S.
Ek

Which one of the following graphs shows the corresponding variation for a metal surface with a
higher work function? The dotted line on each graph below shows the variation for the metal
with the higher work function and the solid line shows the variation of metal S.
A

Ek

Ek

Ek

Ek

THE END

13

Name: ..

HT group: ...

CATHOLIC JUNIOR COLLEGE


JC2 PRELIMINARY EXAMINATION 2008
PHYSICS
Higher 1

8866/ 2

PAPER 2 Structured Questions


Wednesday

27 August 2008
2 hours

Candidates answer on the Question Paper


No additional materials are required.

READ THESE INSTRUCTIONS FIRST


Write your name and tutorial group on all the work you hand in.
Write in dark blue or black pen in the spaces provided on the Question Paper.
You may use a soft pencil for any diagrams, graphs or rough working
Do not use staples, paper clips, highlighters, glue or correction fluid
Section A
Answer all questions
Section B
Answer any two questions
The number of marks is given in brackets [ ] at the end of each question or part question.
For examiners Use
Section A
1
2
3
4
5
Section B
6
7
8
Total

This question paper consists of 20 printed pages.

PHYSICS DATA:
speed of light in free space,

= 3.00 x 108 m s-1

elementary charge,

= 1.60 x 10-19 C

the Planck constant,

= 6.63 x 10-34 J s

unified atomic mass constant,

= 1.66 x 10-27 kg

rest mass of electron,

me

= 9.11 x 10-31 kg

rest mass of proton,

mP

= 1.67 x 10-27 kg

acceleration of free fall,

= 9.81 m s-2

PHYSICS FORMULAE:
s =

u t + a t2

v2 =

u2 + 2 a s

work done on / by a gas,

W =

p V

Hydrostatic pressure

P =

gh

electric potential,

V =

Q
4E o r

resistors in series,

R =

R1 + R2 + ...

uniformly accelerated motion,

resistors in parallel,

1/R =

1/R1 + 1/R2 + ...

Section A
Answer all questions in the spaces provided.
1 Body A, of mass 2.0 kg, has a light spring attached to it. Body B has a mass of 5.0 kg and is
initially suspended by a string of length 0.10 m. Body A moves with a velocity of 3.5 m s-1 over
a frictionless plane directly towards body B as shown in Figure 1.1. The two bodies undergo an
elastic collision, during which the spring on body A is compressed as shown in Figure 1.2 after
which the spring extended. The two bodies separated after collision as shown in Figure 1.3.

Figure 1.1:
Before collision

3.5 m s-1

0.10 m
B

Figure 1.2:
Maximum compression of spring
B

V0
Figure 1.3:
After collision

VA
A

VB
(a) At a certain time during the collision as shown in Figure 1.2, the spring undergoes
maximum compression and the two bodies A and B have a common velocity V0.
(i)
Find the value of Vo.

(ii)

Vo = m s-1
What is the total kinetic energy during the maximum compression of the
spring?

[2]

Total kinetic energy = J

[1]
3

(iii)

Explain why it is less than the initial kinetic energy of Body A.

(iv)

[1]

Find the velocities VA and VB after the collision.

VA = m s-1 and VB = m s-1

[3]

2 (a) Define potential difference.


..
..
..

[2]

(b) The circuit shown in figure 2.1 below can be used as an electronic thermometer. The
battery has negligible internal resistance.

6.0 V

20.0

Figure 2.1
The reading on the digital voltmeter can be converted to give the temperature of the
thermistor T which is used as a temperature sensor.

(i)

Explain why the reading on the voltmeter increases as the temperature of the
thermistor increases.

(ii)

[2]

When the thermistor is at 20.0 C, its resistance is 24.5 . Calculate the


reading on the voltmeter.

Reading on the voltmeter = V

[2]

(c) The battery is replaced with another having the same e.m.f. but an internal resistance
of 3.0 . State and explain the effect, if any, on the measured temperature when the
thermistor is at 20.0 C.
..
..
..
..

[2]

3 The platinum sample is placed in a container with a small aperture. The emitted particle
escapes through the aperture through a series of slits such that it enters the region between
the parallel plates horizontally at point O, as shown in figure 3 below. The line OX is horizontal
and the entire set-up is placed in a vacuum. Mass of an particle is 4.0026u.

Figure 3
The particles, which have a net charge of +2e, are observed to exit from the region between
the plates at point Y, located 4 mm vertically below X. Calculate
(a) the electric field strength between the plates

electric field strength = . N C-1

[2]

(b) the acceleration of the particles between the plates.

acceleration= m s-2

[2]

speed = . m s-1

[3]

(c) the speed of the particles at point O,

4 (a) Define magnetic flux density


..
..

[2]

(b) Figure 4.1 shows a plan view of a solenoid. On Figure 4.1, sketch the pattern of the
magnetic field in the solenoid and near its ends.

Figure 4.1

[2]

(c) A copper wire has a resistivity of 1.7 x 10-8 m and a diameter of 2.0 mm. Find the
resistance of a 10 cm length of copper wire.

resistance = .

[2]

(d) The copper wire in (c) is placed across one end of the solenoid where the magnetic
flux density is 5.0 x 10-4 T. There is a current of 2.8 A in the copper wire. The
diameter of the solenoid is 0.10 m. Find the force acting on the copper wire in (c)
when it is placed just outside the solenoid as shown in figure 4.2.

Figure 4.2

force = .. N

[1]
7

(e) (i)

The density of the copper wire is 8920 kg m-3. Find the weight of the copper
wire in (c).

weight = .. N
(ii)

[2]

State one change that can be done to cause the magnetic force acting on the
wire in (c) to be equal to the wires weight.

[1]

5 When a car has a brake test, two sets of measurements are made:
1.

the maximum braking force on the wheels produced by operating the foot brake,

2.

The maximum braking force produced by operating the hand brake.

Typical data for a car of mass 900 kg are as follows.


1. Foot brake

Maximum braking force/ N


6700

2. Hand brake

2000

In order to determine whether or not the brakes are satisfactory, the data are applied to a
chart (called a nomogram) like the one shown in Figure 5. This chart has three vertical
lines, marked with scales.

Figure 5
The central vertical line is for the maximum braking force. The left line is for the mass of
the car. The right line is for the braking efficiency and also for the stopping distance from
an initial speed of 20 m s-1.
The braking efficiency E is defined by the equation
E=

deceleration of car
x 100
acceleration of free fall

As an example of the use of this chart for the car of mass 900 kg, the figures in the table
show a maximum braking force for the foot brake of 6700 N. The point A corresponding
to the mass and the point B corresponding to the braking force are joined to give a
straight, sloping line. This line is extended to cut the braking efficiency scale at the point
C, and shows that in this particular case the stopping distance S from a speed of 20 m s-1
is about 27 m.
(a) Read from the chart the braking efficiency corresponding to point C.
.. %

[1]

(b) Using the definition of braking efficiency given above, find the deceleration
corresponding to this value of braking efficiency. Give your answer in m s-2.

.. m s-2

[2]

(c) Show, by calculation from the equations of motion, that the deceleration you obtained
in (b) gives a stopping distance of 27 m to 2 sig. fig. from an initial speed of 20 m s-1.

[2]
(d) (i)
(ii)

Draw a line on the chart to represent the results of the hand brake test on the
car of mass 900 kg.

[1]

Using the hand brake alone,


1.

what would be the stopping distance from a speed of 20 ms-1,


. m

2.

[1]

. %

[1]

what is the braking efficiency?

End of Section A

10

Section B

Answer two questions from this section.


6 (a) State the conditions for the equilibrium of a body, acted upon by a number of forces.
..
..
..

[2]

(b) A research submarine of mass 5.0 x 10 kg is lowered into the water by a crane. The
crane consists of an uniform arm, with a length of 4.0 m and a mass of 2.0 x 10 kg.
1.0 m
T

Arm of crane
3.0 m

submarine

50

water

Figure 6.1
(i)

Figure 6.1 above shows the submarine held in place by a horizontal cable
attached to the arm 1.0 m from the top. Calculate the tension T in the cable.

T = .. N

[3]

11

(ii)

When the submarine is totally submerged, it experiences an upthrust of 40 kN


and is held stationary by a vertical cable. Calculate the tension T in the cable.

T = N

[2]

(c) A helicopter is flying horizontally at an altitude of 20 m with a constant velocity of 8.0


m s-1. At that instant, a package of emergency food supplies is ejected horizontally
backward with a speed of 14 m s-1 with respect to the helicopter. Assume air
resistance to be negligible.
(i)

Determine the time taken for the package to reach the ground.

t = .. s
(ii)

[2]

Calculate the magnitude of the velocity of the package just before hitting the
ground.

v = m s-1

[3]

12

(d) When the traffic light turns green, a car accelerates uniformly from rest to a speed of
25.0 m s-1 in 12.0 s. The car then travels with a constant speed of 25.0 m s-1. Also, as
the light turns green, a truck travelling in the same direction with a constant speed of
17.0 m s-1 passes the car.
(i)

On figure 6.2, sketch graphs to show how the speeds of the car and the truck
vary with time during the first 20 s.
speed/ m s-1

time/ s

[3]

Figure 6.2
(ii)

How long since the light turns green will it take for the car to catch up with the
truck?

time taken = .s [2]


(iii)

What is the maximum distance between the car and the truck before the car
catches up with the truck?

maximum distance = . m

[3]
13

7 (a) Explain what is meant by the term progressive as applied to a wave.


..
......

[1]

(b) A Youngs double slit experiment was conducted in which the source is a laser of
wavelength .
The following measurements were obtained from the experiment:
silt separation,
fringe separation,
distance between slits and screen,
(i)

d = (0.50 0.01) mm
x = (2.03 0.01) mm
D = (1.50 0.01) mm

State an appropriate instrument for measuring the slit separation d.

(ii)

[1]

Determine the wavelength and express it with its associated uncertainty.

( ) = .. m

[3]

(c) Two small loud speakers A and B are positioned 1.5 m apart in a large room and are
connected to the same signal generator. The loud speaker emits a note of frequency 3400
Hz.
A microphone connected to an oscilloscope is placed at P which is equidistant from the two
speakers as shown in figure 7.1. Figure 7.2 shows the trace on the oscilloscope.
The speed of sound in air at room temperature = 340 m s-1.
X
P
Y

9.0 m
1.5 m
A
B
Figure 7.1
14

trace height

(i)

Figure 7.2
What adjustments should you make to the oscilloscope to obtain the trace as
shown in figure 7.2?

[1]

(ii)

State clearly what the trace height represents.

(iii)

[1]

As the microphone is moved along the line XY, the trace height passes through
a series of maxima and minima, with a maximum at P. Explain the variation in
trace height as the microscope is moved along XY.

(iv)

[2]

Calculate the approximate distance from P to the first minimum at Q.

[2]
(v)

The frequency of the signal generator is increased gradually. Suggest and


explain qualitatively what an observer would hear if he remained stationary at
some point Q.

15

[3]

(d) In order to investigate stationary waves on a stretched string, a student sets up the
apparatus illustrated in figure 7.3.
pulley

vibrator

weights

Figure 7.3
(i)

What is meant by a node? Explain why a node must exist at the pulley?

(ii)

[2]

The distance between successive nodes on the string is 16.0 cm when the
frequency of the vibrator is 75 Hz. Calculate the speed of the wave on the
string.

speed = . m s-1
(iii)

[2]

On figure 7.3, show the stationary wave on the string when the frequency is
such that the distance between the vibrator and the pulley corresponds to two
wavelengths of the wave on the string.

[2]

16

8 (a) Explain what is meant by the photoelectric effect.

[2]

(b) Figure 8.1 shows the experimental setup used to investigate the photo-electric effect.

Figure 8.1
The frequency f of the incident radiation is first increased to obtain the graph shown
in figure 8.2 where I is the current obtained in the ammeter. The frequency is then
kept constant but the voltage V is varied to obtain the graph shown in Figure 8.3.
The intensity of the radiation was kept constant throughout the experiments.

Figure 8.2

Figure 8.3

17

Deduce from the graphs the following:


(i)

The number of photo-electrons emitted in 1 minute.

number of photons emitted per minute = .. min-1


(ii)

[2]

The work function of the metal.

= . J

(iii)

[1]

= .

[3]

The wavelength of the incident radiation.

18

Energy/ eV

(c)

-0.54

5
4

486.1 nm

3
656.3 nm

-1.51

434.0 nm

-3.40

-13.60
Fig. 8.4

Figure 8.4 represents a typical energy-level diagram (not to scale) for hydrogen atoms.
(i)

Describe what would happen when an atom makes a transition from one
energy level to a lower level.

(ii)

[1]

Explain why each of the transitions gives rise to a spectrum line.

[2]

19

(iii)

Determine the wavelength of the spectrum line which corresponds to transition


P.

[3]
(iv)

An electron of energy 20.0 eV collides with a hydrogen atom in its ground


state. The atom is excited to level 2 and the electron is scattered with reduced
velocity. The atom subsequently returns to its ground state with emission of
radiation. Determine the velocity of the scattered electron.

[3]
(d) Electromagnetic waves have a wave nature as well as a particulate nature. This is
known as wave/ particle duality. Describe a situation in which particles can be shown
to have a wave nature.

[3]

End of Paper

20

CJC Physics H1 Preliminary Examinations 2008


Paper 1

1
2
3
4
5
6
7
8
9
10
11
12
13
14
15
16
17
18
19
20
21
22
23
24
25
26
27
28
29
30

B
A
B
C
A
A
C
C
B
C
D
C
B
D
D
B
C
B
C
C
A
D
D
A
B
B
C
D
C
A

Paper 2 Section A

1(a)(i)

(ii)

By COM,
MAUA = (MB + MA)V0
2(3.5) = (2+5) V0
V0 = 1 m s-1
Total kinetic energy = (MB + MA)V02
= (2+5) (1)2
= 3.5 J

1
1
1

(iii)

This value is less than the initial kinetic energy of A because some of the kinetic 1
energy has been converted to elastic potential energy stored in the
compressed spring.

(iv)

As the collision is elastic, relative speed of approach = relative speed of


separation
UA UB = VB (-VA)
3.5 = VB + VA .. (1)
1

By conservation of momentum,
MA UA = MBVB - MA VA
(2)(3.5) = 5VB 2 VA ..... (2)
(1) x 2 + (2)
14 = 7 VB
VB = 2.0 m s-1 (2 s.f.)

3.5 = 2 + VA
VA = 1.5 m s-1 (2 s.f.)
2(a)

Potential difference between two points in the circuit is where electrical energy 2
is converted to other forms of energy per unit charge delivered from one point to
another.

(b)(i)

As the temperature of T increases, its resistance decreases. This will reduce 2


the pd across T, so that the pd across the resistor 20 increases.

(ii)

1
20
x6
20 + 24.5
1
= 2.7 V
The measured temperature would be lower because the pd across the resistor 2
would be less (ie. 2.53V).

(c)
3(a)
(b)

(c)

V=

E = V/ d = (2x 160 x 1x 103)/ 20 x 10-3


= 1.60 x 107 N C-1

1
1

a = F/ m = qE/ m
= 2 x 1.60 x 10-19 x 1.6 x 107/ 4.0026 x 1.66 x 10-27
= 7.71 x 1014 m s-2

1
1

Consider vertical motion,


Using sy = uyt + ay t2
0.004 = (7.71 x 1014) t2
t = 3.22 x 10-9 s

Consider horizontal motion,


Speed v = sx/ t = 0.040/ 3.22 x 10-9
= 1.24 x 107 m s-1
4(a)

1
1

Magnetic flux density at a point is the magnetic force acting per unit current in 2
a wire of unit length lying at right angles to the magnetic field.

(b)

1 mark for parallel magnetic lines within the solenoid.


1 mark for the direction of the magnetic filed.

1
1
2

(c)

R = l / A
= (1.7 x 10-8)(10 x 10-2) / [()(2.0 x 10-3/2)2]
= 5.41 x 10-4
Force = BIL
= (5.0 x 10-4) (2.8) (0.10)
= 1.4 x 10-4 N

(d)

(e)(i)

(ii)
5(a)
(b)

(c)

1
1

Weight = mg = Vg = LA g
= (8920) (10 x 10-2) [()(2.0 x 10-3/2)2 (9.81)
= 2.75 x 10-2 N

1
1

Increase the value of the current / potential difference across it.

75%

deceleration of car
x 100
acceleration of free fall
deceleration of car
75 =
x 100
9.81
Deceleration of car = 7.36 m s-1
E=

V2 = u2 + 2as
0 = 202 + 2(-7.36)s
stopping distance, s = 27 m

1
1

1
1

(d)(i)

(ii)1.

90

(ii)2.

22%

Paper 2 Section B

6(a)

No resultant force and no resultant torque.

(b)(i)

(ii)

Taking moment about the hinge of the cranes arm


Counter-clockwise moment = Clockwise moment
(5000g sin 50)(4) + (2000g sin 50)(2) = (T sin 40)(3)
T = 93530 = 93.5 kN

1
1

(ii)

1
1

Consider vertical motion and take downwards as positive,


s = ut + at2
20 = 0 + (9.81) t2
t = 2.02 s

1
1

Consider horizontal motion and backwards as positive


Horizontal component of velocity of package just before hitting the ground
= 14 8.0 = 6.0 m s-1 backwards.

For vertical component of velocity,


v y = uy + ay t
vy = 0 + (9.81)(2.02) = 19.8 m s-1

(c)(i)

T + U = Weight of submarine
T = (5000 x 9.81) 40 000
= 9050 N

Vx
Vy
V

Magnitude of velocity,
V = 19.8 2 + 6 2 = 20.7 m s-1
(d)(i)

speed/ m s-1
25.0

car

17.0

truck

0
0

time/ s
12.0

20.0

Fig. 6.1
5

(ii)

Let t be the time taken.


At time t, both vehicles have travelled the same distance.
[ t + (t 12.0)] (25.0) = 17.0 t
t = 18.8 s

(iii) Let t be the time when the speed of the car reaches 17 m s-1.
v = u + at
17.0 = 0 + (25.0/12.0) t
t = 8.16 s
At time t, the distance between the 2 vehicles is a maximum.
Maximum separation = 17.0 8.16 = 69.4 m

1
1
a

7(a)

Progressive implies that the energy is being transferred when the wave is 1
travelling away from the source.

(b)(i)

Travelling microscope

(ii)

xd 2.03 x 0.5
=
= 0.677 mm
D
1.5
x d D
=
+
+

x
d
D
0.01 0.01 0.01
=
+
+
= 0.0316
2.03 0.5 1.5
= 0.0316 x 0.677 = 0.02 mm

( ) = (0.68 0.02) mm = (0.68 0.02) x 10-3 m

Adjust y-sensitivity to get the required height


Adjust time base to get a few cycles

(ii)

amplitude of the resultant waves

(iii)

Constructive interference occurs at maxima where the two waves arrive in


phase and the microphone detects maximum loudness.
1
Destructive interference occurs at minima where the two waves arrive in
antiphase and the microphone detects minimum loudness.
1

(iv)

= v / f = 340 / 3400 = 0.1 m


PQ = x = ( D / a) = (0.1) ( 9) / 1.5 = 0.3 m

1
1

(v)

Alternate loud and soft sound is detected.


As the frequency is increased, wavelength decreases.

(c)(i)

When path difference = 3/2 5/3 , destructive interference occurs and soft 2
sound is heard. When path difference = , 2, 3, constructive interference
occurs and loud sound is heard. Hence point Q passes through a series of
alternate constructive and destructive interference.
Alternate explanation.
The distance between adjacent maximum and minimum decreases as
6

wavelength decreases gradually, according to x = D / a.


Hence the positions of the maxima and minima will be shifted towards the
central axis. Hence Q will passes through alternate maxima and minima.
(d)(i)

A node is a point along a stationary wave with no displacement.

The string is tightly adhered to the rim of the pulley due to the weights, so a 1
node has to be there.
(ii)

Distance between successive nodes = x = 16.0 cm


= 32.0 cm

Speed of the wave = f x = 75 x 32 x 10-2 = 24 m s-1

(iii)

pulley
weights

vibrator

Photoelectric effect is a phenomenon that results in the liberation of electrons 1


from a metal surface when EM radiation of high enough frequency falls on the 1
metal. The electrons emitted are called photoelectrons.

8(a)

(b)(i)

No. of electrons = Q/ e = 2.4 x 10-7 / 1.6 x 10-19 = 1.5 x 1012


(ii)

Q = It = 4.0 x 10-9 x 60 = 2.4 x 10-7 C

= hfo = 6.63 x 10-34 x 5.0 x 1014 = 3.32 x 10-19 J

(iii) Kmax = eVs = 1.6 x 10-19 x 1.1 = 1.76 x 10-19 J

Energy of photons, hf = Kmax +


hc/ = 1.76 x 10-19 + 3.32 x 10-19
= 3.92 x 10-7 m
(c)(i)

It will emit a photon whose energy is equal to the energy difference between the
initial and final states

1
1
1

(ii)

Hydrogen atoms have discrete energy levels and a specific transition between 2 1
energy levels of an atom
emits radiation of a certain wavelength giving rise to a sharp line
1

(iii)

hc 6.63x10 34 3 10 8
=
= 4.09 x 10-19 = - 2.56 eV
9

486.1 10
E4 = -3.40 + 2.56 = - 0.84 eV
E2 E4 =

-19

E = 1.51 0.84 = 0.67 eV = 0.67 x 1.60 x 10 J


hc 6.63x10 34 3 10 8
=
=
= 1.86 x 10-6 m = 1860 nm
-19
E 0.67 x 1.60 x 10

1
1
7

(iv)

Energy of the scattered electron = 20 [13.6 3.40] = 9.8 eV


1
mv 2
2
2E
2 x 9.8 x 1.6 x 10 -19
=
= 1.9 x 106 m s-1
v=
m
9.11 x 10 -31
When electrons are passed through a thin layer of graphite, which acts like a
diffraction grating,
a well formed interference pattern is obtained, similar to that obtained from
interference of waves.
This effect gives evidence for the wave nature of particles.

E=

(d)

2
1
1
1

Name

Class (6C

DUNMAN HIGH SCHOOL


SENIOR HIGH
PRELIMINARY EXAMINATION 2008
YEAR SIX
H1 PHYSICS
______________________________________________________________________________

Date: 23 September 2008 (Tuesday)


Paper 1

Duration: 1 hour

Multiple Choice

READ THESE INSTRUCTIONS FIRST


DO NOT TURN THIS PAGE OVER UNTIL YOU ARE TOLD TO DO SO.
Write your name and class at the top of this page.
There are 30 questions in this paper. Answer all questions.
For each question, there are four possible answers A, B, C and D. Choose the one you
consider correct and record your choice in pencil in the separate Answer Sheet.
Each correct answer will score one mark. A mark will not be deducted for a wrong answer.
Any rough working should be done in this booklet.

This question paper consists of 14 printed pages (including this cover page).

[Turn Over

2
Data

speed of light in free space,

3.00 x 108 m s-1

elementary charge,

1.60 x 10-19 C

the Planck constant,

6.63 x 10-34 J s

unified atomic mass constant,

1.66 x 10-27 kg

rest mass of electron,

me

9.11 x 10-31 kg

rest mass of proton,

mp

1.67 x 10-27 kg

acceleration of free fall,

9.81 m s2

Formulae
uniformly accelerated motion,

1 2
at
2

= ut +

v2

= u2 + 2as

work done on/by a gas,

= pV

hydrostatic pressure,

= gh

electric potential,

resistors in series,

= R1 + R2 + . . .

resistors in parallel,

1
1
1
+
+...
=
R
R1
R2

Q
4 o r

[Turn Over

3
1

A typical human body has a mass of 70 kg. Which of the following estimates could
be correct?
A
B

When running quickly his momentum is 70 N s.

When running at top speed, his de Broglie wavelength is 70 x 10-38 m

When walking normally, his kinetic energy is 70 J.

When standing still, the pressure exerted on the ground is 70 Pa.

The drag force experienced by a car can be expressed by the equation


F = CxAy vz
where C is a dimensionless constant, is the density of air, A is the cross-section
area of the car and v is the speed of the car. What are the possible values of x, y
and z?
x

1
2

1
2

1
2

1
1

1
2
1

Forces of 3 N, 4 N and 5 N are in equilibrium. What is the angle between the 4 N


and the 5 N forces?

37o

53o

127o

143o

[Turn Over

4
4

The graph represents the motion of a ball which is dropped to the ground and then
bounces vertically. The speed immediately after impact is half that immediately
before impact. The time interval PQ is 0.40 s.
velocity

time

What is the maximum velocity, in m s-1, acquired by the ball?


A

25

10

8.0

4.0

A cannon ball is fired horizontally from a cliff on the surface of Earth with a speed of
30 m s1. Assuming that there is no air resistance, what will its speed 4.0 s later?
A

30 m s1

40 m s1

50 m s1

60 m s1

In the absence of air resistance, a stone is thrown from P and follows a parabolic
path in which the highest point reached is Q.
Q
P
The vertical component of acceleration of the stone is
A

zero at Q

greatest at P

greatest at Q

the same at P and Q

[Turn Over

5
7

Newtons third law of motion concerns the forces of interaction between two bodies.
Which of the following statements relating to the law is false?
A
B

The two forces are always opposite in direction.

The two forces are at all times equal in magnitude.

The two forces must act on different bodies

The two forces are equal and opposite so that the bodies are in equilibrium.

A man is inside a lift which is accelerating downwards. Which statement is correct?


A

The force exerted by the lift on the man and the weight of the man are actionreaction forces.

The force exerted by the lift on the man is equal to the weight of the man.

The weight of the man is equal to the gravitational force of attraction of the
Earth on him.

The tension in the lift cable is equal to the weight of the man and lift.

The graph illustrates the variation with time of the force acting on a body.
force / N
6
4
2

time / s

What is the increase in momentum of the body during the first 3 seconds?
A

24 N s

12 N s

6.8 N s

1.5 N s

[Turn Over

6
10

A non-uniform beam hangs horizontally from two spring balances as shown. The
forces recorded by the balances are F and 2F.

2F

non-uniform beam

L
2
L

What is the horizontal distance of the centre of gravity of the beam from the right
hand end of the beam?
A

11

L
6

L
4

L
3

5L
6

A tensile force of 6.0 N extends a given spring by 12 cm. A 3.0 N load is attached to
two such springs arranged as shown.

3.0 N
What is the extension in each spring?
A

3.0 cm

4.0 cm

6.0 cm

12.0 cm

[Turn Over

7
12

Two liquids X and Y are placed in a U-tube. If liquid X has a lower density than
liquid Y, which of the following correctly shows the levels of the two liquids?

13

A 2.0 kg block is pushed against a spring of negligible mass and force constant k =
400 N m-1, compressing it by 22 cm. When the block is released, it moves along a
frictionless horizontal surface and then up a frictionless incline with slope 30o as
shown.
x

30o
22 cm

What is the distance x, in cm, that the block travel up the incline before starting to
slide back down?
A

49

99

197

449

[Turn Over

8
14

A bicycle dynamo is started at time zero. The total energy transformed by the
dynamo during the first 5 seconds increases as shown.
total energy / J

time / s
What is the average power generated at any instant during these 5 seconds?
A

15

0.26 kW

0.30 kW

1.30 kW

An electric motor is required to haul a lift of mass 400 kg up a mine shaft through a
vertical height of 1200 m in 2.0 minutes. What will be the electrical power required if
the overall efficiency is 80%?
A

16

0.10 kW

3.1 kW

4.9 kW

31 kW

49 kW

The diagram shows different regions of air molecules for a progressive sound wave
moving to the right at a certain instant. P, Q, R, S and T are particles at the centres
of these regions.
P

Which of the following statement is true?


A

R is moving towards the right at that instant.

Distance between P and T is one wavelength.

After a period of time, Q moves to the position of S.

The amplitude of R is always zero.


[Turn Over

9
17

If two waves of the same frequency are superposed at a point in phase, the total
intensity at that point is proportional to
A
B

the square of the sum of the two amplitudes.

the square of the mean value of the two amplitudes.

18

the sum of the intensities of the two waves.

the square of the difference of the two amplitudes

In the Young double-slit experiment, the slits are illuminated with white light. Which
one of the following observations is most likely?
A
B

The central fringe is black with coloured fringes on either side.

The central fringe is white with coloured fringes on either side.

19

No fringes are seen.

The central fringe is white with black and white fringes on either side.

Waves from a radio station have a wavelength 300 m. They arrive at a home
receiver 20 km away from the transmitter by two paths. One is a direct path, and the
second is by reflection from a mountain directly behind the home receiver as shown
below.

radio station

receiver
mountain
20 km

What is the minimum distance, in m, from the mountain to the receiver such that
destructive interference occurs at the receiver?
A

600

300

200

75

[Turn Over

10
20

A four-terminal box is connected as shown to a cell and two non-ideal


milliammeters.

mA
3

mA
2

For which of the following circuits will the current in the two milliammeters be the
same?

1
1

[Turn Over

11
21

The power supply for a transistor consists of four dry cells in series. Each cell has
an e.m.f. E and internal resistance r. If one cell is connected the wrong way round,
what would be the e.m.f. and internal resistance of the power supply?
A

3E and 4r

22

2E and 4r
2E and 3r

3E and 3r

A resistor is made from two equal lengths of wire of the same resistivity joined in
series. The first wire X has twice the diameter of the second wire Y. When a current
flows through the resistor, what is the potential difference across X as a fraction of
the total potential difference across the resistor?
A

23

1
5

1
4

1
3

4
5

In the circuit shown, the potential difference between points W and Y is 2.0 V. What
is the potential of X with respect to Z?
X
5.0

5.0

4.0

1.0
Z

2.0 V

0.60 V

0.60 V

2.6 V

2.6 V

[Turn Over

12

24

The diagram shows four identical lamps J, K, L and M which are all lit. Lamp K is
then removed from the circuit.

K
L
M

Which of the following statements correctly reflects the current status of the lamps?
A J, L and M are equally bright.
B J is brighter than before but not as bright as L and M.
C J is less bright than before but not as bright as L and M.
D J is less bright than before but brighter than L and M.

25

Three long straight wires P, Q and R are placed normal to the plane of paper as
shown in the diagram below. Wires P and Q carry currents directed into the paper,
and wire Q carries a current directed out of the paper. All three current have the
same magnitude.
A

C
D

Which arrow best shows the direction of the resultant force on wire R?
[Turn Over

13
26

A plotting compass is placed near a solenoid when there is no current. The


compass needle points due north as shown in the figure

If there is a current from P to Q such that the magnetic field at the compass is equal
to the magnitude of Earths magnetic field at that point. In which direction does the
plotting compass set?

27

Two current carrying wires, P and Q, are placed in a uniform magnetic field on the
same plane as shown below.
X

60o

Q
X

P X

If the length of wire P is twice that of Q, what is the ratio of the force acting on wire
P to wire Q if the current in P is also twice of Q?
P

[Turn Over

14
28

A beam of monochromatic light of wavelength 580 nm is totally reflected at normal


incidence by a plane mirror. If the light exerts a force of 2.50 1020 N on the mirror,
what is the number of photons hitting the mirror per second?
A

1.1 107

29

5.5 106
2.2 107

2.8 107

Which on of the following statements, referring to photoelectric emission, is always


true?
A
B

The number of electrons emitted per second is directly proportional to the


intensity of incident light

For a given metal, there is a minimum frequency, below which no emission


occurs

30

For very low intensity of light, there will not be any emission of photoelectrons

The velocity of the emitted electrons is proportional to the intensity of the


incident light.

The diagram below represents, drawn to scale, the energy levels for an electron in a
certain atom.
E4
E3
E2

Energy

E1
Leanne observed that the emission spectrum of this atom produces a red, green
and a blue line. Which of the following transitions will most likely give rise to these
lines?
Blue

Red

Green

E2 to E1

E3 to E2

E4 to E3

E3 to E1

E4 to E3

E3 to E2

E4 to E2

E3 to E2

E4 to E1

E4 to E1

E2 to E1

E3 to E1
[Turn Over

Name

Class (6C

DUNMAN HIGH SCHOOL


SENIOR HIGH

PRELIMINARY EXAMINATION 2008


YEAR SIX
H1 PHYSICS

___________________________________________________________________

Date: 16 September 2008 (Tuesday)


Paper 2

Duration: 2 hours

Structured Questions

READ THESE INSTRUCTIONS FIRST


DO NOT TURN THIS PAGE OVER UNTIL YOU ARE TOLD TO DO SO.

Write your name and class at the top of this page.


Answer all questions in the spaces provided.
For numerical answers, all working should be shown.

For Examiners Use


Section A
1
2

Section A
Answer all questions.

3
4
5
Section B

Section B
Answer any two questions.
The number of marks is given in brackets [ ] at the end
of each question or part question.

6
7
8
Total

This question paper consists of 23 printed pages (including this cover page).

[Turn Over

2
Data

speed of light in free space,

3.00 x 108 m s-1

elementary charge,

1.60 x 10-19 C

the Planck constant,

6.63 x 10-34 J s

unified atomic mass constant,

1.66 x 10-27 kg

rest mass of electron,

me

9.11 x 10-31 kg

rest mass of proton,

mp

1.67 x 10-27 kg

acceleration of free fall,

9.81 m s2

Formulae

uniformly accelerated motion,

1 2
at
2

= ut +

v2

= u2 + 2as

work done on/by a gas,

= pV

hydrostatic pressure,

= gh

electric potential,

resistors in series,

= R1 + R2 + . . .

resistors in parallel,

1
1
1
+
+...
=
R1
R2
R

Q
4 o r

[Turn Over

3
Section A
Answer all the questions in this section.
1

(a)

(b)

Distinguish between the terms precision and accuracy.

[2]

The relationship between the object distance u, image distance v


and focal length f of a lens is given by the formula
1 1 1
= +
f u v
In a certain experiment, the object distance is found to be
(5.0 0.1) cm and the image distance is (10.0 0.2) cm. How
would you express the focal length of the lens, with its
uncertainty?
[5]

[Turn Over

4
2

During the course of part of the circuit for the Singapore Formula One
motor race, the speed of car A varied in the way shown in Figure 5.1
below. It took 62 seconds to complete that part of the track.
speed / m s1
100
80
60
40
20
0

10

15

20

25 30

35 40

45

50

55

60

time / s

Figure 5.1

(a)

Complete the table below which shows the total distance


traveled over time table below
[3]

Total Distance travelled in first x seconds

Distance / m

5
10
25
30
45
53

[Turn Over

5
(b)

Plot the distance-time graph in Figure 5.2

[2]

distance / m
5000
4000
3000
2000
1000
0

10

15

20

25 30

35

40

45

50

55

60

time / s

Figure 5.2

Plot the acceleration-time graph in Figure 5.3

(c)

[3]

Acceleration / m s2
10
5
0

time / s

5
10
15
0

10

15

20

25 30

35 40

45

50

55

60

Figure 5.3
(d)

Calculate the average speed for the car

[1]

[Turn Over

6
(e)

(f)

Comment on the effect of the condition of tires on the


acceleration-time graph
[1]

Car B starts off 200 m behind the car A. At what average speed
must car B travel in order to overtake car A at t = 50 s?
[2]

(a)

State the conditions for the equilibrium of a body which is acted


upon by a number of forces.
[2]

[Turn Over

7
(b)

The figure shows a crane being used to lift a load of girders, each
of which has a mass of 500 kg.

The jib of the crane has a mass of 2500 kg and the cab has a
mass of 20 000 kg. The centres of gravity of the jib and the cab
are at their mid-points E and F respectively. The hook and
cable have negligible mass.
(i)

When a single girder is lifted,


1.

Calculate the tension in the cable AB when a single


girder is lifted.
[1]

2.

Determine the corresponding tension in the cable


PQR.
[2]

[Turn Over

8
(ii)

For the jib in the position shown,


1.

2.

(a)

determine the tension AB which will just topple the


crane.
[2]

determine the maximum number of girders which


can be lifted without the crane toppling over.
[1]

Define magnetic flux density.

[1]

[Turn Over

9
(b)

A current carrying conductor of length 0.40 m is placed at an


angle of 50o to a uniform magnetic field as shown in Figure 2.1
below.
magnetic field
50o
A

B
conductor
Figure 2.1

(i)

Calculate the magnetic flux density if a force of 0.060 N


acts on the current carrying conductor when a current of
2.0 A passes through it.
[3]

(ii)

What is the direction of the force if the current flows from A


to B?
[1]

[Turn Over

10
5

(a)

(i)

Explain why the maximum possible kinetic energy of a


photoelectron is independent of the intensity of the
incident light
[2]

(ii)

Ultraviolet radiation of frequency 4.85 1015 Hz is incident


on a metal surface and the resulting photoelectrons have
a maximum kinetic energy of 1.25 1018 J. Calculate
1. The energy of the photon of this radiation

[1]

2. The cut-off wavelength of the metal surface

[2]

[Turn Over

11
(b)

(i)

A sodium lamp is found to produce yellow light of


wavelength 589 nm. Explain why light of specific
wavelength is produced in the lamp.
[2]

(ii)

The smallest wavelength of light produced by the sodium


lamp from single electron transition is 242 nm, calculate
the minimum potential difference to accelerate
bombarding electrons to produce the light from sodium
lamp
[1]

End of Section A

[Turn Over

12
Section B
Answer two questions in this section.
6

(a)

A trolley of mass 5.0 kg is moving on a level bench. Two seconds


later, a fan is switched on. The trolley stops after moving for a
further 3.0 seconds. The graph shows the velocity-time graph of
the trolley.

(i)

Explain why the graph is not a single straight line.

(ii)

Determine the frictional force acting on the trolley by the bench


top.
[2]

[3]

[Turn Over

13
(iii)

Calculate the force on the trolley due to the air current produced
by the fan.
[3]

(iv)

If the fan is kept on blowing after the trolley has come to a stop,
the trolley starts moving backwards. If the same frictional force
still acts on the trolley, what is the acceleration of the trolley? [2]

[Turn Over

14
(b)

Sphere P of mass 2.0 kg and sphere Q of mass 1.0 kg are


moving towards each other with speeds 2.0 m s-1 and 1.0 m s-1
as shown.

2.0 m s-1
1.0 m s-1

If the collision is head on, determine the speeds of the spheres


(i)

if the collision is completely inelastic

(ii)

if the collision is elastic

[5]

[Turn Over

15
(c)

The collision turns out to be inelastic and the force between the
spheres during collision varies with time as shown in the graph.
F /N
15

20

40

t /ms

Determine
(i)

the change in momentum of each sphere

(ii)

the speed of P and Q after the collision

(iii)

the total kinetic energy of P and Q after the collision

[5]

[Turn Over

16
7

(a)

(b)

What is meant by the term stationary wave?

[2]

An air column of variable length is made to resonate in its


fundamental mode by a loudspeaker which is connected to a
signal generator. When the length L of the air column is changed,
its resonant frequency f also changes.

loudspeaker

L
1
The graph below shows how L varies with .
f

L/ m

1
/ kHz-1
f

[Turn Over

17
(i)

Sketch the standing wave in the air column in its fundamental


mode of vibration.
[2]

(ii)

Explain why the graph produced


1. is a straight line,

2. does not pass through the origin

[3]

[Turn Over

18
(iii)

(c)

Using the graph, determine the value of the speed of sound in air.
[3]

The diagram shows the arrangement of apparatus to observe


interference of light.

A
S

B
screen

Monochromatic light from the source S falls on two narrow slits A


and B which act as coherent sources and interference fringes are
observed on the screen.
(i)

Explain monochromatic light and coherent sources.

[2]

[Turn Over

19
(ii)

Explain how the apparatus produces interference fringes


on the screen.
[4]

(iii)

State the effect on the brightness and separation of the


fringes if the following changes are made independently.
1.

the screen is moved closer to the two slits.

2.

the source is moved closer to the two slits.

[4]

[Turn Over

20
8

(a)

The graph shows how the current through A and B, a resistor and
an intrinsic semiconductor respectively, changes as the potential
difference across each is increased.

(i)

Describe how the resistance of each component varies as


the potential difference is increased.
[2]

(ii)

Explain why the resistance B varies according to (i) as the


potential difference is increased.
[2]

[Turn Over

21
(iii)

Calculate the potential difference required to pass a


current of 3.0 mA through A and B connected in series. [2]

(iv)

If A and B are connected in parallel to a cell of e.m.f. 8.0 V


of negligible internal resistance, determine the current
drawn from the battery.
[2]

[Turn Over

22
(b)

Define the terms resistance and the ohm.

[2]

(c)

Two resistors having resistances of 1.8 k and 4.7 k are


connected in series with a battery of e.m.f. 12 V and negligible
internal resistance as shown.
4.7 k

1.8 k

12 V
When a particular voltmeter of fixed resistance R, which is known
to be accurately calibrated, is placed across the 1.8 k resistor, it
reads 2.95 V. When placed across the 4.7 k resistor, it reads
7.70 V.
(i)

Why do these two readings not add up to 12 V?

[2]

[Turn Over

23
(ii)

Calculate the resistance R of the voltmeter.

[4]

(iii)

A second identical voltmeter is used so that a voltmeter is


placed across each resistor. What will each voltmeter read?
[4]

End of Paper

[Turn Over

Preliminary Examination

Hwa Chong Institution (College Section)


H1 Physics

2008/C2

Solution
C2 H1 Physics 8866 Preliminary Examination
Paper 1
Qn
1
2
3
4
5

Ans
A
C
D
B
D

Qn
6
7
8
9
10

Ans
D
A
C
B
B

Qn
11
12
13
14
15

Ans
C
C
D
A
A

Qn
16
17
18
19
20

Ans
D
B
D
C
A

Qn
21
22
23
24
25

Ans
C
A
B
D
A

Qn
26
27
28
29
30

Ans
D
B
A
C
B

Candidates Name : ____________________________

Name of Tutor: ___________

Centre Number/Index Number : __________

CT Group : 07S_____

HWA CHONG INSTITUTION

H1 PHYSICS 8866
Paper 2
Preliminary Examination
C2

12 September 2008

Duration: 2 hrs

INSTRUCTIONS TO CANDIDATES

Do Not Open This Booklet Until You Are Told To Do So.


1.

Write your name, index number and CT class clearly on the top of
this cover page. Write the model number of the calculator you are
using in the box provided at the bottom of this page.

Paper 1
(MCQ)

/ 30

Paper 2
Section A (Short Structured)

2. There are 5 structured questions including 1 data analysis


question in Section A. Write all your answers in the spaces
provided on the question paper. Answer ALL questions.

Q1

/9

Q2

/8

3. There are 3 questions in Section B. Answer ANY TWO


questions. Circle the question number of the question you have
attempted on this cover page.

Q3

/7

Q4

/6

4. For numerical answers, all workings should be shown.


5. A data and formulae list is provided on page 2.

Q5(Data
Analysis)

Section B (Long Structured)


[Choose 2]

6. You are advised to spend about one hour on each section.


7. You are reminded of the need for clear presentation and good
English.
8. Please clear the memory of your graphic calculator.
9. The total mark for this paper is 80 marks.
10. The number of marks is given in brackets [ ] at the end of each
question or part question
.

/ 10

Q6

/ 20

Q7

/ 20

Q8

/ 20

Deductions
Total

Calculator
Model

This question booklet consists of 23 printed pages (including this page)

/ 80

Preliminary Examination

Hwa Chong Institution (College Section)


H1 Physics

DATA AND FORMULAE


Data
speed of light in free space
elementary charge
the Planck constant
unified atomic mass constant
rest mass of electron
rest mass of proton
acceleration of free fall

c
e
h
u
me
mp
g

=
=
=
=
=
=
=

3.00 108 m s-1


1.60 10-19 C
6.63 10-34 J s
1.66 10-27 kg
9.11 10-31 kg
1.67 10-27 kg
9.81 m s-2

Formulae
uniformly accelerated motion
work done on/by a gas
resistors in series
resistors in parallel
hydrostatic pressure

s
v2
W
R
1/R
p

=
=
=
=
=
=

ut + 1 at2
2
u2 + 2as
pV
R1 + R2 + ...
1/R1 + 1/R2 + ...
gh

2008/C2

Preliminary Examination

Hwa Chong Institution (College Section)


H1 Physics

2008/C2

Section A
Answer ALL questions from this section
1

A student times the fall of a small steel ball. Data for the time t taken for the ball to fall a vertical
distance h from rest are given below.

h = 266 1 cm
t = 0.740 0.005 s
(a)

Use these data to determine


(i)

a value, to 3 significant figures, of the acceleration of free fall g.

acceleration = ______________m s-2 [2]


(ii)

the percentage uncertainty, to two significant figures, of


1.

the distance h,

uncertainty = ______________% [1]


2.

the time t,

uncertainty = ______________% [1]

Preliminary Examination

(b)

Hwa Chong Institution (College Section)


H1 Physics

2008/C2

Use your answers in (a) to determine the actual uncertainty in the value of g.
Hence give a statement of g, with its uncertainty, to an appropriate number of significant
figures.

g = ___________ ___________ m s-2 [3]


(c)

Suggest two reasons why, in this experiment, although the value of t is precise, it may
not be accurate.
1.____________________________________________________________________
______________________________________________________________________
2.____________________________________________________________________
___________________________________________________________________ [2]

Preliminary Examination

Hwa Chong Institution (College Section)


H1 Physics

2008/C2

A sphere of mass m travelling in a straight line with speed u collides head-on with a stationary
board of a larger mass, M. The collision is perfectly elastic. The final velocities of the sphere
and the board are v1 and v2 respectively.

M
(a)

Explain what is meant by a perfectly elastic collision.


______________________________________________________________________
___________________________________________________________________ [1]

(b)

By applying Principle of Conservation of Linear Momentum, show that

mM
v1 =
m+ M

(c)

u .

[3]

Describe the subsequent motion of the sphere after the collision if m is negligible
compared to M.
______________________________________________________________________
______________________________________________________________________
___________________________________________________________________ [2]
5

Preliminary Examination

(d)

Hwa Chong Institution (College Section)


H1 Physics

2008/C2

If m is negligible compared to M and the number spheres of mass m colliding with the
board every second is N, write down the expression for the average force acting on the
board of mass M.

average force = ________________ [2]

Preliminary Examination

(a)

Hwa Chong Institution (College Section)


H1 Physics

2008/C2

State what is meant by a progressive wave.


______________________________________________________________________
___________________________________________________________________ [2]

(b)

Two sources S1 and S2 produce waves of the same frequency on the surface of some
water.
(i)

State three conditions that must be satisfied for waves from the two sources to
produce an observable interference pattern on the water surface.
1. ______________________________________________________________
________________________________________________________________
2. ______________________________________________________________
________________________________________________________________
3. ______________________________________________________________
_____________________________________________________________ [3]

(ii)

Wavefronts produced by the two sources are illustrated in Fig 3.1.

Fig. 3.1
Waves of maximum amplitude are observed along the directions AB and CD,
among others.
1.

Determine the number of directions between AB and CD along which


maximum amplitude occurs.

2.

number = ________ [1]


On Fig. 3.1, draw a line to show one direction along which the waves have
minimum amplitude.
[1]
7

Preliminary Examination

(a)

Hwa Chong Institution (College Section)


H1 Physics

2008/C2

Calculate the speed of electrons such that they can be used to measure the distance
between adjacent atoms in typical crystals (about 0.1nm).

Speed = _____________ m s-1 [2]


(b)

Calculate the kinetic energy of such an electron.

Kinetic energy = _____________ J [1]


(c)

Calculate the energy of a photon which has the same wavelength as the electron.

Energy = _____________ J [1]


(d)

Explain whether protons of identical kinetic energy as the electron would make a more
or less effective probe of small-scale structures.
______________________________________________________________________
______________________________________________________________________
___________________________________________________________________ [2]

Preliminary Examination

Hwa Chong Institution (College Section)


H1 Physics

2008/C2

Solar cells are used in some appliances for the generation of electrical energy. When light
energy is incident on the surface of such a cell, an e.m.f. is generated between the terminals of
the cell. Connection of a resistor between these terminals will result in a current and electrical
power dissipation in the resistor.
The variation with output potential difference V of the current I from a solar cell may be
investigated using the circuit of Fig. 5.1.

Fig. 5.1
The ammeter has negligible internal resistance and the voltmeter has a very high resistance.
Light of constant intensity is incident on the solar cell. The I/V characteristic of one type of
solar cell, when it is illuminated with a certain intensity of light, is shown in Fig. 5.2.

Fig. 5.2

Preliminary Examination

(a)

(i)

Hwa Chong Institution (College Section)


H1 Physics

2008/C2

Determine the current from the solar cell for an output potential difference of
400 mV.

current = ______________ A [1]


(ii)

From the graph, find the e.m.f. of the solar cell.

e.m.f. of solar cell = _____________mV [1]


(iii)

Use your answers to (i) and (ii) to determine the internal resistance of the cell, r,
at an output potential difference of 400 mV.

r = _____________ [2]
(b)

(i)

Determine the power dissipation in the load resistor for point P on Fig. 5.2.

Power dissipation at P = ______________ mW [2]


(ii)

(c)

On Fig. 5.2, shade an area that represents the power dissipation calculated
in (i).

A number of identical solar cells of a different type each produce an output power of
75 mW at an output potential difference of 0.50 V.
Each cell may be represented by the symbol shown in Fig. 5.3.

Fig. 5.3

10

[1]

Preliminary Examination

Hwa Chong Institution (College Section)


H1 Physics

2008/C2

Draw suitable arrangements of solar cells so that the cells may be used to provide
(i)

a power of 150 mW at a potential difference of 1.0 V

[1]

(ii)

a power of 150 mW at a potential difference of 0.50 V

[1]

(iii)

a power of 300 mW at a potential difference of 1.0 V.

[1]

End of Section A

11

Preliminary Examination

Hwa Chong Institution (College Section)


H1 Physics

2008/C2

Section B
Answer any TWO questions from this section
6

(a)

A student makes the following statements of Newtons laws of motion:

First Law: Every body continues in its state of motion unless it is acted upon by a
resultant external force.
Second Law: The acceleration of a body is inversely proportional to the resultant
force acting on the body. The direction of the acceleration is in the same direction as the
force.
Third Law: Action and reaction forces always occur in pairs and are equal in
magnitude and opposite in direction.
(i)

The statement of the First Law is incomplete in two aspects. Identify the two
aspects in which it is incomplete and hence rewrite it with the appropriate
amendments.
________________________________________________________________
________________________________________________________________
________________________________________________________________
_____________________________________________________________ [2]

(ii)

The statement of the Second Law is wrong in two aspects. Identify the mistakes
made in the statement and hence rewrite it with appropriate corrections.
________________________________________________________________
________________________________________________________________
________________________________________________________________
_____________________________________________________________ [2]

(iii)

The statement of the Third Law is correct, but fails to emphasise an important
aspect of the action and reaction forces. Identify this aspect and rewrite this
Third Law to make this emphasis.
________________________________________________________________
________________________________________________________________
________________________________________________________________
_____________________________________________________________ [2]
12

Preliminary Examination

Hwa Chong Institution (College Section)


H1 Physics

2008/C2

(b) State the two conditions for equilibrium of an object which is acted on by a number of forces.
__________________________________________________________________________
__________________________________________________________________________
_______________________________________________________________________ [2]
(c)

A uniform block is standing on the floor of a moving trailer that is moving towards the right at
an acceleration a of 2.00 m s-2. (Fig. 6.1)

a = 2.00 m s-2

block

Trailer

Block

Fig. 6.1
Fig. 6.2 shows the block on the floor of the moving trailer. The dimensions of the block are
25.0 cm by 12.0 cm and its mass is 0.800 kg.
Only three forces act on the block :

the normal contact force N on the block by the floor (not drawn in the figure),

the frictional force F on the block by the floor, and

the weight W of the block .

The centre of mass of the block is indicated by O .

12.0 cm

25.0 cm

F
P

W
Fig. 6.2

13

Floor of moving trailer

Preliminary Examination

Hwa Chong Institution (College Section)


H1 Physics

2008/C2

Using Newton's 1st Law of Motion, explain why the frictional force F acts in the
direction as indicated in Fig. 6.2.

(i)

___________________________________________________________________
___________________________________________________________________
________________________________________________________________ [2]

(ii)

Determine the value of F.

F = __________ N [1]
(iii) 1.

By considering moments about O for each of the forces shown in Fig. 6.2, explain
why the line of action of the normal contact force N cannot pass through the
centre of mass of the block in order that it does not topple.
________________________________________________________________
________________________________________________________________
________________________________________________________________
_____________________________________________________________ [2]

2.

Copy Fig. 6.2 onto the space below.


Sketch on your figure a labelled arrow to represent the normal contact force N.
Please pay particular attention to the position of the line of action of this force. [2]

14

Preliminary Examination

3.

Hwa Chong Institution (College Section)


H1 Physics

2008/C2

Determine the horizontal distance x between the line of action of N and centre of
mass O.

x =__________ m [3]

(iv)

If the acceleration is increased gradually, the block will finally topple backwards
about the point P. Explain why that happens.

___________________________________________________________________
___________________________________________________________________
___________________________________________________________________
________________________________________________________________ [2]

15

Preliminary Examination

(a)

Hwa Chong Institution (College Section)


H1 Physics

2008/C2

Define the magnetic flux density.


______________________________________________________________________
______________________________________________________________________
______________________________________________________________________
______________________________________________________________________
____________________________________________________________________ [2]

(b)

Fig. 7.1 shows a rear-view cross-section of the body of a railway carriage and of an
electric cable under the floor of the carriage. The cable carries a current of 80.0 A
towards the front of the carriage. A magnetic compass is held horizontally at P, 1.5 m
above the cable.

Fig. 7.1
(i)

The magnetic flux density of the field due to a long straight wire is given by

B=

0 I
.
2r

Calculate the flux density BC of the magnetic field at P due to the current in the
cable. (Given that o is 4 x 10-7)
B

BC = T [2]
B

16

Preliminary Examination

Hwa Chong Institution (College Section)


H1 Physics

2008/C2

(ii)

Copy Fig. 7.1 onto the space below.


Draw the magnetic field pattern around the cable ensuring that at least one field
line passes through the point P.
[2]

(iii)

The flux density BH of the horizontal component of the Earth's magnetic field is
1.8 10 5 T . Assume that this acts in the direction of true North, and that there
are no other magnetic fields apart from that of the current in the cable. Calculate
the resultant horizontal magnetic flux density at P, and state the direction in
which the compass points, when the carriage is oriented with its front:
B

1.

towards the east

Resultant horizontal magnetic fields = .. T


Direction: [3]

17

Preliminary Examination

Hwa Chong Institution (College Section)


H1 Physics

2.

2008/C2

towards the north

Resultant horizontal magnetic fields = .. T


Direction: .. [3]

(c)

Fig.7.2 below shows a cylindrical aluminium bar A of length 10.0 cm long resting on two
horizontal aluminium rails which can be connected to a battery. A magnetic field of flux
density 0.40 T, acts perpendicularly into the paper. Assume that frictional force between
bar A and the rails is negligible.

S1

Top View

Y
Fig. 7.2

(i)

Describe the motion of bar A immediately after the switch S1 is closed.


________________________________________________________________
________________________________________________________________
_____________________________________________________________ [2]

18

Preliminary Examination

(ii)

Hwa Chong Institution (College Section)


H1 Physics

2008/C2

If the ends of the rails (X and Y) can be lifted perpendicular to the plane of the
paper, calculate the angle to the horizontal to which the rails must be raised to
keep bar A stationary. The mass of bar A is 5.0 g and the current passing
through it is 4.0 A.

Y
Side View

v
B

= __________ [4]
(d)

An electron with charge e and mass m is projected with kinetic energy K into the region
between two plates as shown in figure below. The distance between the plates is d. The
magnetic field between the plates is B and acts in the direction as shown in the figure.
If the electron is to just miss collision with the opposite plate, sketch the path of the
electron, in the figure below.
[2]

19

Preliminary Examination

(a)

2008/C2

State what is meant by the photoelectric effect.


______________________________________________________________________
______________________________________________________________________
____________________________________________________________________ [2]

(b)

The experimental results of the photoelectric effect yield a linear relation between the
stopping potential Vs and the frequency f of the incident electromagnetic radiation. Write
down the equation for Vs as a function of f defining any other terms used in the
equation.

[2]

Fig 8.1 shows how the stopping potential varies with light of different frequencies for a
sodium surface. The point P refers to the last data point on the graph. The light used is
produced by the excitation of gaseous atoms in the discharge tube.

The stopping potential as a function of frequency


for a sodium surface.
3.5

3.0

V (volts)

(c)

Stopping potential

Hwa Chong Institution (College Section)


H1 Physics

2.5
2.0
1.5
1.0
0.5
0.0
0.0

2.0

4.0

6.0

8.0

Frequency f ( x 10

Fig. 8.1
20

10.0
14

Hz)

12.0

14.0

Preliminary Examination

(i)

Hwa Chong Institution (College Section)


H1 Physics

2008/C2

Using the graph and data points provided. Calculate the following and explain
clearly how each value is obtained.
1. The ratio of the Planck constant to the electronic charge ( h/e ),

2. The work function of sodium,

work function of sodium = ___________ J


3. The de Broglie wavelength of the most energetic electron emitted in this
experiment,

(ii)

de Broglie wavelength = _____________ m


[8]
The work function that you have calculated differs from the energy required to
remove the outermost electron of an isolated sodium atom. Explain why this is
so.
________________________________________________________________
________________________________________________________________
________________________________________________________________
_____________________________________________________________ [2]
21

Preliminary Examination

(d)

Hwa Chong Institution (College Section)


H1 Physics

2008/C2

Fig. 8.2 shows a high voltage supply set up to produce energetic electrons to bombard
the cool sodium gas in the discharge tube, giving rise to a line spectrum when observed
through a diffraction grating. Figure 8.3 shows some energy levels of sodium.
n=5
n=4
n=3

-1.98 eV

n=2

High
voltage
supply

-1.42 eV
-1.56 eV

-3.07 eV

n=1

-5.17 eV

Observer

Sodium gas

Collimator slit
Diffraction
Grating

Fig. 8.2

Fig. 8.3

Given that the bombarding electrons have kinetic energy of 3.70 eV,
(i)

Deduce the number of spectra lines observed.

Number of spectra lines = ___________ [1]


(ii)

Sketch the positions of the lines on the spectrum below, indicating clearly
the various transitions. The line due to the transition from n = 2 to n = 1
has been drawn for you.

n=2
to n = 1
Increasing frequency

(iii)

[2]

What is the range of kinetic energy of the recoiling electrons after they
have excited the sodium atoms?

Range = ______ eV KE ______ eV [1]

22

Preliminary Examination

Hwa Chong Institution (College Section)


H1 Physics

(iv)

2008/C2

If another cool gas with energy levels as shown in Fig. 8.4 is placed
between the discharge tube and the collimator slit, how many lines will be
observed? Explain your answer.
n=4

-0.48 eV

n=3

-0.90 eV

n=2

-1.49 eV

n=1

-3.00 eV

Fig. 8.4
__________________________________________________________
__________________________________________________________
__________________________________________________________
__________________________________________________________
_______________________________________________________ [2]

End of Section B

23

Preliminary Examination

Hwa Chong Institution (College Section)


H1 Physics

2008/C2

Suggested Solution
C2 H1 Physics 8866 Preliminary Examination 2008
Paper 2
1(a) (i)

s = g t2

266 = g (0.740)2
g = 9.72 m s-2

(ii)

1/266 x 100% = 0.38%

(iii)

0.005/0.740 x 100% = 0.68%

(b)

g/g = h/h + 2 t/t


g = 0.2 ms-2 (1sf)
g = 9.7 0.2 m s-2

(c)

1. zero error in the timer


2. miscalibration of the timer

2 (a)
(b)

A perfectly elastic collision is one in which the total kinetic energy of the
system is conserved.
Conservation of Linear Momentum,
mu = mv1 + Mv2
Relative speed of approach = relative speed of separation
0 - u = -(v2 v1)
v 2 = u + v1
hence mu = mv1 + M (u + v1 )
v1 = (

(c)

mM
)u
m+M

If m is much smaller than M, v1 = -u . The sphere with lighter mass m will


recoil with the about the same speed in the opposite direction after the
collision.

(d) Average force


= change in momentum x collisions per unit time
= (mu (-mu)) x N
= 2Nmu
3 (a)

A progressive wave is one where the wave profile moves along the direction
of propagation and transfers energy from the source to points around it. The
wave motion is caused by vibrations or oscillations.

(b)(i) Conditions for observable Interference pattern on water surface:


The waves must overlap or superpose.
The two sources must be coherent, i.e. there must be a constant
phase difference or relation between them. Being coherent also
implies that the two sources must be of the same wavelength and
same frequency.

Preliminary Examination

Hwa Chong Institution (College Section)


H1 Physics

2008/C2

(No marks awarded if merely states having same freq or wavelength)


The amplitudes of the two sources must be approximately the
same, then the regions of constructive and destructive interference
will be distinct.
(ii)

1. 7
2. Any line that is halfway in between two constructive interference lines.

4 (a)

For the electron to have a wavelength of 0.1nm, its momentum is p =


Thus its speed is v =

(b)
(c)
(d)

6.63 10 34
p
h
= 7.29 10 6 m s 1
=
=
31 10 10
m m 9.1 10

)(

2
1 2 1
mv = 9.1 10 31 7.29 10 6 = 2.42 10 17 J
2
2
c
6.63 10 34 3 10 8
= 1.99 10 15 J
Energy of the photon = h =
10

10

Kinetic energy

)(

Protons. They have a larger mass and a smaller de Broglie wavelength.

5 (a)(i) From the graph, at p.d. = 400 mW, current = 130 mA = 0.13 A.
(ii) As the solar cell behaves like an emf source, V = E Ir.

V= E when I = 0, and that is when V = 550 mV.


(iii) V = E Ir. At V = 400 mV, I = 130 mA, r = 1.1538 = 1.15 (3 sf)
(b)(i) For point P, V. = 520 mV, I = 75 mA
Hence power dissipated in load resistor = IV = 39 mW
(ii) rectanglular area under IV graph at P.
(c)(i) two cells in series.
(ii) two cells in parallel.
(iii) two branches of two cells in series in parallel arrangement.

Preliminary Examination

Hwa Chong Institution (College Section)


H1 Physics

2008/C2

6 (a)(i) The body can also remain in a state of rest (i.e. remain stationary) if no
resultant force acts on it. The uniform motion is in a straight line (velocity is
constant) if no resultant force acts on it.
1st Law : Every body continues in its state of rest or uniform motion in a
straight line unless it is acted upon a resultant external force.
(ii) The acceleration of the body is not inversely proportional to the force acting
on it. Second Law gives a relationship between rate of change of momentum
and force and not a direct relationship between force and acceleration.
2nd Law: The rate of change of momentum of a body is proportional to the
resultant force acting on it and occurs in the direction of the resultant force.
(iii)Action and reaction pairs act on different bodies.
If body A exerts a force on body B then B would exert an equal in magnitude
by opposite in direction force on body A.
(b) When an object is in equilibrium, the resultant forces must be zero and the
resultant torque (moment) on the body about any point/axis must be zero.
(c) (i)

As the lorry accelerates to the right, the block tends to stay in its original
position due to inertia (Newton's 1st Law). Concept of inertia
Therefore, the block is moving towards the left relative to the lorry.
The frictional force on the block by the floor of the lorry therefore acts to
oppose this relative motion and hence directs to the right.

(ii)

The frictional force provides the force to accelerate the block with the
lorry.
F = ma = (0.800 kg)( 2.00 m s-2) = 1.60 N

(iii)

1.

The force F will causes an anticlockwise moment about O.


Since the block is not to topple, the normal contact force N has to
contribute a clockwise moment to ensure that the net torque on the
block about O is zero.
The line of action of N is therefore shifts left of O to provide this
counter torque.

2.

Preliminary Examination

Hwa Chong Institution (College Section)


H1 Physics

2008/C2

x
y

3.

Vertically, the net force on the block is 0.


Therefore, N = W
Since the block is in rotational equilibrium, taking moments about O,
Sum of anticlockwise moments = Sum of clockwise moments

(iv)

Fy = Nx
x =

Fy Fy
(1.60 N 12.5 cm)
=
=
= 2.55 cm
N W (0.800 kg 9.81 m s 2 )

As the acceleration increases, F will increase to keep the block moving


with the lorry and hence the anticlockwise moment provided by F on the
block also increases.
To counter this torque, N continually shifts leftwards to provide a larger
clockwise moment till it eventually moves to the left edge of the block.
If the acceleration increases further, the anticlockwise moment provided
by F will be larger than the clockwise moment provided by the block and
the block topples backward.

7 (a)

The magnetic flux density is defined as the force per unit length per unit
current acting on an infinitely long current carrying conductor placed at
perpendicular to the magnetic field.

4 10 7 80
(b) (i) B c = 0 I =
= 1.1 10 5 T
2r

2 1.5

(ii)

Preliminary Examination

(iii) 1. BR

Hwa Chong Institution (College Section)


H1 Physics

2008/C2

= 1 .8 10 5 - 1.110 5 [1]
= 0.7 10 5 T [1] towards North

2. Magnetic field due to cable and Earth at right angles to each other,
hence add vectorially:
Resultant field BR = 1.8 10 5 2 + 1.1 10 5 2
= 2.1 10 5 T
Direction is 310 east of north.
(c) (i) Bar A moves to the right with constant acceleration.
(ii) Force experienced by bar A in B-field = BIL
Consider forces acting on the bar parallel to the rails,

BIL cos = mg sin


tan = BIL / mg
= tan-1 (BIL / mg)
-1

= tan [(0.40)(4.0)(0.100) / (0.0050)(9.81)] = 73 0


(d) To just miss the opposite plate, the electron must move in a circular path with
radius d.

8 (a) The photoelectric effect refers to the emission of electrons from a cold metallic
surface when electromagnetic radiation of sufficient frequency (higher than
the threshold frequency) is incident on it.
(b) From Einstein's photoelectric equation
Max KE of photoelectron = Energy of photon - Work function

eVs = hf -
h
Vs = f
e e

h = Planck constant
e = electronic charge

= work function, the minimum energy needed to liberate an electron from a


metal surface.

Preliminary Examination

Hwa Chong Institution (College Section)


H1 Physics

2008/C2

(c)(i)
1.

Using

hf = hf o + eVs

Vs =

h
h
f fo
e
e

The gradient of the (Vs - f ) graph gives the ratio h/e.


Using co-ordinates (5.51014, 0.5) and (11.81014, 3.2)
h
e

0.53.2
( 5.511.8 )1014

4.29 10-15 JsC-1

=
2.

The intercept on the frequency axis will give the threshold frequency, f0 =
4.4 1014 Hz
Work function, = hf o
=(6.63 10-34)(4.4 1014) J
= 2.7 10-19 J

Alternatively,
Use co-ordinates (5.61014, 0.5) and (11.81014, 2.9) to obtain threshold frequency.
f intercept, (fo, 0)
0 0.5
f o 5.61014

0.5 2.9
(5.6 11.2)1014

f0 = 4.4 1014 Hz
3. The most energetic electron will be stopped by largest stopping p.d. which
is 3.2 V.
Using

eV =

1
2

mv 2 ,

p=

hc

p = mv = 2meV
hc
de Broglie =
2meV

We obtain

6.6 10 34 3 10 8
2 9.11 10 31 3.2

= 0.21 m
(ii) The work function is the minimum amount of energy required for a conducting
electron to be removed from the attraction of a metallic lattice (to remove an
electron that is delocalised within the metal lattice). This is different from the
amount of energy required to remove an outermost electron from the
attraction of the nucleus in an isolated gaseous atom, involving the ionization
energy.

Preliminary Examination

Hwa Chong Institution (College Section)


H1 Physics

2008/C2

(d) (i) 6
(ii)

4 to 2

3 to 2

4 to 2

n=2
to n = 1

3 to 1

4 to 1

Increasing frequency

(iii) 0.09 eV KE 1.60 eV


(iv) 4 lines, because the photons of energy 1.51 eV (from n = 1 to n = 2) and 2.10
eV (from n = 1 to n = 3) will be absorbed. Only photons of energy
corresponding exactly to the difference between the ground state and an
excited state will be absorbed.

- End of Paper -

INNOVA JUNIOR COLLEGE


JC 2 PRELIMINARY EXAMINATIONS 2
in preparation for General Certificate of Education Advanced Level

Higher 1
CANDIDATE
NAME
CLASS

INDEX NUMBER

PHYSICS

8866/01

Paper 1 Multiple Choice

18 September 2008
1 hour

Additional Materials:

Multiple Choice Answer Sheet.

READ THESE INSTRUCTIONS FIRST


Write in soft pencil.
Do not use staples, paper clips, highlighters, glue or correction fluid.
Write your name, class and index number on the Answer Sheet in the spaces provided.
There are thirty questions on this paper. Answer all questions. For each question there are
four possible answers A, B, C and D.
Choose the one you consider correct and record your choice in soft pencil on the separate
Answer Sheet.
Read the instructions on the Answer Sheet very carefully.
Each correct answer will score one mark. A mark will not be deducted for a wrong answer.
Any rough working should be done in this booklet.

This document consists of 14 printed pages and 0 blank pages.


Innova Junior College
IJC 2008

8866/01/Prelim2

[Turn Over

Data
speed of light in free space,

c = 3.00 x 108 m s-1

elementary charge,

e = 1.60 x 10-19 C

the Planck constant,

h = 6.63 x 10-34 J s

unified atomic mass constant,

u = 1.66 x 10-27 kg

rest mass of electron,

me = 9.11 x 10-31 kg

rest mass of proton,

mp = 1.67 x 10-27 kg
g = 9.81 m s-2

acceleration of free fall,

Formulae
s = ut + at2

uniformly accelerated motion,

v2 = u2 + 2as
work done on/by a gas,

W = pV

resistors in series,

R = R1 + R2 +
1/R = 1/R1 + 1/R2 +

resistors in parallel,

p = gh

hydrostatic pressure,

IJC 2008

8866/01/Prelim2

[Turn Over

3
1.

The light year is often used as a unit of measurement in astronomy. What is a


light year?
A
B
C
D

2.

What is a realistic estimate of the magnitude of the momentum of an Olympic


sprinter?
A

3.

3 x 108 years
3 x 108 x (365 x 24 x 60 x 60) years
3 x 108 metres
3 x 108 x (365 x 24 x 60 x 60) metres

300 N s

800 N s

3000 N s

D 8000 N s

In manufacturing a guitar, physical quantities such as pitch range, scale length


and string thickness are of paramount importance. A well-known equation that
relates the physical quantities mentioned above is
f =

1 T
2L

where f is the frequency of the note in hertz, L is the scale length in metres, T is
the tension in newtons and is the mass per unit length.
Given that
f = (123 1) Hz,
T = (0.0370 0.0001) N,
L = (0.5398 0.0001) m
What is the percentage uncertainty in , the mass per unit length?
A
4.

0.40 %

1.1 %

1.9 %

2.3 %

An object is travelling between 2 points along the x-axis. Which position-time


(x - t) graph below is not physically possible?
x /m

x /m

t/s

t/s
B

A
x /m

x /m

t/s

t/s
D

C
IJC 2008

8866/01/Prelim2

[Turn Over

4
5.

The diagram below represents the path of an object after it was thrown. At point
P, the object reaches the highest height. Air resistance is negligible.
P
Q

Which of the following statements best describes the motion of the object as it
travels from P to Q?
A
B
C
D
6.

The velocity of the object at P is zero.


The velocity of the object decreases.
The acceleration of the object remains the same.
The acceleration of the object increases.

A rigid ball is released from rest at a distance above a hard surface. The
velocity-time (v t) graph of the ball over 3 bounces is as shown in the diagram
below.
v / m s-1

t/s

Which graph best represent the acceleration-time (a - t) graph of the ball?


a / m s-2

a / m s-2

t/s

t/s

a / m s-2

a / m s-2

t/s

IJC 2008

t/s

8866/01/Prelim2

[Turn Over

5
7.

Which of the following free body diagrams best represents a cart slowing down
as it travels to the right on a horizontal surface?

8.

The diagram below shows a crate on a frictionless plane inclined at an angle


to the horizontal. The crate is pushed at constant speed up the incline from
point M to point N by force F which is acting parallel to the plane.

N
F

M
If angle is increased and force F is applied parallel to the plane, such that the
crate moves from M to N at the same constant speed, what would be the
effects on the magnitude of force F plane and the total work W done on the
crate.
A
B
C
D

W would remain the same and the magnitude of F would decrease.


W would remain the same and the magnitude of F would increase.
W would increase and the magnitude of F would decrease.
W would increase and the magnitude of F would increase.
A 1.0 kg and 5.0 kg masses are tied with a 1.0 m length string. They are
released vertically from rest at the top of a tall building. Neglecting air
resistance, what is the tension in the string 10 s after release? (take
acceleration of free fall to be 10 m s-2)

9.

1.0 kg mass
5.0 kg mass
tall building

Figure not drawn to scale

0N

IJC 2008

10 N

8866/01/Prelim2

50 N

100 N

[Turn Over

6
A number of forces are acting on the objects below. Which of the objects
below is/are in rotational equilibrium?

10.

10 N

10 N

20 N

10 N

10 N
10 N
(i)

(ii)

10 N

10 N
10 N
10 N
10 N

(iii)
A
B
C
D
11.

When a horizontal force of 1.0 N is applied on an object of mass 2.0 kg, it


moves across a rough horizontal surface at a constant horizontal velocity of
2.0 m s-1. What is the change in the kinetic energy of the mass during its
movement over the 6.0 m distance?
A

12.

(i) only
(ii) only
(iii) only
(ii) and (iii) only

0J

3.0 J

C 4.0 J

D 6.0 J

The diagram below shows a 100 g apple attached to a branch of a tree 2.0 m
above a spring on the ground below. The apple falls and hits the spring,
compressing it 0.10 m from its equilibrium position. If all of the gravitational
potential energy of the apple on the tree is transferred to the spring when it is
compressed, what is the spring constant of this spring in N m-1?

2.0 m
2.0 m

3.9 102

IJC 2008

4.1 102

8866/01/Prelim2

3.9 105

D 4.1 105

[Turn Over

7
13.

The graph below shows the profile of a progressive transverse wave at a


particular instant of time. The particles P, Q, R and S all oscillate with uniform
amplitude.
y-displacement
S
P

x-displacement

R
Q
What one of the following statements is correct?
A
B
C
D
14.

Both P and R have zero total energy.


S has maximum total energy.
Q has minimum total energy.
All the particles have the same total energy.

The diagram shows two oscillations.


displacement

time

What is the phase difference between the oscillations?

15.

rad

rad

rad

rad

speed

Radio waves and gamma rays traveling in space have the same
A

frequency

IJC 2008

period

8866/01/Prelim2

wavelength

[Turn Over

8
16.

The table below shows four pairs W, X, Y and Z of polarising sheets cut into
uniform circles. Each pair is mounted on the path of initially unpolarised light.
The polarising direction of each sheet is indicated by the dashed line.
W

X
30

45

30

30

Z
60

30

60

30

Which of the following row gives the correct order for the intensity of the
emergent light in decreasing order?
A
B
C
D
17.

XYZW
YWZX
XZWY
YXWZ

A stationary sound wave was set up in a closed pipe. Which of the following
row correctly describes the physical changes for the displacements of the air
particles and the air pressure at the open and closed end of the pipe?
Open End

Closed End

Displacement antinode,
Pressure node

Displacement node,
Pressure antinode

Displacement node,
Pressure antinode

Displacement antinode,
Pressure node

Displacement antinode,
Pressure antinode

Displacement node,
Pressure node

Displacement node,
Pressure node

Displacement antinode,
Pressure antinode

IJC 2008

8866/01/Prelim2

[Turn Over

9
18.

The diagram below represents shallow water waves of constant wavelength


passing through two small openings. Which statement best describes the
interference at point P?

A
B
C
D
19.

It is constructive at all times


It is constructive sometimes and destructive at others.
It is destructive at all times.
It is neither constructive nor destructive.

In the Youngs double slit arrangement shown below, a pattern of equally


spaced parallel interference fringes appears on the screen. The slit width is a
while the separation between the slits is d. The distance from the screen to the
slits is unchanged.

Interference
fringe pattern
on the screen

d
a

coherent light

Which one of the following changes would cause the separation of interference
fringes to be doubled?
A
B
C
D

Slit width is a/2


Slit width is 2a
Slit separation is d/2
Slit separation is 2d

IJC 2008

8866/01/Prelim2

[Turn Over

10
20.

The diagram below shows the cross-section of an electric cable which consists
of a strong core conductor X surrounded by six other conductors of Y. The
resistivity for X is four times that of Y. The cross-sectional area for each
conductor is the same.

Y
Y

Y
X

Y
Y

Given that the resistance per metre for X is R, what is the net resistance per
metre of the electric cable in terms of R?
A
21.

0.40R

2.5R

25R

The electrical resistance of the thermistor decreases as temperature increases.


Which of the following reasons best explains this?
A
B
C
D

22.

0.040R

There is an increase in the vibrations of the metallic ions.


There is a decrease in the vibrations of the metallic ions.
There is an increase in the number of charge carriers per unit volume.
There is a decrease in the number of charge carriers per unit volume.

In which of the following circuits would the ammeter show the greatest current
reading?
R

IJC 2008

8866/01/Prelim2

[Turn Over

11
23.

The diagram shows three resistors of resistance 4 , 10 and 6 connected


in series. A potential difference of 10 V is maintained across them, with point Q
being earthed.

Which graph represents the change in potential along the resistor network?

24.

What is the equivalent resistance of the circuit arrangement between M and N?


10

N
10

5.0

IJC 2008

20
10

8866/01/Prelim2

30

40

[Turn Over

12
25.

A long, horizontal, straight wire is placed parallel to the plane of a coil as shown
in the diagram. Both the coil and the wire carry currents in the directions
indicated.

Coil
X
P
Y
wire

currents
What is the direction of the force at point P on the wire due to these currents?
A
B
C
D
26.

in the direction of X
in the direction of Y
out of the plane of the paper
into the plane of the paper

X, Y and Z are three long parallel wires carrying currents in a vertical plane as
shown.
X

10A

10A

20A

The resultant force on wire Y is


A
B
C
D

zero
towards X
towards Z
perpendicular to the paper

IJC 2008

8866/01/Prelim2

[Turn Over

13
27.

The diagram below shows a proton moving at velocity v in a uniform magnetic


field of magnetic flux density B.

proton
Deduce the subsequent path of the proton.
A
B
C
D

stationary
straight
circular
helical

28. The energy levels of an atom of an element are shown in the following diagram,
with values given in eV. Which energy transitions will produce photons of
wavelength 620 nm?
A

B
0
C

D
-1.0 eV

-3.0 eV

-10 eV

IJC 2008

8866/01/Prelim2

[Turn Over

14
29.

An energy level diagram for an atom is shown in the diagram. The electron
transitions give rise to the emission of a spectrum of lines of wavelength 1, 2,
3, 4 and 5.

What can be deduced from the diagram?


A
B
C
D

30.

1 > 2
3 = 4 + 5
4 is the shortest of the five wavelengths
The transition corresponding to wavelength 3 represents the
ionisation of the atom.

The diagram shows part of a typical line emission spectrum. This spectrum
extends through the visible region of the electromagnetic spectrum into the
ultraviolet region.

Which statement is true for emission line X of the spectrum?


A
B
C
D

It has the longest wavelength and is at the ultraviolet end of the


spectrum.
It has the highest frequency and is at the ultraviolet end of the
spectrum.
It has the shortest wavelength and is at the red end of the spectrum.
It has the lowest frequency and is at the red end of the spectrum.

END OF PAPER

IJC 2008

8866/01/Prelim2

[Turn Over

Innova JC Prelim 2
H1 Physics Paper

1 (Solutions)
Q1
Q2
Q3
Q4
Q5
Q6
Q7
Q8
Q9
Q10

D
B
C
C
C
D
B
D
A
B

Q11
Q12
Q13
Q14
Q15
Q16
Q17
Q18
Q19
Q20

A
B
D
B
D
C
A
C
C
A

Q21
Q22
Q23
Q24
Q25
Q26
Q27
Q28
Q29
Q30

C
D
D
A
A
B
D
C
A
D

1.

Answer D
A light year is the distance travelled by light in 1 year

2.

Answer B
The mass of the sprinter is estimated to be 80 kg and his velocity is 10 m s-1.

3.

Answer C

T
4f 2L2

f
L
T
100% =
+2
+2
100%
f
L

0.0001

1
0.0001
100% =
+ 2
+ 2 0.5398 100%

123

0.0370

100% = 1.9%

4.

Answer C
At any instance, a newtonian object cannot be at 2 positions.

5.

Answer C
From P to Q, the object is undergoing free-fall (air resistance is negligible), hence the
acceleration is 9.81 m s-2.

6.

Answer D
During rising and falling, acceleration is the same (free-fall). At point of impact, there is
a sudden reversal of acceleration to reverse the velocity. However, due to energy lost,
the reversing acceleration gets smaller with subsequent impact.

7.

Answer B
For the cart to decelerate as it moves to the right, the net force should be acting to the
left.

8.

Answer D
To move the object at a constant speed up the slope, F is equal to the component of
weight of the crate along the slope (F = W sin ).
Hence if is increased, a larger F is required.
Work done would also increase for the same displacement as applied force F is larger.
1

9.

Answer A
Both objects accelerate at free fall, they are still travelling with the same speed at any
instance. Hence tension is 0 N.

10.

Answer B
Taking moment about the geometrical centre of the object, we can see that
in (i), the moment produced by the 20 N is not cancelled
in (iii), the vertical (up and down) forces form an anticlockwise couple and the
horizontal (left and right) forces form a clockwise couple. However, the magnitude of
the anticlockwise couple is larger than that of the clockwise couple due to a farther
perpendicular distance from the geometrical centre.

11.

Answer A
Since the object is moving at constant velocity, there is no change in the kinetic energy.
In fact, all the work done by the applied force has been used to do work against friction.

12.

Answer B
Considering the apple and the spring as a system,
Initial energy of the system = final energy of the system
GPEi of apple + EPEi of spring = GPEf of apple + EPEf of spring
mghi + 0 = 0 + k x2
(0.100) (9.81) (2.0 + 0.10) = k (0.10)2
k = 410 N m-1

13.

Answer D
The energy of a wave at any point is indicated by the amplitude at the point. A larger
amplitude suggests larger energy. As the amplitude is the same for all the particles, the
total energy of each is the same.
Note:
The instantaneous y-displacement may vary but the sum of the KE and PE is the same.
For example, at P and R, KE is maximum but PE is zero. At Q and S, KE is zero but
PE is maximum.

14.

Answer B

t
=
2 T
1

=8
2
T

= rad
4

15.

Answer D
Radio waves and gamma rays are electromagnetic waves. They travel with the same
speed of 3.0 108 m s-1.

16.

Answer C
The angular differences between the polarising axes are as follows.
W
o
60

X
o
15

Y
o
90

Z
o
30

Malus law for the polarised light emerging from a pair of polarisers is given as

I = I o cos 2
where

Io: intensity of the unpolarised incident light.


I : intensity of the polarised emerging light
: the angle between the polarising axis or directions.

Thus, in decreasing order of intensity for the emergent light, the pairs are X, Z, W and
Y.
17.

Answer A
At the open end of the pipe is a displacement antinode (motion of air particles is
maximum) and a pressure node (changes in air pressure is minimum).
At the closed end of the pipe is a displacement node (motion of air particles is
minimum) and a pressure antinode (changes in air pressure is maximum).

18.

Answer C
At P, the waves meet at antiphase all the time, hence destructive interference occurs.

19.

Answer C
=

d
D

The fringe separation x is given by

Since the wavelength and the distance D between the screen and the slits are
unchanged, reducing the slit separation to d/2 will cause the fringe separation to
double.
The slit width does not affect the interference fringe separation. However, the intensity
of the fringes will be reduced if the slit width is narrowed.
20.

Answer A
Let
Rnet
R
RY

= net resistance per metre of whole cable.


= resistance per metre of conductor X.
= resistance per metre of conductor Y

R = 4RY
The conductors are connected parallel to each other.
1
1 6 1
6
1
25
= +
= +
= (1+ 24 ) =
Rnet R RY R R/4 R
R

Rnet =

1
R = 0.040R
25

21.

Answer C
For thermistors, as the temperature increases, the vibrations of the metallic ions
increase, causing more resistance to the flow of charge carriers. However, the number
of charge carriers per unit volume also increases and this factor, being more significant
would mean that the overall resistance of the thermistor decrease.

22.

Answer D
The effective resistance in D is the lowest, hence the current in the circuit is the
greatest.

23.

Answer D
p.d. across 4 = 2 V
p.d. across 10 = 5 V
p.d. across 6 = 3 V
Hence, the potential value at P is 2.0 V, Q is 0 V (earthed), R is -5 V and S is -8 V.
Answer A
Current bypasses the 20 resistor. Therefore it can be ignored.

01 01
1 1

0.5
1

24.

R = 5.0
25.

Answer A
Direction of magnetic field due to the coil is into the paper. Using Left Hand Rule
direction of force on wire at P is upwards

26.

Answer B
X

Force by Z
Force by X

10A

10A

20A

The force on Y due to Z is to the left.


The force on Y due to X is also to the left.
Hence the resultant force is to the left (towards X)
27.

Answer D
As the proton has a velocity component (v sin ) that is perpendicular to the magnetic
field, it experiences a magnetic force which is always perpendicular to the velocity
(Flemings Left Hand Rule). This causes the proton to move in a circle while traveling
horizontally to the right with a constant velocity of v cos . The overall path is helical.

28.

29.

Answer C
E = hc/
E = hc/
E = [(6.63x10-34)(3x108)] / 621 x10-9
E = 3.2 x10-19 J
E = 2.0 eV
Answer A
E1 =

hc

and E2 =

hc

Since E1 < E2, 1 > 2


30.

Answer D
Emission line X corresponds to the photon emitted for the smallest E between 2
energy levels. Hence it is of the lowest frequency.

INNOVA JUNIOR COLLEGE


JC 2 PRELIMINARY EXAMINATION 2
in preparation for General Certificate of Education Advanced Level

Higher 1
CANDIDATE
NAME
CLASS

INDEX NUMBER

PHYSICS

8866/02

Paper 2 Structured Questions

12 September 2008
2 hours

Candidates answer on the Question Paper.


No Additional Materials are required.
READ THESE INSTRUCTIONS FIRST

Do not open this booklet until you are told to do


so.
Write your name, civics group and index number in the
spaces at the top of this page.
Write in dark blue or black pen on both sides of the
paper.
You may use a soft pencil for any diagrams, graphs or
rough working.
Do not use staples, paper clips, highlighters, glue or
correction fluid.

EXAMINERS USE ONLY


MARKS
SECTION A
Q1

/6

Q2

/6

Q3

/6

Q4

/6

Q5

/ 16

SECTION B

Section A
Answer all questions.

Q6
Q7

You are advised to spend about one hour on each


section.
At the end of the examination, fasten all your work
securely together.
The number of marks is given in brackets [ ] at the end
of each question or part question.

/ 20

Q8

Section B
Answer any two questions.

/ 20

/ 20

Significant
figures

Paper 2 TOTAL

/ 80

Paper 1 TOTAL

/ 30

Grand Total

/ 110

PERCENTAGE

This document consists of 23 printed pages and 1 blank page.


Innova Junior College
IJC 2008

8866/02/Prelim2

[Turn Over

2
Data
speed of light in free space,

c = 3.00 x 108 m s-1

elementary charge,

e = 1.60 x 10-19 C

the Planck constant,

h = 6.63 x 10-34 J s

unified atomic mass constant,

u = 1.66 x 10-27 kg

rest mass of electron,

me = 9.11 x 10-31 kg

rest mass of proton,

mp = 1.67 x 10-27 kg

acceleration of free fall,

g = 9.81 m s-2

Formulae
uniformly accelerated motion,

s = ut + at2
v2 = u2 + 2as

work done on/by a gas,

W = pV

resistors in series,

R = R1 + R2 +

resistors in parallel,
hydrostatic pressure,

1/R = 1/R1 + 1/R2 +


p = gh

3
Section A (40 Marks)
Answer all the questions in this section.

For
Examiners
Use

1 (a) Why is it considered good experimental practice to take several readings for
the diameter of the cylinder, rather than to rely on a single measurement?
.
. [1]
(b) In order to determine the density of the metal cylinder, the student measured
the height h, of the cylinder with a ruler, the diameter d, of the cylinder with a
vernier calipers and its mass m, with an electronic balance. The readings were
as follows:
height h of cylinder = (12.0 0.1) cm
mass m of cylinder = (536 1)g
diameter d of the cylinder = (2.53 0.01) cm
(i)

Using his readings, find the density of the metal cylinder.

= ... g cm -3 [2]
(ii)

Calculate the absolute uncertainty in the density of the cylinder and


hence express the density and its associated uncertainty to an
appropriate number of significant figures.

= ... g cm-3

= (... ...) g cm -3 [3]


IJC 2008

8866/02/Prelim2

[Turn over

4
2 (a) Two planks, X and Y, each of length 2.40 m and weight 100 N, are joined endto-end by a hinge at A to form a structure as shown in Fig. 2.1. The structure
stands on a frictionless horizontal surface. A light rope of length 1.20 m is
attached to the midpoints of the two planks.

For
Examiners
Use

A
2.40 m

1.20 m

Fig. 2.1
(i)

Find the normal force exerted on the structure by the surface.

(ii)

Find the tension in the rope.

normal force = .. N [1]

tension = .. N [2]
(iii)

Hence find the force exerted by the hinge on plank X.

force exerted by the hinge on plank X =.. N [1]

[Turn over
IJC 2008

8866/02/Prelim2

5
(b) The structure in part (a) is then placed on a smooth surface that is concave
upwards so that equilibrium is still maintained (see Fig. 2.2.)

For
Examiners
Use

Fig. 2.2
(i)

Draw arrows on Fig. 2.2 to represent the forces acting on the structure.
[1]

(ii)

Explain why the normal force on the structure by the concave surface in
Fig. 2.2 will be greater than the normal force by the horizontal surface in
Fig. 2.1.
.
. [1]

[Turn over
IJC 2008

8866/02/Prelim2

6
3

Fig. 3.1 shows an arrangement to investigate the interference between


microwaves that meet after being reflected from two parallel surfaces. The
hardboard sheet reflects some energy while allowing some to pass through it
and the metal plate reflects all the energy that falls on it.
transmitter
and receiver

For
Examiners
Use

movement
of metal
plate

microwave
generator
and
receiver

hardboard sheet
(partial reflector)

metal plate
(total reflector)

Fig. 3.1
(a) As the metal plate is moved towards the hardboard sheet the received signal
strength goes through a series of maximum and minimum values. Explain why
the received signal strength is a minimum for certain positions of the metal
plate.
.
.
.
.
.
.. [3]
(b) The wavelength of the microwaves used in the experiment is 28 mm. A
maximum signal is observed when the metal plate is in position X.
(i)

Calculate the least distance that the metal plate will have to be moved
so that another maximum is observed.

least distance = .. mm [2]

[Turn over
IJC 2008

8866/02/Prelim2

7
(ii)

When the metal plate moves at a constant velocity the received signal
strength varies at a constant frequency. Calculate this frequency when
the metal plate is moved at 0.50 m s-1.

For
Examiners
Use

frequency = .. Hz [1]

[Turn over
IJC 2008

8866/02/Prelim2

8
4 (a) Fig. 4.1 shows a cross-section of a conductor carrying a current which flows
out of the plane of the paper at right angles. The conductor is placed between
the poles of a permanent magnet and it experiences a force.

For
Examiners
Use

Conductor carrying current out of the paper

Fig 4.1

(i)

On Fig 4.1, draw the magnetic flux pattern of the combined magnetic
field of the current- carrying conductor and the permanent magnet and
indicate the direction of the force on the conductor.
[1]

(ii)

State the energy change that has taken place.


.. [1]

(b) A beam of electrons is generated inside an evacuated flask which is placed in a


magnetic field of flux density 6.5 x 104T. The electrons are travelling at right
angles to the magnetic field, which is directed into the paper, as shown in
Fig. 4.2.

Initial direction of the


electrons
electron generator

magnetic field
flask
Fig. 4.2
(i)

On Fig. 4.2, sketch the path taken by the beam of electrons.

[1]

[Turn over
IJC 2008

8866/02/Prelim2

9
(ii)

Explain why the electrons move in a path as drawn is (b)(i).


.

For
Examiners
Use

.....
.
.....
.
. [3]

[Turn over
IJC 2008

8866/02/Prelim2

10
5

The internal resistance of a source dissipates power whenever we attempt to


transfer power from the source to a load. In this question, well investigate this
situation using the circuit in Fig. 5.1 where a source of e.m.f. VS and internal
resistance RS is connected across a load of resistance RL.

For
Examiners
Use

i
RS
RL
VS

Fig. 5.1
(a) (i)

Write down an expression for the current drawn from the source, i, in
[1]
terms of VS , RS and RL .

(ii)

Hence, show that the power dissipated due to the internal resistance,
PS , is given by

PS =

VS 2

R
RS 1 + L
RS

[2]

[Turn over
IJC 2008

8866/02/Prelim2

11
(b) (i)

Fig. 5.2 shows an incomplete table for PL


to different values of

and

PS corresponding

RL
. PL is the useful power developed across the
RS

load. Complete the table in Fig. 5.2.

For
Examiners
Use

[3]

RL
RS

PL

PS

0.0

0.000

0.4

VS 2
0.204 x
RS

1.00 x

VS 2
RS

0.8

0.247 x

VS
RS

1.2

0.248 x

VS
RS

0.207 x

VS 2
RS

1.6

0.237 x

VS 2
RS

0.148 x

VS 2
RS

2.0

0.222 x

VS 2
RS

3.0

0.188 x

VS
RS

4.0

0.160 x

VS
RS

VS
RS

5.0

0.139 x

6.0

V2
0.122 x S
RS

0.020 x

VS 2
RS

7.0

0.109 x

VS 2
RS

0.016 x

VS 2
RS

Fig. 5.2
(ii)

For a particular source of e.m.f. Vo and internal resistance Ro, why is the
total power supplied by the source, PL + PS , not constant for all values
of

RL
?
RS

.
.
.. [1]

[Turn over
IJC 2008

8866/02/Prelim2

12

(iii)

Fig. 5.3.
P/

For
Examiners
Use

RL
has been drawn on Fig. 5.3. Using data
RS
R
from the table in Fig. 5.2, sketch the graph of PS against L on
RS

The graph of PL against

[1]

VS 2
RS

1.1
1.0
0.9
0.8
0.7
0.6
0.5
0.4
0.3
0.2
0.1
RL
RS

0.0
0

Fig. 5.3

[Turn over
IJC 2008

8866/02/Prelim2

13
(c) Using Fig. 5.3, answer the following questions, giving your answers in terms of
VS , RS and/or RL where appropriate.
(i)

For
Examiners
Use

When a source has a fixed internal resistance RS , what is the condition


for maximum power transfer to a load?
.
.. [1]

(ii)

What is this maximum power transferred to the load?

maximum power = .. [1]

(iii)

Efficiency is the ratio of useful power developed across the load to total
power generated by the source. What is the efficiency corresponding to
maximum power transfer to the load?

efficiency = .. % [2]

(iv)

What happens to the efficiency when RL<< RS?


.
.
.
.. [2]

(v)

What happens to the efficiency when RL>> RS?


.
.
.
.. [2]

[Turn over
IJC 2008

8866/02/Prelim2

14
Section B (40 Marks)

For
Examiners
Use

Answer two questions from this section.

6 (a) (i)

In the study of kinematics, we normally adopt a coordinate system (e.g.


x,y and z) as a frame of reference. Explain why it is necessary.
.
.. [1]

(ii)

Jack and Jill traveled from A to B during the same time interval. Jack
took the curved road but Jill took the straight road as shown in Fig. 6.1.
Jack

Jill

Fig. 6.1
1.

Define average velocity during a time interval.

.
.. [1]

2.

Discuss if there is any difference between the average velocity


of Jack and Jill.

.
.
.
.. [2]

[Turn over
IJC 2008

8866/02/Prelim2

15
3.

A third person, Peter decides to move from B to A in an erratic


fashion as in Fig. 6.2. Calculate his average velocity between X
and Y.

For
Examiners
Use

Displacement / m

150

100

50

Y
B

10

20

30

time / s

Fig. 6.2

average velocity = .. m s-1 [2]

(iii)

Distinguish between negative acceleration and deceleration.


.
.. [1]

(b)

Two particles of masses m1 and m2 moving along the same straight line on a
smooth surface with velocities u1 and u2 respectively, collide elastically and
move off with velocities v1 and v2.

(i)

Explain what is meant by an elastic collision.

.............. [1]

[Turn over
IJC 2008

8866/02/Prelim2

16
(ii)

Using the answers in (b)(i), show that for the above collision,
v2 v1 = u1 u2.

(iii)

Hence, show that


v1 =

(iv)

[3]

For
Examiners
Use

m1 m2
2m2
u1 +
u2
m1 + m2
m1 + m2

[2]

Using the result from (b)(iii), discuss the motion of a particle of mass m
after it collides head-on with a stationary particle of mass 99m, paying
particular attention to the direction and magnitude of its velocity after
collision as compared to that before collision.

.
.
.. [2]

[Turn over
IJC 2008

8866/02/Prelim2

17
(c)

A person of mass 75 kg jumps off a tall bridge with a bungee cord (a heavy
stretchable cord) tied around his ankle. He falls for 15 m before the bungee
cord begins to stretch. The cord obeys Hookes law with force constant
k = 200 N m1. The total mechanical energy of the system is assumed to be
constant at any time.

(i)

For
Examiners
Use

Calculate the maximum distance below the bridge the person will fall.
(Ignore the mass of the cord.)

maximum distance = .. m [3]

(ii)

How far below the bridge will the person be when he eventually comes
to a stop?

distance below the bridge = .. m [2]

[Turn over
IJC 2008

8866/02/Prelim2

18
7 (a) (i) Define diffraction.
.....

For
Examiners
Use

....... [1]

(ii)

Given an explanation for each of the following:

1. One can hear around corners, but not see around corners.

... [1]

2. Two flashlights held close together do not produce observable


interference pattern on a distant screen.

... [1]

(iii)

Describe the changes to the peak intensity and width of the central
maximum of the single-slit diffraction pattern when the width of the slit is
made narrower.

.... [2]

(b) (i)

Coherent light rays of wavelength strike a pair of silts separated by


distance d at angle of 1 as shown in Fig. 7.1 below. Assume an
interference maximum is formed at an angle 2 and the screen is at a
great distance from the slits.

2
1
d

Fig. 7.1
Derive the condition for constructive interference in this case in terms of d,
1, 2 , and n where n is an integer.
[2]

[Turn over
IJC 2008

8866/02/Prelim2

19
(ii)

Fig.7.2 shows a radio-wave transmitter and a receiver separated by a


distance 3000 m apart and both has a height 80 m above the ground. The
receiver can receive signals both directly from the transmitter and indirectly
from signals that reflect from the ground. Assume that the ground is level
and a 180 phase shift occurs upon reflection.

For
Examiners
Use

3000 m

80 m

Receiver

Transmitter

Fig.7.2
Determine the longest wavelengths that interfere

1. constructively and

wavelength = .. m [2]

2. destructively.

wavelength = .. m [2]

[Turn over
IJC 2008

8866/02/Prelim2

20
(c) (i) In a Youngs experiment, two narrow and parallel slits separated by
0.250 mm are illuminated by green light ( = 546.1 nm). The interference
pattern is observed on a screen 1.20 m away from the plane of the slits.

For
Examiners
Use

Determine the distance

1.

between the first order maxima on either sides of the central maximum

distance = .. m [2]

2.

between the first order maximum on one side and the second order
minimum on the other side of the principal axis.

distance = .. m [2]

(ii) It is possible to observe the interference effect in (c)(i) using electrons


which are particles. Explain the necessary modifications required for the
experimental set-up.
.
.

.... [3]

(iii) Describe the changes to the fringe pattern observed in (c)(i) if the number
of parallel slits is increased.
.

..... [2]
IJC 2008

8866/02/Prelim2

[Turn over

21
8 (a) In a photoelectric emission experiment, a metal plate is irradiated with light of
wavelengths 1 = 388 nm and 2 = 283 nm. The stopping potentials recorded
are V1 and V2 respectively.
(i)

For
Examiners
Use

Which stopping potential has the larger magnitude?


stopping potential: .. [1]

(ii)

If the larger stopping potential is twice of the other, find the work
function of the metal in eV.

work function = .. eV [3]

(b)

In another experiment, a light source of constant power 0.20 W emits light of


wavelength 480 nm uniformly in all directions. A metal plate with a small
surface area is placed 30 cm from this source and facing the source directly.
Electrons are emitted from the metal surface. At this wavelength, only one in
eight of the incident photons succeeds in ejecting a photoelectron from the
metal. Assuming that all the emitted electrons are collected, the current
detected is 8.5 A.

(i)

Calculate the power of the light incident on the metal plate.

power = .. W [3]

[Turn over
IJC 2008

8866/02/Prelim2

22
(ii)

If the metal plate is moved closer to the light source by 10 cm, find the
percentage increase in the incident power.

For
Examiners
Use

percentage increase = .. % [3]

(c)

Fig. 8.1 below represents the lowest energy levels of the mercury atom with
corresponding quantum number n and energy measured in eV.

Energy

-1.56 eV

-2.48 eV

-3.71 eV

-5.52 eV

-10.4 eV

Fig. 8.1
(i)

Distinguish between emission line spectrum and absorption line


spectrum.
.
.
. [2]

IJC 2008

8866/02/Prelim2

[Turn over

23
(ii)

Considering transitions between only these 5 levels, state

1.

the total number of possible emission transitions that can be


produced.

For
Examiners
Use

number of transitions = . [1]

2.

the spectral emission transition that has the shortest wavelength.


transition = .. [1]

(iii)

Calculate the wavelengths of the lines A and B arising from the


transitions marked A and B respectively.

A = ........................ m [1]
B = ........................ m [1]
(iv)

Cool mercury vapour at low pressure is bombarded with electrons


of kinetic energy E. State the transition(s), if any, that would be
expected to be observed if E has the value of

1.

4.00 eV
transition (s): ........................................................................[1]

2.

7.00 eV
transition (s): ........................................................................[1]

(v)

If an electron of kinetic energy 5.0 eV collides elastically with the


mercury atom in the ground state, calculate the kinetic energy of the
electron after collision.

kinetic energy = ........................ eV [2]

END OF PAPER
IJC 2008

8866/02/Prelim2

[Turn over

Innova JC Prelim 2
H1 Physics Paper

2 (Solutions)

Section A
1(a) By finding the average of several readings, random error will be reduced.
(b)(i)
m
m
4m
= =
=
2
V d
d 2h
h

4
4 536
=
2
( 2.53 ) 12.0

[M1]

= 8.89 g cm3
(ii)

[B1]

[A1]

m
d h
+2
+
m
d
h

d h
m

=
+2
+
d
h
m

1
0.01 0.1
=
+ 2
8.885
+
536
2.53 12.0

= 0.16 g cm3
0.2 g cm3 (to 1 s.f.)
= ( 8.9 0.2 ) g cm

2(a)(i)
(ii)

[M1]

[A1]

[A1]

magnitude of normal reaction = magnitude of weight = 200 N

[A1]

[M1]

(Substitute N = mg = 100 N)
T (1.20) sin 60 = 100(1.20) cos 60
T = 57.7 N
(iii)

Considering the forces on just one plank.


Taking moments about A,
T (1.20)sin 60 + mg(1.20) cos 60 = N(2.40) cos 60

[A1]

Considering the sum of horizontal components:


Fx = T
Fx = 57.7 N
Force exerted by hinge = 57.7 N

[A1]

IJC 2008

8866/02/Prelim2/Solutions

(b)(i)

mg

[B1]
Deduct 1 mark for each wrong/missing force
(ii)

The normal reaction force, N in (b) now has a vertical component of the same
magnitude of N in (a) and an additional horizontal component. Hence, by summing
up the components, N in (b) will be larger than N in (a)
[B1]

3(a) There is a path difference between the two reflected waves which arrive at the
receiver.
[B1]
Reflected waves are coherent.

[B1]

And any one of these:


[B1]
Minima occurs when the path difference is a odd number of half wavelengths
Minima is caused by destructive interference producing no resultant amplitude.
For a minima to occur, the waves arrive in anti-phase or, the crest meets the trough.
(A labelled diagram can be drawn to show destructive interference.)

(b) (i)

(ii)

When the path difference is integer multiples of wavelength (n), maxima is


obtained. Hence, to get the next maxima, the path difference needs to be
reduced by one.
Let d be the minimum distance moved by plate
=2d
[C1]
d = / 2 = 14 mm
[A1]

f = 0.50 / 14 x 10-3 = 35.7 Hz

[A1]

4(a)(i)

Force
correct magnetic flux pattern and correct direction of the force [B1]

(ii)

Electrical energy to mechanical energy. [B1]

IJC 2008

8866/02/Prelim2/Solutions

(b)(i)
Initial direction of the
electrons
Electron generator

magnetic field
flask
[B1]
(ii)

Using Flemings Left Hand Rule, the electrons experiences a magnetic force
directed to the right at the instance it enters the magnetic field.
[B1]
As the electrons moves in the deflected path, the magnetic force acting on it
remains at right angle to its velocity.
[B1]
Thus, the magnetic force provides the acceleration for the electrons to move in a
circular path.
[B1]

5(a)(i) i =

Vs
Rs + RL

[A1]

(ii)

PS = i 2RS

[M1]

=
=

VS RS
(RS + RL )2

[M1]

VS 2RS
R
RS 2 (1 + L )2
RS

VS 2
=
R
RS (1 + L )2
RS

IJC 2008

[A0]

8866/02/Prelim2/Solutions

(b)(i)

RL
RS
0.0

PL

PS

0.000

0.4

0.204 x

VS 2
RS

0.8

0.247 x

VS 2
RS

1.00 x

VS 2
RS

0.510 x

VS 2
RS

0.309 x

VS 2
RS

1.2

0.248 x

VS 2
RS

0.207 x

VS 2
RS

1.6

0.237 x

VS 2
RS

0.148 x

VS 2
RS

2.0

0.222 x

VS 2
RS

0.111 x

VS 2
RS

3.0

0.188 x

VS 2
RS

0.063 x

VS 2
RS

4.0

0.160 x

VS 2
RS

0.040 x

VS 2
RS

5.0

0.139 x

VS 2
RS

0.028 x

VS 2
RS

6.0

VS 2
0.122 x
RS

7.0

0.109 x

VS 2
RS

VS 2
0.020 x
RS
0.016 x

VS 2
RS
[B3]

Deduct 1 m for each wrong answer


(ii) Even as R0 is fixed, as RL changes, the total resistance varies. The total current in the
circuit changes.
[B1]
Hence the total power is not constant.

IJC 2008

8866/02/Prelim2/Solutions

(iii)
P/

VS 2
RS

1.1
1.0
0.9
0.8
0.7
0.6
0.5
0.4
0.3
0.2
RL
RS

0.1
0.0
0

8
correct graph [B1]

(c)(i) PL is maximum when

RL
=1
RS

[B1]

(ii) Maximum power = 0.25

IJC 2008

VS
RS

8866/02/Prelim2/Solutions

[B1]

(iii)

V
Since PL = PS, total P = PL + PS = 0.50 S
RS
VS 2
RS
100%
Efficiency =
VS 2
0.50
RS
0.25

[M1]

= 50%

[A1]
2

(v)

[B1]

Hence, efficiency tends to zero.

(iv)

V
PL tends to zero & PS tends to S
RS

[B1]

PS << PL
Efficiency tends to 100%.

[B1]
[B1]

IJC 2008

8866/02/Prelim2/Solutions

Section B
6(a)(i) It helps us define the starting point for measuring any quantity.
OR
To give a precise location or positioning of any moving object.
(ii)

[B1]

1. Average velocity is the change in displacement divided by the time interval OR


Change of displacement over the time interval
[B1]
2. Since Jack and Jill started and end off at the same point, their displacements are
the same.
[B1]
As they also took the same time interval, their average velocities are the same
too.
[B1]
3.

90 190
28 8
100
=
= 5.0 m s1
20

Average velocity =

[C1]
[A1]

(iii)

Deceleration is the slowing down of speed while negative acceleration simply means
the acceleration is in the negative direction with no indication if it is speeding up or
slowing down.
[B1]

(b)(i)

An elastic collision is a collision in which total kinetic energy and the total momentum
of the colliding particles is conserved.
[B1]

(ii)

By the law of conservation of momentum,


m1u1 + m2u 2 = m1v 1 + m2v 2
m1(u1 v 1 ) =

m2 (v 2 u 2 ) (1)

Since the collision is elastic, the total KE is conserved:


1
1
1
1
2
2
2
2
m1u1 + m2u 2 = m1v 1 + m2v 2
2
2
2
2
1
1
2
2
2
2
m1(u1 v 1 ) =
m2 (v 2 u 2 ) (2)
2
2

[M1]

[M1]

Dividing (2) by (1),


u1 + v 1 = v 2 + u 2
Hence, u1 u 2 = v 2 v 1

(iii)

[M1]
[A0]

v 2 = u1 u 2 + v1 (3)

[M1]

Substitute (3) into (1)


m1(u1 v1 ) =

m2 (u1 u2 + v1 u2 )

(m1 + m2 ) v1 = ( m1 m2 ) u1 + 2m2
v1 =

IJC 2008

m1 m2
2m2
u1 +
u2
m1 + m2
m1 + m2

8866/02/Prelim2/Solutions

[M1]
[A0]

(iv)

Let m1 be the particle of mass m and m2 be the particle of mass 99m. Then u2 = 0.

2 99m
m 99m
(0)
u1 +
100m
100m
= 0.98 u1

v1 =

[B1]

So the particle moves off in the direction opposite to its original direction with a speed
slightly less than (98%) its original speed.
[B1]
(c)(i)
Let the lowest point be h + 15 m from top of bridge and take it as reference, GPE =0.
By conservation of mechanical energy,
KE1 + GPE1+ EPE1 = KE2 + GPE2+ EPE2
1 2
kx
2
75 9.81( h + 15 ) = 100h 2
0 + mgh + 0 = 0 + 0 +

[M1]

100h 735.75h 11036.25 = 0


735.75 ( 735.75)2 4(100 11036.25)
2(100)
h = 14.81 or 7.45( rejected )
h=

Distance below bridge = 15 +14.8 = 29.8 m


(ii)

[M1]
[A1]

When it comes to stop, the system would be in equilibrium.


mg = kx
75 9.81 = 200 x
x = 3.679 m

[M1]

Final displacement = 15 + 3.679 = 18.7m

[A1]

IJC 2008

8866/02/Prelim2/Solutions

7(a) (i)
(ii)

Diffraction refers refers to the bending of waves around an object or [B1]


spreading of waves through an aperture.
1. The wavelength of sound is very much longer than the width of [B1]
the corners that causes diffraction (spreading) of sound waves
around them. However, the wavelength of light is very much
shorter and little diffraction occurs.
2. The two light sources are not coherent. They do not have a [B1]
constant phase relation.

(iii)

(b)

The intensity of the central maximum decreases


[B1]
and its width becomes broader (distance between the two first [B1]
minimum increases)

(i)
Path difference = d sin 1 d sin 2
= n
(ii)

Path difference = 2

15002 + 802 - 3000

= 4.3 m
Since the sources are now 180o out of phase,
1. For constructive interference path difference =
= 2 ( 4.26) = 8.5 m
.2 For destructive interference path difference =
= 4.3 m
(c)

(i)

[B1]
[B1]
[M1]
[M1]
[A1]
[A1]

d = 0.250 10-3 m; = 546.1 10-9 m; D = 1.20 m


The fringe separation, x = D
d
= 2.62 10-3 m
1. Distance between the first order maximum (bright line) on both
sides
[M1]
=2x
-3
[A1]
= 5.24 10 m
2. Distance between the first order maxima on one side and second
order bands
[M1]
= 2.5 x
[A1]
-3
= 6.55 10 m

(ii)

Increase the velocity (kinetic energy) of electron by an accelerating [B1]


potential difference.
Replace the parallel slits with crystal with a lattice structure.

[B1]

The matter wavelength associated with the electron must be of the [B1]
same order as the lattice spacing.
(iii)

IJC 2008

The fringes will become more intense


and sharper.
8866/02/Prelim2/Solutions

[B1]
[B1]

8 (a) (i)
(ii)

hc

[A1]

V2

1
hc

= + eV1 ........... (1)


= + e ( 2V1 ) ............(2)

[M1]

From (1),

1 hc
............(3)

e 1

Sub (3) into (2),

V1 =

hc

= + 2

2
1

2hc hc
=

hc

2
1

= 6.63 10 34 3.0 108

388 109 283 10 9

-19
=3.22 10 J

)(

[M1]

= 2.02eV
(b) (i)

[A1]

Let rate of emission of electrons be ne.

8.5 10 6
1.6 10 19
= 5.31 1013 s 1

ne =

[M1]

Let rate of incidence of photons of photons be np.


np = 8ne
= 4.25 1014 s 1

[M1]

Incident power, Pi =

np hc

( 4.25 10 )( 6.63 10 )(3.0 10 )


=
( 480 10 )
14

34

= 1.76 104 W

IJC 2008

8866/02/Prelim2/Solutions

[A1]

10

(ii) Let P0 be the power of the source and A be the surface area of the metal plate.

Pi =

P0 A
4 (0.30 )

Let Pi be the new incident power on the plate.


P0 A
Pi =
2
4 ( 0.20 )
Pi ( 0.30 )
Therefore,
=
Pi ( 0.20 )2
P Pi
Percentage increase = i
100%
Pi
2

= i 1 100%
Pi

2
0.30

=
1 100%
2
0.20

= 125%

[M1]

[M1]
[A1]

(c)(i) Emission line spectrum: bright spectrum lines on dark background


Absorption line spectrum: dark spectrum lines on bright background

[B1]
B1]

(ii) 1. Including the 2 transitions that are already marked, there are altogether 10 transitions
that can be produced.
[B1]
5
Alternatively, use C 2 = 10 to obtain total possible number of transitions between 5
given energy levels.
2.
(iii)

The transition that has the smallest wavelength will correspond to the largest energy
1
[B1]
change. Therefore, it is from 5
For transition A,

For transition B,

=
(iv)

)(

)(

hc
6.63 10 34 3.00 10 8
=
= 6.87 10 7 m
E (5.52 3.71) 1.60 10 19

[A1]

hc
6.63 10 34 3.00 10 8
=
= 3.14 10 7 m
19
E (5.52 1.56) 1.60 10

[A1]

No transition will be observed.

[B1]

2.
(v)

1.

3 1, 3 2 and 2 1

[B1]

Kinetic energy of the electron is

= 5.00 - (10.4 - 5.52)


= 0.12 eV

IJC 2008

[M1]
[A1]

8866/02/Prelim2/Solutions

11

Jurong Junior College H1 Physics Preliminary Examination 2008

JURONG JUNIOR COLLEGE


Name: _________________________

Class: 07S______

Preliminary Examination 2008

H1 Physics (8866/01)
PAPER 1

Multiple Choice
August 2008
1 hour

Additional Material :

Multiple Choice Answer Sheet

READ THESE INSTRUCTIONS FIRST


Write in soft pencil.
Do not use staples, paper clips, highlighters, glue or correction fluid.
Write your name and class on the Answer Sheet and Question Paper in the spaces
provided.
There are thirty questions on this paper. Answer all questions.
For each question there are four possible answers A, B, C and D. Choose the one you
consider correct and record your choice in soft pencil on the separate Answer Sheet.
Each correct answer will score one mark. A mark will not be deducted for a wrong answer.
Any rough working should be done in this booklet.

(This question paper consists of 10 printed pages)

Page 1 of 10

Jurong Junior College H1 Physics Preliminary Examination 2008

Data and Formulae

Page 2 of 10

Jurong Junior College H1 Physics Preliminary Examination 2008

The radiancy, RT, is defined as the total energy emitted per unit time per unit area from a
blackbody at thermodynamic temperature T. It was first stated empirically in 1879 in the
form given by
RT = T4
where is the Stefan-Boltzmann constant. Which of the following is a unit for ?
A

J s m-2 K-4

W m-2 K-4
kg s-3 K4

J s-1 m-2 K4

Which pair includes a vector quantity and a scalar quantity?


A
B

Force and kinetic energy

Power and speed

Displacement and acceleration

Work and potential energy

The power loss P through a resistor is found by measuring the potential difference V
across the resistor and the current I through it. The equation is given by P = VI. The
voltmeter has a 4% uncertainty and the ammeter reading has a 3% uncertainty. What is
the uncertainty in the power calculated?
A

3%

4%

7%

12 %

When comparing systematic errors and random errors, the following pairs of properties of
errors in an experimental measurement may be contrasted.
P1:

error can possibly be eliminated

P2:

error cannot possibly be eliminated

Q1: error is of constant sign and magnitude


Q2: error is of varying sign and magnitude
R1:

error will be reduced by averaging repeated measurements

R2:

error will not be reduced by averaging repeated measurements

Which properties apply to random errors?


A

P1, Q1, R2

P1, Q2, R2

P2, Q2, R1

P2, Q1, R1

Page 3 of 10

Jurong Junior College H1 Physics Preliminary Examination 2008

A sandbag is released from a hot air balloon when it is 10 m above the ground and
ascending at a velocity of 4.0 m s-1. Which of the following graphs best describes how the
velocity of the sandbag will vary with time?
A

A basketball is projected with an initial speed of 10.0 m s-1 at an angle of 30 above the
horizontal, towards a vertical wall, which is located at a horizontal distance 10.0 m away
from the ball. Determine the final speed of the ball when it hits the wall.
A
B

6.33 m s-1

8.06 m s-1

7.

1.15 m s-1

10.7 m s-1

An aircraft in level flight is moving with constant velocity relative to the ground. The
resultant force acting on the aircraft is equal to
A

the weight of the aircraft.

the resultant of the air resistance and the thrust of the engines.

the resultant of the air resistance and the weight of the aircraft.

zero.

Page 4 of 10

Jurong Junior College H1 Physics Preliminary Examination 2008

8.

The diagram shows two masses 10 kg and 20 kg respectively, joined by a light string
which passes over a light frictionless pulley. The acceleration of free fall is g.

The acceleration of the masses is


A

2g
3

9.

g
3
g

3g
2

Which of the following is a statement of the principle of conservation of momentum?


A
B

In an elastic collision, momentum is constant.

The momentum of an isolated system is constant.

10

Momentum is the product of mass and velocity.

The force acting on a body is proportional to its rate of change of momentum.

The graph shows the variation of force F acting on a body with time t.
F/N
5

t/s

The change in momentum of the body during this 4 s period is


A

2.5 N s

5.0 N s

7.5 N s

12.5 N s

Page 5 of 10

Jurong Junior College H1 Physics Preliminary Examination 2008

11

Newton's third law of motion applies to an action-reaction pair of forces. Which statement
concerning the forces is not correct?
A

The two forces are at all time equal in magnitude

The two forces must act on different bodies

The two forces are always in opposite direction.

The two forces are equal and opposite so the bodies are in equilibrium.

12

A ball is released from rest on the top of a rough slope. Which of the following best show
the direction of the force exerted on the ball by the slope at the position shown in the
diagram?

13

A body of mass 2.0 kg initially at rest, slides down a 10.0 m long incline plane, from the
top of the inclined plane that is 1.0 m high. If the body experiences a constant resistive
force of 0.5 N over the slope, what is the kinetic energy of the body at the base of the
plane?
A
B

14.6 J

19.1 J

14

9.6 J

24.6 J

An electric motor is required to haul a load of mass 400 kg up a vertical height of 1200 m
in 2.0 minutes at constant velocity. Find the total power wasted if the overall efficiency of
the motor is 80%.
A

7.8 kW

9.8 kW

39 kW

49 kW
Page 6 of 10

Jurong Junior College H1 Physics Preliminary Examination 2008

15

A car engine provides 25 kW of useful power for a car travelling at constant speed. The
driving force on the car is 500 N. At what speed does the car travel?
A
B

4.2 m s-1

25 m s-1

16

0.05 m s-1

50 m s-1

An opaque object 10 cm wide casts a shadow when placed in a beam of light but has little
effect in blocking a beam of sound emitted by a small source of frequency 100 Hz. This is
because
A
B

sound waves are longitudinal whereas light waves are transverse.

sound waves travel at a much slower speed than light.

17

sound waves have a much longer wavelength than light waves.

sound is a pressure wave whereas light is an electromagnetic wave.

Which of the following gives an estimate of the minimum frequency of visible light?
A
B

8 x 1014 Hz

4 x 1018 Hz

18

4 x 1014 Hz

8 x 1018 Hz

A sound wave of frequency 500 Hz is travelling in a gas at a speed of 300 m s-1. When
the phase difference between two points is

7
radians, what is the distance between the
4

two points in mm?


A

0.525

1.050

525

1050

Page 7 of 10

Jurong Junior College H1 Physics Preliminary Examination 2008

19

Two sources of waves are said to be coherent if


A
B

they have same frequency and amplitude.

they have a constant phase difference.

20

they are in phase.

they can interfere constructively.

The diagram below shows the interference pattern in a ripple tank when the probes P and
Q are vibrating in phase and at a frequency of 20 Hz.

The lines of the pattern are equally spaced between P and Q and are 1 cm apart. The
speed of the water waves in the ripple tank is
A

0.2 m s-1

21

0.1 m s-1
0.4 m s-1

0.8 m s-1

The current in a component is reduced uniformly from 100 mA to 20 mA over a period of


8.0 s. What is the charge that flows during this time?
A

320 mC

22

160 mC
480 mC

640 mC

A wire carries a current of 2.0 amperes for 1.0 hour.


How many electrons pass a point in the wire in this time?
A

7.2 103

23

1.2 1015
1.3 1019

4.5 1022

A 12 V battery supplies a steady current of 200 A for 2.5 ms. The energy supplied by the
battery is
A

3J

6J

62 J

300 J

Page 8 of 10

Jurong Junior College H1 Physics Preliminary Examination 2008

24

The diagram shows a circuit where two resistors each of 5 are connected across three
cells of negligible internal resistance.
2V

2V

1V

5
What is the potential difference across XY?
A

1.5 V

25

0.5 V
2.5 V

3.5 V

In the circuit below, a 9.0 V supply of negligible internal resistance is joined to two
resistors of values 10 k and 30 k in series. A voltmeter of resistance 60 k is
connected in parallel with the resistor of value 30 k.

What is the reading on the voltmeter?


A

6.0 V

26

2.7 V
6.8 V

8.1 V

If a stationary electron is subjected to a uniform magnetic field it will be


A

accelerated in the direction of the field.

caused to move in a circular path.

caused to move in an elliptical path.

unaffected.

Page 9 of 10

Jurong Junior College H1 Physics Preliminary Examination 2008

27

A metallic rod PQ of density , area of cross-section A and length L, is suspended


horizontally by two wires. A uniform magnetic field B is directed perpendicular to the
length of the rod. A certain current I passed along the rod as shown removes the tension
in the supporting wires. The current I is proportional to
A
C

28

B
D

A
A

1
AL
AL

Which of the following observations regarding the photoelectric effect is not predicted by
the classical theory of electromagnetic radiation?
A
B

Photoelectric emission is observable using any part of the electromagnetic radiation.

The maximum kinetic energy of the photoelectrons is dependent on the intensity of


radiation.

29

The rate of photoelectric emission is proportional to the intensity of the incident


radiation.

Irrespective of the intensity of the radiation, photoelectric emission is instantaneous.

Photons, each of energy E and speed c, are incident normally on the solar panel of a
satellite at a rate of N per second. Assuming complete absorption of the photons, what is
the force exerted on the solar panel?
A

30

NE
c
Ec
N

NEc

Nc
E

Transitions between three energy levels in a particular atom give rise to the three spectral
lines of frequencies, in increasing magnitudes f1, f2 and f3. Which one of the following
equations correctly relates f1, f2 and f3?
A
C

1
1
1
=
+
f1
f2 f3
f1 = f 3 + f 2

f 1 = f 3 f2

f3 = f2 f1

Page 10 of 10

Jurong Junior College

H1 Physics Preliminary Examination 2008 (Paper 2)

JURONG JUNIOR COLLEGE


Name: _________________________

Class: 07S______

Preliminary Examination 2008

H1 Physics (8866/02)
PAPER 2

Structured Questions
August 2008
2 hours

Candidates answer on the Question Paper.


No additional materials are required.

READ THESE INSTRUCTIONS FIRST


Do not open this booklet until you are told to do so.
Write your name and class in the spaces provided at the top of this page
and on all the work you hand in.
Write in dark blue or black pen in the spaces provided on the Question
Paper.
You may use a soft pencil for any diagrams, graphs or rough working.
Do not use staples, paper clips, highlighters, glue or correction fluid.
This paper consists of 2 sections. All answers will be written in spaces
provided on the Question Paper.
Section A (40 marks)
Answer all questions.

For Examiners
Use
Section A

1
2
3
4
Section B

Section B (40 marks)


Answer any two questions.

You are advised to spend about one hour on each section.


At the end of the examination, fasten all your work securely together.
The number of marks is given in brackets [ ] at the end of each question or
part question.

7
Total

(This question paper consists of 19 printed pages including this page)


Page 1 of 19

Jurong Junior College

H1 Physics Preliminary Examination 2008 (Paper 2)

Data and Formulae

Page 2 of 19

Jurong Junior College

H1 Physics Preliminary Examination 2008 (Paper 2)

Section A
Answer all the questions in this section.

A car starts from rest and travels upwards along a straight road inclined at 5.0o to the
horizontal, as illustrated below.
The length of the road is 450 m and the car has mass 800 kg. The speed of the car
increases at a constant rate to reach 28 m s-1 at the top of the slope and it takes 32.1 s.
The constant power output of the engine is 2.50 x 104 W.
450 m

5.0o

(a)

Determine, for this car traveling up the slope,


(i)

[1]

(ii)

(b)

The gain in kinetic energy,

The gain in gravitational potential energy.

[2]

(i)

Use your answer in (a) to determine the useful output power of the car.

[2]

Page 3 of 19

Jurong Junior College

(ii)

H1 Physics Preliminary Examination 2008 (Paper 2)

Hence, determine the efficiency of the engine.

[1]

A wire frame ABCD is supported on two knife-edges P and Q so that the section PBCQ of
the frame lies within a solenoid as shown in Fig.2.

Fig. 2
Electrical connections are made to the frame through the knife-edges so that the part
PBCQ of the frame and the solenoid can be connected in series with a battery. When there
is no current in the circuit, the frame is horizontal.
(a)

When the frame is horizontal and a current passes through the frame and solenoid,
what can you say about the direction of the force, if any, due to the magnetic field of
the solenoid acting on
(i)

side BC,

Page 4 of 19

Jurong Junior College

H1 Physics Preliminary Examination 2008 (Paper 2)

(ii)

(b)

side PB?

[3]

(i)

The solenoid has 700 turns m-1 and carries a current of 3.5 A. Given that the
magnetic flux density B on the axis of a long solenoid is
B = 0 n I,
calculate the magnetic flux density in the region of side BC of the frame. [Take
0 = 4 x 10-7 H m-1]

(ii)

Side BC has length 5.0 cm. Calculate the force acting on BC due to the
magnetic field in the solenoid.

(iii)

A small piece of paper of mass 0.10 g is placed on the side DQ and positioned
so as to keep the frame horizontal. Given that QC is of length 15.0 cm, how far
from the knife-edge must the paper be positioned?
[5]

Page 5 of 19

Jurong Junior College

H1 Physics Preliminary Examination 2008 (Paper 2)

In order to verify Einsteins study of the photoelectric effect, the apparatus was set up as
illustrated in Fig. 3.1. Electromagnetic radiation is incident on the surface of a metal A.
Electrons are emitted from the surface and these are attracted to a positively charged
electrode.

Fig.3.1
(i)

When the intensity of the electromagnetic radiation is I0 and the potential difference
recorded is V0, the current recoded in the microammeter is 1.5 A. Calculate the
number of electrons collected by the electrode per second.
[2]

(ii)

(a)

The above experiment is repeated by bringing the source of electromagnetic


radiation closer to the metal surface, and the same potential difference V0 is applied.
State and explain the observation made in the current recorded in the
microammeter.

(b)

The experiment set-up shown in Fig.3.1 is modified to measure the stopping voltage VS
required just to prevent electrons from reaching the electrode. The stopping voltage VS
corresponding to two values of wavelength are measured and the results are shown in
Table 3.2.

Page 6 of 19

[2]

Jurong Junior College

H1 Physics Preliminary Examination 2008 (Paper 2)

Stopping voltage, VS / V

Wavelength, / nm

1.00

280

0.25

400
Table. 3.2

(i)

State the modification to Fig. 3.1 needed to measure the stopping potential VS.

[1]

(ii)

What is the maximum kinetic energy of an electron emitted from the metal surface by
radiation of wavelength 280 nm?
[1]

(iii)

Calculate the work function energy of the metal surface.

[2]

(iv)

Using the answers to b(ii), sketch the graph of maximum kinetic energy versus
wavelength in Fig. 3.3. Label this graph as A.

[2]

maximum kinetic energy / J

wavelength / nm
Fig. 3.3
(v)

The experiment in (b) is repeated using metal B of higher work function energy than
metal A. On the axes in Fig. 3.3, sketch the graph for this new metal. Label this
graph as B.
[1]

Page 7 of 19

Jurong Junior College

H1 Physics Preliminary Examination 2008 (Paper 2)

Two prominent commercial airplane manufacturers are Boeing and Airbus. Singapore
International Airlines has at least 18 Airbus and 76 Boeing airplanes in full flight
operations. The general specifications and flight information of a typical small size Airbus
is given below:
Mass of airplane, including crew and equipment

45 000 kg

Maximum capacity of fuel tanks (including reserve fuel)

25 000 kg

Maximum number of passengers

200

Average mass of a passenger with baggage

100 kg

Average use of fuel per kilometre

5.0 kg

Safety reserve of fuel at the end of journey

3000 kg

Take-off speed

75 m s-1

Length of runway used Changi Airport

1500 m

Many factors are taken into consideration to ensure safety in air travel. Some factors
include maximum mass of load carried by the airplane, length of runway, airport design as
well as fuel capacity. It is a requirement that every commercial airplane maintain a certain
amount of reserve fuel at the end of each journey as a safety measure.
a)

Calculate the total mass of the airplane before take-off assuming that it is carrying
the maximum number of passengers and that its fuel tanks are filled to full capacity.
[1]

b)

(i)

The range of an airplane refers to the maximum distance it can fly without
utilizing the reserve fuel. Calculate the range for the airplane from the
information given.

[1]

(ii) What is the minimum further distance the airplane can fly if the airport it is
scheduled to land is closed due to poor weather conditions?

[2]

Page 8 of 19

Jurong Junior College

(c)

(i)

H1 Physics Preliminary Examination 2008 (Paper 2)

Calculate the acceleration of the airplane before leaving the ground if it uses the
full length of the runway for take-off.
[2]

(ii) Calculate the force required to provide this acceleration.

(d)

[1]

There is always a safety buffer distance for emergency braking at the end of the
runway if the pilot decides to abort take-off. Supposing that just before leaving the
ground, the pilot discovers that there is something wrong with the airplane and he
decides to abort the take-off. Calculate
(i)

the braking force needed to stop the airplane given that the deceleration of the
airplane is 2.8 m s-2
[2]

(ii) the time taken to bring the airplane to a stop.

[2]

(e)

In certain airports, the runway used is only 1200 m. Using the same force calculated
in c(ii), what is the maximum mass that can be accelerated to take-off speed if the
pilot uses the full length of the runway for take-off?
[2]

(f)

Discuss two disadvantages of using a short runway for take-off.

Page 9 of 19

[2]

Jurong Junior College

H1 Physics Preliminary Examination 2008 (Paper 2)

Section B
Answer two questions in this section.

A tennis ball at a given instant just before it is struck by a tennis racket, possesses a
horizontal momentum of 2.4 N s and kinetic energy of 45 J.
(a)

Why is it correct to give the direction of the momentum but not of the kinetic energy?
[1]

(b)

Write down in term of the mass m and the velocity v of a body, expressions for
(i)

the momentum,

(ii) the kinetic energy.

(c)

[2]

Calculate the mass and velocity of the tennis ball.

[4]

Page 10 of 19

Jurong Junior College

(d)

H1 Physics Preliminary Examination 2008 (Paper 2)

Tennis star Roger Federer then strikes the tennis ball with his racket. Assuming that
Roger is able to exert a constant force of 60 N in opposite direction to that of the
tennis ball, bringing the ball to rest momentarily, calculate
(i)

the time the ball takes to stop,

(ii) the total distance the tennis ball travels while stopping.

(e)

The force of 60 N then continues to act on the tennis ball for a further 0.06 s.
Calculate
(i)

the new momentum of the ball,

(ii) the new velocity of the ball

(f)

[4]

[3]

Calculate the increase in kinetic energy of the ball for the whole time that the force is
applied to it and hence deduce the mean power being delivered to the ball while it is
in contact with the racket.
[4]

Page 11 of 19

Jurong Junior College

H1 Physics Preliminary Examination 2008 (Paper 2)

(g)

Suggest why, in reality, it is impossible for Roger Federer to apply a constant force
to the ball.
[2]

a)

With the aid of a diagram, explain the principle of superposition of waves.

b)

A beam of laser of wavelength 600 nm is directed onto an opaque slide with a pair of
narrow slits at a distance of 0.30 mm apart. An interference pattern consisting of
bright and dark fringes is observed at a distance of 1.0 m away from the double-slit.
Calculate

[2]

i)

the frequency of light from the laser,

[1]

ii)

the spacing between the bright fringes.

[2]

Page 12 of 19

Jurong Junior College

H1 Physics Preliminary Examination 2008 (Paper 2)

The double-slit arrangement, shown in Fig. 6.1 is now used to measure the speed of
solid particles carried along in a gas stream in a transparent pipe. The pipe is placed
at a distance of 1.0 m from the double-slit. A photo detector picks up light scattered
from the particles in the pipe and is found to be flashing at a rather constant
frequency of 5.0 kHz.
Gas stream

Laser

Solid
particles

1.0 m

Fig. 6.1
iii)

Explain why light flashes and state what happens to the flash rate if the speed
of gas stream increases.
[2]

iv)

What is the speed of gas stream? State one assumption that you have made in
your calculation.
[3]

v)

Explain why there should be only a few particles in the gas stream.

Page 13 of 19

[1]

Jurong Junior College

H1 Physics Preliminary Examination 2008 (Paper 2)

c)

displacement

distance
A

Fig. 6.2
Fig. 6.2 shows how displacement of air molecules varies with distance from a
loudspeaker L at one instant. Displacements away from the speaker are taken as
positive, and towards the source as negative. [Speed of sound in air is 330 m s-1].
i)

At which of the four points are the instantaneous pressures at their maximum
and minimum respectively? Indicate your answer in the table below.
[2]
Maximum Pressure

ii)

Minimum Pressure

If the frequency of the speaker is 400 Hz, calculate the time required for the
sound to travel the distance AD.
[2]

Metal Plate
Transmitter
D

Fig. 6.3
d)

A transmitter sends a beam of microwaves in the direction TM which is


perpendicular to the stationary metal plate at M as shown in Fig. 6.3.
i)

A microwave detector D is moved from T to M and records a sequence of


intensity maxima and minima. Explain this.

Page 14 of 19

[2]

Jurong Junior College

ii)

a)

H1 Physics Preliminary Examination 2008 (Paper 2)

If the distance between the first and eleventh intensity maxima detected is 30
cm, calculate the frequency of the microwave transmitter.
[3]

On the axes in Fig. 7.1 sketch I-V characteristics for two components, A and B, both
of which obey Ohms law.
Component B has a lower resistance than component A. Label your characteristics
clearly as A and B.
[2]

Fig. 7.1

Page 15 of 19

Jurong Junior College

b)

H1 Physics Preliminary Examination 2008 (Paper 2)

The graph in Fig. 7.2 shows the I-V characteristics for two conductors, the tungsten
filament of a lamp and a length of constantan wire.

Fig. 7.2
i)

State, with a reason, which conductor obeys Ohms law across the full voltage
range.
[2]
conductor:
reason:

ii)

....................................................................................
....................................................................................

Calculate the resistance of the tungsten filament when V = 1 V and V = 10 V.


[2]

iii)

Explain why the values of resistance, calculated in part (b)(ii), differ from each
other.
[2]

Page 16 of 19

Jurong Junior College

iv)

c)

H1 Physics Preliminary Examination 2008 (Paper 2)

Use the graph to determine the resistivity of constantan, given that the wire is
[3]
0.80 m long with a uniform cross-sectional area of 6.8 108 m2.

2 students living in neighboring rooms in a hostel to prepare for their final year
examinations decided to save some money by connecting their ceiling lights in
series to the AC supply. They agreed that each would install a 100 W, 240V bulb in
their own room and they would pay equal shares of the electricity bill. However, both
decided to try to get better lighting at the others expense. A installed a 200 W, 240V
bulb and B installed a 50 W, 240V bulb.
Whose room is dimmer as a result? Explain your answer clearly.
[3]

Page 17 of 19

Jurong Junior College

d)

H1 Physics Preliminary Examination 2008 (Paper 2)

Referring to Fig. 7.3, the 15 V source is of negligible internal resistance.


Switch K

A
6.0

E
2.4

6.0

15 V

6.0

6.0

9.0

Fig. 7.3
i)

With switch K open, find the effective resistance between point C and point D.
[2]

ii)

With switch K closed and given that the potential at point A is 3 V, determine
the potential at point C
[2]

Page 18 of 19

Jurong Junior College

e)

H1 Physics Preliminary Examination 2008 (Paper 2)

A brightly lit string of 100 Christmas tree light bulbs stops working when one bulb
blows. All the light bulbs have identical resistances and are connected in series.
A technician uses a voltmeter to measure the potential difference across every bulb
one by one till he finds and replaces the damaged one. How would he know which
bulb is damaged?
[2]

Page 19 of 19

Answers for H1 Prelim Exam Paper 1 (2008)

Physics H1 Prelim Exam (2008)


Solutions (Paper 1)
1

11

21

12

22

13

23

14

24

15

25

16

26

17

27

18

28

19

29

10

20

30

Qn Suggested Solution
1
R
4
= T
Given RT = T
4

Answer

Js m
K4

Unit of =

-2

-4

=Wm K

Force : vector quantity , kinetic energy: scalar quantity

P = VI

P
V
I
100% =
100% +
100%
P
V
I

= 4% + 3%
= 7%
4

Random error is of varying sign and magnitude and can be reduced by averaging
repeated measurements but cannot possibly be eliminated.

When the sandbag was released initially, it had a velocity of 4 ms-1 and the velocity
would then decrease due to constant acceleration in opposite direction to the initial
velocity. The gradient which reflects the acceleration should be constant.

Vx = 10 cos 30 = 8.67 ms-1


t = 10/ 8.67 = 1.15s

v y = u y + at = 10sin 30 + ( 9.81)(1.15)

vy = -6.28

v = vx 2 + v y 2 = 10.7 ms-1
7

Constant velocity implies zero acceleration which means no resultant force.

Using F=ma on the entire system:


10 x g = 30 a

a=

1
g.
3

D
A

By definition the total momentum in an isolated system is zero.

Page 1 of 3

Answers for H1 Prelim Exam Paper 1 (2008)

Qn Suggested Solution
10 Change in momentum = Area under the F-t graph
= 5 + 2.5 = 7.5 N s
11

Answer
C

The bodies are in equilibrium only if the resultant forces acting on them are zero.
N3L does not imply that, since action and reaction forces must act on separate
bodies.

12

Reaction force on ball


by slope
Frictional force on ball
by slope

13

Gain of kinetic energy + work done in overcoming friction = Loss in PE (mgh)


KE + (0.5)(10) = (2)(9.81)(1)
KE = 14.6 J

Power used = work done/ time

14

mgh (400)(9.81)(1200)
=
=39.2 kW
t
(2 60)

80% - 39.2 kW
20% - 9.8 kW

P = Fv

15

v = 50 ms
16

17

25000 = 500v

Diffraction occurs when the size of the obstacle is of the same order of magnitude
as the wavelength of wave. Hence for sound waves which have a much larger
wavelength than light waves, no shadow is formed, as the waves will spread out
around the obstacle.

For visible light, 400 nm 700 nm


Hence, minimum freq, f =

18

-1

3 x108
4 x 10 14 Hz.
700 x109

Wavelength = 300/500 = 0.6 m; (change in x)/0.6 =

7
x 1/(2p)
4

Hence, change in x = 7/8 x 0.6 = 0.525 m = 525 mm.


19

If the two sources are coherent two sources have a constant phase difference.

20

A stationary wave is set up between probes P and Q with distance of 2 consecutive


nodes = 1 cm.

= 1 cm
C

= 2 cm
Thus speed of wave = f = 20 x 0.02
= 0.4 m s-1

Page 2 of 3

Answers for H1 Prelim Exam Paper 1 (2008)

Qn Suggested Solution
21 Q = I dt = Area under I-t graph = x (100 + 20) x 10 -3 x 8 = 480 mC.

Answer
C

22

N = (2 x 60 x 60)/(1.6 x 10-19) = 4.5 x 10 22

23

E = 12 x 200 x 2.5 x 10-3 = 6 J

24

Vx = 1 V ; Vy = 0.3 x 5 = 1.5 V Hence Vxy = 1.5 1 = 0.5 V

25

Equivalent resistance of voltmeter and 30 k resistor in parallel is 20 k.


Hence by Potential Divider principle, p.d. across voltmeter is 2/3 of 9 V = 6 V

26

Magnetic force on electron F = Bqvsin


Since the electron is stationary, velocity v = 0 F = 0 N. Thus the electron is
unaffected.

27

For the tension in the wires to be equal to zero, the magnetic force acting on the rod
must balance the weight of the rod.

ALg
Ag
mg
=
=
BL
BL
B
Since g and B are constant, I = A
BIL = mg = ALg

28

29

I=

Classical wave theory predicts that there is an observable time interval between the
incidence of the wave and the emission of the electrons since the energy of the
electromagnetic radiation comes in continuous manner.
Energy of a photon E = hf =

hc

By de Broglie principle, momentum of each photon p =

E
c

Thus the rate of change of momentum of the incident photons = Np =

NE
c

30
hf3

hf2

hf1

hf3 = hf1 + hf2


f3 = f1 + f2
f 1 = f 3 f2

Page 3 of 3

Marking Scheme for H1 Prelim Exam (2008)

Physics H1 Prelim Exam (2008)


Marking Scheme
Section A
Qn

Suggested Solution

1(a)(i)

Remarks

The gain in kinetic energy,

KE =

1
mv 2
2

1
(800)(28) 2
2

[1] ans

= 3.14 105 J
(a)(i)

The gain in gravitational potential energy,

GPE = mg h

= (800)(9.81)(450sin 5)
= 3.08 105 J

(b)(i)

Useful output power of car

KE + GPE
3.14 105 + 3.08 105
=
time
32.1

= 1.94 104 W
Efficiency of the car energy

1.94 104
100 %
2.50 104

= 77.6%

(ii)
(b)(i)

(iii)

[1] ans

A vertical force is acting on the side BC which is perpendicular to the


magnetic field. The direction of the magnetic force acting on the side BC cab
be up or down depends on the direction of the current flowing in BC and
direction of the magnetic field.

[1] Answer
[1] Reason

No magnetic force is acting on the side PB since it is parallel to the


magnetic field.

[1] ans

Magnetic flux density B = 0 n I


= 4 x10-7 x 700 x 3.5
= 3.08 x10-3 T

(ii)

[1] for subst


[1] for ans

= useful power output/ power output


=

2(a)(i)

[1] for ans

= gain in energy /time


=

(b)(ii)

[1] for subst

[1[ ans

Magnetic force acting on BC F = BIL


= 3.08 x10-3 x 3.5 x 0.05
= 5.39 x 10-4 N

[1] sub
[1] ans

By Principle of Moment,
Anticlockwise moment = Clockwise moment
0.10 x 10-3 x 9.81 x d = 5.39 x 10-4 x 0.15
d = 8.24 x 10-2 m

[1] sub
[1] ans

Page 1 of 6

Marking Scheme for H1 Prelim Exam (2008)

Qn

Suggested Solution

3(a)(i)

Current I =

Q
t

Remarks
[1] Substn

Ne
t

1.5 10 6
N I
=
=
= 9.38 x 1012 s-1
19
t
e 1.6 10
(ii)

(b)(i)
(ii)

(iii)

[1] Ans

The current recorded in the microammeter increases. This is because when


the source of electromagnetic radiation is brought closer to the metal, the
intensity of the electromagnetic radiation at the metal surface increases
(or the number of photons reaching the metal surface increases).

[1] state

Connect the electrode to the negative terminal of the battery.

[1] ans

KEmax = eVs
= 1.6 x 10-19 x 1.00
= 1.6 x 10-19 J

[1] ans

[1] expl

Using KEmax = hf -

hc

=
=

KE max

6.63 1034 3 108


1.6 1019
280 10 9

= 5.50 x 10-19 J

[1] sub
[1] ans
[1] graph

(iv)

[1] label the


points

(v)

Using metal B of a higher work function, the KEmax of the emitted electrons will
be lower. The curve will be similar to curve A but closer to both axes as
shown above.

[1] graph

4 (a)

M = 45000 + 25000 + 200(100) = 90 000 kg

[1] ans

(b) (i)

Range = (25 000 3000) / 5.0 = 4400 km

[1] for ans

Fuel available = 3000 kg


Max Distance it can fly = 3000/5.0 = 600 km

[1] for subst


[1] for ans

(ii)
(c)(i)

v 2 = u 2 + 2as

75 2 = 0 2 + 2(a )(1500)
a = 1.88 m s-2

(ii)
(d)(i)

[1] for subst


[1] for ans

F = ma = (90 000)(1.88) = 170 000 N

[1]

Braking force

[1] for subst

= 90 000 x 2.8
Page 2 of 6

Marking Scheme for H1 Prelim Exam (2008)

Qn

Suggested Solution
= 250 000 N
(ii)
(e)

v = u + at
t = 27 s

Remarks
[1] for ans

0 = 75 + (-2.8)t

v 2 = u 2 + 2as

[1] for subst


[1] for ans

75 2 = 0 2 + 2(a)(1200)

a = 2.34 m s-2
Mass, m = F/a = 170 000 / 2.34 = 73 000 kg
(f) 1. A larger acceleration is required to bring the aircraft to take-off speed if it
is carrying maximum no. of passengers (Greater force acts on passengers
or Higher fuel consumption).
2. In order to maintain the normal acceleration, the total mass of the plane
must be reduced
fewer passengers (hence, profitability reduces), OR
carry less fuel which leads to a shorter range (however, the plane
cannot change the requirements for safety reserve fuel)

[1] for a
[1] for mass
[1] for each
disadvantage
max [2]

Section B
Qn

Suggested Solution

Remarks

5(a) Momentum is a vector quantity with both magnitude and direction while kinetic
energy is a scalar quantity with only magnitude only.
(b)(i) Momentum = mv

[1] for ans

(ii)
Kinetic energy =
(c)

1 2
mv
2

[1] for ans

mv = 2.4---------(1)
[1] for
equations

1 2
mv = 45------(2)
2

( 2 ) : v = 37.5 ms
(1)

[1] ans

-1

[1] for
subst
[1] for ans

using (1), m(37.5) = 2.4


m = 0.064 kg
(d)(i)

[1] ans

F=

mv mu
t

60 =

0 2.4
t

[1] for
equation

t = 0.04 s
(ii)

[1] for ans

1
S = (u + v)t
2
1
S = (37.5 + 0)(0.04) = 0.75 m
2

[1] for
subst

Page 3 of 6

[1] for ans

Marking Scheme for H1 Prelim Exam (2008)

Qn

Suggested Solution

(e)(i)

F=

Remarks

mv mu
t

60 =

mv 0
0.06

[1] subst

mv = 3.6 Ns ( in opposite direction to the initial momentum)


mv =3.6
(0.064) v = 3.6
v = 56.3 ms-1 ( in opposite direction to the initial velocity)

(ii)

(f)
New kinetic energy =

1 2 1
mv = (0.064)(56.3) 2
2
2

= 101 J
Change in K.E = 101- 45 = 56 J
Mean power = Change in K.E/ time

Power = force x velocity


Since the velocity is changing, a continuously changing power must be supplied
to apply a constant force on the ball. However, the time taken is too short to make
any changes to the power supplied. Hence, in practice, the power supplied by a
person is approximately constant.

6 (a) i)

(b)
i)

[1] for ans

[1] for ans


[1] for ans
[1] for
subst
[1] for ans

56.3
= 563 W
0.04 + 0.06
(g)

[1] for ans

[1] eqn
[1] for
explanation

The principle of superposition states that when two or more waves of the [1] defn
same kind meet at a point, the resultant displacement is the vector sum [1]
of their individual displacements at that point.
appropriate
diagram

f =

[1] answer

3 x108
= 5.0 x 1014 Hz
9
600 x10

iii)

iv)

(600 x109 )(1.0)


0.30 x103

[1] substn

= 2.0 x 10-3 m = 2.0 mm

ii)

[1] answer

Using x =

As a particle moves across the fringes, it will reflect light when it passes
the bright fringes, but no light is reflected when it passes the dark
fringe. Hence, the light flashes.
Flash rate increases as the particle moves faster.

[1] reason
[1] answer

Flashing occurs are frequency = 5.0 kHz.


Hence, time between 2 flashes = T =

1
1
=
= 2.0 x 10-4 s
3
f 5.0 x10

[1]
calculation
of time

Speed of gas stream = speed to travel across 2 adjacent bright fringe

distance 2.0 x103


=
=
= 10 m s-1
4
time
2.0 x10
Assumption : one particle passes through at one time, or the particle is

Page 4 of 6

[1] answer

Marking Scheme for H1 Prelim Exam (2008)

Qn

Suggested Solution
moving at the same speed as the gas-stream.

Remarks
[1] valid
assumption

v)

[1] valid
explanation

c) i)

If too many particles are moving in the gas stream, their reflection will be
confusing to keep track and count.
Minimum Pressure

B
ii)

Maximum Pressure

Period =

1
1
=
= 2.5 x 10-3 s
f 400

Time to travel distance AD =


d) i)

ii)

[1] answer

When the microwave is reflected back by the metal plate, it will superpose
with the microwaves from the transmitter and form stationary waves.
The maxima are the antinodes where the displacements are maximum,
while the minima are the nodes, where there is minimum displacements.

[1]

Between the 1st and 11th maxima are 5


Hence 5 = 30 cm
= 6 cm

[1]

3x108
= 5 x 109 Hz
2
6 x10

2 straight lines (not parallel to each other) passing through the origin.
Line A has a smaller gradient then line B.

b) (i) Conductor:
Reason:
(ii)

[1] period

3
3
T = (2.5 x103 ) = 1.88 x 10-3 s
4
4

Frequency of microwave, f =

7 a)

[1]
[1]

constantan
constant gradient / constant resistance

[1]

[1] subst
[1] answer

[1]
[1]
[1]
[1]
[1] answer

V 1.0
=
= 1.4
I 0.7
V 10.0
V = 10 V; R =
= 4.4
=
I 2.25
V = 1 V; R =

[1] answer

(iii) At V = 10 V, a much larger current would have heated the tungsten filament and
caused more lattice vibrations.
This leads to increased collisions between free electrons and the atomic cores of
the atom, thereby increasing its resistance.

[1]

(iv) From the graph, the resistance of constantan is 12/2.2 = 5.45


R = (L)/A
5.45 = ( x 0.8)/(6.8 x 10-8)

[1] ans
[1] formula

resistivity of constantan = 4.63 x 10 -7 m


c)
Resistance of the 200 W bulb =
Resistance of the 50 W bulb =

[1]

[1] answer

V 2 2402
=
200
P

2402
> resistance of 200 W bulb.
50

For 2 resistors connected in series, the power dissipated across each resistor
Page 5 of 6

[1]
[1] Reason

Marking Scheme for H1 Prelim Exam (2008)

Qn

Suggested Solution
depends on its resistance as the same current passes through each bulb..
A s room will be dimmer.

d) (i) 6.0 // 9.0

R = 3.6

6.0 // (3.6+2.4)

1 1
Hence, Reff = +
6 6
(ii)

I=

Remarks
[1] Ans

[1]
1

= 3.0

[1] Ans
[1]

V 15
=
=1A
R 15

VAC = IR = (1)(6.0) = VA VC
[1]

Hence, VC = 9 V
e) As the circuit is broken, the p.d. across all the undamaged bulbs should be zero,
while the p.d. across the damaged bulbs should be 240 V

Page 6 of 6

[1]
[1]

Class

AdmNo

Candidate Name:

Promotional Examinations 2008


Pre-university 2
PHYSICS

8866/01

Higher 1
Thursday

12 September

1 hour

READ THESE INSTRUCTIONS FIRST


Write in soft pencil.
Do not use staples, paper clips, highlighters, glue or correction fluid.
Write your name, class and admission number on the Optical Mark Reader (OMR) Form in
the spaces provided unless this has been done for you.
There are thirty questions on this paper. Answer all questions. For each question there are
four possible answers A, B, C, and D.
Choose the one you consider correct and record your choice in soft pencil on the separate
Optical Mark Reader (OMR) Form.
Read the instructions on the Optical Mark Reader (OMR) Form very carefully.
Each correct answer will score one mark. A mark will not be deducted for a wrong answer.
Any rough working should be done in this booklet.

FOR EXAMINER'S USE


Paper 1
(

/ 30 ) x 33%

Paper 2 Section A
(

/ 40 ) x 33.5%

Paper 2 Section B
(

/ 40 ) x 33.5%

TOTAL

%
%

This question paper consists of 14 printed pages.


[Turn over

2
DATA AND FORMULAE
Data
8

-1

speed of light in free space,

3.00 x 10 m s

elementary charge,

the Planck constant

1.60 x 10 C
6.63 x 10-34 Js

Unified atomic mass constant

1.66 x 10-27 kg

rest mass of electron,

me

9.11 x 10

rest mass of proton,


acceleration of free fall,

mp

-19

1.67 x 10

-31
-27

-2

kg
kg

9.81 m s

uniformly accelerated motion,

ut + at

work done on/by a gas,

v
W

u + 2as
pV

hydrostatic pressure,

gh

Electric potential

Q / 4or

resistors in series,

R1 + R2 + ....

resistors in parallel,

1/R =

Formulae

2
2

1/R1 + 1/R2 + ....

3
1

A metal sphere of radius r is dropped into a tank of water. As it sinks at speed v, it experiences
a drag force F given by F = k r v, where k is a constant.
What are the SI base units of k?
A
B

kg m-2 s-2

kg m-1 s-1

kg m2 s-1

kg m s-2

In an experiment to determine the acceleration of free fall g , the period of oscillation T and
length l of a simple pendulum were measured. The uncertainty in the measurement of l was
estimated to be 4%, and that of T , 1%.
The value of g was determined using the formula

g=

4 2 l
.
2

What is the uncertainty in the calculated value for g?


A
B

3%

5%

2%

6%

An object has an initial velocity u. It is subjected to a constant force F for t seconds, causing a
constant acceleration a. The force is not in the same direction as the initial velocity.
A vector diagram is drawn to find the final velocity v.

What is the length of side X of the vector diagram?


A

Ft

at

u + at

4
4

A tennis ball is released from rest at the top of a tall building.


Which graph best represents the variation with time t of the acceleration a of the ball as it falls,
assuming that the effects of air resistance are appreciable?

The diagram shows a velocity-time graph for a car.

What is the distance travelled between time t = 0 and t = 4 s?


A

2.5 m

3.0 m

20 m

28 m

5
6

A particle is moving in a straight line with uniform acceleration.


Which graph represents the motion of the particle?

A boy kicks a football with an initial velocity of 28.0 m s-1 at an angle of 30.0 above the
horizontal.
What is the highest elevation reached by the football in its trajectory?
A 11.2 m
B 10.0 m
C 12.7 m
D 9.40 m

The graph shows the variation with time of the momentum of a ball as it is kicked in a straight
line.

Initially, the momentum is p1 at time t1. At time t2 the momentum is p2.


What is the magnitude of the average force acting on the ball between times t1 and t2?

6
9

The diagram shows the masses and velocities of two trolleys about to collide.

After the impact they move off together.


What is the total kinetic energy of the trolleys after the collision?
A 1.3 J
B 12 J
C 18 J
D 19 J

10

A force F is applied to a freely moving object. At one instant of time, the object has velocity v
and acceleration a.
Which quantities must be in the same direction?
A

a and v only

a and F only

v and F only

v, F and a

7
11

The diagram shows a sign of weight 20 N suspended from a pole, attached to a wall. The pole
is kept in equilibrium by a wire attached at point X of the pole.

The force exerted by the pole at point X is F, and the tension in the wire is 40 N.
Which diagram represents the three forces acting at point X?

12

A rigid uniform bar of length 2.4 m is pivoted horizontally at its mid-point.

Weights are hung from two points of the bar as shown in the diagram. To maintain horizontal
equilibrium, a couple is applied to the bar.
What is the torque and direction of this couple?
A

40 N m clockwise

40 N m anticlockwise

80 N m clockwise

80 N m anticlockwise

8
13

A spring of unextended length 0.50 m is stretched by a force of 2.0 N to a new length of


0.90 m. The variation of its length with tension is as shown.

How much elastic potential energy is stored in the spring?


A
B

0.80 J

0.90 J

14

0.40 J

1.8 J

A ball is thrown vertically upwards. Neglecting air resistance, which statement is correct?
A The kinetic energy of the ball is greatest at the greatest height attained.
B By the principle of conservation of energy, the total energy of the ball is constant throughout
its motion.
C By the principle of conservation of momentum, the momentum of the ball is constant
throughout its motion.
D The potential energy of the ball increases uniformly with time during the ascent.

15

In many old-style filament lamps, as much as 65 J of energy is emitted as thermal energy for
every 8 J of energy emitted as light.
What is the efficiency of the lamp, as the percentage of electrical energy converted to light
energy?
A

7%

11 %

92 %

93 %

9
16

The diagram shows a transverse wave on a rope. The wave is travelling from left to right.
At the instant shown, the points P and Q on the rope have zero displacement and maximum
displacement respectively.

Which of the following describes the direction of motion, if any, of the points P and Q at this
instant?
Point P
A

Downwards

Stationary

Stationary

Downwards

Stationary

Upwards

17

Point Q

Upwards

Stationary

A displacement-time graph for a transverse wave is shown in the diagram.

The phase difference between X and Y can be expressed as n .


What is the value of n?
A

1.5

2.5

3.0

6.0

10
18

A light meter measures the intensity I of the light falling on it. Theory suggests that this varies as
the inverse square of the distance d.

Which graph of the results supports this theory?

19

Stationary transverse waves are set up in a stretched string as shown.

The distance between the fixed points is 60 cm. For one particular frequency, the pattern shown
is formed.
What is the wavelength?
A

20 cm

40 cm

60 cm

120 cm

11
20

The potential difference between point X and point Y is 20 V. The time taken for charge carriers
to move from X to Y is 15 s, and, in this time, the energy of the charge carriers changes by 12 J.
What is the current between X and Y?
A
B
C
D

21

0.040 A
0.11 A
9.0 A
25
A

What is a correct statement of Ohms law?


A

The potential difference across a component equals the current providing the resistance
and other physical conditions stay constant.

The potential difference across a component equals the current multiplied by the resistance.

The potential difference across a component is proportional to its resistance.

The potential difference across a component is proportional to the current in it providing


physical conditions stay constant.

22

The graph shows how the electric current I through a conducting liquid varies with the potential
difference V across it.
At which point on the graph does the liquid have the smallest resistance?

23

Which graph shows the I V characteristic of a filament lamp?

12
24

The diagram shows a parallel combination of three resistors. The total resistance of the
combination is 3 .

What is the resistance of resistor X?


A 2
B 3
C 6
D 12

25

When four identical lamps P, Q, R and S are connected as shown in diagram 1, they have
normal brightness.

The four lamps and the battery are then connected as shown in diagram 2.
Which statement is correct?
A

The lamps do not light.

The lamps are less bright than normal.

C The lamps have normal brightness.


D

The lamps are brighter than normal.

13
26

A wire carrying a 2-A current is placed at an angle of 60 with the respect to a magnetic field of
strength 0.2 T. If the length of the wire is 0.6 m what is the magnitude of the magnetic force
acting on the wire?
A

0.20 N

0.40 N

0.60 N

0.80 N

27

The following diagrams show an electron passing through a magnetic field. Which diagram
shows the possible path of the electrons as they pass through the field?

28

A clean plate, made of metal with work function energy of 2.36 eV, is illuminated with ultra violet
light of wavelength 370 nm. What is the kinetic energy of emitted photoelectrons?
A

1.00 eV

3.36 eV

5.38 eV

5.72 eV

14
29

The result of an experiment to investigate the energy of photoelectrons emitted from a metallic
surface is shown below.

The gradient of the graph depends on the


A
B

wavelength of the incident radiation

work function of the irradiated surface.

D
30

intensity of the incident radiation.

ratio of the Planck constant to the electronic charge.

The diagram shows five energy levels of an atom. Five possible transitions between the levels
are indicated. Each transition produces a photon of definite energy and frequency.

Which spectrum corresponds most closely to the transitions shown?

END OF PAPER

Class
Candidate Name:

Adm. No.

________________________________

Promotional Examinations 2008


Pre-university 2
PHYSICS

8866/02

Higher 1
Thursday

12 September

2 hours

Paper 2 Structured questions


Candidates answer on the Question Paper.
No Additional Materials are required.
READ THESE INSTRUCTIONS FIRST
Write your name, class and admission number on all the work you hand in.
Write in dark blue or black pen on both sides of the paper.
You may use a soft pencil for any diagrams, graphs or rough working.
Do not use staples, paper clips, highlighters, glue or correction fluid.
Sections A
Answer all questions.
Sections B
Answer any two questions.

For examiners use


Section A
1
2

At the end of the examination, fasten all your work securely


together.

The number of marks is given in brackets [ ] at the end of each


question or part question.

Section B

5
6
7
Total

This document consists of 24 printed pages and 2 blank pages.


[Turn over

Data
speed of light in free space

= 3.00 x 108 m s-1

elementary charge

= 1.60 x 1019 C

the Planck constant

= 6.63 x 1034 J s

unified atomic mass constant

= 1.66 x 1027 kg

rest mass of electron

me = 9.11 X 1031 kg

rest mass of proton

mp = 1.67 X 1027 kg

acceleration of free fall

g = 9.81 m s2

Formulae
uniformly accelerated motion,

s = ut +

1 2
at
2

v2 = u2 + 2as
work done on/by a gas

W = pV

hydrostatic pressure

p = gh

resistors in series,

R = R1 + R2 + ...

resistors in parallel

1/R = 1/R1 + 1/R2 + ...

BLANK PAGE

[Turn over

4
Section A
Answer all the questions in this section.
1.

(a)

State Newtons second law of motion.


.........................................................................................................................................
.........................................................................................................................................
.........................................................................................................................................

(b)

[1]

Fig. 1.1 shows 2 objects m1 and m2 connected by an inextensible string that passes
through a light frictionless pulley. You can assume all surfaces to be smooth and effects of
friction can be neglected in this question.
m1
A
m2

ground
Fig. 1.1
(i)

Explain why the acceleration of m2 is less than the acceleration of free fall g.

.........................................................................................................................................
.........................................................................................................................................
(ii)

Show that the acceleration a of m2 is

[1]

m2
g where g is the acceleration due to
m2 + m1

free fall.

[3]

5
(iii)

If the string is cut at point A, state and explain the subsequent motions of m1 and
m2.

.........................................................................................................................................
.........................................................................................................................................
.........................................................................................................................................
.........................................................................................................................................

[Turn over

[2]

6
2.

(a)

A force F is acting on a body that is moving with velocity v in the direction of the force.
Derive an expression relating the power P dissipated by the force to F and v.

[2]
(b)

A stone of mass 56 g is thrown horizontally from the top of a cliff with a speed of 18 m s-1,
as illustrated in Fig. 2.1.

Fig. 2.1
The initial height of the stone above the level of the sea is 16 m. Air resistance may be
neglected.
(i)

Calculate the change in gravitational potential energy of the stone as a result of


falling through 16 m.

change = ...............................................J [2]

7
(ii)

Calculate the total kinetic energy of the stone as it reaches the sea.

kinetic energy = .............................................. J [3]

(c)

Use your answer in (b)(ii) to show that the speed of the stone as it hits the water is
approximately 25 m s-1.

[1]

[Turn over

8
3.

In the circuit of Fig. 3.1, the battery has an e.m.f. of 3.00 V and an internal resistance r. R is a
variable resistor. The resistance of the ammeter is negligible and the voltmeter has an infinite
resistance.

Fig. 3.1
The resistance of R is varied. Fig. 3.2 shows the variation of the power P dissipated in R with the
potential difference V across R.

Fig. 3.2

(a)

Use Fig. 3.2 to determine


(i)

the maximum power dissipation in R,

maximum power = .................................................. W [1]

(ii)

the potential difference across R when the maximum power is dissipated.

potential difference = ................................................... V [1]

(b)

Hence, calculate the resistance of R when the maximum power is dissipated.

resistance = ............................................. . [2]

(c)

Use your answers in (a) and (b) to determine the internal resistance r of the battery.

r = ............................................. [3]

[Turn over

10
(d)

By reference to Fig. 3.2, it can be seen that there are two values of potential difference V
for which the power dissipation is 1.05 W. State, with a reason, which value of V will result
in less power being dissipated in the internal resistance.
.........................................................................................................................................
.........................................................................................................................................
.........................................................................................................................................
.........................................................................................................................................

(e)

[3]

The variable resistance R is then removed from the circuit shown in Fig. 3.2 and replaced
with the arrangement of three similar lamps A, B and C as shown in Fig. 3.3 using the
connectors X and Y. The arrangement contains three switches, S1, S2 and S3.

Fig. 3.3

11
One of the lamps is faulty. In order to detect the fault, the resistance across X and Y is
calculated using the ammeter and voltmeter in the circuit.
Fig. 3.4 shows the resistance across X and Y for different switch positions.
resistance

Fig. 3.4
(i)

Identify the faulty lamp, and the nature of the fault.


faulty lamp: ...............................................................................................................
nature of fault: .......................................................................................................... [2]

(ii)

Determine the resistance of one of the non-faulty lamps.

resistance = . [1]

(iii)

If all the lamps, A, B and C are not faulty, calculate the effective resistance across X
and Y when only switch S1 is opened.

effective resistance = . [2]

[Turn over

12
4.

(a)

Two wires X and Y, which are at right angles to the plane of the paper, carry currents I1 and
I2 out of the plane of the paper.

Fig. 4.1
(i)

On Fig. 4.1, draw and label the direction of the magnetic field, B which I1 causes at
wire Y.
[1]

(ii)

On the same figure, draw and label the direction of the force, FY acting on wire Y.

(iii)

[1]

State how (ii) is determined.

.........................................................................................................................................
.........................................................................................................................................

(iv)

[1]

1. State Newtons third law of motion.

.........................................................................................................................................
.........................................................................................................................................

[1]

2. On Fig. 4.1, use this law and your answer in (a)(ii) to draw and label the direction
of the force FX, due to the current in wire Y, on wire X.
[1]

13
(b)

A small square coil has sides of length L and is mounted so that it can pivot freely about a
horizontal axis PQ, parallel to one pair of sides of the coil, through its centre (see Fig. 4.2).

coil

Q
rider
x

P
S
Fig. 4.2
The coil is situated between the poles of a magnet which produces a uniform magnetic field
of flux density B. The coil is maintained in a vertical plane by moving a rider of mass M
along a horizontal beam attached to the coil. When a current I flows through the coil,
equilibrium is restored by placing the rider a distance x along the beam from the coil.
Starting from the definition of magnetic flux density, show that B is given by the expression.

B=

Mgx
IL2

[3]
(i)

If the current is supplied by a battery of constant e.m.f. and negligible internal


resistance, state and explain the effect on x if the coil is replaced with a similar wire
with length

L
.
2

.........................................................................................................................................
.........................................................................................................................................
.........................................................................................................................................
.........................................................................................................................................

[Turn over

[2]

14
Section B
Answer TWO questions from this section.
5.

A rod AB is hinged to a wall at A. The rod is held horizontally by means of a cord BD, attached to
the rod at end B and to the wall at D, as shown in Fig. 5.1.

Fig. 5.1
The rod has weight W and the centre of gravity of the rod is at C. The rod is held in equilibrium by
a force T in the cord and a force F produced at the hinge.

(a)

Explain what is meant by


(i)

the centre of gravity of a body,

.........................................................................................................................................
.........................................................................................................................................
.........................................................................................................................................

(ii)

[2]

the equilibrium of a body.

.........................................................................................................................................
.........................................................................................................................................
.........................................................................................................................................

[2]

15
(b)

The line of action of the weight W of the rod passes through the cord at point P. Explain
why, for the rod to be in equilibrium, the force F produced at the hinge must also pass
through point P.
.........................................................................................................................................
.........................................................................................................................................
.........................................................................................................................................
.........................................................................................................................................

(c)

The forces F and T make angles and respectively with the rod and AC = (

[2]

2
)AB, as
3

shown in Fig. 5.1. Write down equations, in terms of F, W, T, and to represent


(i)

the resolution of forces horizontally,

.........................................................................................................................................

(ii)

the resolution of forces vertically,

.........................................................................................................................................

(iii)

[1]

the taking of moments about A.

.........................................................................................................................................

(d)

[1]

[1]

A ball of the same weight as the rod, W, is then hanged using a string at C. Comment on
the changes to the magnitudes and directions of T and F if the rob remains horizontal after
the ball is hanged at B.
.........................................................................................................................................
.........................................................................................................................................
.........................................................................................................................................

[Turn over

[2]

16
(e)

The string used to hang the ball to the rod is then cut and the ball is dropped onto a
horizontal plate as shown in Fig. 5.2.

Fig. 5.2
Just before impact with the plate, the ball of mass 35 g has speed 4.5 m s1. It bounces
from the plate so that its speed immediately after losing contact with the plate is 3.5 m s-1.
The ball is in contact with the plate for 0.14 s.
Calculate, for the time that the ball is in contact with the plate,
(i)

the average force, in addition to the weight of the ball, that the plate exerts on the
ball,

magnitude of force = .................................... N


direction of force = ........................................[4]

17
(ii)

the loss in kinetic energy of the ball.

loss = ....................................... J [2]

(iii)

State and explain whether linear momentum is conserved during the bounce.

.........................................................................................................................................
.........................................................................................................................................
.........................................................................................................................................
.........................................................................................................................................
.........................................................................................................................................
.........................................................................................................................................

[Turn over

[3]

18
6.

(a)

(i)

Explain what is meant a photon.

.........................................................................................................................................
.........................................................................................................................................

(ii)

[2]

Show that the photon energy of light of wavelength 350 nm is 5.68 x 10-19 J.

[1]
(iii)

State the value of the ratio

ratio = . [1]

19
(b)

Two beams of monochromatic light have similar intensities. The light in one beam has
wavelength 350 nm and the light in the other beam has wavelength 700 nm.
The two beams are incident separately on three different metal surfaces. The work function
of each of these surfaces is shown in Fig. 6.1.

Fig. 6.1
(i)

Explain what is meant by the work function of the surface.

.........................................................................................................................................
.........................................................................................................................................
.........................................................................................................................................
(ii)

[2]

State which combination, if any, of monochromatic light and metal surface could
give rise to photoelectric emission. Give a quantitative explanation of your answer.

.........................................................................................................................................
.........................................................................................................................................
.........................................................................................................................................
.........................................................................................................................................
(c)

[3]

Electromagnetic radiation of frequency f and intensity I, when incident on a metal surface,


causes n electrons to be ejected per unit time. The maximum kinetic energy of the electron
is Emax.
State and explain the effect, if any, on n and Emax when
(i)

the intensity is reduced to

1
but the frequency f is unchanged,
2

.........................................................................................................................................
.........................................................................................................................................
.........................................................................................................................................
(ii)

[2]

the frequency f is increased but the intensity I is not changed.

.........................................................................................................................................
.........................................................................................................................................
.........................................................................................................................................

[Turn over

[2]

20
(d)

(i)

State the de Broglie relation, explaining any symbols you have used.

.........................................................................................................................................
.........................................................................................................................................
.........................................................................................................................................
(ii)

[2]

An electron of mass m has kinetic energy E. Show that the de Broglie wavelength
of this electron is given by

2mE

[2]
(iii)

Calculate the potential difference through which an electron, initially at rest, must be
accelerated so that its de Broglie wavelength is equal to 0.40 nm (the diameter of
an atom).

potential difference = . V [3]

21
7.

(a)

(i)

Explain what is meant by the frequency of vibration of an object.

.........................................................................................................................................
.........................................................................................................................................

(ii)

[1]

Describe, using labelled sketches where appropriate, the motion and phase of
particles in a progressive longitudinal wave.

.........................................................................................................................................
.........................................................................................................................................
.........................................................................................................................................
.........................................................................................................................................
.........................................................................................................................................
.........................................................................................................................................
.........................................................................................................................................
.........................................................................................................................................
.........................................................................................................................................
.........................................................................................................................................
.........................................................................................................................................
.........................................................................................................................................
.........................................................................................................................................
.........................................................................................................................................

[Turn over

[3]

22
(b)

Fig. 7.1 (not drawn to scale) shows an experimental arrangement for studying the
transmission of light by a double slit.

Fig. 7.1
Monochromatic light from a laser falls normally on two narrow, closely spaced parallel slits.
The intensity of light transmitted is studied by moving a small light senor along the line PQ,
at a perpendicular distance of 6.0 m from the slits. Fig. 7.2 shows how the light intensity
varies with distance y from the mid-point O.

Fig. 7.2

23
Explain why the two light waves from the slits produce a minimum intensity at X.

(i)

.........................................................................................................................................
.........................................................................................................................................
.........................................................................................................................................

(ii) 1.

Point O is equidistant from the slits. State, in terms of the wavelength , the path
difference between the waves arriving at Y.

.........................................................................................................................................

2.

[1]

What is the phase difference, in radians, between the waves arriving at point Y?

.........................................................................................................................................

(iii)

[2]

[1]

The spacing of the slits in the experiment was 0.20 mm. Use this, together with
information from Fig. 7.1 and Fig. 7.2, to calculate the wavelength of the light.

wavelength of light = . m [3]

(iv)

Describe and explain changes (if any) in Fig. 7.2 if one of the slits is painted over so
that it transmits only half the light intensity of the other?

.........................................................................................................................................
.........................................................................................................................................
.........................................................................................................................................

[Turn over

[2]

24
(c)

A long tube, fitted with a tap, is filled with water. A tuning fork is sounded above the top of
the tube as the water is allowed to un out of the tube, as shown in Fig. 7.3.

Fig. 7.3

Fig. 7.4

A loud sound is first heard when the water level is as shown in Fig. 7.3, and then again
when the water level is as shown in Fig. 7.4.
Fig. 7.3 illustrates the stationary wave produced in the tube.
(i)

On Fig. 7.4,
1. sketch the form of the stationary wave set up in the tube,
2. mark, with the letter N, the positions of any nodes of the stationary wave.

(ii)

[1]
[1]

The frequency of the fork is 512 Hz and the difference in the height of the water
level for the two positions where a loud sound is heard is 32.4 cm.
Calculate the speed of sound in the tube.

Speed = . m s-1 [3]

25
(iii)

The length of the column of air in the tube in Fig. 7.3 is 15.7 cm.
Suggest where the antinode of the stationary wave produced in the tube in Fig. 7.3 is
likely to be found.

.........................................................................................................................................
.........................................................................................................................................
.........................................................................................................................................

END OF PAPER

[Turn over

[2]

26

BLANK PAGE

Millennia Institute
Physics 8866
PROMOTIONAL EXAMINATION 2008
PU 2 (H1)
Mark Scheme
Paper 1
1

10

11

12

13

14

15

16

17

18

19

20

21

22

23

24

25

26

27

28

29

30

Paper 2 Section A
1

(a) Newtons second law of motion states that the rate of change of momentum of an
object is directly proportional to the resultant force acting on that object and has the
same direction as the force.
B1
(b) (i) m2 is accelerating towards the ground with a = g -

T
which is smaller than g due
m

to the presence of T (tension).


(ii) Considering m1:
T = m1a
--- (1)
Considering m2:
m2g T = m2a ---(2)

B1
B1

Sub (1) into (2): m2g - m1a = m2a


a=

B1

B1

m2 g
(shown)
m1 + m2

(iii) m1 will move forward with a constant speed (B1) and m2 will fall with acceleration

m2 g
. (B1)
m1 + m2
2

(a) either work/time or power = (force distance)/time


to give power = force velocity

M1
A1

(b) (i) (change in) potential energy = mgh


= 0.056 x 9.8 x 16
= 8.78 J (allow 8.8)

C1
A1

(ii) (initial) kinetic energy = mv2


= x 0.056 x 182
= 9.07 J (allow 9.1)
total kinetic energy = 8.78 + 9.07 = 17.9 J

C1
C1
A1

(c) kinetic energy = mv2


17.9 = x 0.056 x v2 and v = 25(.3) m s-1
3

(a)
(i) 1.13 W
(ii) 1.50 V

B1

B1
B1

(b) power = V2 / R or power = VI and V = IR


R = 1.502/1.13
= 1.99 .
(c) either E = IR + Ir
I = 1.5 / 2.0 (=0.75 A)
3.0 = 1.5 + 0.75r
r = 2.0

or

C1
A1

voltage divided between R and r


p.d. across R = p.d. Across r = 1.5

C1
C1

so R = r = 1.99 .

A1

(d) larger p.d. across R means smaller p.d. across r


smaller power dissipation at larger value of V
since power is VI and I is same for R and r
(e)
(i) lamp C
lamp is shorted

M1
A1

(ii) 15

B1

(iii) effective resistance = 15/(1 + 1) + 15


= 22.5
4

M1
A1
A1

M1
A1

(a) (i) Correct direction


A1
(ii) Correct direction
A1
(iii) Flemings left hand rule
A1
(iv) 1. For every action force, there is an equal and opposite reaction force.
2. Correct direction
A1
(b) Taking moments about the centre of the coil,
[1]

2 x FEM (

L
) = Mg x
2
Mgx
B=
IL2

[1]
[1]

(i) Since L2 x [1] , if L is reduced by twice, x will be reduce by four times [1].

A1

Paper 2 Section B
5

(a)(i) point at which whole weight of body


may be considered to act
(ii) sum of forces in any direction is zero
sum of moments about any point is zero

M1
A1
B1
B1

(b) either:
T and W have zero moment about P
M1
so F must have zero moment, i.e. pass through P
A1
or:
if all pass through P, distance from P is zero for all forces (M1)
so sum of moments about P is zero
(A1)
(c) (i) Fcos = Tcos
B1
(ii) W = Fsin + Tsin
B1
(iii) 2W = 3Tsin
B1
(d) magnitude of T and R increases.
No change in the direction of F and T.

B1
B1

(e) (i) p = 0.035 (4.5 + 3.5) OR


a = (4.5 + 3.5)/0.14
= 0.28 N s
= 57.1 m s-2
Force = p/ t (= 0.28/0.14) OR
F = ma (= 0.035 x 5751) (allow e.c.f.)
= 2.0 N
= 2.0 N
A1
upwards
B1
(ii) loss = x 0.035 (4.52 - 3.52)
C1
= 0.14 J
A1
(iii) e.g. plate (and Earth) gain momentum
i.e. discusses a 'system'
equal and opposite to the change for the ball
i.e. discusses force/momentum
so momentum is conserved
i.e. discusses consequence
6

(i) Quantum/packet/discrete amount of energy


mention electromagnetic radiation
(ii) E = hf = h

6.63 10 34 3 108
=
350 10 9

= 5.68 x 10-19 J

[M1]

B1
M1
A1
A1
A1

C1
C1

h
(iii) ratio =

700
v

350

350 1
=
700 2

[A1]

(b) (i) energy of photon to cause emission of electron from surface


either with zero kinetic energy or photon energy is minimum.

[B1]
[B1]

(ii) correct conversion eV to J or J to eV seen


photon energy must be greater than work function
350 nm wavelength and potassium metal

[B1]
[B1]
[A1]

(c) (i) photons have same energy so Emax unchanged


[B1]
intensity OR number of photons per unit time is halved, so n OR n reduced [B1]
(allow 1 mark for statement that Emax unchanged and n reduced)
(ii) photons have higher energy so Emax increases
[B1]
but fewer photons per unit time so n decreases
[B1]
(allow 1 mark for statement that Emax increases and n reduced)
(allow any argument based on increased efficiency)
(d) (i) =

h
h
or =
p
mv

where - wavelength, h plancks constant, p or mv momentum


(ii) E = mv2

2E
m
h
=
=
mv

v=

[M1]

h
2E
m
m

[M1]

h
(shown )
2mE
22 m
2
W E
= = h
(iii) V =
Q Q
Q
2 (0.40 10 9 ) 2 9.11 10 31
(6.63 10 34 ) 2
=
1.6 10 19
=

= 9.4 V

[M2]
[A1]

(a) (i) number of complete oscillations per unit time.


(ii)
correct diagram [1]

[B1]

Longitudinal waves are waves in which the motion of disturbance from source is in the
[B1]
same direction as the direction of wave travel.
or Longitudinal wave is a series of compression and rarefactions that propagrate in a
material
the phase difference between a rarefaction and adjacent compression is radian. [B1]
(b) (i) At x, the 2 waves will be out of phase [B1],
forming destructive interference [B1]. Therefore, the superposed wave is a minima.
(ii) 1.

2. 3

(iii) x = 1.8 10 2 m

[M1]
2

xa 1.8 10 0.20 10 3
=
6.0
D

[M1]

= 6.0 x 10-7 m

(iv) No change in the shape of the graph or no change in the fringe separation [B1]
Dark fringe will become brighter and the bright lines will become dimmer [B1]
(c) (i) 1. Correct sketch
[A1]
2. correct N located [A1]
(ii) distance between 2 nodes = 0.324 m
= 0.324 x 2 = 0.648 m
V = f = 512 x 0.648
= 332 m s-1

[M1]
[M1]
[A1]

(iii) distance between 2 nodes is 0.324 m.


Distance between node and antinode =

1
(distance between 2 nodes)
2

= 0.162 m
0.162 m above the water surface
or 0.005 m above the open end

[B1]
[B1]

Preliminary Examinations 2008


JC 2 Physics 8866/01
Higher 1
Paper 1 Multiple Choice

18 September 2008

1 hour

INSTRUCTION TO CANDIDATES
Do not open this booklet until you are told to do so.
Paper 1
There are thirty questions on this paper. Answer all questions. For each question, there are
four possible answers labelled A, B, C and D. Choose the one you consider correct and record
your choice in soft pencil on the separate answer sheet.
Read very carefully the instructions on the answer sheet.
Write your name and class in the spaces provided on the optical marksheet

Shade your Index Number column using the following format:


1) first 2 digits is your index number in class (eg 5th student is shaded as 05);
2) ignore the last row of alphabets;

This question paper consists of 16 printed pages

Preliminary Examination

Meridian Junior College


JC2 Physics 2008

18 September 2008

DATA AND FORMULAE


Data
speed of light in free space

3.00 x 108 m s-1

elementary charge

1.60 x 10-19 C

the Planck constant

6.63 x 10-34 J s

unified atomic mass constant

1.66 x 10-27 kg

rest mass of electron

me

9.11 x 10-31 kg

rest mass of proton

mp

1.67 x 10-27 kg

9.81 m s-2

acceleration of free fall

Formulae
uniformly accelerated motion

s =

ut +

1
2

at2

v2

u2 + 2as

p V

hydrostatic pressure

gh

resistors in series

R1 + R2 +

1/R

1 / R1 + 1 / R2 +

work done on/by a gas

resistors in parallel

Preliminary Examination

Meridian Junior College


JC2 Physics 2008

18 September 2008

A quantity x is measured many times. The number of measurements, N, giving a


value x is plotted against x. The true value of the quantity is xo.
Which result could be described as precise but not accurate?

Which of the following quantities are all scalars?


A impulse, friction, spring energy
B kinetic energy, momentum, power
C force, gravitational potential energy, distance
D work, pressure, charge

Preliminary Examination

Meridian Junior College


JC2 Physics 2008

18 September 2008

A student attempts to measure the diameter of a steel ball by using a metre rule to
measure four similar balls in a row.

6
c

The student estimates the positions on the scale to be as follows:


X = (1.0 0.2) cm
Y = (5.0 0.2) cm
What is the diameter of a steel ball together with its associated uncertainty?
A 1.0 0.05 cm

B 1.0 0.1 cm

C 1.0 0.2 cm

1.0 0.4 cm

A car is travelling along a straight road. The graph shows the variation with time t
of its acceleration a during part of the journey.
a
V

Z
t

At which points on the graph does the car have its greatest velocity and greatest
displacement?
Greatest velocity

Greatest displacement

Preliminary Examination

Meridian Junior College


JC2 Physics 2008

18 September 2008

Three rocks P, Q and R of masses m/2, m and 2m respectively are thrown off the
edge of a cliff with the same speed v in different directions as shown in the diagram
below. The surface of the ground on which the rocks landed is flat. Ignore the effect
of air resistance.

edge of cliff

Q
R

Which one of the following statements best represents the situation?

A Rock R will reach the ground with the highest speed because it was thrown
downwards and it has a larger mass.
B Rock P will reach the ground with the highest speed because it travels a greater
distance downwards, thereby gaining more kinetic energy.
C Rock P, Q and R will reach the ground with the same speed because the effects
of mass and thrown angles are not significant for parabolic motions.
D Need more information like the thrown angles and to decide the answer.

When a man is standing in an ascending lift, the magnitude of the force exerted on the
mans feet by the floor is always

A equal to the magnitude of the force exerted on the lift floor by his feet.
B equal to the magnitude of his weight.
C less than the magnitude of his weight.
D equal to what it would be in a stationary lift.

Preliminary Examination

Meridian Junior College


JC2 Physics 2008

18 September 2008

If we assume that the masses of the ropes and pulley are negligible, what will be the
tension found in the angled rope shown in the diagram below?
ceiling

angled rope

pulley

(Mg + mg)2 + (mg)2

(Mg)2 + (mg)2

C Mg + mg

Mg mg

The diagram below shows a velocity (v) time (t) graph of a 3.0 kg battery operated
toy car. From t = 0.0 s to 4.0 s, the toy car experiences a horizontal driving force F
from its battery and frictional force f. After t = 4.0 s, the battery of the toy car
suddenly fails and the car is brought to rest by the frictional force f.
v / m s-1
12

0.0

10

4.0

t/s

What is the value of F ?


A 3.0 N

B 9.0 N

C 15 N

36 N

Preliminary Examination

Meridian Junior College


JC2 Physics 2008

18 September 2008

In the diagram below, balls B, C, D, E and F stay at rest on a horizontal surface and
they are lined up side-by-side to each other in a straight line. Ball A is moving
towards ball B along the same straight line with a speed of 10 m s-1. Finally, ball A
collides elastically with ball B. The friction between the sides of the balls and the
surface is negligible.
10.0 m s-1
A

The mass of ball A and F is m, while the mass of the other balls is 4m each.
Which one of the following statements is true?
A Only one ball moves after the collision.
B Two balls move after the collision.
C Three balls move after the collision.
D Ball F will move with a speed of 10 m s-1.

10

A ball of mass m released from a height h0 above a horizontal surface rebounds to


a height h1 after one bounce. The graph that relates h0 to h1 is shown in the diagram
below.
h1 / cm

80

100

h0 / cm

If the ball was dropped from initial height h and made three bounces, what is the
total loss in kinetic energy of the ball immediately after its third impact with the
surface?
A 2(0.2)mgh

B 3(0.2)mgh

C (1-(0.8)2)mgh

(1-(0.8)3)mgh

Preliminary Examination

11

Meridian Junior College


JC2 Physics 2008

18 September 2008

In the diagram below, an object of mass m, originally at rest, slides down the top of
a smooth plane inclined at an angle of . At time t, it falls a vertical distance of h
metres and gains a velocity v.

Which of the following represents the instantaneous power of the object at time t?
Ignore air resistance.

mg 2gh sin

mgh +

1 2
mv
2

mg 2gh sin t

12

mgv sin
t

Which of the following is a property of transverse wave, but not a longitudinal wave?
A Diffraction
B Refraction
C Reflection
D Polarisation

13

Consider a sound beacon emitting sound waves deep inside the ocean. Which of
the following is true of the waves as it travels through the water? Assume negligible
energy losses due to attenuation (energy loss).
Nature of the wave

Amplitude (with distance d from the source)

transverse

proportional to 1/d

transverse

proportional to 1/d2

longitudinal

proportional to 1/d

longitudinal

proportional to 1/d2

Preliminary Examination

14

Meridian Junior College


JC2 Physics 2008

18 September 2008

A stationary sound wave has a series of nodes. The distance between the first and
the sixth node is 30.0 cm.
What is the wavelength of the sound wave?
A 5.0 cm
B 6.0 cm
C 10.0 cm
D 12.0 cm

15

The diagram shows the displacements at the same instant of two waves, P
and Q, of equal frequency and having amplitudes Y and 2Y, respectively.

2Y

The waves are superimposed to give a resultant wave. What is the amplitude of the
resultant wave and what is the phase difference between the resultant wave and
wave P?
amplitude of
resultant wave

(phase difference between resultant wave and


wave P)/radians

3Y

3Y

Preliminary Examination

Meridian Junior College


JC2 Physics 2008

18 September 2008

16 A high voltage is applied across the terminals of a gas discharge tube such that the gas
ionizes. The electrons move towards one end of the discharge tube at a rate of
2.5 x 1015 s-1 while the positive ions move to the other end of the discharge tube at a rate
of 2.5 x 1015 s-1.
What is the value of the current flowing in the discharge tube?
A 0A
B 8.0 x 10-4 A
C 5.0 x 1015 A
D 3.1 x 1015 A

17

Which of the following options will double the original resistance of a wire?
Radius of Wire

Length of Wire

Halve

Halve

No change

Halve

Halve

18

No change

Halve

Double

Which part of the circuit has the smallest current flowing through it?

3.0 V

10

10

20

Preliminary Examination

Meridian Junior College


JC2 Physics 2008

18 September 2008

19
A1
A2
A3
S1

In the circuit shown above, the lamps have identical resistances and the ammeters have
negligible resistances. What will happen when the switch S1 is closed?

Reading of ammeter A1 increases while that of A2 and A3 remain unchanged.

Reading of ammeter A1 increases while that of A2 and A3 decreases.

Reading of ammeter A1 increases while that of A2 and A3 increases.

Reading of ammeter A1 decreases while that of A2 and A3 increases.

20
A
10.0

10.0
10.0

10.0

10.0

Determine the effective resistance between terminals A and B.


A

5.0

10

11

20

35

Preliminary Examination

Meridian Junior College


JC2 Physics 2008

18 September 2008

21
E1 = 3.0 V, r1= 1.0
G

2.0

E2 unknown, r2= 3.0

Two batteries E1 and E2, with internal resistances r1 and r2 respectively, are connected
as shown in the circuit above. Determine the e.m.f. of E2 when no current flows through
the galvanometer G.
A

22

3.0 V

4.5 V

C 6.0 V

D 7.5 V

An electron is fired horizontally into a long thin solenoid as shown below. The magnetic
flux density at the centre of the solenoid is given as B = 0 nI , n = number of turns per
unit length. The solenoid has 7 turns in one centimeter and is supplied by a current of
3.5 A. The velocity of the electron is 2.5 x 107 m s-1.

Determine the magnetic force experienced by the electron when it is in the middle of the
solenoid.
A

0N

1.2 x 10-18 N

12

1.2 x 10-16 N

1.2 x 10-14 N

Preliminary Examination

23

Meridian Junior College


JC2 Physics 2008

18 September 2008

The graph of magnetic flux density B, against the distance x from a point outside a long
solenoid carrying a constant current is shown below.
B

A B C D
Which of the position marks the end of the solenoid?

24 The diagram below shows a cross-sectional view of four long straight current-carrying
wires. The wires are aligned parallel to each other and are perpendicular to the plane of
the page, passing through the points P, Q, R, S at the corners of a square. O is the
intersection of the diagonals of the square.
P

If the magnetic flux density at the point O is zero, which of the following statements must
be true?
A

The currents in all four wires must be in the same direction.

The currents in all four wires must be of the same magnitude.

The current in P must be in the same direction as that in R and the current in Q
must be in the same direction as that in S

The current in P must be in the opposite direction as that in R and the current in Q
must be in the opposite direction as that in S.

13

Preliminary Examination

Meridian Junior College


JC2 Physics 2008

18 September 2008

25

photocell

meter
Light falls on the photo-sensitive metal surface of a photocell. A battery and a sensitive
meter are connected to the photocell as shown. Which of the following statements is
true?
A

The number of electrons emitted from the metal surface per second is proportional
to the potential difference between the metal surface and the anode.

The maximum energy of the electrons emitted is independent of the particular metal
used.

26

No deflection is observed in the meter unless the wavelength of light is below a


maximum value.

No deflection is observed in the meter until after a considerable time when the metal
surface has heated up.

A laser beam of power P has wavelength . What is the rate of photons produced?
A

hc
P

Phc

14

P
hc

Phc

Preliminary Examination

27

Meridian Junior College


JC2 Physics 2008

18 September 2008

In a typical photoelectric effect experiment, monochromatic light is shone on a metal


surface. The graph below shows how the photocurrent I varies with the potential
difference V applied across the emitter and collector.
I

V
Vo

If the frequency and intensity of the incident light is increased, which of the following
correctly shows the new I-V graph (dotted line) ?

V
Vo

Vo

Vo

Vo

15

Preliminary Examination

28

Meridian Junior College


JC2 Physics 2008

18 September 2008

An atom makes a transition from a state of energy E2 to one of lower energy E1.
Which of the following gives the wavelength of the radiation emitted, in terms of the
Planck constant h and the speed of light c?

29

E2 E1
hc

hc
E2 E1

hc hc

E2 E1

c
h(E 2 E1 )

White light from a tungsten filament lamp is passed through sodium vapour and viewed
through a spectrometer. Which of the following best describes the spectrum that would
be seen?

A Coloured lines on a black background.


B White lines on a black background.
C Dark lines on a coloured background.
D Dark lines on a white background.

30

A surface is bombarded normally by photons of frequency f. On average, n photons


strike a unit area of the surface each second. Given that h is the Plancks constant,
assuming that the photons are absorbed by the surface, what is the pressure exerted on
the surface?

2nhf

2nhf
c

nhf
c

hf

~ End of Paper ~

16

Class

Reg Number

Candidate Name _____________________________

Preliminary Examinations 2008


JC 2 Physics 8866/02
Higher 1
Paper 2

8 September 2008

2 hours

INSTRUCTION TO CANDIDATES
This booklet contains Sections A and B of Paper 2.
Do not open this booklet until you are told to do so.
Section A
Answer all questions.
Examiners Use
Section B
Section A
Answer any two questions. Please note that the marker
will mark and allocate marks for only the first two
questions if you attempt more than two questions.

The number of marks is given in brackets [ ] at the end


of each question or part question. Marks will be
deducted if units are not stated where necessary or if
answers are not quoted to the appropriate number of
significant figures.
All working for numerical answers must be shown. You
are reminded of the need for good English and clear
presentation of your answers.

/7

Q3

/ 12

Q4
INFORMATION FOR CANDIDATES

/7

Q2
You are advised to spend about one hour on each
section. Write your answers on this question booklet in
the blanks provided.

Q1

/7

Q5

/7
Section B

Circle the questions you attempted


Q6

/ 20

Q7

/ 20

Q8

/ 20

Deductions
Total

This question paper consists of 25 printed pages

/ 80

Preliminary Examination

Meridian Junior College


JC2 H1 Physics 2008

8 September 2008

DATA AND FORMULAE


Data
speed of light in free space

3.00 x 108 m s-1

elementary charge

1.60 x 10-19 C

the Planck constant

6.63 x 10-34 J s

unified atomic mass constant

1.66 x 10-27 kg

rest mass of electron

me

9.11 x 10-31 kg

rest mass of proton

mp

1.67 x 10-27 kg

9.81 m s-2

acceleration of free fall

Formulae
uniformly accelerated motion

s =

ut +

1
2

at2

v2

u2 + 2as

p V

hydrostatic pressure

gh

resistors in series

R1 + R2 +

1/R

1 / R1 + 1 / R2 +

work done on/by a gas

resistors in parallel

2
[Turn Over

Preliminary Examination

Meridian Junior College


JC2 H1 Physics 2008

8 September 2008

Section A
Answer all the questions in the spaces provided.

Fig. 1.1 (not drawn to scale) shows a projectile shot off at an angle of 60o with
respect to the horizontal with a speed of 10 m s1. The projectile landed t seconds
later at point A along the slope. The slope has an angle of 45o to the horizontal as
shown.
10 m s1

60o
O

45o

Figure 1.1

Assume air resistance is negligible.


(i)

Using s = ut + at2, write down the equations in terms of t for the projectiles
1. horizontal displacement, sx,
2. vertical displacement, sy.

[1]
(ii)

Show that the ratio of sy to sx when the stone hits the slope at A is equal to 1.

[1]

Preliminary Examination

(iii)

Meridian Junior College


JC2 H1 Physics 2008

8 September 2008

Hence calculate the time taken to hit point A.

t = . s

(iv)

[2]

Calculate the speed of impact v at A.

v = . m s-1

[3]

Preliminary Examination

Meridian Junior College


JC2 H1 Physics 2008

8 September 2008

Fig 2.1 shows a helicopter of mass m hovering above ground. The rotating blades
of the helicopter move through an effective area A, and gives an average speed of v
m s-1 to the air.

helicopter hovering

ground

Figure 2.1
(i)

State the volume of air moved per second in terms of A and v.


Volume of moving air per second = ..

(ii)

[1]

Assuming the density of air is , derive an expression for the force F exerted
on the air by the helicopter blades in terms of A, v and .

[2]
(d)

Assume the helicopter hovers at the same spot, draw a free body diagram and
label all the forces acting on the helicopter in the space below.

[1]

Preliminary Examination

(e)

Meridian Junior College


JC2 H1 Physics 2008

8 September 2008

Given m = 500 kg, A = 30 m2 and = 1.3 kg m-3, calculate v.

-1

v = . m s

[3]

A photovoltaic cell is a device which converts light energy directly to electrical


energy. A potential difference is developed between its two terminals. The
magnitude of the p.d. depends on the intensity of the light incident on the surface of
the cell and on the current of the cell.
The variation of the potential difference V across the cell with current I can be
investigated using the circuit shown in Fig. 3.1.

Photovoltaic
cell

Figure 3.1
R is a variable resistor, the voltmeter has infinite resistance and the resistance of
the ammeter is negligible.

Preliminary Examination

Meridian Junior College


JC2 H1 Physics 2008

8 September 2008

Fig. 3.2 shows the variation of V with I for a particular cell of surface area
4.0 x 10-4 m2 when illuminated normally with light of intensity 1100 W m-2.

600
550
500
450
400
V/mV

V/ mV 350
300
250
200
150
100
50
0
0

10 20

30 40 50

60 70 80 90 100 110 120 130 140


I/mA

I/ mA

Figure 3.2

(a)

Use the data supplied in Fig. 3.2 to complete the table below;

V/mV
250
350
450
470
480
510

I/mA

P/mW

130
120

45.50
54.00

110

52.80
[2]

Preliminary Examination

(b)

(i)

Meridian Junior College


JC2 H1 Physics 2008

8 September 2008

Using the values in the table in (a), draw a graph of P against V on the
axes given below in Fig. 3.3

P/ mW

55

50

45

40

35

30
200

250

300

400

350

450

500

V/ mV

Figure 3.3

[2]
(ii)

Determine the maximum power output Pmax of the cell from the graph
drawn in Fig. 3.3.

Pmax = .. m W

(iii)

[1]

Hence calculate Imax for this maximum power output.

Imax = .. A

[2]

Preliminary Examination

(iv)

Meridian Junior College


JC2 H1 Physics 2008

8 September 2008

Using the value of the maximum power output Pmax and any other data,
calculate the maximum efficiency of conversion of light energy into
electrical energy.

Maximum Efficiency = .. %
(c)

[3]

Another different type of solar cell produces an output power of


250 mW at an output potential of 500 mV.
Determine how you can obtain a maximum output power of
approximately 0.75 W at 1.0 V using a number of these photovoltaic
cells in a suitable arrangement. Draw the arrangement in the space
provided below.

[2]
9

Preliminary Examination

(a)

(i)

Meridian Junior College


JC2 H1 Physics 2008

8 September 2008

Define the magnetic flux density of a magnetic field.


.
.
. [1]

(ii)

Hence or otherwise, show that the units of flux density can be


expressed as N s C-1 m-1.

[2]
(b)

A twin core electrical cable made up of two wires as shown in Fig 4.1, is used
to supply current of 0.50 A to a household lamp. The distance between the
centres of the wires is 0.50 cm.
0.50 cm
Electrical
insulation

Wires

Figure 4.1: Top View

(i)

Given that the magnetic flux density for a long wire carrying current
is B =

0 I
, where d is the distance between the two wires, and I is the
2d

current in the wire, calculate the force per unit length between the
wires.

Force per unit length = . N m-1 [3]


10

Preliminary Examination

(ii)

Meridian Junior College


JC2 H1 Physics 2008

8 September 2008

Given that the current in both wires are moving in the opposite
direction, draw the magnetic forces acting on the wires, clearly
indicating the direction of the current in the wires.

[1]

(a)

Fig 5.1 below shows the energy levels of the hydrogen atom.
eV
0
-0.54

n=
n=5

-0.84

n=4

-1.5

n=3

-3.4

n=2

-13.6

n=1

Figure 5.1

(i)

Determine the ionization energy of hydrogen atom in joules.

Ionization energy= .. J
11

[2]

Preliminary Examination

(ii)

Meridian Junior College


JC2 H1 Physics 2008

8 September 2008

Hence find the minimum wavelength of photon that the hydrogen


atom needs to absorb to be ionized.

Wavelength = . m

(b)

[2]

Vega is the brightest star in the constellation Lyra and the fifth brightest star in
the sky. Vega has a ring of gas around it. Fig. 4.2 shows the variation in
intensity against wavelength for part of the visible spectrum of Vega.

Intensity

Wavelength/nm
Figure 4.2

The absorption lines in Fig. 4.2 are due to excited gas atoms around Vega.
(i)

Explain how the absorption spectrum is formed.


.
.
.
. [2]

12

Preliminary Examination

(ii)

Meridian Junior College


JC2 H1 Physics 2008

8 September 2008

Sketch in the space below, using the axes provided, a labelled


graph of intensity against wavelength of the emission spectrum
for the same gas atoms in b(i).

Intensity

Wavelength /nm
[1]

13

Preliminary Examination

Meridian Junior College


JC2 H1 Physics 2008

8 September 2008

Section B
Answer two questions from this section.
6

Define momentum.

(a)

(b)

[1]

A ping-pong (table-tennis) ball and a bowling ball are rolling in the same
direction on a frictionless surface. Both balls have the same momentum.

(i)

State which of the balls has a higher velocity.

(ii)

[1]

If the same external force, F, is exerted on both balls individually to


stop the balls, comment on the time intervals to stop the ping-pong
ball and the bowling ball. (Which ball will take a longer time or shorter
time to stop? Or will they take the same time?) Explain.

.......

(iii)

[2]

Show that the deceleration of the bowling ball aB as it stops is given


by aB =

MP
aP
MB

where ap is the deceleration of the ping-pong ball, MP is the mass of


ping-pong ball and MB is the mass of the bowling ball.

[2]

14

Preliminary Examination

(iv)

Meridian Junior College


JC2 H1 Physics 2008

8 September 2008

Sketch a graph to show how the velocity of the ping-pong ball and
bowling ball changes with time from the moment the external force F
is applied on each ball to the time when they stop.
Label the graph of the ping-pong ball, P and the graph of
bowling ball, B.

Velocity/m s-1

time/s

[2]
(v)

Hence state which of the balls will have a longer stopping distance.

(c)

(i)

[1]

State the conservation of momentum.

15

[2]

Preliminary Examination

(ii)

Meridian Junior College


JC2 H1 Physics 2008

8 September 2008

The bowling ball of mass 1.50 kg is now rolled on a smooth track with a
constant velocity. A steel ball of mass 1.00 kg is also moving at a constant
velocity and collides with the bowling ball at t =0 s.
The graph in Fig. 6.1below shows their individual momentum versus time.
Complete the graph of the bowling ball for t >0 s.

momentum/ kg ms-1

8.00
6.00
Steel ball

4.00
2.00
0
- 2.00
- 4.00
- 6.00

Bowling ball

- 8.00
time/ s
-2.0

-1.0

1.0

Figure 6.1

16

2.0

[2]

Preliminary Examination

(iii)

Meridian Junior College


JC2 H1 Physics 2008

8 September 2008

Using the data in Fig. 6.1, calculate the average force experienced by
the steel ball during the collision.

Average force = ...N


(d)

[2]

The same bowling ball has fallen into a trench. In an attempt to get the ball out of
the trench, a student uses some pulleys, an inextensible string and a batteryoperated toy car.
The toy car of mass mc is initially at rest on one side of the trench which has a
rough surface. The pulley is set up vertically 30.0 cm above the car and the taut
string is tied to the back of the car as shown in Fig. 6.2(a).

30.0 cm

40.0 cm
rough surface

rough surface

(a)

(b)
Figure 6.2

The boy was able to lift the bowling ball when he starts the car. When the toy car
has moved a distance of 40.0 cm in the horizontal direction, the bowling ball is
raised by height h as shown in Fig 6.2 (b).

17

Preliminary Examination

(i)

Meridian Junior College


JC2 H1 Physics 2008

8 September 2008

Show that the new length of string between toy car and pulley, l, in
Fig 6.2 (b) is 50.0 cm.

l = . cm
(ii)

Hence calculate the height h at which the bowling ball is raised.

h = . cm
(iii)

[1]

[2]

Calculate the gain in gravitational potential energy of the bowling ball


given that mass of bowling ball is 1.50 kg.

Gain in gravitational potential energy .. J

18

[2]

Preliminary Examination

(a)

Meridian Junior College


JC2 H1 Physics 2008

8 September 2008

A length of wire is maintained taut between two points M and P as shown in


Fig. 7.1. A signal generator which produces alternating current of variable
frequency is passed through the wire and a pair of magnets is placed on
either side of the wire as shown.

Figure 7.1

The frequency of the signal generator is gradually adjusted to 10 Hz (i.e. the


current in the wire changes direction 10 cycles per second). A stationary
wave is set up as shown in Fig. 7.1.
(i)

Explain why the wire is observed to vibrate when an alternating


current is passed through it.
.
.
.
.

(ii)

[2]

Given that the horizontal length kept taut between M and P is 0.60 m,
determine wavelength of the stationary wave formed.

wavelength = m

19

[1]

Preliminary Examination

(iii)

Meridian Junior College


JC2 H1 Physics 2008

8 September 2008

Determine the speed of the wave.

speed = m s-1
(iv)

[2]

Sketch the stationary wave formed in the space below when the
frequency of the generator is adjusted to 30 Hz (alternating current
changes direction 30 cycles per second).

[2]
(b)

Fig. 7.2 shows an experimental arrangement for observing light wave


interference fringes using a double-slit method. A single slit so is placed in
front of an extended light source S which produces monochromatic light of
wavelength .

so

light source
S

s1
0.50 mm

s2

2.0 m

Figure 7.2

The double slits have a separation of 0.50 mm and the screen is placed
2.0 m from it. Fringes are observed on the screen, the central one being
located at C.

20

Preliminary Examination

(i)

Meridian Junior College


JC2 H1 Physics 2008

8 September 2008

Given that the distance from the fourth bright fringe on one side of the
pattern to the fourth bright fringe on the other side of the pattern is
found to be 12.8 mm, calculate the fringe separation of the
interference pattern.

Fringe separation = m

(ii)

Determine the wavelength of the light.

wavelength = m

(iii)

[2]

[2]

Explain why single slit so is needed.


.
.

(iv)

[1]

State the experimental advantage in determining the fringe separation


in the way used in b(i).
.
.

(v)

[1]

State and explain what change will be observed, if any, in the


interference pattern when each of the following changes is made
separately.
1. increasing the distance between the double slit and the screen.
.
.
.

21

[2]

Preliminary Examination

Meridian Junior College


JC2 H1 Physics 2008

8 September 2008

2. white light is used instead of monochromatic light source.


.
.
.

(c)

[2]

Fig. 7.3 shows a similar set using a double slit and a beam of helium atoms
all travelling with the same velocity.

Source of
Helium atoms
O

Detector move
in direction of
arrow
single slit

double slits

Figure 7.3

It is observed that the intensity of the beam measured by the detector varies
along the direction moved by the detector. The detector detects a maximum
at point O and subsequently alternating regions of minima and maxima.
(i)

What does the above observation tell you about the moving helium
atoms?
.
.

(ii)

[1]

Given that the wavelength required to produce such a pattern is


approximately 1 x 10-10 m, calculate the speed of the helium atoms
given that the mass of the atoms is 6.6 x 10-27 kg.

speed = m s-1

22

[2]

Preliminary Examination

(a)

(i)

Meridian Junior College


JC2 H1 Physics 2008

8 September 2008

Distinguish between e.m.f. of a battery and potential difference


across a resistor in terms of energy considerations.

(ii)

[2]

Two equations for the power P dissipated in a resistor are P = I2R


and P = V2/R. The first suggests that the greater the resistance R
of the resistor, more power is dissipated. However, the second
equation suggests that the greater the resistance, less power is
dissipated. Is there an inconsistency? Explain your answer.

(b)

[3]

A 230 V electric kettle transfers 7.0 x 105 J of energy in 5.0 minutes.


(i)

Calculate the current in the kettle.

Current = A
(ii)

[2]

Power = .W

[2]

Calculate the power rating of the kettle.

23

Preliminary Examination

(iii)

Meridian Junior College


JC2 H1 Physics 2008

8 September 2008

Show that the effective resistance of the heating coil in the kettle
is 23 .

[1]
(c)

A cylindrical copper rod is resting freely on two horizontal smooth


conducting rails which are connected to a battery of negligible internal
resistance. A magnetic flux density of 0.55 T is applied perpendicularly
into the paper as shown in Fig. 8.2. The copper rod has a mass of 40 g.
The resistance of the wires, conducting rails and copper rod are
negligible.

24.0 V

Conduction
rail

15.0 cm

D
Copper rod

R1 = 6.0

Figure 8.2 : Top View

Copper rod

R1

Conduction
Rail

Figure 8.3 Side View

(i)

Draw a clearly labelled diagram to illustrate the direction of the


magnetic force F on the copper rod when a current I flows
through the copper rod when the switch is closed.

[2]
24

Preliminary Examination

(ii)

Meridian Junior College


JC2 H1 Physics 2008

8 September 2008

When the switch is closed, current flows through the copper rod.
Given that the battery has an e.m.f. of 24.0 V and the value of the
resistor R1 is 6.0 , calculate the value of this current through the
copper rod.

Current = A

(iii)

Calculate the magnetic force on the copper rod.

Force = N

(iv)

[2]

The conduction rails must be tilted to keep the copper rod


stationary when the switch is closed. State which ends of the rail
need to be raised. (AB or CD).

(v)

[2]

[1]

Hence draw a labelled free body diagram of the copper rod when
it is stationary with the conduction rails tilted.

[3]

~ End of Paper ~

25

Preliminary Examination Answers

Meridian Junior College


JC2 Physics 2008

18 September 2008

MJC Prelim 2008


H1 Paper 1 Answers
1
2
3
4
5
6
7
8
9
10

11
12
13
14
15
16
17
18
19
20

B
D
B
D
C
A
B
C
C
D

A
D
C
D
B
B
C
B
A
B

21
22
23
24
25
26
27
28
29
30

D
A
B
C
A
C
A
B
C
C

Reasoning for MCQ


MCQ 1:
reasoning:

Precise but not accurate. Small distribution but far from accurate value.

MCQ 2:
reasoning:

Work, pressure, charge

MCQ 3:
reasoning:

Length XY =5.0 -1.0 = 4.0 = d + d + d +d


XY = 0.2 + 0.2 = 0.4 = d + d + d +d
Diameter of steel ball, d = 4.0/4 =1.0 cm
d = 0.4 /4 = 0.1
therefore, d = 1.0 0.1 cm

MCQ 4:
reasoning:

At X, car starts to decelerate (negative acceleration) hence X is where velocity is max. Looking at
area under the graph (= change in velocity), it is maximum at X (beyond X area is negative).
Beyond X, car decelerates with maximum deceleration at Y before the magnitude of deceleration
decreases and it travel at constant velocity at Z. Therefore maximum displacement is at Z.

Preliminary Examination Answers

MCQ 5:
reasoning:

Meridian Junior College


JC2 Physics 2008

18 September 2008

using COE,
loss of gPE = gain in KE
mgh = mvf2 - mvi2
hence vf = (2gh + vi2)1/2
since the h and vi are the same for all, the final v will also be the same.

MCQ 6:

reasoning: By Newtons 3rd law, opposite and equal forces: the force exerted on the mans feet by the
floor is always equal to the force exerted on the lift floor by his feet.

Preliminary Examination Answers

Meridian Junior College


JC2 Physics 2008

MCQ 7:
B
reasoning:
Considering FBD of mass m:

18 September 2008

Let the tension in the angled rope be T.

When mass m is in equilibrium,

Tcos = Mg ------- (1)


Tsin = mg -------- (2)
(1)2 + (2)2 :
T =

Mg

mg

(Mg)2 + (mg)2

PS: We should not assume a bigger object always weigh more the question does not specify
what materials are the boxes made of.

MCQ 8:
reasoning:

Between t = 4.0 s to 6.0 s,


a=

12
= 2.0 m s-2 (deceleration)
6.0

frictional force f = ma = (3.0 kg)(2.0 m s-2) = 6.0 N


Between t = 0.0 s to 4.0 s,
12
a=
= 3.0 m s-2 (acceleration)
4.0
The pulling force F has to overcome frictional force f to produce this acceleration.
F f = ma = (3.0 kg)(3.0 m s-2) = 9.0 N
F 6.0 N = 9.0 N
F = 15 N

Preliminary Examination Answers

MCQ 9:
reasoning:

Meridian Junior College


JC2 Physics 2008

18 September 2008

In elastic collision, there is both conservation of momentum and kinetic energy.


Between ball A and B:
Since mass of ball A < mass of ball B, ball A is going to rebound back.
Rebound velocity of ball A: v 1 =

m 4m
3
(10 m s-1 ) =
(10) = 6.0 m s-1
5
m + 4m

Using COM:
m(10 m s-1) + 0 = m(-6 m s-1) + (4m)v2, velocity of ball B, v2 = 4 m s-1 or
Alternatively: velocity of ball B: v 2 =

2m
2
(10 m s-1 ) = (10) = 4.0 m s-1
5
m + 4m

Ball B will then transfer all its gain momentum to ball C and since they are of the same mass, the
final velocity of ball B is zero. Ball C now gains a speed of 4.0 m s-1. Ball C will do the same to ball
D and ball D will do the same to ball E (chain of elastic collisions).
Between ball E and F:
When ball E collides with ball F, it already has an initial velocity = 4.0 m s-1.
Since mass of ball E > mass of ball F, both ball E and ball F will move.
Velocity of ball E: v 1 =

4m m
3
(4.0 m s-1 ) = (4.0) = 2.4 m s-1
4m + m
5

Velocity of ball F: v 2 =

2(4m )
8
(4.0 m s-1 ) = (4.0) = 6.4 m s-1
m + 4m
5

PS: Unlike balls B, C and D, ball E actually moves together with ball F in the same direction.
Altogether, there will be three balls moving after the collision because we also need to include
ball A, which rebounds.

MCQ 10:
reasoning:

The kinetic energy of the ball immediately after its first impact with the surface = 0.8 mgh
k.e. = mgh 0.8 mgh = 0.2 mgh.
the kinetic energy of the ball immediately after its third impact with the surface = (0.8)3mgh

So, the total loss in kinetic energy of the ball after its total impact with the surface
= mgh - (0.8)3mgh = (1- (0.8)3)mgh

loss of

Preliminary Examination Answers

MCQ 11:
reasoning:

Meridian Junior College


JC2 Physics 2008

18 September 2008

The constant force pulling the object down the slope, F = mgsin
Instantaneous power, P = Fv = mgsinv
But v = 2gh {from mgh = mv2}, P = Fv = mgsinv = mg sin 2gh

MCQ 12:
D
From the notes: Longitudinal wave cannot be polarised.

MCQ 13:

Sound wave is a longitudinal wave. Intensity, I

1
where r is the distance away from source.
r2

Since I (Amplitude ) 2 ,

Amplitude

1
r

MCQ 14:
D
Distance between 6 nodes = 2 = 30 cm
Therefore 1 wavelenght = 12 cm

MCQ 15:
B
Amplitude of resultant wave = | Y+ (-2Y) | = Y since P and Q are out of phase.
Resultant wave will be in phase to Q. Hence it is out of phase to P.

MCQ 16: Ans: B. I = Q/t = Nq /t = (2.5 x 1015 + 2.5 x 1015) x 1.60 x 10-19 = 8.0 x 10-4 A
MCQ 17: Ans: C. Given R = (L)/A, where A = r2, option c will double the original resistance, since
the area has been reduced by and the length has been reduced by giving a net increment of
twice the original value.
MCQ 18: Ans: B. A and D will have the same current that is split into B and C. Since B has a higher
resistance, it will have the smallest current.
MCQ 19: Ans: A. When switch S1 is closed, the effective resistance reduces and hence A1
increases. However, since the voltage drop across the parallel circuit of the lamps is the same, A2
and A3 will have the same current readings.
MCQ 20: Ans: B. Since the circuit given is symmetrical, the equivalent resistance across AB is just 20
resistors in parallel. Therefore answer is 10 .
MCQ 21: Ans: D. Since no current flows in the upper circuit, the voltage drop across 2.0 resistor
must be 3.0 V. Therefore current in lower circuit is 3.0/2.0 = 1.5 A. Thus
E2 = 1.5 x 3.0 + 3.0 = 7.5 V.
5

Preliminary Examination Answers

Meridian Junior College


JC2 Physics 2008

18 September 2008

MCQ 22: Ans: A, Since the path of the electron is parallel to the magnetic field lines at the middle
of the solenoid, there is no magnetic force acting on the electron. Sin 0 = 0.
MCQ 23: Ans: B. At the end of the solenoid, the magnetic flux density is half that of the uniform
field in the middle.
MCQ 24: Ans: C. The must be true condition is that current in diagonally opposite wires need to be
of the same direction and magnitude.
MCQ 25: Ans: A.
There will be no photoemission and photocurrent unless f is above threshold frequency or
wavelength less than threshold wavelength, which is a maximum value of wavelength.
MCQ 26: Ans: C

Power = Total Energy / time

Nhc
where N is the no. of photons produced.
t
N P
=
t
hc

P=

MCQ 27: Ans: A.


h

f , as frequency is increased, the stopping potential gets bigger (larger in


e
e
magnitude. more negative)

From Vs =

As intensity increases, the rate of incident photon increases, hence the rate of photoelectron
emitted increases and photocurrent increases.
MCQ 28: Ans: B

hc

= E2 E1

hc
E2 E1

MCQ 29: Ans: C


The spectrometer setup disperses the white light into its component colours, forming a coloured
background. Sodium vapour absorbs photons of specific frequencies corresponding to the
difference in energy levels of the atoms. These missing frequencies are manifested as dark lines on
a coloured background.

Preliminary Examination Answers

Meridian Junior College


JC2 Physics 2008

MCQ 30: Ans C

Momentum of a photon: p = mc =

hf
c

hf
0|
c
Change in momentum per unit area per unit time =
nhf
=
c
|n

18 September 2008

Preliminary Examination Answers

Meridian Junior College


JC2 Physics 2008

MJC Prelim 2008


H1 Paper 2 Answers
Section A
1

Fig. 1.1 (not drawn to scale) shows a projectile shot off at an angle of 60o with
respect to the horizontal with a speed of 10 m s1. The slope has an angle of 45o to
the horizontal. The projectile landed t seconds later at point A along the slope.

10 m s1

60o
O

45o

Figure 1.1 (not drawn to scale)


If we assume the effect of air resistance is negligible,
(i)
(i)

Using s = ut + at2, write down two equations for


1. horizontal displacement, sx, with time,
2. and the vertical displacement, sy, with time.
Using s = ut + at2
+: sx = (10 cos 60o) t =5 t
+: sy = (10 sin 60o) t + (-9.81) t2 = 8.66 t 4.905 t2
[1]- for both equations to be correct.

(ii)

Show that the ratio of sy to sx when the stone hits the slope at A is equal to 1.
Sy /Sx = tan 45o [1]
Sy /Sx = 1

Preliminary Examination Answers

(iii)

Meridian Junior College


JC2 Physics 2008

Hence calculate the time taken t to hit point A.


Taking the directions into consideration, use Sx = - Sy
10 cos 60 t = - (10 sin 60 t + (-9.81)t2) [1]
t = = 2.78 [1]

(iv)

Calculate the speed of impact at A.

vx = 10 cos 60

v y = 10sin 60(2.78) + (1/ 2)(9.81)(2.78) 2

2
2
v = vx + v y

= .

[1]

[1]

= 19.3 m s-1

[1]

[2]

Preliminary Examination Answers

Meridian Junior College


JC2 Physics 2008

Fig 2.1 shows a helicopter of mass m which starts from rest hovering above ground.
The rotating blades of the helicopter move through an area, A, and gives an
average speed of v m s-1 to the air.

helicopter hovering

ground

(i)

Find the volume of moving air in 1 second in terms of A and v.


Volume of moving air in 1 second
= cross-sectional area of air flow x velocity of air flow
= Av
[1]
.
Volume of moving air in 1 s = ____________

(ii)

[1]

Assuming the density of air is , derive an expression for the force F exerted
on the air by the helicopter blades in terms of A, v and .
Force on the air
m v
)
t
= mass of moving air per unit time x change in velocity of air
[1]
= (volume of moving air per unit time x density of air) x change in
velocity of air
= Av x x v
[1]
= Av2

= momentum change per unit time of the air molecules (i.e.,

(d)

[2]

Assume the helicopter stays at the same spot, draw and label all the forces
acting on the helicopter in the diagram below.
Force on the
blades by air

mg
3

[1]

Preliminary Examination Answers

(e)

Meridian Junior College


JC2 Physics 2008

Given m = 500 kg, A = 30 m2 and = 1.3 kg m-3, calculate v.


Since the weight of the helicopter is mg and the magnitude of the
upward force is Av2 :
Force on the air by the blades = Force on the blades by air [1]
mg = Av2
(500 kg)(9.81 m s-2) = (30 m2)(1.3 kg m-3)v2
[1]
v=

(a)

(500)(9.81)
(30)(1.3)

= 11 m s-1

[1]

[3]

Use the data supplied in Fig. 3.2 to complete the values current I and the
corresponding power P in the table below;

V/mV

I/mA

P/mW

250
350
450
470
480
510

133
130
120
114
110
70

33.25
45.50
54.00
53.58
52.80
35.70
[2]

[1]- for all correct values of current.


[1]- for all correct values of power.
(b)

(i)

Draw a graph of P against V on the axes given below in Fig. 3.3


[1]- for plotting all the 6 points correctly.
[1] for the correct shape of graph.

(ii)

Determine the maximum power output Pmax of the cell from the graph
drawn in Fig. 3.3.
From the graph;
Maximum power = 54.25 mW [1]

(iiii)

Hence calculate Imax for this maximum power output.


Vmax = 460 mV
Imax = 54.25 x 10-3 / 460 x 10-3 [1]
= 0.118 A [1]

Preliminary Examination Answers

(ii)

Meridian Junior College


JC2 Physics 2008

Using the value of the maximum power output Pmax and any other data,
calculate the maximum efficiency of conversion of light energy into
electrical energy.
Pinput = 1100 4.0 104 [1]

Emax =

Pmax
100%
Pinput

Pmax
100%
Pinput

54.25 10 3
100%
1100 4.0 10 4

[1]

= 12. 3 % [A1]
[3]
Maximum Efficiency = .. %
(c)

Another different type of solar cell produces an output power of


250 mW at an output potential of 500 mV.
Determine how you can obtain a maximum output power of
approximately 0.75 W at 1.0 V using a number of these photovoltaic
cells in a suitable arrangement. Draw the arrangement in the space
provided below.

Considering power only


For each cell,
Pmax = 250 10 3 W
However required Ptotal = 0.750W and you will need three cells of
0.250 W each to obtain 0.750 W.
Considering V only
Required Voltage = 1.0 V

1.0
=2
500 10 3
Each cell at 500 10 3 V at max power, 1.0 V can be obtained with
2 cells in series. Therefore two of the cells (out of 3 cells
needed to obtain the required power) have to be in parallel.
[1] : To realise you need 3 cells to obtain P =0.750 W and two of
the cells need to be in parallel and arrange in series with the third
cell.

Preliminary Examination Answers

Meridian Junior College


JC2 Physics 2008

[2]
Arrangement:
For drawing of arrangement.
[1]

1.0 V, 0.750 W

0.50 V,
0.250 W

0.50 V, 0.500 W

(a)

(i)

Define the magnetic flux density of a magnetic field.


The magnetic flux density of a magnetic field is defined as the force
per unit length per unit current on a wire placed at right angle to the
field. [B1]
[1]

(ii)

Hence, or otherwise, show that the units of the flux density can be
N s C-1 m-1.

B=

F
, or F = BIL
LI

[1]

since unit of Force is N, unit of current is Cs-1 and unit of length is


m, therefore, units of flux density can be N s C-1 m-1 [1]
Unit of B

[2]

= unit of F
LI
=

N
[B1]
m(Cs 1 )

= N s C-1 m-1

Preliminary Examination Answers

(b)

Meridian Junior College


JC2 Physics 2008

A twin core electrical cable shown in figure 3.1 is used to supply a current of
0.50 A to a household lamp. The distance between the centres of the wires is
0.50 cm.
0.50 cm
Electrical
insulation

Wires

Figure 3.1

(i)

Calculate the force per unit metre between the wires, given the
magnetic flux density formula is B =

0 I
, where d is the distance
2d

between the two wires.


The magnetic flux density experience by one conductor due to the
other is calculated by B =
Thus

B=

0 I
.
2 d

0 I 4 x107 x0.50
=
2 d 2 x0.50 x102 [1]

= 2.0 x105 T
F
= BI
l
Therefore = 2.0 x10 = 5 x 0.50 [1]
= 1.0 x10 5 Nm 1 [1]
[3]
(ii)

Given that the current in both wires are moving in the opposite
direction, draw the magnetic forces acting on the wires, clearly
indicating the direction of the current in the wires.

The two forces must be in opposite direction and magnitude must


be the same. [1]
[1]

Preliminary Examination Answers

(i)

Meridian Junior College


JC2 Physics 2008

Determine the ionization energy of hydrogen in Joules.

Ionization energy = 13.6 x 1.6 x 10-19 [M1]


= 2.18 x 10-18 J [A1]

(ii)

[2]

Ionization energy= .. J
Hence find the maximum wavelength of photon that the hydrogen
atom needs to absorb to have an electron at n=1 to be ionized.

2.18 x 10-18 = h

[1]

= 9.10 x 10-8
[1]

[2]

Wavelength = . m
(i)

Explain how absorption lines are produced by the hydrogen


atoms.

Energy level in the hydrogen atoms are quantized or hydrogen atoms


have discrete energy elvels. [1]
When electromagnetic radiation passes through the hydrogen gas, photons
of energy corresponding to the exact difference in energy between two
energy levels in the atom are absorbed and are dispersed when the atom
de-exites. [1] This results in lines missing from the spectrum after the
radiation passes through the gas.
(ii)

Sketch a labelled graph of intensity against wavelength for


emission spectrum of hydrogen in the space provided below.

Intensity

wavelength
410 434 486

656

[1] for 4 peaks and labelling all wavelengths.


[1]

Preliminary Examination Answers

(a)

Meridian Junior College


JC2 Physics 2008

Define momentum.

[1]

Momentum is the product of mass and velocity.


(b)

A ping-pong (table-tennis) ball and a bowling ball are rolling in the same
direction on a frictionless surface. Both balls have the same
momentum.
(i)

State which of the balls has a higher velocity.


Ping-pong ball [1]

(ii)

[1]

If the same external force, F, is exerted on both balls individually


to stop the balls, comment on the time intervals to stop the pingpong ball and the bowling ball. (Which will take a longer time or
shorter time to stop? Or will they take the same time to stop).

[2]

From (a), we know that the force is equal to the rate of change of
m(v f -v i ) m v
momentum of a body (or object), i.e., F =
=
(t f -t i )
t
Since we applied the same force to both the ping-pong ball and
the bowling ball, the two balls would experience the same rate of
change of momentum, i.e., both of them are losing momentum at
the same rate. [1]
Henceforth, if both balls have the same momentum initially, it will
take the same amount of time to stop each of them.
[1]
(ii)

Show that the deceleration of the bowling ball aB as it stops is


given by aB =

MP
aP
MB

where ap is the deceleration of the ping-pong ball, MP is the mass


of ping-pong ball and MB is the mass of the bowling ball.
external force on bowling ball = M B aB
external force on ping pong ball = M P aP [1]
Since the same external force is applied on them;
M P aP = M B aB [1]
hence,

aB =

MP
aP
MB

[2]

Preliminary Examination Answers

(iv)

Meridian Junior College


JC2 Physics 2008

Sketch a graph to show how the velocity of the ping-pong ball and
bowling ball changes with time from the moment the external force
F is applied on each ball to the time when they stop. Label the
graph of the ping-pong ball, P and the graph of bowling ball, B.

velocity

P
B
time
[1] starting velocity of P higher than B drawn.
[1]- B having a lower deceleration than P and both B and P
stopping at the same time.
(v)

Hence state which of the balls will have a longer stopping


distance.

[1]

Ping pong ball cover a larger distance [1]

(c)

(i)

State the conservation of momentum.


[2]
The principle of conservation of momentum states that the total
momentum of a system of objects remains constant provided no
resultant external force acts on the system.

(ii)

Complete the graph of the bowling ball for t >0 s.

[2]

Drawing shows
[1] final momentum of bowling ball approximately 7 kg ms-1
[1] time for collision approximate 1.5 s
(iii)

Calculate the average force experienced by the bowling ball


during the collision.
Average force = 8 (-3) / 1.5 [1]
= 7. 33 N [1]

10

[2]

Preliminary Examination Answers

Meridian Junior College


JC2 Physics 2008

(d)
(i)

Show that the new length of string between toy car and pulley, l,
in Fig 6.2 (b) is 50.0 cm

[1]

l = 50.0 cm
30.0 cm
y = 30.0 cm

20.0 cm
x = 40.0 cm

l = (y2 + x2) = (302 + 402)


= 50.0 cm

(ii)

[1]

Hence calculate the height h at which the bowling ball raised.


h = (302 + 402) - 30 [M1] = 20.0 cm [1]
The original length of the string is 30.0 cm. After the car moved a
distance of 40.0 cm in the x-direction, the length of the string increased
to 50.0 cm. There is an increment of 20.0 cm. This 20.0 cm increment
is also the same height gained by the bowling ball.

(iii)

Calculate the gain in gravitational potential energy of the bowling ball


given that mass of bowling ball is 1.50 kg.

gain in g.p.e of bowling ball


= mass of bowling ball g increment in height
= (1.5)(9.81 m s-2)(0.20 m)
[1]
= 2.94 J

11

[1]

Preliminary Examination Answers

(i)

Meridian Junior College


JC2 Physics 2008

Explain why the wire is observed to vibrate when an alternating


current is passed through it.
The current carrying wire experiences a magnetic force [1] in the
vertical plane. Since the current is alternating and changes direction,
the direction of force alternates and causes the wire to vibrate. [1]

[2]
(ii)

Given that the horizontal length kept taut between M and P is 0.6 m,
determine wavelength of the stationary wave.

0.6 =

2
= 1.20 [1]
[1]
wavelength = m

(iii)

Determine the speed of the wave travelling in the wire.


V = f = (10)1.20 [M1] = 12.0 [1]

speed = m s-1
[2]

12

Preliminary Examination Answers

(iv)

Meridian Junior College


JC2 Physics 2008

Sketch the stationary wave formed when the frequency of the


generator is adjusted to 30 Hz in the space provided below.
[1]- correct waveform drawn for stationary wave. (dotted)
[1]- drawn stationary wave with 3 antinodes and 4 nodes, with 2 of the
nodes at M and P.
[2]

(i)

Given that the distance from the fourth bright fringe on one side of the
pattern to the fourth bright fringe on the other side of the pattern is
found to be 12.8 mm, calculate the fringe separation of the
interference pattern.
8x = 12.8 mm [1]
x = 1.6 mm = 1.6 x 10-3

Fringe separation = m

(ii)

[2]

Determine wavelength of the light.

x=

D
a

1.6 103 =

(2.0)
0.5 103

[2]

[1]

4 x 10-7 m [1]
wavelength = m

(iii)

Explain why single slit so is needed.


Single slit is needed to produce a coherent source so that the rays
that reaches the double slits are coherent.
[1]

(iv)

State the experimental advantage in determining the fringe separation


in the way used in (iii).
Random error is reduced with the method in (iii) [1]
[1]

13

Preliminary Examination Answers

(v)

Meridian Junior College


JC2 Physics 2008

State and explain what change will be observed, if any, in the


interference pattern when each of the following changes is made
separately.
1. increasing the distance between the double slit and the screen.

From x =

D
a

, when D is increased, the fringe separation will

increase. [1] No change in the contrast of the bright and dark fringes.
[1]

[2]

2. white light is used instead of monochromatic light source.


When white light is passed through, a spectrum of light [1] of the
different colours will be obtained , with the central fringe being white.
[1]
[2]
(i)

What does the above observation tell you about the moving helium
atoms?
The atoms have a wave-like property. [1]
[1]

(ii)

Given that the wavelength required to produce such a pattern is


approximately 1 x 10-10 m, calculate the speed of the helium atoms
given that the mass of the atoms is 6.6 x 10-27 kg.

p=

mv =

6.6 1027 v =
[1]

h
1.0 1010
[2]

v = 1.00 x 103

m s-1

[1]

speed = m s-1

14

Preliminary Examination Answers

(a)

(i)

Meridian Junior College


JC2 Physics 2008

Distinguish between the e.m.f. of a battery and p.d. across a


resistor in terms of energy considerations.

E.m.f. of a source is the ability of the source to generate


electrical energy from other forms of energy and to supply
it to the circuit. [1]

whereas p.d. across a part of a circuit or device, measures


the ability of the device or part of a circuit at converting
electrical energy to other forms of energy. [1]
[2]

(ii)

Two equations for the power P dissipated in a resistor are P = I2R


and P = V2/R. The first suggests that the greater the resistance R
of the resistor, more power is dissipated. However, the second
equation suggests that the greater the resistance, less power is
dissipated. Is there inconsistency? Explain your answer.
Since the same current is flowing through the resistor irregardless of
which formula we use, [1]
the voltage drop V across resistor increases when resistance increases
because of V = RI, [1]
If we substitute V2 = (RI)2, back into the P = V2/R, we would get the
same conclusion as P = I2R . Hence there is no inconsistency. [1]

(b)

If student just answer yes or no without justification, no credit.


A 230 V electric kettle transfers 7.0 x 105 J of energy in 5.0 minutes.
(i)

[3]

Calculate the current in the kettle.


E = VIt

7.0 x 105 = 230 x I x 5.0 x 60


I = 10 A.
(ii)

(iii)

[1]

[1]

[2]

Calculate the power rating of the kettle.


P = work done per unit time
= 7.0 x 105 / (5.0 x 60)
[1]
[2]
= 2300 W or 2330 W.
[1]
Show that the effective resistance of the heating coil in the kettle
is 23 .
R = V/I = 230/10 = 23
Or
P = I2R or V2/R,
R = 23
[1]

[1] for the correct substitution.


15

Preliminary Examination Answers

(b)

Meridian Junior College


JC2 Physics 2008

A cylindrical copper rod is resting freely on two horizontal smooth


conducting rails which can be connected to a battery to drive a current
through AB. A magnetic flux density of 0.55 T acts perpendicularly into
the paper. The setup is shown in fig 8.2. The battery has an e.m.f. of
24.0 V with negligible internal resistance. The value of the resistor R1 is
6.0 . The copper rod has a mass of 40 g. The resistance of the wires,
conducting rails and copper rod are negligible.

C
0

Conduction
rail

15.0 cm

24.0 V
A
D
R1 = 6.0

Copper rod

Figure 8.2 : Top View

Copper rod

R1

Conduction
Rail
D

Figure 8.3 Side View


(i)

Draw a clear diagram to illustrate the direction of the magnetic


force F on the conductor when a current, I is flowing through the
copper rod, when the switch is closed. The magnetic field, B acts
perpendicularly into the paper.

I
F

[1] for labelling the force and current.


[1] for correct direction of current and force relatively.
16

[2]

Preliminary Examination Answers

Meridian Junior College


JC2 Physics 2008

[2]
(ii)

When the switch is closed, a current flows through the copper rod.
Calculate the value of this current.
V = RI
I = 24/6.0
= 4.0 A

(iii)

[1]
[1]

[2]

Calculate the magnetic force on the copper rod AB


Fm = BIL
= 0.55 x 4.0 x 0.15
[1]
= 0.33 N
[1]
Allow ECF is current from part (bi) is wrong.

[2]

(iv)

The conduction rails must be tilted to keep the copper rod


stationary when the switch is closed. State which ends of the rail
need to be raised. (AB or CD)
CD [1]

(v)

The conduction rails must be tilted to keep the copper rod AB


stationary, when the switch is closed. Draw the free body diagram
of this setup.
[1] Both forces must be in the correct directions
ECF can be given for Fm from part (bii) since this
N
part tests their knowledge of FBD

[1]

[1] The length of Fg must be longer than the


length of Fm.

Fm

[1] Normal force, N


Fg

17

[3]

Subject Class
2PH1A

Registration Number

Name

NATIONAL JUNIOR COLLEGE


JC 2 PRELIMINARY EXAMINATION
PHYSICS

8866/01

Higher 1

9 September 2008
1 hour

Paper 1 Multiple Choice


Additional Materials:

Multiple Choice Answer Sheet

READ THESE INSTRUCTIONS FIRST


Write in soft pencil.
Do not use staples, paper clips, highlighters, glue or correction fluid.
Write your name, Subject Class and Registration Number on the Answer Sheet in the
spaces provided unless this has been done for you.
There are thirty questions on this paper. Answer all questions. For each question there
are four possible answers A, B, C and D.
Choose the one you consider correct and record your choice in soft pencil on the
separate Answer Sheet.
Read the instructions for filling up the Answer sheet carefully.
Each correct answer will score one mark. A mark will not be deducted for a wrong answer.
Any wrong working should be done in this booklet.
Instructions for filling up the Multiple Choice Answer Sheet

This document consists of 10 printed pages including this cover page.


NJC2008

8866/01/08

[Turn over

Data

speed of light in free space,

c = 3.00 x 108 ms-1

permeability of free space,

0 = 4 x 10-7 Hm-1

permittivity of free space,

0 = 8.85 x 10-12 Fm-1

elementary charge,

e = 1.60 x 10-19 C

the Planck constant,

h = 6.63 x 10-34 Js

unified atomic mass constant,

u = 1.66 x 10-27 kg

rest mass of electron,

me = 9.11 x 10-31 kg

rest mass of proton,

mp = 1.67 x 10-27 kg

molar gas constant,

R = 8.31 JK-1mol-1

the Avogadro constant,

NA = 6.02 x 1023 mol-1

the Boltzmann constant,

k = 1.38 x 10-23 JK-1

gravitational constant,

G = 6.67 x 10-11 Nm2kg-2

acceleration of free fall,

g = 9.81 ms-2

Formulae

1 2
at
2

uniformly accelerated motion,

s = ut +

work done on/by a gas,

W = p V

hydrostatic pressure

p = gh

gravitational potential,

displacement of particle in s.h.m.,

x = x0 sin t

velocity of particle in s.h.m.,

v = v 0 cos t and v = x 0 x 2

resistors in series,

R = R1 + R2 +

resistors in parallel,
electric potential,

Gm
r
2

1
1
1
=
+
+ ...
R R1 R2
Q
V =
4 0 r

alternating current/voltage,

x = x0 sin t

Transmission coefficient

2
T = ex p(-2kd) Where k = 8 m(U E )
2

radioactive decay,

x = x0 exp (-t)

decay constant,

0 . 693
t1
2

NJC2008

8866/01/08

1.

The ability for a coil of wire to have an current induced in it due to the presence of a
changing current is measured by its inductance, L. Given that a coil (or circuit) has a large
inductance, a small rate of change of current, dI

dt

, in it will induce a large e.m.f, . The

equation relating the inductance of the coil to the induced e.m.f and the rate of change of
current is given as

L=

dI

The SI unit for L is


A

2.

kgms-1A

kgm2s-2A-2

kgms-2A-1

(20.579 0.009) cm3


(20.58 0.03) cm3
(20.6 0.1) cm3
(20.6 0.2) cm3

An object of mass 2.0kg moves at a speed of 3.0 ms-1 at a bearing of 120 (at a clockwise
angle of 120o with respect to the North). A force of 1.0 N, at a bearing of 060, acts on the
object for 2.0 s. What is the final velocity of the object?
A
B
C
D

4.

A student measured the diameter of a ping pong ball as 3.40 cm using a pair of vernier
calipers. The uncertainty in the measurement is 0.01 cm. How should the total volume of
the ping pong ball be expressed?
A
B
C
D

3.

kgm2s-2A-1

dt

3.6 ms-1 at a bearing of 106


3.6 ms-1 at a bearing of 286
2.6 ms-1 at a bearing of 161
2.6 ms-1 at a bearing of 341

60

1.0 N

120
3.0 ms-1

A toy rocket is launched vertically from Earth with a constant acceleration. After some time,
the fuel is used up and the rocket falls freely back to Earth. Which one of the velocity-time
graphs best represents the journey? (Ans: A)

NJC2008

8866/01/08

5.

A cart is released from rest at point A of the track with side view as shown in Figure 5.

Figure 5

The cart makes dots at a constant rate on a strip of paper laid along the track as the cart
moves from point A to B. The tape is then removed from the cart and laid flat. In each
picture below, the tape is oriented so that the first dot that was made on the left. Vertical
grid lines have also been added to help you gauge the distance between the dots.
Neglecting friction, which of the pictures below would most possibly represent the pattern of
dots that is made on the tape? (Ans:A)

6.

A missile of mass 2.0kg was released from an airplane flying horizontally at a speed of 20
ms-1. Find the kinetic energy of the missile after 5.0 s.
A

7.

53 J

400 J

2406 J

2806 J

A space shuttle of mass m re-enters the Earths atmosphere at an angle to the horizontal.
Air resistance acting on the space shuttle causes it to travel at constant speed v.
The heatshield of the shuttle dissipates heat at a rate of P, so that the mean temperature
of the shuttle remains constant. Taking g as the relevant value of acceleration due to free
fall, which expression is equal to P?
A
C

8.

mgv
1 2
mv
2

B
D

mgv sin
1 2 2
mv sin
2

At take-off, the motor blades of a helicopter propel 2400 kg of air vertically downwards in
one second. The air initially at rest, is given a speed of 11.0 ms-1. The mass of the
helicopter is 2500 kg. What is the acceleration of the helicopter, as it starts to rise?
A

0.0400 ms-2

0.750 ms-2

9.56 ms-2

10.6 ms-2

NJC2008

8866/01/08

9.

A bullet of mass 20 g is fired horizontally from a rifle at a lead block of uniform mass 500 g
which is resting on a platform. The bullet hits the centre of the block with a velocity of 230
ms-1, and the impact causes the block to slide along the platform. If the bullet remains
embedded in the block after impact and the block experiences a constant frictional force of
30 N throughout its motion, how far away from its initial position does it stop?

230 ms-1

?
A 0.68 m

10.

D 17.6 m

19.6 N

8.84 kN

8.86 kN

8.88 kN

Two boys, Ah Hock and Ah Seng, both wearing roller skates and initially at rest, push each
other apart on a smooth surface. Ah Hock has a mass of 50 kg and Ah Seng has a mass of
70 kg. After the boys push each other apart, Ah Hock has a speed of 6.0 ms-1. As the boys
move apart, Ah Seng has
A
B
C
D

12.

C 15.8 m

An object of mass 2.0 kg is released at a height of 1.0m. When it hits the ground, it took
1.0 ms to come to rest. What is the force of impact on the object?
A

11.

B 0.71 m

less kinetic energy than Ah Hock


more kinetic energy than Ah Hock
less momentum than Ah Hock
more momentum than Ah Hock

Two snow-trucks tow a housing unit at constant speed to a new location in the Antarctica
as shown below. The sum of forces FA and FB exerted on the unit by the horizontal cables
is parallel to the line L, and FA = 4500 N. Determine FB.

L
30o
50o

FB

FA
Housing
Unit

A 1700 N
NJC2008

B 2500 N

Top
View

C 6900 N
8866/01/08

D 9000 N

13.

A cup is filled to the brim with water and a floating ice cube. Resting on top of the ice cube
is a small stone. When the ice melts, which of the following occurs?

A
B
C
D

14.

Water overflows the cup.


The water level decreases.
The water level remains the same.
Not enough information given to make a conclusion.

A ball of mass m = 0.260 kg, starting from rest falls a vertical distance h = 55.0 cm before
striking a vertical coiled spring which it compresses an amount x =15.0 cm. Determine the
spring constant of the spring. Assume the spring has negligible mass, remains stationary
and obeys Hookes Law.
A
B

11.9 Nm-1

125 Nm-1

15.

9.35 Nm-1

159 Nm-1

Two waveforms X and Y are displayed on a Cathode Ray Oscilloscopes screen. Which of
the following statements is correct?

A
B
C
D

NJC2008

X Y has a phase difference of / 4.


X Y has a phase difference of / 2.
XY has a path difference of / 2.
XY has a path difference of .

8866/01/08

16.

Light from an unpolarised source is allowed to fall on a piece of Polaroid P and then on a
second Polaroid Q. In the position shown in the diagram, the intensity of the light emerging
from Polaroid Q is a maximum. As Q is slowing rotated, the intensity of light emerging is
reduced to half the maximum value at angle 1 and to a minimum at angle 2.

Q
Which of the following gives the correct values of angles 1 and 2 ?

A
B
C
D

17.

It refers to the speed of the stationary wave.


It refers to the speed of the particles in the stationary wave.
It refers to the speed of the individual progressive wave that forms the stationary
wave.
It refers to the rate at which energy is transferred in the stationary wave.

Consider a tube of length 32.8 cm which is open at both ends. When a vibrating tuning fork
is held close to one end of the tube, a loud sound is detected. If the velocity of sound in air
is 345 ms-1, what is the frequency of the fork?
A
B
C
D

19.

2
90o
90o
180o
90o

In what way is the symbol v in the equation v = f , where the symbols have their usual
meanings, relevant to a stationary wave?
A
B
C

18.

1
30o
45o
45o
60o

789 Hz
526 Hz
263 Hz
175 Hz

White light diffracted by a double slit falls on a white screen. Which of the following
statements is true?
A
B
C
D

NJC2008

The central band is white.


The centre of the diffraction pattern is white.
The first maximum is closer to the centre for red light than for blue light
The first minimum is closer to the centre for red light than for blue light.

8866/01/08

20.

Figure 20 represents a longitudinal wave travelling from the left to the right at a frequency
of 100 Hz. Two particles in the medium are labelled X and Y.

Figure 20
If the distance between X and Y is 100 m, what is the speed of the wave in the medium?
A 2000 ms-1

21.

B 2222 ms-1

C 3333 ms-1

D 4000 ms-1

In the circuit shown, filament lamps L1 and L2 are identical and are seen to be marked 6V,
18W.

Which of the following statements is true?


A
B
C
D

When switch S is closed, lamp L1 and lamp L2 glow with less than its normal
brightness.
When switch S is closed, lamp L2 glows with its normal brightness.
When switch S is opened, lamp L1 glows with its normal brightness.
When switch S is opened, lamp L1 glows with more than its normal brightness.

22.

In the circuit above, the equivalent resistance across XY is


A

NJC2008

10

8866/01/08

40

64

23.

In the circuit shown below, a 10.0 V power supply with negligible internal resistance is
joined to two 30 k resistors and an unknown resistor R in series. A voltmeter of resistance
120 k is connected in parallel with one of the resistors.
10.0 V

R
30 k

30k

What is the value of R if the voltmeter reads 3.0 V?


A

24.

40 k

78 k

320 k

Two parallel wires repel each other with a force F when the same current passes through
them. If the current is doubled and the distance between the wires is also doubled, the
force of repulsion will then be
A

25.

26 k

F /4

F /2

2F

4F

In the diagram below, P is a circular coil of wire with its plane horizontal carrying a current I.
QR is a conducting rod placed within the coil, supported by 2 horizontal parallel rails which
are perpendicular to QR and carry a current I as shown.
P
Q
R

I2

I1
The rod QR will experience
A
a magnetic force along the direction QS.
B
a magnetic force along the direction TR.
C
a magnetic force perpendicularly upwards from the plane of the coil.
D
no magnetic force at all.

26.

The energy of dissociation (i.e. to break up) of AgBr is 1.00 10 5 J mol-1. Find the
wavelength of a photon that is just able to dissociate a molecule of silver bromide.
A

2.00 10 30 m

1.20 10 6 m

8.35 10 5 m

5.03 10 29 m

NJC2008

8866/01/08

10

27.

Two beams, P and Q, of light of the same wavelength, fall upon the same metal surface
causing photoemission of electrons. The photoelectric current produced by P is four times
that produced by Q.
What is the ratio

A
28.

Wave amplitude of beam Q


B

at the metal surface ?

The diagram shows some energy levels of a certain atom. Transition X results in the
emission of a photon of wavelength 600 nm. Which transition (Q to T) would result in the
emission of a photon of wavelength 300 nm?

29.

Wave amplitude of beam P

When a beam of white light passes through iodine vapour, the spectrum of the emergent
light shows dark lines. Which of the following statements is true?
A
B
C
D

The iodine vapour absorbs from the light all frequencies except those which it emits.
The iodine vapour emits less energy than it absorbs.
The iodine vapour absorbs the same frequencies as it emits.
The iodine vapour causes diffraction of the white light.

30.
Figure 30
X
Y
Figure 30 above shows two adjacent lines in the spectrum of a hot gas. Line X is brighter
than line Y because
A
B
C
D

line X has a higher frequency


line X has a greater wavelength
for line X, more electrons undergo transitions between the two states involved in the
emission,
for line X, electrons undergo transitions between two states of greater energy
difference.
END of PAPER ONE

NJC2008

8866/01/08

Subject Class
2PH1A

Registration Number

Name

NATIONAL JUNIOR COLLEGE


JC 2 PRELIMINARY EXAMINATION
PHYSICS

8866/02

Higher 1

9 September 2008
2 hours

Paper 2 Structured Questions


Candidates answer on the Question Paper.
No Additional Materials are required
READ THESE INSTRUCTIONS FIRST
Write your Subject Class, Registration Number and Name on all the work you hand in.
Write in dark blue or black pen on both sides of the paper.
You may use a soft pencil for any diagrams, graphs or rough working.
Do not use staples, paper clips, highlighters, glue or correction fluid.
Section A
Answer all questions.
Section B
Answer any two questions.
The number of marks is given in brackets [ ] at the end of each question or part question.
You may lose marks if you do not show your working or if you do not use appropriate units.

For Examiners Use


Section A
1
2
3
4
5
Section B
6
7
8
Total
This document consists of 19 printed pages including this cover page.
NJC2008

8866/02/08

[Turn over

Data

speed of light in free space,

c = 3.00 x 108 ms-1

permeability of free space,

0 = 4 x 10-7 Hm-1

permittivity of free space,

0 = 8.85 x 10-12 Fm-1

elementary charge,

e = 1.60 x 10-19 C

the Planck constant,

h = 6.63 x 10-34 Js

unified atomic mass constant,

u = 1.66 x 10-27 kg

rest mass of electron,

me = 9.11 x 10-31 kg

rest mass of proton,

mp = 1.67 x 10-27 kg

molar gas constant,

R = 8.31 JK-1mol-1

the Avogadro constant,

NA = 6.02 x 1023 mol-1

the Boltzmann constant,

k = 1.38 x 10-23 JK-1

gravitational constant,

G = 6.67 x 10-11 Nm2kg-2

acceleration of free fall,

g = 9.81 ms-2

Formulae

1 2
at
2

uniformly accelerated motion,

s = ut +

work done on/by a gas,

W = p V

hydrostatic pressure

p = gh

gravitational potential,

displacement of particle in s.h.m.,

x = x0 sin t

velocity of particle in s.h.m.,

v = v 0 cos t and v = x 0 x 2

resistors in series,

R = R1 + R2 +

resistors in parallel,

1
1
1
=
+
+ ...
R R1 R2
Q
V =
4 0 r

electric potential,

Gm
r

alternating current/voltage,

x = x0 sin t

Transmission coefficient

2
T = ex p(-2kd) Where k = 8 m(U E )
2

radioactive decay,

x = x0 exp (-t)

decay constant,

0 . 693
t1
2

NJC2008

8866/02/08

[Turn over

3
Section A

Answer all the questions in this section.


1. When an object moves relative to a fluid, the fluid exerts a retarding force on the object. This drag
force F is due to the viscosity of the fluid. Under non-turbulent conditions, the drag force on a
sphere moving in a tube of fluid is given by
F = 6rv
where r is the radius of the sphere, is the viscosity of the fluid and v is the velocity of the sphere.
(a) Show that the base units of viscosity are kg m-1 s-1.

[1]

(b) A sphere of diameter (2.0 0.1) cm falls under non-turbulent conditions through a fluid of
viscosity (0.13 0.02) kg m-1 s-1. Using a ruler and a stopwatch, a student from 2ph2I measured
the velocity through the liquid to be 2.7 m s-1 and estimates that the percentage uncertainty in
this measurement to be 5%.
Determine the drag force F acting on the sphere? State your answer with its uncertainty.

(i)

F = ..N

[3]

State 1 source of error in the measurement of velocity and suggest an improvement to


overcome this error.

(ii)

[2]
2. (a) State 2 assumptions that you usually make in solving a projectile motion problem near the
surface of the earth?
1.

jh

2.

jh
[2]

NJC2008

8866/02/08

[Turn over

4
2. (b) A projectile is fired at a falling target as shown in the above figure. The projectile leaves the gun
at the same instant that the target is dropped from rest. Neglect air resistance. Assume that the
gun is initially aimed at the target.

(i)

Write down the expressions to show the vertical displacement and horizontal
displacement of the projectile at the point of collision.

[2]

(ii)

Write down an expression to show the vertical displacement of the target which reaches
the point of collision.

[1]

(iii)

Hence show that the projectile will hit the target.

[3]

3. (a) Explain what is meant by moment of a force.

[1]
NJC2008

8866/02/08

[Turn over

5
3. (b) The telescoping arm ABC is used to provide an elevated platform for construction workers. The
workers and platform together have a mass of 204 kg and their combined centre of gravity is
located directly above C. Given that the direction of the force exerted on the telescopic arm
ABC at B by the single hydraulic cylinder BD is in the direction from D to B, find the force on the
arm at A for the position when = 20o. This is shown in Fig. 3.1. You may neglect the weight of
the telescopic arm in your calculations.

1.72m

Fig. 3.1

1.76 m

Force at arm A = ..N [6]

NJC2008

8866/02/08

[Turn over

6
4. (a) A battery of e.m.f. 10.0 V and internal resistance 0.500 is connected to a resistor of
resistance 4.50 . Determine

(i)

the current in the circuit,

Current = .. A [1]
(ii)

the potential difference across the 4.50 resistor and

Potential difference = .. V [1]


(iii)

the power supplied to the 4.50 resistor.

Power = .. W [1]
(b)

Another battery of e.m.f. 2.00 V and negligible internal resistance is inserted in series within
the circuit, but in the reverse polarity to the original battery. What is the power dissipated by
the internal resistor of the original battery?

Power = .. W [2]
NJC2008

8866/02/08

[Turn over

7
4. (c) In the figure 4.1 below, the internal resistance of the 4.0 V cell is 2.00 .

12.0 V driver cell


(with negligible internal resistance)

R1

R2

contact
C

B
4.0 V cell

centre zero
milliammeter
Figure 4.1

(i)

Determine the ratio of R1 to R2 where the ammeter will show zero deflection.

R1
= .. [1]
R2
(ii)

Now, the contact is placed at position D. State the deflection in the milliameter and
calculate the current flowing through BC.

Deflection = ..
Current = .. A [3]
(iii)

If R1 and R2 represent filament bulbs, calculate the potential difference, V AD across


both R1 and R2 if the filament bulb R2 is spoilt and causes a break in the circuit.

V AD = .. V [1]
NJC2008

8866/02/08

[Turn over

8
5. (a) State two conditions for a charged particle to feel a force in a magnetic field.

1.

jh

2.

jh
[2]

(b) In 1991, Japan successfully completed the building of the first working prototype of a magnetohydrodynamic propulsion (or electromagnetic propulsion) boat, Yamato 1. The boat is propelled
by applying a magnetic field to seawater which causes the induction of a magnetic force. This
eliminates the need of screw propellers which are noisy and can be easily picked up by sonar
operators.
+ ve

30.0 cm

10.0 cm
y

- ve
Figure 5.1
Picture of Yamato 1

Figure 5.2
Simplified picture of electromagnetic propulsion tube

Figure 5.2 is a simplified picture of the electromagnetic propulsion tube engine. A pair of very
powerful electromagnets is placed across the propulsion tube. Seawater is propelled through
the tube when the engine is switched on. The cross-section of the tube is a rectangle of side
30.0 cm by 10.0 cm.
(i)

Draw in the Figure 5.2, the direction of the current flow when the engine is on. Label it
as I.
[1]

(ii)

State the property of sea water that allows current to be conducted.

[1]
(iii)

In order to propel the seawater towards the right in the tube (in the positive x-direction),
state the direction of the magnetic field in the tube.

[1]

NJC2008

8866/02/08

[Turn over

(iv)

9
Given that the current supplied is adjusted to be 10.0 A, calculate the magnetic flux
density required to provide an average output force of 16.0 kN.

Magnetic flux density = ..T [2]


(v)

Discuss, qualitatively, a possible disadvantage of this propulsion technology.

[2]

NJC2008

8866/02/08

[Turn over

10
Section B

Answer two questions from this section.


6. (a) A golfer stands on an extensive area of level ground and practice his swing. The golfer has a
mass of 80 kg and golf balls have mass 40 g. A golf ball is struck hard by the club begins its
flight with speed 60 ms-1 and an angle of 20o to the ground. Ignore the effects of air resistance
in the following calculations.
(i)

Calculate
1.

the impulse applied to the ball by the club,

Impulse = ..N s [2]


2.

the time for which the ball is in the air,

Time = .. s [2]
3.

the distance from the golfer to the point where the golf ball first returns to the
ground.

Distance = ..m [1]


(ii)

NJC2008

Draw and label a free body diagram showing the forces which act on the golf club at the
moment the golfer strikes the golf ball. Provide suitable explanation.
[3]

8866/02/08

[Turn over

6. (b) (i)

11
Two particles, M1 with a speed of 3.0 m s-1 and M2 with a speed of 5.0 m s-1, travel
towards each other and collide head-on. M1 rebounds with a speed of 10 m s-1. If the
total kinetic energy is unchanged after the collision, find the final velocity of M2 using the
conservation laws.

Velocity = ..m s-1 [5]


(ii)

A child of mass 40 kg stand at one end of a plane of mass 20 kg and length 6.0 m. The
plank is situated on perfectly smooth rollers as shown.

The child walks carefully to the other end of the plank with a velocity of 1.0 m s-1 to the
right relative to the floor. What velocity does this action give to the plank?

Velocity = ..m s-1 [3]


NJC2008

8866/02/08

[Turn over

12
6. (c) In each of the following situations, use Newtons laws of motion to explain whether or not a net
force is acting on the body. If no net force is acting, describe how the forces are balanced. If a
net force is acting, explain the origin of the net force and state its direction.

(i)

A raindrop which falls at terminal speed.

[2]
(ii)

A ping pong ball which collides obliquely (at an angle) with a smooth wall.

[2]

NJC2008

8866/02/08

[Turn over

13
7. (a) State the principle of superposition for waves.

[1]
(b) A student uses the apparatus shown in Figure 7.1 to determine the speed of sound in air. A
loudspeaker, L, is connected to the signal generator and a microphone, M, is connected to the
Y-input and the earth terminal of a CRO. The loudspeaker and the reflecting plate R are placed
on a metre rule (not shown) with the microphone M between them. The axis of the loudspeaker
is along the metre rule and perpendicular to the reflecting plate.

Figure 7.1

(i)

With the signal generator set at a certain frequency, briefly describe the procedures you
would follow to set up stationary wave between the loudspeaker and the reflecting plate.
[2]

(ii)

With the stationary waves formed, the microphone M is moved along the axis of the
loudspeaker and the trace on the CRO screen is seen to vary in size. Consecutive
positions of M are recorded whenever the trace has its smallest amplitude. The
procedure is repeated for different signal frequencies. The results are recorded in Table
7.1 below.

Frequency of signal
generator/ kHz

Positions of M corresponding to the


smallest amplitude of the CRO trace/ cm

2.500

27.0, 34.0, 41.0, 47.9, 54.5, 61.6, 68.7

2.941

16.0, 22.0, 28.1, 33.7, 39.6, 45.7, 51.5

3.571

18.7, 23.7, 28.4, 33.3, 38.4, 43.5, 48.1

4.545

31.8, 35.8, 39.8, 43.6, 47.4, 51.5, 55.5

6.250

26.4, 29.4, 32.4, 35.2, 38.2, 41.5, 44.7

NJC2008

8866/02/08

Wavelength /
cm

Period
T/s

[Turn over

7. (b) (ii)

14
1. Complete Table 7.1 and plot a graph of wavelength against period T.

[5]

2. Hence find the speed of sound in air.

Speed of sound in air = ..m s-1 [2]


NJC2008

8866/02/08

[Turn over

7. (b) (ii)

15
3. The experiment cannot be performed with sound frequencies of uniform intervals.
Explain.

[1]
4. The graph drawn in (b)(ii)1. does not pass through the origin. Explain.

[1]
(iii)

Suggest an advantage and a disadvantage of performing this experiment outdoor.

Advantage:

jh

Disadvantage:

jh

[2]
(c) When a monochromatic light strikes a thin film of soapy water nearly perpendicularly, it will be
reflected from the surface of the bubble (light ray 1). However refraction also occurs and some
light enters the film. Part of this light reflects from the bottom surface of the film and passes
back through the film, eventually re-entering the air (light ray 2). This results in interference of
light.
Ray 1

Ray 2

Thin Film of Soapy water

(i)

Use the principle of superposition to account for the observed phenomenon of


interference of light due to a thin film of soapy water.

[2]
NJC2008

8866/02/08

[Turn over

7. (c) (ii)

16
A soap bubble can be perceived as a hollow sphere made from a thin film of soapy
water with air in the middle. Under natural conditions, it is observed that the soap
bubbles have a multicoloured appearance that often changes while you are watching
them. Explain this observation.

[2]
(iii)

Suggest and explain briefly one practical use for light interference.

[2]

NJC2008

8866/02/08

[Turn over

17
8. (a) In 1914, James Franck and Gustav Hertz performed an experiment which demonstrated the
existence of excited states in mercury atoms, helping to confirm the quantum theory which
predicted that electrons occupied only discrete, quantized energy states.

Figure 8.1
Schematic diagram of Franck and Hertz apparatus

Figure 8.2
Graph of anode current Ia against V1

Electrons emitted at the cathode C are accelerated by a potential difference of V1 toward


a positively charged grid G, in a glass tube filled with mercury vapor at a pressure of about
100 N m-2. Beyond the grid is an anode A, held at a voltage of V2 of 1 V negative with respect to
the grid.
A graph of anode current Ia against V1 is shown in Figure 8.2.
The values of accelerating voltage where the current dropped gave a measure of the energy
necessary to force an electron to an excited state.
(i)

1. Account for the shape of the graph when V1 is between P and Q.

[2]
2. As shown in Figure 8.2, when the accelerating voltage reaches 4.9 V, the current
sharply drops, indicating the sharp onset of a new phenomenon which takes enough
energy away from the electrons that they cannot reach the collector. Suggest with
explanation what the new phenomenon is.

[2]
NJC2008

8866/02/08

[Turn over

8. (a) (i)

18
3. As the mercury atoms de-excites to their ground state, they emit radiation. Using the
values from the graph in Figure 8.2, find the wavelength of the radiation emitted by
the mercury atoms as they return to their ground state.

Wavelength = ..m [2]


4. Can the radiation calculated in (a)(i)3. be detected? Explain.

[2]
(ii)

1.

The energy level diagram shows that mercury has another excitation potential at
6.7 V. Why is this not shown by the graph in Figure 8.2?

[2]
2.

What would happen to a photon of energy 9.0 eV when it collides with a mercury
atom? Explain your answer briefly.

[2]
NJC2008

8866/02/08

[Turn over

19
8. (b) The Colleges electron diffraction tubes in the Physics Laboratories usually direct a beam of
electrons at a thin layer of graphite. The transmitted electrons form a diffracted pattern on the
end of the tube that is a series of concentric rings.

Filament

Anode

Graphite film

diffraction
pattern

Cathode
+V
(i)

Fluorescent screen

Explain the formation of the diffraction pattern.

[3]
(ii)

The atomic spacing of the thin layer of graphite is approximately to be 0.2 nm. What
should Ms. Ng (the laboratory technician) set the accelerating voltage to be?

Voltage = .. V [3]
(iii)

Explain qualitatively how the pattern will change if the accelerating voltage is increased?

[2]

NJC2008

8866/02/08

End

NATIONAL JUNIOR COLLEGE

PHYSICS DEPARTMENT

2008 JC2 H2 Physics Prelim Paper Solution


Paper 1
(1-5)
(6-10)
(11-15)
(16-20)
(21-25)
(26-30)

CDAAA
DBBAD
ACBDB
BCBBD
BBACA
BCACC

Paper 2
1(a) = F / 6rv
Units of = kg m s-2 / m m s-1 = kg m-1 s-1 (shown)

(b)(i)
F = 6rv = 6 (3.142) (0.13) (0.01) (2.7) = 0.06617 N

F r v
=
+
+
F

r
v
F
0.02 0.001
=
+
+ 0.05
0.06617052 0.13 0.01
F
= 0.15384 + 0.1 + 0.05
0.06617052
F = 0.02 N
F = (0.06 0.02) N

(ii)

Measurement of the timing might not be accurate as it will be hard to determine when
the object has passed the length use of light gates
Any other reasonable (workable) answers.

2. (a)
1. Air resistance is negligible.
2. The acceleration of free fall is uniform.
2. (b) (i)
Taking upwards as positive and the time of collision to be t. The displacement is considered to
be zero at the gun level.

1
s y = (vo sin ) t gt 2
2
sx = (vo cos ) t
(ii) Taking upwards as positive and the time of collision to be t. The displacement is considered
to be zero at the gun level and the targets initial vertical displacement is d.

1
s 'y = d gt 2
2
FOR INTERNAL USE ONLY

NATIONAL JUNIOR COLLEGE

(iii)

PHYSICS DEPARTMENT

Hence show that the projectile will hit the target.


For the projectile to hit the target, s y = s 'y . Which means by comparison, (vo sin ) t = d
Since sx = (vo cos ) t , t =

sx
vo cos

Since the horizontal displacement is sx and the vertical initial vertical displacement of
target is d, tan =

d
sx

(vo sin ) t = (vo sin )


3. (a)

sx
d
= sx tan = sx ( ) = d (Shown)
vo cos
sx

Moment of a force is the given by the product of the force and the perpendicular distance
measured from the pivot to the line of application of the force.

3. (b) Let tan = 1.72/1.76

= 44.3o

Let Rx and Ry be the horizontal and vertical component forces at A respectively.


Sum of forces vertically:
Fsin 44.3o + Ry = 2001
0.7 F + Ry = 2001 [1]

Sum of forces horizontally:


Fcos 44.3o = Rx
0.72 F = Rx [2]

Ry
y

B
2001 N

Taking moment about A:


2001 x 5 cos 20o +Fcos 44.3o x 0.82 = Fsin 44.3o x 2.26
9402 = 0.992 F
F = 9477 N = 9480 N (3 sf)

1.72 m

Equation [2]: 0.72 F = Rx


Rx = 6823 N = 6820 N (3 sf)

Rx
0.9 m

1.76 m

Equation [1]: 0.7 F + Ry = 2001


Ry = -4633 N = -4630 N (Ry should be point downwards instead)
The resultant force at A: R = (Rx2 + Ry2) = 8247 N = 8250 N
Tan = 4633/ 6823
R
The resultant force
o
Ry
acts at an angle of = 34.1 measure

clockwise with respect to the horizontal as shown


in the vector diagram.

2.26 m

Rx

4. (a)
Current in the circuit, treating r as internal resistance,
I = E/(R+r) = 10.00/(4.50 + .50) = 2.00 A
(ii) P.d across R,
= IR = (2.00)(4.50) = 9.00 V
(iii) Power supplied to R,
= VI = (9.00)(2.00) = 18.0 W
FOR INTERNAL USE ONLY

NATIONAL JUNIOR COLLEGE

PHYSICS DEPARTMENT

I = [E(original) E(new)]/(R+r)
= [10 2]/(4.50+0.50) = 1.6 A

(b)

Power dissipated by Internal Resistor = I2r = (1.62)(0.5)


= 1.28 W

4. (c)

12.0 V driver cell


(with negligible internal resistance)

R1

R2

contact
C

B
4.0 V cell

centre zero
milliammeter
Figure 4.1

R1
4 1
=
=
R2 12 3

(i)

(ii)
Towards right

I BC =
(iii)

12 4
= 1.5 A
2
12.0 V

5. (a) 1. The charge has to be non-stationary


2. The charge has to have a component of velocity that is non parallel to the direction of the
magnetic field.
(b)
(i)

Pointing downwards

(ii)
The sea water can be electrically charged.
(iii) In order to propel the seawater towards the right in the tube (in the positive x-direction), state
the direction of the magnetic field in the tube.
In should be in the positive z direction.
(iv)
FOR INTERNAL USE ONLY

NATIONAL JUNIOR COLLEGE

PHYSICS DEPARTMENT

F = BIL
B=

F 16 103
=
= 5.333 103 T
IL 10(0.30)

(v) A current of 10 A is very large, thus a large power supply is required. Power loss due to this
current will also be very high.
The large magnetic field required may interfere with the electronic equipment on the ship.
Section B
6. (a) (i)
1.
Impulse = m(v f vi ) = (0.04)(60 0) = 2.4 Ns
2.

v = u + at
0 = 60sin 20o (9.81)t
t=

60sin 20o
= 2.092s
9.81

time in the air = 2t = 4.184 s


3.

sx = (60 cos 20o )(4.1834) = 235.9m


(ii)

Force on club by
man

Force on club by
Earth
Force on club by
golf ball

6. (b) (i)
By conservation of linear momentum,

3M 1 5M 2 = 10 M 1 + M 2 v
M2 =

13M 1
v+5

By conservation of kinetic energy,

FOR INTERNAL USE ONLY

NATIONAL JUNIOR COLLEGE

PHYSICS DEPARTMENT

9M 1 + 25M 2 = 100 M 1 + M 2 v 2
M2 =

91M 1
91M 1
=
2
v 25 (v + 5)(v 5)

Hence,

13M 1
91M 1
=
v + 5 (v + 5)(v 5)
91
v5 =
13
v = 2ms 1

(ii)

By conservation of Linear Momentum,


0 = 40(1.0) + 20v
v = -2.0 ms-1

6. (c)
(i)
No net force as raindrop is falling at constant velocity.
The weight of the raindrop is balanced by the drag force on the rain drop (and the upthrust on
raindrop due to air).
(ii)
There is a net force on the ball.
During collision, the horizontal component of the net force is due to the impact on the wall.
Direction of the force is away and perpendicular to the wall
The vertical component of the net force is due to the Earths gravitational pull. Direction of the
force is towards the centre of Earth.
7. (a)
The resultant displacement caused by two waves arriving at a point is the vector sum of the
displacements caused by each wave at that point.
(b)
(i)
With the signal generator on, and the experiment set up as in Fig 7.1, shift the reflecting plate
along the axis until a varying signal can be found using the microphone. Make fine adjustments
to the position of the reflecting plate so that there are regions of resonance and nodes.
With the ruler, measure the distance the microphone need to move to locate two consecutive
loud signals.
7. (b) (ii)

1. Plotting of graph
2.

Speed of sound = gradient = 309.1 ms-1

7. (b) (ii)
3.
The experiment requires stationary waves to be formed.
The formation of the stationary waves require the wavelength of the waves to fit the boundary
condition caused by the distance traveled by the wave.
Hence the sound frequencies are of non-uniform intervals. (The wavelengths are of equal
intervals)
FOR INTERNAL USE ONLY

NATIONAL JUNIOR COLLEGE

PHYSICS DEPARTMENT

4.
This might be because of errors due to the sound being reflected by surfaces other than the
reflecting plate, causing the stationary waves to be set up to be disturbed.
(iii)
Advantage: Experiments using smaller values of frequencies can be done as there can be a
longer distance between the signal generator and the reflecting plate.
There are also less reflective surfaces to interfere with the actual signal.
Disadvantage: External factors like wind will affect the experiment.
(c)
(i)
As the two light rays comes from the same source, they are coherent.
Ray 2 travel a longer distance (approximately twice the thickness of the film), there will be a
path difference between ray 1 and ray 2.
When the path difference is n, where is the wavelength of light, the amplitudes of the wave
add up and constructive interference occur.
When the path difference is n, where is the wavelength of light, the amplitudes of the wave
subtract and destructive interference occur.
7. (c) (ii)
The multicoloured appearance is due to the interference pattern set up by the different
wavelengths in white light.
The appearance changes as the soap bubbles thickness is always changing (due to wind,
surface tension etc.) thus the conditions for interference changes.
(iii)
Measurements of wavelength, size of objects etc.
Any reasonable answer.
8. (a)
1.
Electrons are accelerated in the Franck-Hertz apparatus. The collected current rises with
accelerated voltage as the amount of charge reaching the anode per unit time increases with
the velocity of the electrons.
2.
Energy from the accelerated electrons are passed to the atomic electrons of mercury (through
inelastic collisions). The sudden onset suggest that the mercury electrons cannot accept energy
until it reaches the threshold for elevating them to an excited state. i.e. excitation of the mercury
electrons.
3.
hc / = 4.9 x 1.6 x 10-19
= 2.54 x 10-7 m
4.
254 nm is in the region of Ultra Violet light. Hence it cannot be detected by the naked eye but it
can be detected by photographic film or material with silver salts.
(ii)
1.
As an excited state is unstable, the excited mercury electron would have de-excite and return to
the ground state almost immediately, hence there will not be a dip at 1.8 V.
FOR INTERNAL USE ONLY

NATIONAL JUNIOR COLLEGE

PHYSICS DEPARTMENT

There will also not be a dip at 6.7 V as inelastic collisions between the accelerated electrons
and mercury atoms would have taken place before that. Hence the probability of an electron
accelerated at 6.7 V colliding with the mercury atoms is very low.
2.
Nothing happens. As a photon is a quantum of energy, it will not be able to transfer part of its
energy to the mercury atom.
8. (b)

(i)
As electrons have wavelike behaviour as describe by the de Broglies equation p = h/.
Hence as it approaches the graphite film, it will undergo diffraction between the Carbon atoms.
When the diffraction patterns formed superimpose on each other, the diffraction rings are
formed.
(ii)
For observable diffraction, the atomic spacing should be the same as wavelength.
Using eV = P2/2m where P =h/
V =37.7 V

(iii)
If accelerating voltage is increased, the momentum of the electrons will increase, which by de
Broglies equation means that the wavelength will decrease.
Hence the diffraction at each slit (between the Carbon atoms) will decrease, causing the rings to
be closer.

FOR INTERNAL USE ONLY

NANYANG JUNIOR COLLEGE


JC 2 Preliminary Examination
Higher 1

PHYSICS

8866/01

Paper 1 Multiple Choice

18 Sep 2008
1 hour

Additional Materials:

Multiple Choice Answer Sheet

READ THESE INSTRUCTIONS FIRST


Write in soft pencil.
Do not use staples, paper clips, highlighters, glue or correction fluid.
Write your name, class and tutors name on the Answer Sheet in the spaces provided unless this has been
done for you.

There are thirty questions on this paper. Answer all questions. For each question there are four possible
answers A, B, C and D.
Choose the one you consider correct and record you choice in soft pencil on the separate Answer Sheet.

Read the instructions on the Answer Sheet very carefully.

Each correct answer will score one mark. A mark will not be deducted for a wrong answer.
Any rough working should be done in this booklet.

This document consists of 8 printed pages.


[Turn over

DATA AND FORMULAE


Data
8

speed of light in free space

3.00 x 10 m s

elementary charge

1.60 x 10

the Planck constant

unified atomic mass constant

me

rest mass of electron

mp

rest mass of proton

19

C
34

Js

6.63 x 10
27

kg

1.66 x 10
31

kg

9.11 x 10
27

kg

1.67 x 10
2

9.81 m s

ut + at2

v2

u2 + 2as

pV

hydrostatic pressure

gh

resistors in series

R1 + R2 + ....

1/R

1/R1 + 1/R2 + ....

acceleration of free fall


Formulae
uniformly accelerated motion

work done on/by a gas

resistors in parallel

The expected emission spectrum of a diluted gas is shown below :

However when an experiment is conducted to obtain the emission spectrum of the same gas,
the following spectrum (over the same frequency range) is obtained :

Which of the following best describe the error(s) committed ?


A
Random Error
Systematic Error

The air flow over the wing of an aircraft can be described by the equation
P + dV2 = (constant, k)
where P, d and V are the pressure, density and velocity of the air.
Which of the following statement best describes the given equation ?
A
B
C
D

The (constant, k) is dimensionless.


The (constant, k) must have a unit.
This equation is definitely dimensionally inconsistent.
This equation is wrong because P and dV2 are two different quantities.

3
3

Two quantities X and Y are related by the equation : X = Y-2.


If the percentage uncertainty of X is 6 %, what is the percentage uncertainty of Y ?
A

3%

12%

18%

The average acceleration is 40 ms-2.


The instantaneous acceleration is 40 ms-2.
The acceleration is constant throughout.
The acceleration of the plane causes a net force acting on it.

If a car can be brought to rest from 15 ms-1 in a distance of 12 m, what would be the braking
distance if it is traveling at 30 ms-1 ? Assuming braking force and road conditions are the same.
A

A plane makes a U-turn along a semi-circular path length 500 m with an average speed of 100
ms-1. Which of the following best describes the acceleration of this plane during this U-turn ?
A
B
C
D

2%

18 m

24 m

36 m

48 m

A ball is released from rest above a horizontal surface. The graph shows the variation with
time of its velocity.
Velocity

Y
time
X

Which of the following best describes the motion of the ball ?


A
B
C
D

Area X is the same as area Y.


There is air drag acting on the ball.
The acceleration is not constant.
The ball makes elastic collision with the surface.

A non-uniform object is balanced on a knife-edge. The part of the object on the left hand side
of the knife-edge is P while the other is Q. If this object broken into part P and Q, what is the
ratio : mass of P : mass of Q ? (Assume that P and Q are individually uniform).
4x
P

2:1

2x
Q

1:2

4:1

1:4

4
8

A rod is leaning against a cylinder which is fixed to the ground. The rod is in a plane which
contains a circular cross-section of the cylinder. All surfaces are rough. Which of the following
diagrams best shows the forces acting on the rod ?

2.0 N

Equal and opposite forces of magnitude 2.0 N are applied to the ends
of a ruler of length 0.30 m, creating a couple as shown in the diagram.
What is the magnitude of the torque of the couple on the ruler when it
is in the position shown?
A
C

0.23 Nm
0.60 Nm

B
D

50 o

0.46 Nm
0.92 Nm

50 o
2.0 N

10 The diagram shows three identical blocks that undergo elastic collisions on a frictionless
surface. In situations 1 and 2, two of the blocks are glued together. In all three situations, the
initially moving blocks have the same velocity v. Rank the situations according to the speed of
the rightmost block after the collisions, greatest first.
v

(1)
A

1, 2, 3

v
(2)

1, 3, 2

(3)
3, 1, 2

11 An object of mass 4.0 kg is initially moving with 3.0 m s-1 in the xdirection. A force of 8.0 N acts in the y-direction for 2.0 s. What is the
final velocity of the object?
A 5.0 ms-1 53o above x-axis

B
D

5.0 ms-1 38o above x-axis

4.0 kg
3.0 m s-1

7.0 ms-1 53o above x-axis

C 7.0 ms-1 in Y-axis

3, 2, 1

8.0 N

5
12 A man exerts an 80 N force to move a 10 kg box up a 30o rough
slope at a constant speed. What is the magnitude of the frictional
force ?
(take g = 10 N kg-2)
A
C

60 N
30 N

B
D

40 N
80 N

13 A metal sphere of mass m is moving through a viscous liquid. When it reaches a constant
downward velocity v, which of the following describes the changes with time in the kinetic
energy and gravitational potential energy of the sphere?

Kinetic energy
no change and equal to mv2

Gravitational potential energy


decease at a rate of mgv

no change and equal to mv2

decrease at a rate of (mv2 mgv)

Increase at a rate of mgv

Decrease at a rate of mgv

Increase at a rate of mgv

Decrease at a rate of mv2 mgv

14 A constant external force of magnitude 120 N is exerted on a 20 kg box, which is on a rough


horizontal surface. The force pushes the box a distance of 8.0 m, in a time interval of 4.0 s,
and the speed changes from v1 = 0.5 ms-1 to v2 = 3.5 ms-1.
120 N
35

120 N

8.0 m

v1 = 0.5 ms-1

v2 = 3.5 ms-1

The work done against friction is closest to


A
430 J
B
670 J

840 J

870 J

15 A motor using electrical energy at the rate of 400 W raises a block of weight 120 N. If the block
moves 8.0 m vertically in 4.0 s, the efficiency of the motor is
A

60%

48 %

30%

24%

16 A point source of sound emits energy uniformly in all directions at a constant rate and a
detector placed 3.0 m from the source measures an intensity of 3.0 Wm-2. The power of the
source is then doubled.
What intensity would the detector measure if it is placed at a distance 5.0 m from the source?
A

4.3 Wm-2

3.6 Wm-2

2.5 Wm-2

2.2 Wm-2

6
17 A transverse wave of wavelength is progressing along a horizontal rope from point P to point
Q. If the points are

5
apart, which of the following correctly describes the particle at point P
4

at an instant when the particle at point Q is below the equilibrium position but moving upwards?

A
B
C
D

Displacement of particle at P
Above equilibrium position
Above equilibrium position
Below equilibrium position
Below equilibrium position

Movement of particle at P
Downwards
Upwards
Upwards
Downwards

18 A piece of glass tubing with both ends open is


closed at one end by covering it with a sheet of
metal as shown in the diagram below. The
fundamental frequency is found to be 280 Hz.

Metal sheet
Closed pipe

If the metal sheet is now removed, what is the


new fundamental frequency of the resulting open
tubing ? Treat the speed of wave to be constant.
A

140 Hz

280 Hz

420 Hz

560 Hz

19 Light of wavelength 600 nm falls on a double slits, forming fringes 3.00 mm apart on a screen.
What would be the fringe spacing if the wavelength is 450 nm.
A

0.75 mm

1.50 mm

2.25 mm

20 Two loudspeakers are placed 1.2 m apart on two corners


of a rectangular floor as shown below. Both speakers are
operating in phase and are giving out a steady frequency
of 400 Hz. Point X is 1.6 m from one speaker. The speed
of sound can be taken as 320 ms-1

4.00 mm

1.6 m

1.2 m

Which of the following is correct about point X?


A
B
C
D

It has maximum intensity


It has intensity between minimum and maximum.
It has minimum intensity
It has intensity that varies from minimum to maximum periodically.

21 Two cells of emfs E1 and E2 , and internal resistances r1 and r2 , are


connected to a load of resistance R in the circuit shown. If the current
flowing in the circuit is I and E1 < E2 , what is the magnitude of the
terminal p.d. across the cell E1?
A
B
C
D

E1
E1
E2
E2

Ir1
Ir2
I (r1 + r2 + R)
Ir2 IR

E2, r2

E1 , r1

7
22 Which of the following statements regarding the variation of electrical resistivity of filament with
temperature is true?
A

Electrical resistivity increases with an increase in temperature because fewer free


electrons are available for conduction.

Although scattering of electrons by lattice ions increases with temperature, electrical


resistivity decreases due to a significant increase in free electrons.

Electrical resistivity increases with an increase in temperature because the mean time
between collisions of electrons with lattice ions increases.

Electrical resistivity can be decreased by applying a greater potential difference.

23 Two identical heaters of V volt and W watt rating are to be connected to a dc source of V volt
first in series and then in parallel. Which of the following statements is correct?
A
B
C
D

The total power drawn in each case is the same.


The power drawn is higher in the parallel connection.
There is more current drawn from the source in the series connection.
The total power drawn is 2W in the series connection.
1

24 Refer to the circuit given below. Find the total current


through the 5.0 V cell?.
A
C

1.5 A
5.0 A

B
D

3.5 A
6.5 A

2.0 V

5.0 V
2

25 Find the force on the segment YZ of the


wire as shown below if B = 0.3 T. Assume
the current in the wire is 5 A
A
B
C
D

0.15 N into the page


0.15 N out of the page
1.5 N into the page
1.5 N out of the page

Y
35 o 12 cm
B = 0.3 T

Y
26 Three parallel conductors, carrying currents, pass
vertically through the three corners of an equilateral
triangle XYZ. It is required to produce a resultant
magnetic field at O in the direction shown. What must
be the directions of the currents?

A
B
C
D

Into the page


X
Z
X and Z
X, Y and Z

Out of the page


Y and Z
X and Y
Y
None

O
Resultant
magnetic field

8
27 A particle, moving through a certain region of space, experiences a non-zero magnetic force.
Which of the following is possible?
A

A magnetic field exists in that region and changes the speed of the particle.

A magnetic field exists in this region and the particles velocity is parallel to the magnetic
field.

A magnetic field exists in this region and the particle is moving at right angle to the
magnetic field.

A magnetic field exists in this region and the particle is moving in the direction opposite
to the magnetic field.

28 In a photoelectric experiment, ultraviolet radiation is illuminated on a clean metal housed in an


evacuated chamber. If the intensity of the radiation is reduced by half, which of the following
will be reduced by approximately the same factor ?
1.
2.
3.
4.
A

The energy of each photon


The number of incoming photons per unit time.
The photoelectric current.
The maximum kinetic energy of the emitted electrons.
1, 2, 3

1, 4

2, 3

All

29 If the de Broglie wavelength of a free proton with a kinetic energy of 4 eV is , what would be
the de Broglie wavelength of this proton if its kinetic energy is 64 eV ?
A

/ 16

/4

16

30 The diagram (drawn approximately to scale) shows some of the energy levels of an atom.
Transition P results in the emission of a photon of wavelength 4 x 107 m.
A
B
P

Which one of the transitions A, B, C or D could result in the emission of a photon of


wavelength 8 x 107 m ?

~ End of Paper ~

NANYANG JUNIOR COLLEGE


JC 2 Preliminary Examination
Higher 1
CANDIDATE
NAME

TUTORS
NAME

CLASS

PHYSICS

8866/02

Paper 2 Structured Questions

12 Sep 2008
2 hours

Candidates answer on the Question Paper.


No Additional Materials are required
READ THESE INSTRUCTIONS FIRST
Write your name and class on all the work you hand in.
Write in dark blue or black pen on both sides of the paper.
You may use a soft pencil for any diagrams, graphs or rough working.
Do not use staples, paper clips, highlighters, glue or correction fluid.
Section A
Answer all questions.

Section B
Answer any two questions.

For Examiners Use


Section A

You are advised to spend about one hour on each section.


At the end of the examination, fasten all your work securely together.
The number of marks is given in brackets [ ] at the end of each question or part
question.

1
2
3
4
5
Section B
6
7
8
Total

This document consists of 13 printed pages.


[Turn over
1

DATA AND FORMULAE


Data
8

speed of light in free space

3.00 x 10 m s

elementary charge

1.60 x 10

the Planck constant

unified atomic mass constant

me

rest mass of electron

mp

rest mass of proton

=
=
=
=

19

C
34

Js

6.63 x 10
27

kg

1.66 x 10
31

kg

9.11 x 10
27

kg

1.67 x 10
2

9.81 m s

ut + at2

v2

u2 + 2as

pV

hydrostatic pressure

gh

resistors in series

R1 + R2 + ....

1/R

1/R1 + 1/R2 + ....

acceleration of free fall


Formulae
uniformly accelerated motion

work done on/by a gas

resistors in parallel

Section A
Answer all the questions in this section.
You are advised to spend about 1 hour in this section.
1

(a)

A horizontal spring of spring constant 4.5 Nm-1 is compressed by 9.0 cm. The
compressed spring is use to project a uniform sphere of mass 600 g.
(i)

Prove that the initial velocity of the sphere is 0.25 ms-1. Neglect air resistance.

[2]

(ii) This spring-sphere system is originally designed to hit a target placed at a distance
of W as shown. What should W be in order for the sphere to hit the target ?
[3]
sphere

35o

target

(b)

When this experiment is performed, the sphere


will hit the target at slight different positions. In
one particular experiment, the results of 6
launches were shown. State and give reason(s)
what kind of error or errors have appeared in
this experiment.
[2]

(a)

Distinguish between electromotive force (e.m.f.) and potential difference.

(b)

Given a network of identical resistors of as shown below. Each resistor has a resistance
of 4 .
B
C
A

Calculate the total effective resistance across

(c)

Theoretically, all sphere


should hit this point.

[2]

F
(i) AE

(ii) BC

[4]

Explain why the total resistance across any 2 points in the above circuit is less than 5
[1]

(a)

Define magnetic flux density.

[2]

(b)

Figure below shows two long straight wires carrying currents I1 and I2 where I1 > I2 . Both
currents flow out of the plane of the paper. The point P is at a distance d as shown. The
two wires are separated by a distance D.
d

I1

xP

I2
D

(i)

Indicate the direction of the resultant magnetic field B at the midpoint on the above
diagram.
[1]

(ii) Derive an expression, in terms of I1 and I2, for the distance d from I1 where the
magnitude of the resultant magnetic field is zero.
[3]
Note: The magnetic flux density B at a perpendicular distance r from a wire carrying
a current I is given by
I
B= o
2 r

(iii) If a charge is placed at point P, explain whether there will be a magnetic force acting
on the charge.
[1]

(a)

A world-class individual pursuit cyclist of mass 70 kg


(including the bicycle) is cycling at a constant speed of
55 km h-1. Given that he is delivering an average
power of 520 W, find the drag force acting on him.
[2]

(b)

In trying to overtake the front cyclist, he needed to move up along a certain direction that
makes an angle of 2o with the horizontal. Given that he is still moving with the same
speed, find the new power deliver by him.
[2]

(c)

In a four man team pursuit cycling as


shown below, the first cyclist delivered
an average power of 600 W. Given the
second, third and fourth cyclist deliver
power at a capacity of 75%, 65% and
60% of 600 W respectively.

55 kmh-1

(i) Find the drag force acting on the fourth cyclist when the group is moving at 60 km h-1.
[1]

(ii) Explain why the last cyclist required the least power to move at constant speed
compared to the other cyclist.
[1]

(iii) Suggest why it is important for the four cyclists to remains close to each other
during the entire race.
[1]

5
The walls of music halls are acoustically covered
with a sound-absorbing panel that is able to absorb certain
sound frequencies more than others.
One particular design is to use the resonance of a
perforated panel as shown in figure 5.1.
It is found that this panel resonates at a particular
frequency and hence can absorb sounds of that
frequency more than others. A simple formula for this
resonant frequency f is :
f

5000

x
h (t + 0 .8 d )
Fig. 5.1

where

h
t
d
x

=
=
=
=

depth of airspace in mm,


thickness of panel in mm,
hole diameter in mm,
percentage of panel surface area occupied by holes (e.g. x = 10 %)

An acoustic engineer decides to improve on the formula. Using hardboard panels having holes
of the same diameter d, the resonant frequencies f for different x values were measured. The
results are shown in table 5.1 :
2
540

x/%
f / Hz

4
750

6
930

8
1070

10
1190

12
1300

Table 5.1

A graph of f against x is then plotted.


(a) Suggest a reason (together with explanation) why the engineer refuses to accept the given
formula totally.
[2]

(b) (i)

Plot a graph of f against x on the grid given using the data from table 8.1. Find the rate
of change of resonant frequency with the percentage of panel surface area occupied

df
, when x = 5%.
dx

by holes i.e.

[4]

f l Hz
1900
1700
1500
1300
1100
900
700
500
300
0

x/%

10

15

(b) (ii) Briefly describe what is the effect of hole on resonant frequency ?

[2]

(b) (ii) Discuss the random error revealed by your f against x graph. State one like cause of
this error.
[2]

Using a panel with x fixed at 10% and d fixed at 5


mm, the engineer decided to investigate the
effect of filling the air space with 2 different
absorbers. He obtained the absorption
characteristics as shown in figure 5.2.

Fig. 5.2

Absorption coefficient is an indication of how


much sound wave can be absorbed i.e. the
higher the absorption coefficient, the stronger the
absorption.
(c) Which of the two absorbers would be
preferred in a room in which
(i) absorption over a wide frequency range
was required,

(ii) sounds below 500 Hz were of low intensity and needed emphasizing ?

[2]

Section B
Answer any two questions in this section.
You are advised to spend about 1 hour in this section.
6

(a) (i)

State Newtons second law of motion.

[1]

(ii)

What is the momentum of a 0.15 kg tennis ball which is moving with a speed of 30
ms-1.
[1]

(iii)

This ball is struck by a racket such that after the impact, it moves in the opposite
direction to that of its approaching path. The graph shows the force of racket acting
on the ball.
F/N

300
200
100

t/s

0
0.1

0.2

Calculate
1. the change in momentum of the ball.

2. the average force of racket on the ball during impact.

3. the speed of the ball after impact.

[5]

(iv) If the strings of the racket are loosen, draw another graph on the same axes to show
how the force of racket on the ball would be during impact. Assume that the speed of
the ball after impact is the same as what you have found in (6ai-3).
[2]
8

(b) A uniform flexible tube of rubber (fig. 6bi) is bent into a symmetrical arch (fig. 6bii). The
arch is placed on a rough surface and stay at equilibrium.
Fig. 6bi

Fig. 6bi

(i) Draw a free body diagram of the arch in the space below. The label as well as the
lines of action of each force must be shown clearly.
[2]

(ii) The mass of the arch is 50 kg. If a big bird of mass 6.0 kg perches on
the top of the arch, what will be the effect on the magnitude of each of
the force you have draw in part (6bi).
[3]

(c)

A uniform vertical wheel of radius 5.0 m turn at a uniform rate of 3 rounds per minute.
There is a 2.4 kg bucket at one end of the wheel.
(i) What is the kinetic energy of the bucket ?

[2]
5.0 m
bucket

(ii) What is the largest possible increase in gravitational potential energy of this bucket ?
[1]

(iii) Deduce the maximum instantaneous power supplied to the bucket and its
corresponding position of the bucket. Explain your answer.
[3]

(a) (i)

(ii)

State the principle of superposition

[2]

State the two conditions for interference

[2]

(b) Two radio aerial are emitting radio signals that are in phase and coherent. They are
separated by a distance d. A signal is received from an aircraft passing directly overhead
at speed v and height h along a line parallel to that between the aerials as shown below.

2PQ = QR

d
Aerial 1

Aerial 2

The signals from the two aerials are added through an electronic device and the resultant
signal is recorded.
(i)

Given that Q is equal distance from both aerials and R, Q and P represent the points
where the resultant signal received are maxima, what are the possible path
difference of the signals received by the two aerial at these points ? (2PQ = QR)
Explain your answers clearly.
[3]

(ii) Explain why the intensity of the signal fluctuates?

10

[2]

(iii) Given that the frequency of the signals emitted by both aerials is 1.5 x 108 Hz and v
= 200 ms-1 , d = 15 m and h = 1500 m, calculate the time for the plane to fly from Q
to R.
[3]

(iv) In the event that one of the aerial malfunction, suggest why a plane flying directly
over may still detect fluctuation in the signal received.
[2]

(c)

A wire 2.4 m long is fixed at bit ends. When the wire is plucked, a transverse wave of
speed 320 ms-1 is propagated along the wire and forms a standing wave. Three antinodes
are formed.
(i) Explain briefly how standing waves are formed along the wire.
[2]

(ii) Sketch the appearance of the vibrating wire.

[2]

(iii) What is its frequency of vibration?

[2]

11

(a) (i)

A 20 W point source of light emits monochromatic light of wavelength 530 nm. What
is the intensity of the light at a distance 5.0 m from the source ?
[2]

(ii)

In 1 m2 of area at this distance, many photons are detected per second ?

(iii)

A 3.0 cm2 metal is place 5.0 m from this point source. What is the threshold
frequency if the work function energy of this metal is 2.8 x 10-19 J.
[1]

(iv)

Hence use the concept of conservation of energy to explain why photoelectric is


possible ? Show any necessary calculation.
[2]

(v)

What is the stopping potential for this photoelectric effect experiment ?

[1]

(vi)

If the probability of ejecting the electron is 0.05, calculate the electron current.

[3]

[2]

(b) In the production of emission and absorption line spectra, briefly describe the difference in
the following areas :
(i)

appearance of spectrum

(ii)

excitation mechanism

12

[4]

(c) Fig. 8ci illustrates a set of 5 concentric lines to represent the four lowest energy levels of
an isolated atom. The innermost line represents n = 1 state while the outermost line
represents n = 5 state. Diagrams are not drawn to scale.
n=4
n=3

Balmer
Series

n=5

n=2

n=1

Lyman
Series
Paschen
Series
Fig. 8cii

Fig. 8ci

(i)

Why the total energy of an electron residing on one of these energy state must be
negative ?
[1]

(ii) Fig. 8cii shows three series of transitions by the single electron in the hydrogen atom.
Draw the resulting spectrum due to all these transitions on fig. 8ciii. The relative
position of each spectral line with respect to the rest must be well illustrated. Label
your series.
[3]

Fig. 8ciii
Longest wavelength End

Shortest Wavelength End

(iii) In reality it is hard to obtain isolate hydrogen atoms, usually two hydrogen atoms will
react to become one hydrogen molecule. Describe one change to your spectrum in Fig.
(8ciii) if liquid hydrogen (consisting of hydrogen molecules) is used.
[1]

~ End of Paper ~
13

NANYANG JUNIOR COLLEGE


JC 2 Preliminary Examination
Higher 1

PHYSICS

8866/01

Paper 1 Multiple Choice

18 Sep 2008
1 hour

Additional Materials:

Multiple Choice Answer Sheet

READ THESE INSTRUCTIONS FIRST


Write in soft pencil.
Do not use staples, paper clips, highlighters, glue or correction fluid.
Write your name, class and tutors name on the Answer Sheet in the spaces provided unless this has been
done for you.

There are thirty questions on this paper. Answer all questions. For each question there are four possible
answers A, B, C and D.
Choose the one you consider correct and record you choice in soft pencil on the separate Answer Sheet.

Read the instructions on the Answer Sheet very carefully.

Each correct answer will score one mark. A mark will not be deducted for a wrong answer.
Any rough working should be done in this booklet.

This document consists of 11 printed pages.


[Turn over

DATA AND FORMULAE


Data
8

speed of light in free space

3.00 x 10 m s

elementary charge

1.60 x 10

the Planck constant

unified atomic mass constant

me

rest mass of electron

mp

rest mass of proton

19

C
34

Js

6.63 x 10
27

kg

1.66 x 10
31

kg

9.11 x 10
27

kg

1.67 x 10
2

9.81 m s

ut + at2

v2

u2 + 2as

pV

hydrostatic pressure

gh

resistors in series

R1 + R2 + ....

1/R

1/R1 + 1/R2 + ....

acceleration of free fall


Formulae
uniformly accelerated motion

work done on/by a gas

resistors in parallel

The expected emission spectrum of a diluted gas is shown below :

However when an experiment is conducted to obtain the emission spectrum of the same gas,
the following spectrum (over the same frequency range) is obtained :

Which of the following best describe the error(s) committed ?


A
Random Error
Systematic Error

The position of each line is consistently shifted to one side Systematic error
The lines are thicker which shows that there is a wider spread of reading random error

Ans: D

The air flow over the wing of an aircraft can be described by the equation
P + dV2 = (constant, k)
where P, d and V are the pressure, density and velocity of the air.
Which of the following statement best describes the given equation ?
A
B
C
D

If [constant] = [P] = [dV2],


The (constant, k) is dimensionless.
this equation is dimensionally correct.
The (constant, k) must have a unit.
Ans: B
This equation is definitely dimensionally inconsistent.
2
This equation is wrong because P and dV are two different quantities.

3
3

Two quantities X and Y are related by the equation : X = Y-2.


If the percentage uncertainty of X is 6 %, what is the percentage uncertainty of Y ?
A 2%

3%

X = Y-2
Y / Y

= (0.5) X / X

12%

18%

Y = ( / X)
3%

Ans: B

A plane makes a U-turn along a semi-circular path length 500 m with an average speed of 100
ms-1. Which of the following best describes the acceleration of this plane during this U-turn ?
A
B
C
D

The average acceleration is 40 ms-2.


The instantaneous acceleration is 40 ms-2.
The acceleration is constant throughout.
The acceleration of the plane causes a net force acting on it.

Average acceleration = (final velocity initial velocity) / time = 200 / 5 = 40 ms-2

If a car can be brought to rest from 15 ms-1 in a distance of 12 m, what would be the braking
distance if it is traveling at 30 ms-1 ? Assuming braking force and road conditions are the same.
A 18 m

24 m

v2 = u2 + 2as
0 = (15)2 + 2(-a)(12) ---- (1)
Upon solving, s = 48 m

Ans. A

36 m

0 = (30)2 + 2(-a)(s)

48 m

---- (2)
Ans: D

A ball is released from rest above a horizontal surface. The graph shows the variation with
time of its velocity.
Velocity

Y
time
X

Which of the following best describes the motion of the ball ?


A
B
C
D

Area X is the same as area Y.


There is air drag acting on the ball.
The acceleration is not constant.
The ball makes elastic collision with the surface.

Area X represents the distance the ball travels immediately after the first impact to the peak. If the
ball were to make the second impact, it must travel the same distance down i.e. area Y. Ans: A

4
7

A non-uniform object is balanced on a knife-edge. The part of the object on the left hand side
of the knife-edge is P while the other is Q. If this object broken into part P and Q, what is the
ratio : mass of P : mass of Q ? (Assume that P and Q are individually uniform).
4x
P

A 2:1

= 0
mpg (2x) - mQg (x) = 0

2x
Q

1:2

4:1

2mp = mQ

1:4

mp : mQ

= 1:2
Ans: B

A rod is leaning against a cylinder which is fixed to the ground. The rod is in a plane which
contains a circular cross-section of the cylinder. All surfaces are rough. Which of the
following diagrams best shows the forces acting on the rod ?

The surface of the cylinder is rough. The friction on the rod should be up along the rod to prevent the rod
slipping down. There is no friction at the contact with the cylinder in (A) and the friction is down the rod in
(C). The friction at the ground is to the right so it is wrong. The three forces must also intersect at one
point.
Ans: B

Equal and opposite forces of magnitude 2.0 N are applied to the ends of a ruler of length 0.30
m, creating a couple as shown in the diagram. What is the magnitude of the torque of the
couple on the ruler when it is in the position shown?
A 0.23 Nm
C 0.60 Nm

B
D

2.0 N

0.46 Nm
0.92 Nm

T = F d = 2.0 x (0.30 x sin 50o)

= 0.46 N m

50 o
Ans : B
50 o
2.0 N

5
10 The diagram shows three identical blocks that undergo elastic collisions on a frictionless
surface. In situations 1 and 2, two of the blocks are glued together. In all three situations, the
initially moving blocks have the same velocity v. Rank the situations according to the speed of
the rightmost block after the collisions, greatest first.
v

(1)
1, 2, 3

v
(2)

1, 3, 2

Case

(3)
3, 1, 2

(1)

Situation

Velocity of
righmost block

(3)

2mv = 2mv1 + mv2


2v = 2v1 + v2
v 0 = v2 - v1
v = v2 - v1
v2 =

3, 2, 1

(2)

Conservation of
total momentum
Elastic Collision

mv = mv1 + 2mv2
v = v1 + 2v2
v 0 = v2 - v1
v = v2 - v1

4v
3

v2 =

mv = mv1 + mv2
v = v1 + v2
v 0 = v2 - v1
v = v2 - v1

2v
3

v2 = v

You can also see the answer directly using a bit of common sense without using all the above rigorous
mathematics. The key point is it is elastic collision.
Case 1: Larger mass collide with smaller mass, expect smaller mass to move faster than v.
Case 2: Smaller mass collide with a larger mass, expect larger mass to move slower than v.
Case 3: Equal masses collide elastically, expect the velocity to be the same.

11 An object of mass 4.0 kg is initially moving with 3.0 m s-1 in the xdirection. A force of 8.0 N acts in the y-direction for 2.0 s. What is the
final velocity of the object?
A 5.0 ms-1 53o above x-axis

B
D

4.0 kg

5.0 ms-1 38o above x-axis

3.0 m s-1

7.0 ms-1 53o above x-axis

C 7.0 ms-1 in Y- axis

F = ma

a = 8.0 / 4.0 = 2.0 m s-2


vy = uy + ayt = 0 + 2.0 x 2.0 = 4.0 m s-1
Resultant velocity = 5.0 m s-1
= tan-1 (4.0/3.0) = 53o

8.0 N

in y-dir

Ans: A

12 A man exerts an 80 N force to move a 10 kg box up a 30o rough


slope at a constant speed. What is the magnitude of the frictional
force ?
(take g = 10 N kg-2)
A 60 N
C 30 N

B
D

40 N
80 N

Along the slope :


F = ma
80 - f - mgsin30o = ma

80 - f - (10)(10)sin30o = 0

f = 30 N

Ans B

Ans : C

6
13

A metal sphere of mass m is moving through a viscous liquid. When it reaches a constant
downward velocity v, which of the following describes the changes with time in the kinetic
energy and gravitational potential energy of the sphere?

Kinetic energy
no change and equal to mv2

Gravitational potential energy


decease at a rate of mgv

no change and equal to mv2

decrease at a rate of (1/2 mv2 mgv)

Increase at a rate of mgv

Decrease at a rate of mgv

Increase at a rate of mgv

Decrease at a rate of mv2 mgv

Since velocity is constant, there should be no change in kinetic energy.


As it is falling with a speed v, the rate of decrease of GPE = mgh/t = mgv

Ans:A

14 A constant external force of magnitude 120 N is exerted on a 20 kg box, which is on a rough


horizontal surface. This force pushes the box a distance of 8.0 m, in a time interval of 4.0 s,
and the speed changes from v1 = 0.5 ms-1 to v2 = 3.5 ms-1.
120 N
35

120 N

8.0 m

v1 = 0.5 ms-1

v2 = 3.5 ms-1

The work done against friction is closest to


A
430 J
B
670 J

840 J

870 J

Work done by P = Gain in KE + work done against friction


120(8.0) cos35o = [ (1/2)(20)(3.5)2 - (1/2)(20)(0.5)2] + WD against friction
WD against friction = 670 J

15

Ans: C

A motor using electrical energy at the rate of 400 W raises a block of weight 120 N. If the
block moves 8.0 m vertically in 4.0 s, the efficiency of the motor is
A

60%

48 %

30%

24%

Power output = mgh/t = (120)(8)/4 = 240 W


Hence efficiency = 240 / 400 = 60 %

Ans: A

16 A point source of sound emits energy uniformly in all directions at a constant rate and a
detector placed 3.0 m from the source measures an intensity of 3.0 Wm-2. The power of the
source is then doubled.
What intensity would the detector measure if it is placed at a distance 5.0 m from the source?
A

4.3 Wm-2

3.6 Wm-2

2.5 Wm-2

At 3.0 m, intensity, I = P/4r2 = P/432 = 3.0 W m-2


When power doubled,
Intensity = 2P/452 = (0.72)( P/452) = 0.72(3) = 2.2 Wm-2

2.2 Wm-2

Ans: D

7
17

A transverse wave of wavelength is progressing along a horizontal rope from point P to


point Q. If the points are

5
apart, which of the following correctly describes the particle at
4

point P at an instant when the particle at point Q is below the equilibrium position but
moving upwards?

Movement of particle at P
Downwards
Upwards
Upwards
Downwards

Displacement of particle at P
Above equilibrium position
Above equilibrium position
Below equilibrium position
Below equilibrium position

A
B
C
D

Q
Ans: B

18

A piece of glass tubing with both ends open is closed at one end by covering it with a sheet
of metal as shown in the diagram below. The fundamental frequency is found to be 280 Hz.
Metal sheet
Closed pipe

If the metal sheet is now removed, what is the new fundamental frequency of the resulting
open tubing ? Treat the speed of wave to be constant.
A

140 Hz

280 Hz

For closed pipe, = 4L, f = 280 Hz

420 Hz

560 Hz

For open pipe, = 2L, f = 560 Hz


Ans: D

19

Light of wavelength 600 nm falls on a double slits, forming fringes 3.00 mm apart on a
screen. What would be the fringe spacing if the wavelength is 450 nm.
A

0.75 mm

X = D/ a
X ,

1.50 mm

2.25 mm

Hence x = 450/600 x (3.00) = 2.25

4.00 mm

Ans: C

8
20

Two loudspeakers are placed 1.2 m apart on two corners of a rectangular floor as shown
below. Both speakers are operating in phase and are giving out a steady frequency of 400
Hz. Point X is 1.6 m from one speaker. The speed of sound can be taken as 320 ms-1
1.6 m

1.2 m

Which of the following is correct about point X?


A
B
C
D

It has maximum intensity


It has intensity between minimum and maximum.
It has minimum intensity
It has intensity that varies from minimum to maximum periodically.
v = f , = 320 / 400 = 0.8 m
Path diff = 2.0 1.6 = 0.4 m = (0.5)(0.8) =
Hence destructive interference at X

21

Ans:

Two cells of emfs E1 and E2 , and internal resistances r1 and r2 , are connected to a load of
resistance R in the circuit shown. If the current flowing in the circuit is I and E1 < E2 , what is
the magnitude of the terminal p.d. across the cell E1?

E2, r2
A
B
C
D

E1
E1
E2
E2

Ir1
Ir2
I (r1 + r2 + R)
Ir2 IR

E1 , r1

R
Ans:

22 Which of the following statements regarding the variation of electrical resistivity of filament with
temperature is true?
A Electrical resistivity increases with an increase in temperature because fewer free electrons
are available for conduction.
B Although scattering of electrons by lattice ions increases with temperature, electrical
resistivity decreases due to a significant increase in free electrons.
C Electrical resistivity increases with an increase in temperature because the mean time
between collisions of electrons with lattice ions increases.
D Electrical resistivity can be decreased by applying a greater potential difference.
Ans: C

9
23

Two identical heaters of V volt and W watt rating are to be connected to a dc source of V
volt first in series and then in parallel. Which of the following statements is correct?
A
B
C
D

The total power drawn in each case is the same.


The power drawn is higher in the parallel connection.
There is more current drawn from the source in the series connection.
The total power drawn is 2W in the series connection.

For series connection, total power = 2 x (V/2)2 / R = V2 / 2R


For parallel connection, total power = 2 x V2/R = 2V2/R (4 times more)

24

Ans: B

Refer to the circuit given below. Find the total current through the 5.0 V cell?.
1

A
B
C
D

1.5 A
3.5 A
5.0 A
6.5 A

2.0 V

5.0 V
2

Apply V = IR to 1 resistor, I = 5 A
Apply V = IR to 2 resistor, I = 1.5 A
Hence total current = 6.5 A

25

Ans: D

Find the force on the segment YZ of the wire as shown below if B = 0.3 T. Assume the
current in the wire is 5 A
Y
35 o 12 cm
B = 0.3 T

A
B

0.15 N into the page


0.15 N out of the page

F = BIL = (0.3)(5)(0.12cos35o) = 0.147 N


Using FLFR, direction of force = out of page

C
D

1.5 N into the page


1.5 N out of the page

Ans: B

10
26

Three parallel conductors, carrying currents, pass vertically through the three corners of an
equilateral triangle XYZ. It is required to produce a resultant magnetic field at O in the
direction shown. What must be the directions of the currents?
Y

O
Resultant
magnetic field

X
A
B
C
D

Into the page


X
Z
X and Z
X, Y and Z

Out of the page


Y and Z
X and Y
Y
None

By using right hand grip rule, the B-field at Y is OUT and IN for X and Z
Ans: C

27

A particle, moving through a certain region of space, experiences a non-zero magnetic


force. Which of the following is possible?
A

A magnetic field exists in that region and changes the speed of the particle.

A magnetic field exists in this region and the particles velocity is parallel to the
magnetic field.

A magnetic field exists in this region and the particle is moving at right angle to the
magnetic field.

A magnetic field exists in this region and the particle is moving in the direction
opposite to the magnetic field.

The charged particle must not move along the direction of B field to experience a force. Ans: C

28

In a photoelectric experiment, ultraviolet radiation is illuminated on a clean metal housed in


an evacuated chamber. If the intensity of the radiation is reduced by half, which of the
following will be reduced by approximately the same factor ?
1.
2.
3.
4.

The energy of each photon


The number of incoming photons per unit time.
The photoelectric current.
The maximum kinetic energy of the emitted electrons.

1, 2, 3

1, 4

2, 3

Ans : C

All

11
29

If the de Broglie wavelength of a free proton with a kinetic energy of 4 eV is , what


would be the de Broglie wavelength of this proton if its kinetic energy is 64 eV ?
A

/ 16

K = p2 / 2m = (h/)2 / 2m
K 1 / 2
K1 / K = ( / 1)2

64 / 4 = ( / 1)2

30

/4

1 = / 4

16

Ans : B

The diagram (drawn approximately to scale) shows some of the energy levels of an atom.
Transition P results in the emission of a photon of wavelength 4 x 107 m.
A
B
P

Which one of the transitions A, B, C or D could result in the emission of a photon of


wavelength 8 x 107 m ?
The larger the transition gap, the smaller is the wavelength of photon.

~ End of Paper ~

Ans : B

NANYANG JUNIOR COLLEGE


JC 2 Preliminary Examination
Higher 1
CANDIDATE
NAME

TUTORS
NAME

CLASS

PHYSICS

8866/02

Paper 2 Structured Questions

12 Sep 2008
2 hours

Candidates answer on the Question Paper.


No Additional Materials are required
READ THESE INSTRUCTIONS FIRST
Write your name and class on all the work you hand in.
Write in dark blue or black pen on both sides of the paper.
You may use a soft pencil for any diagrams, graphs or rough working.
Do not use staples, paper clips, highlighters, glue or correction fluid.
Section A
Answer all questions.

Section B
Answer any two questions.

For Examiners Use


Section A

You are advised to spend about one hour on each section.


At the end of the examination, fasten all your work securely together.
The number of marks is given in brackets [ ] at the end of each question or part
question.

1
2
3
4
5
Section B
6
7
8
Total

This document consists of 13 printed pages.


[Turn over
1

DATA AND FORMULAE


Data
8

speed of light in free space

3.00 x 10 m s

elementary charge

1.60 x 10

the Planck constant

unified atomic mass constant


rest mass of electron

me

rest mass of proton

mp

19

C
34

Js

6.63 x 10
27

kg

1.66 x 10
31

kg

9.11 x 10
27

1.67 x 10

kg

9.81 m s

ut + at2

v2

u2 + 2as

pV

hydrostatic pressure

gh

resistors in series

R1 + R2 + ....

1/R

1/R1 + 1/R2 + ....

acceleration of free fall


Formulae
uniformly accelerated motion

work done on/by a gas

resistors in parallel

Section A
Answer all the questions in this section.
You are advised to spend about 1 hour in this section.
1

(a)

A horizontal spring of spring constant 4.5 Nm-1 is compressed by 9.0 cm. The
compressed spring is use to project a uniform sphere of mass 600 g.
(i)

Prove that the initial velocity of the sphere is 0.25 ms-1. Neglect air resistance.
mv2 = kx2
(0.600) v2 = (4.5)(0.090)2

By conservation of energy :

[2]

v = 0.246 ms-1

(ii) This spring-sphere system is originally designed to hit a target placed at a distance
of W as shown. What should W be in order for the sphere to hit the target ?
[3]
sphere
sy = uyt + ayt2
+ve
W sin35o = (9.81) t2 ------------- (1)
W
+ve
sx = uxt + axt2
W cos35o = (0.25) t ------------- (2)
35o

target
(1) / (2) :

tan35o = (9.81) t2 / (0.27) t

t = 0.0307 sec

sub back to get W = 0.0089 m

(b)

When this experiment is performed, the sphere will hit the target at slight different
positions. In one particular experiment, the results of 6 launches were shown. State and
give reason(s) what kind of error or errors have appeared in this experiment.
[2]
Random Error - The points are scattered.

Theoretically, all sphere


should hit this point.

Systematic Error - The points are biased to


one side of the targeted point.

(a)

Distinguish between electromotive force (e.m.f.) and potential difference.

[2]

e.m.f. is the energy converted to electrical from other forms per unit charge,
p.d. is the energy converted from electrical to other forms per unit charge.

(b)

Given a network of identical resistors of as shown below. Each resistor has a resistance
of 4 .

Calculate the total effective resistance across


1.90 ,
(c)

(i) AE

(ii) BC

[4]

Explain why the total resistance across any 2 points in the above circuit is less than 5
[1]

Across any two different points, there will be parallel connections of resistors. Since the minimum
value of each branch in parallel connection is 4 , the maximum value of the resultant parallel
connection is certainly less than 4 .

(a)

Define magnetic flux density.

[2]

The magnetic flux density at a point is defined as the force acting per unit current per unit length
of a conductor when the conductor is placed at right angles to the magnetic field.

(b)

Figure below shows two long straight wires carrying currents I1 and I2 where I1 > I2 . Both
currents flow out of the plane of the paper. The point P is at a distance d as shown. The
two wires are separated by a distance D.

I1

xP

I2
D

(i)

Indicate the direction of the resultant magnetic field B at the midpoint on the above
diagram.
[1]
3

(ii) Derive an expression, in terms of I1 and I2, for the distance d from I1 where the
magnitude of the resultant magnetic field is zero.
[3]
Note: The magnetic flux density B at a perpendicular distance r from a wire carrying
a current I is given by

B=
B1 =

o I1
2 d

o I
2 r

B2 =

o I 2

2 (D d )

When resultant B-field = 0, IB1I = IB2I


(D - d)I1 = I2d

d =

DI1
I1 + I 2

(iii) If a charge is placed at point P, explain whether there will be a magnetic force acting
on the charge.
[1]
No unless the charge is moving with a component of its velocity perpendicular to the magnetic field direction.

(a)

A world-class individual pursuit cyclist of mass 70 kg


(including the bicycle) is cycling at a constant speed of
55 km h-1. Given that he is delivering an average
power of 520 W, find the drag force acting on him.
[2]

55 kmh-1

Since kinetic energy is constant and the cyclist is cycling


on a level road, the power used is only used to overcome
drag force.
55 kmh-1 = 55 000 / 3600 ms-1
Power to overcome drag = Power supplied = Fdrag v

Fdrag = 34 N
520 = Fdrag (55 000 / 3600)

(b)

In trying to overtake the front cyclist, he needed to move up along a certain direction that
makes an angle of 2o with the horizontal. Given that he is still moving with the same
speed, find the new power deliver by him.
[2]
Power supplied = Power to overcome drag + Rate of increase of G.P.E.
P = Fdrag v + mgv sin2o
P = (34)(55 000 / 3600) + (70)(9.81)( 55 000 / 3600) sin2o
P = 887 W

(c)

In a four man team pursuit cycling as shown below, the first cyclist delivered an average
power of 600 W. Given the second, third and fourth cyclist deliver power at a capacity of
75%, 65% and 60% of 600 W respectively.

(i) Find the drag force acting on the fourth cyclist when the group is moving at 60 km h-1.
[1]
Power supplied = Power to overcome drag
Power supplied = Fdrag v

(0.6)(600) = Fdrag (60 000 / 3600)

Fdrag = 21.6 N

(ii) Explain why the last cyclist required the least power to move at constant speed
compared to the other cyclist.
[1]
The first cyclist faces the full effect of the air resistance on him. By cycling behind the lead
cyclist, the effect of air resistance is gradually diluted down the cyclists.

(iii) Suggest why it is important for the four cyclists to remains close to each other
during the entire race.
[1]
The dilution of air resistance is effective if the cyclists are close together. With a smaller drag force, the whole
group can collectively cycle at a higher speed with the same power so as to maximize their chance of winning.

The walls of music halls are acoustically covered with a


sound-absorbing panel that is able to absorb certain
sound frequencies more than others.
One particular design is to use the resonance of a
perforated panel as shown in figure 5.1.
It is found that this panel resonates at a particular
frequency and hence can absorb sounds of that
frequency more than others. A simple formula for this
resonant frequency f is :
f

5000

x
h (t + 0 .8 d )

Fig. 5.1

where h = depth of airspace in mm,


t = thickness of panel in mm,
d = hole diameter in mm,
x = percentage of panel surface area occupied by holes (e.g. x = 10 %)
An acoustic engineer decides to improve on the formula. Using hardboard panels having holes
of the same diameter d, the resonant frequencies f for different x values were measured. The
results are shown in table 5.1 :
x/%
f / Hz

2
540

4
750

6
930

8
1070

10
1190

12
1300

Table 5.1

A graph of f against x is then plotted.


(a) Suggest a reason (together with explanation) why the engineer refuses to accept the given
formula totally.
[2]
The formula is dimensionally inconsistent.
Unit of f is not the same as the unit(s) of the term(s) on the RHS.

(b) (i)

Plot a graph of f against x on the grid given using the data from table 8.1. Find the rate
of change of resonant frequency with the percentage of panel surface area occupied

df
, when x = 5%.
dx

f/Hz

by holes i.e.

[4]

Graph of f / Hz against x / %

1500

(df/dx)x = 5%
= Tangent to the graph at x = 5%
= 80 to 86 Hz

1300
1100
900
700
500
300
100
0

x/ %

10

15

(b) (ii) Briefly describe what is the effect of hole on resonant frequency ?

[2]

Resonant frequency increase with the hole occupancy.


The increase is non-linear.

(b) (ii) Discuss the random error revealed by your f against x graph. State one like cause of
this error.
[2]
All the points are close to the best fit line
Hence good precision (small random error)
1) The holes might not be of uniform size.
2) There could be other sound waves that could interfere with the one under studied.
3) Slight fluctuations in air temperature could affects sound properties.

Using a panel with x fixed at 10% and d fixed at 5


mm, the engineer decided to investigate the
effect of filling the air space with 2 different
absorbers. He obtained the absorption
characteristics as shown in figure 5.2.

Fig. 5.2

Absorption coefficient is an indication of how


much sound wave can be absorbed i.e. the
higher the absorption coefficient, the stronger the
absorption.
(c) Which of the two absorbers would be
preferred in a room in which
(i) absorption over a wide frequency range
was required,
(ii) sounds below 500 Hz were of low
intensity and needed emphasizing ? [2]
(i)

Absorber 2

(ii)
6

Absorber 1

Section B
Answer any two questions in this section.
You are advised to spend about 1 hour in this section.
6

(a) (i)

State Newtons second law of motion.

[1]

The rate of change of momentum of an object is directly proportional to the net external force acting on
it and the change in momentum take place in the direction of the net external force.

(ii)

What is the momentum of a 0.15 kg tennis ball which is moving with a speed of 30
ms-1.
[1]
P = mv = (0.15)(30) = 4.5 Ns

(iii)

This ball is struck by a racket such that after the impact, it moves in the opposite
direction to that of its approaching path. The graph shows the force of racket acting
on the ball.
F/N

300
A impulse graph with longer time
duration and lower maximum
such that the area under graph is
the same as before.

200
100

t/s

0
0.1

0.2

Calculate
1. the change in momentum of the ball.
Change in momentum = Impulse = area under F-t graph = (250)(0.16 - 0.04) = 15 Ns

2. the average force of racket on the ball during impact.


An average force of constant magnitude Fave can deliver the same
impulse over the same time interval.

Fave = 125 N
(Fave)(0.16 - 0.04) = 15

3. the speed of the ball after impact.


P = Pf - Pi
15 = Pf - (- 4.5)
Pf = 10.5 Ns
mvf = 10.5 Ns
vf = 70 ms -1

[5]

Pi
+
Pf

(iv) If the strings of the racket are loosen, draw another graph on the same axes to show
how the force of racket on the ball would be during impact. Assume that the speed of
the ball after impact is the same as what you have found in (6ai-3).
[2]

(b) A uniform flexible tube of rubber (fig. 6bi) is bent into a symmetrical arch (fig. 6bii). The
arch is placed on a rough surface and stay at equilibrium.

Fig. 6bi

Fig. 6bi

(i) Draw a free body diagram of the arch in the space below. The label as well as the
lines of action of each force must be shown clearly.
[2]

Force of pillar
on arch, R1

Force of pillar
on arch, R2

Weight of
arch, W

(ii) The mass of the arch is 50 kg. If a big bird of mass 6.0 kg perches on
the top of the arch, what will be the effect on the magnitude of each of
the force you have draw in part (6bi).
[3]
Weight of arch, W will not be affected.
There is an additional downward force of bird on the arch. Hence the vertical component of each R will
increase.
This additional force will tend to flatten the arch to turn it back into a straight tube. Hence the frictional
force on the ground will increase to oppose this. Horizontal component of R will increase.

(c)

A uniform vertical wheel of radius 5.0 m turn at a uniform rate of 3 rounds per minute.
There is a 2.4 kg bucket at one end of the wheel.
(i) What is the kinetic energy of the bucket ?

[2]

The bucket travels a distance of 3 x the circumference of the wheel in 60


second. Hence speed = 3 ( )(10.0) / 60 = 1.57 ms-1
K.E. of bucket = mv2 = (2.4)(1.57)2 = 2.96 J

5.0 m

bucket
(ii) What is the largest possible increase in gravitational potential energy of this bucket ?
[1]
Largest increase in G.P.E. occurs when the bucket moves from the bottom to the top of the wheel.
Largest increase in G.P.E. = mgh = (2.4)(9.81)(10) = 235 J

(iii) Deduce the maximum instantaneous power supplied to the bucket and its
corresponding position of the bucket. Explain your answer.
[3]
Assuming no dissipative forces, by conservation of energy, since the K.E. remains the same, all the
energy supplied should ideally results in the increase of G.P.E.
Power supplied = rate of increase of G.P.E = mg(vertical component of the velocity)
Hence maximum power = mgv = (2.4)(9.81)(1.57) 37.0 W
The bucket should be at the mid point of the circular motion and on its way up.

(a) (i)

State the principle of superposition

[2]

When waves meet, the resultant displacement at a point in the vector sum of all the individual wave
displacements and the waves proceed as if the rest are not there.

(ii)

State the two conditions for interference


8

[2]

The waves must be coherent.

Their amplitudes must be similar.

(b) Two radio aerial are emitting radio signals that are in phase and coherent. They are
separated by a distance d. A signal is received from an aircraft passing directly overhead
at speed v and height h along a line parallel to that between the aerials as shown below.

2PQ = QR

d
Aerial 1

Aerial 2

The signals from the two aerials are added through an electronic device and the resultant
signal is recorded.
(i)

Q:
P:
R:

Given that Q is equal distance from both aerials and R, Q and P represent the points
where the resultant signal received are maxima, what are the possible path
difference of the signals received by the two aerial at these points ? (2PQ = QR)
Explain your answers clearly.
[3]
Path difference = 0. It is in the middle of the two aerials and the waves must arrive in phase.
Path difference = n where n is an integer larger than 0.
Path difference = 2n since 2PQ = QR (n is an integer larger than 0)

(ii) Explain why the intensity of the signal fluctuates?

[2]

As the aircraft moves, the path difference of the waves from the two aerial changes continuously.
If the path difference is k where k is an integer, the waves arrive in phase and reinforce each other.
Hence a maximum signal is detected.
If the path difference is (k ) where k is an integer, the waves arrive out of phase and cancel each
other. Hence a minimum signal is detected.

(iii) Given that the frequency of the signals emitted by both aerials is 1.5 x 108 Hz and v
= 200 ms-1 , d = 15 m and h = 1500 m, calculate the time for the plane to fly from Q
to R.
[3]
Given x = D/d
x = (3 x 108 / 1.5 x 108)(1500)/15 = 200 m = PQ
QR = 2PQ = 400 m
t = 2.0 s

(iv) In the event that one of the aerial malfunction, suggest why a plane flying directly
over may still detect fluctuation in the signal received.
[2]
The signal from one aerial may be reflected by surrounding objects resulting in interference near the plane

(c)

A wire 2.4 m long is fixed at bit ends. When the wire is plucked, a transverse wave of
speed 320 ms-1 is propagated along the wire and forms a standing wave. Three antinodes
are formed.
(i) Explain briefly how standing waves are formed along the wire.
[2]

At the point where the wire is plucked, two progressive waves are generated and propagated away from the
source towards the 2 end points and reflected back moving towards each other. This will results in two
progressive wave of the same speed, amplitude and frequency traveling in opposite direction superpose forming
a stationary wave.

(ii) Sketch the appearance of the vibrating wire.

[2]

(iii) What is its frequency of vibration?

[2]

(1.5) = 2.4 m

= 1.6 m
f = v / = 320 / 1.6 = 200 Hz

(a) (i)

A 20 W point source of light emits monochromatic light of wavelength 530 nm. What
is the intensity of the light at a distance 5.0 m from the source ?
[2]

A point source emit light uniformly will form a spherical radiation profile.
Area of sphere of 5.0 m = 4r2 = 4(5.0)2 m2.
Intensity = Power / Area = 20 / 4(5.0)2 = 0.0637 Wm-2.

(ii)

In 1 m2 of area at this distance, many photons are detected per second ?

[2]

Intensity = Power / Area = (total energy) / (time)(area)


0.0637 = (no. of photon)(energy of one photon) / (second)(area)
0.0637 = [(no. of photon)/(second)] [(energy of one photon) / (area)]
0.0637 = [(no. of photon)/( second)] [(hc/) / (area)]
0.0637 = [(no. of photon)/( second)] [(6.63 x 10-34)(3 x 108) / (530 x 10-9) / (1)]
(no. of photon)/( second) = 1.70 x 1017

(iii)

A 3.0 cm2 metal is place 5.0 m from this point source. What is the threshold
frequency if the work function energy of this metal is 2.8 x 10-19 J.
[1]

hfo =
(6.63 x 10-34) fo

2.8 x 10-19

fo
10

4.22 x 1014 Hz

(iv)
o = c / fo

Hence use the concept of conservation of energy to explain why photoelectric is


possible ? Show any necessary calculation.
[2]
= (3 x 108) / (4.22 x 1014) = 7.10 x 10-7 m = 710 nm

The energy of each photon is more than enough to break the bond between the electron and the
lattice. Hence photoelectric effect is possible.

(v)

What is the stopping potential for this photoelectric effect experiment ?

hc/ = + eVs
(6.63 x 10-34)(3 x 108) / (530 x 10-9)

(vi)

= (2.8 x 10-19)

[1]

(1.6 x 10-19) Vs Vs = 0.596 V

If the probability of ejecting the electron is 0.05, calculate the electron current.

[3]

In an area of 3.0 x 10-4 m2, there are (1.70 x 1017)(3.0 x 10-4) incoming photons per sec.
There are (0.05)(1.70 x 1017) (3.0 x 10-4) electrons ejected per sec.
Each electron carries a charge of 1.6 x 10-19 C,
= 4.08 x 10-7 A
Electron current = (1.6 x 10-19) (0.05)(1.70 x 1017) (3.0 x 10-4)

(b) In the production of emission and absorption line spectra, briefly describe the difference in
the following areas :
(i)

appearance of spectrum

(ii)

excitation mechanism

[4]

Emission spectrum :

Isolated coloured lines against dark background.


Electrons from the electric current making collisions with electrons in the gas
atoms to excite the latter to higher energy states.

Absorption spectrum:

Relatively dark lines against a background with continuous spectrum


Photons from a white light source collide with the electrons in the gas atoms to
excite the latter to higher energy states.

(c) Fig. 8ci illustrates a set of 5 concentric lines to represent the four lowest energy levels of
an isolated atom. The innermost line represents n = 1 state while the outermost line
represents n = 5 state. Diagrams are not drawn to scale.
n=4
n=3

Balmer
Series

n=5

n=2

n=1

Lyman
Series
Paschen
Series

Fig. 8ci

11

Fig. 8cii

(i)

Why the total energy of an electron residing on one of these energy state must be
negative ?
[1]

The electron is bounded to the atom and could not escape from the atom. Hence the total
energy of this electron must be negative as a free electron has a total energy of > 0 J.

(ii) Fig. 8cii shows three series of transitions by the single electron in the hydrogen atom.
Draw the resulting spectrum due to all these transitions on fig. 8ciii. The relative
position of each spectral line with respect to the rest must be well illustrated. Label
your series.
[3]
Fig. 8ciii
Longest wavelength End
Paschen Series

Shortest Wavelength End


Balmer Series

Lyman Series

Fig. 8ciii
Longest wavelength End

Shortest Wavelength End

(iii) In reality it is hard to obtain isolate hydrogen atoms, usually two hydrogen atoms will
react to become one hydrogen molecule. Describe one change to your spectrum in Fig.
8ciii if liquid hydrogen (consisting of hydrogen molecules) is used.
[1]
Some or all of these isolated lines in the spectrum will become bands or thicker lines.
~ End of Paper ~

12

PIONEER JUNIOR COLLEGE


JC2 PRELIMINARY EXAMINATION
PHYSICS
Higher 1

8866/01

Paper 1 Multiple Choice


17 September 2008
1 hour
Additional Material:

Multiple Choice Answer Sheet

READ THESE INSTRUCTIONS FIRST


Write in soft pencil.
Do not use staples, paper clips, highlighters, glue or correction fluid.
Write your name, class and index number on the Answer Sheet in the spaces provided.
There are thirty questions on this paper. Answer all questions. For each question there
are four possible answers A, B, C and D.
Choose the one you consider correct and record your choice in soft pencil on the
separate Answer Sheet.
Read the instructions on the Answer Sheet very carefully.
Each correct answer will score one mark. A mark will not be deducted for a wrong
answer.
Any rough working should be done in this booklet.

This document consists of 13 printed pages.


[Turn over

2
Data
speed of light in free space,

c = 3.00 10 8 m s1

elementary charge,

e = 1.60 10 19 C

the Planck constant,

h = 6.63 10 34 J s

unified atomic mass constant,

u = 1.66 10 27 kg

rest mass of electron,

me = 9.11 10 31 kg

rest mass of proton,

m p = 1.67 10 27 kg

acceleration of free fall,

g = 9.81 m s2

Formulae

uniformly accelerated motion,

1 2
at
2
v 2 = u 2 + 2as
s = ut +

work done on/by a gas,

W = pV

hydrostatic pressure,

p = gh

electric potential,

V =

resistors in series,

R = R1 + R 2 + ...

resistors in parallel,

1/ R = 1/ R1 + 1/ R 2 + ...

Q
4 o r

3
1

The diagram shows the position of the meniscus of the mercury in a mercury-in-glass
thermometer.

30

32

34

36

38

40

42

44

T / C

Which of the following best expresses the indicated temperature with its uncertainty?
A (39.0 0.5) oC
C (39.0 0.3) oC

B (39.00 0.25) oC
D (39 1) oC

The gravitational field strength at a point is defined as the gravitational force per unit
mass acting at that point.
What is the SI base unit for gravitational field strength?
A m s2

B N kg1

C m2 s2

D kg m s2

A quantity x is measured many times and the number N of measurements giving a


value x is plotted against x. The true value of the quantity is x o .
Which graph best represents accurate but not precise measurements?
A

xo

xo

xo

xo

x
[Turn over

4
4

The acceleration-time graph of an object moving in a straight line is as shown.


acceleration
P
Q

time

If the object starts it motion from rest, at which point is the object moving with the
largest speed?
A P

B Q

C R

D S

A projectile is launched at an angle as shown in the diagram.

Ignoring air resistance, what fraction of its initial kinetic energy does the projectile have
at the top of its trajectory?
A 0

cos

cos 2

sin 2

Which of the following is not a valid example of action and reaction to which Newtons
third law of motion applies?
A The forces of repulsion between an atom in the surface of a table and an atom in
the surface of an apple resting on the table.
B The forces of attraction experienced by each of two parallel wires carrying currents
in the same direction.
C The gravitational force on a pendulum bob suspended by a string and the tension
in the string.
D The forces of attraction between an electron and a proton in a hydrogen atom.

5
7

An object of mass m is hanging by a string from the ceiling of a lift. The lift is
accelerating upwards.
The tension in the string is
A zero.
B less than mg.
C exactly mg.
D greater than mg.

A pendulum is swinging in a vertical plane. The diagram shows the position of the
pendulums bob of mass, m at a particular instant when the pendulum is moving
towards the equilibrium position.

Which of the following diagrams represents the threads tension, T and the
components of the gravitational force acting on the bob at that instant?
A

mg sin
mg cos
mg cos

mg sin

mg cos

mg sin

mg sin

mg cos
[Turn over

6
9

A heavy uniform plank of length L is supported by two forces F1 and F2 at points of


L
L
and
from its end as shown in the diagram.
distances
4
8
L
L
4

L
8

F1

F2

What is the ratio F2 : F1 ?


A 1:3

10

B 2:3

C 3:2

D 3:8

A body moves from X to Y along a track. Its kinetic energy at Y is 25 kJ, and its
potential energy is 30 kJ more than at X.
If the work done by friction along XY is 10 kJ, what is the kinetic energy of the body at
X?
Y

A 35 kJ

11

B 45 kJ

C 55 kJ

D 65 kJ

A car of mass 1500 kg is accelerated from rest to a speed of 100 km h1 on level


ground.
Given that the time taken is 12.1 s, calculate the maximum power delivered by the
engine.
A 1.72 kW

B 47.8 kW

C 95.7 kW

D 620 kW

7
12

A 0.20 kg mass, with an initial kinetic energy of 2.80 J, moves up a smooth slope.
1.0 m

30

What is the kinetic energy of the mass after it has moved a distance of 1.0 m?
A 1.10 J

13

B 1.82 J

C 2.68 J

D 4.50 J

A small source of sound radiates energy equally in all directions. The intensity of the
sound 1.5 m away from the source is 0.18 W m2.
If the power of the source is tripled, the intensity at a distance 3.0 m away from the
source is
A 0.090 W m2.

14

B 0.14 W m2.

C 0.18 W m2.

D 0.28 W m2.

The diagram below shows a displacement-distance graph of a progressive transverse


wave at a particular time. The arrows show the directions in which some of the
particles on the wave are moving at that instant.
displacement / m

4
2
2
4

Q
0

10

distance / m

Which of the following statements about the wave is correct?


A The wave has amplitude 8 m and the phase of the particles on the wave is
constantly changing.
B Energy is being transferred by the wave from the right to the left.
C The wave is moving to the right and its wavelength is 3 m.
D Particles P and Q have a phase difference of radians at the instant shown.

[Turn over

8
15

The diagram shows a cross-section of a water wave moving from right to left.
Which point on the wave is moving upward with maximum velocity?
direction of wave motion
D
B
A
C

16

A standing wave is set up on a stretched string XY as shown in the diagram. P, Q, R


are points on the string.
P
Q
Y

X
R
Which of the following statement is true?
A The distance between X and Q is one wavelength.

B P is always at maximum amplitude while Q is always at zero displacement.


C P and R always move in the same direction.
D P and R are always in anti-phase with each other.

17

In a double-slit experiment, light of wavelength 600 nm gives a fringe separation of


0.40 mm on a screen. With a different monochromatic source, the fringe separation
becomes 0.33 mm on the same screen.
The wavelength of the second source is
A 50 nm.

B 495 nm.

C 660 nm.

D 727 nm.

9
18

The diagram shows a double-slit experiment and the light source from S1 and S2 are
assumed to be of equal amplitude. M is at the centre of the zeroth order bright fringe.
The light intensity at M for the zeroth order fringe is 2I.

monochromatic
light source

S1
M
S2

Which of the following is the best estimate for the light intensity at M when S2 is
covered?
A

19

0.5 I

B I

2I

4I

Two wires A and B, each of the same length and the same material, are connected in
parallel to a battery. The diameter of A is twice that of B.
What fraction of the total current passes through A?
A

20

1
2

2
3

3
4

4
5

Two squares X and Y are cut from a uniform metal plate as shown in the diagram. The
sides of each square are of length L and 2L respectively.

2L

What is the ratio of the resistances

1
4

2L

1
2

RX
between the shaded edges of the squares?
RY
C 1

[Turn over

10
21

A cell of e.m.f. E is connected in series with a resistor R.

The potential difference across R is VR . The potential difference across the internal
resistance r of the cell is Vr .
What is the energy developed across the resistor R in driving unit charge across it?
A VR V r

22

B VR

Er

ER

In the circuit shown below, resistors A and B, of resistances 2R and R respectively, are
connected to a 6 V battery of negligible internal resistance. When a voltmeter is
connected across A, it gives a reading of 3.0 V.
6V

What is the reading of the voltmeter when it is connected across B?


A 0.25 V

B 1.5 V

C 2.0 V

D 4.0 V

11
23

The resistors P and Q in the circuit have equal resistances. The resistance of P is
twice that of R.

Q
The cell, of negligible internal resistance, supplies a total power of 12 W.
What is the power dissipated by heating in resistor Q?
A 2W

24

B 3W

C 4W

D 6W

A potential difference is applied between the electrodes of a gas discharge tube so that
the gas is ionised. The gas carries a current of 8.16 mA and the number of electrons
passing any point in the gas per unit time is 2.58 10 16 s1.
If the charge on each positive particle is 3.2 10 19 C, what is the number of positively
charge particles passing any point in the gas per unit time?
A 1.26 10 16 s1
B

2.58 1016 s1

3.84 1016 s1

D 10.3 1016 s1

[Turn over

12
25

The diagram below shows three arrangements of circular loops, centered on vertical
axes and carrying identical currents in the directions indicated.

Which of the following gives the magnetic field strengths at the midpoints between the
loops on the axes, in order of increasing magnitudes?
A 1, 2, 3
B 2, 1, 3
C 2, 3, 1
D 3, 1, 2

26

A wire of length 5.0 cm is placed at right angles to a magnetic field of flux density
0.040 T. The wire makes an angle with the line PQ and carries a current of 6.0 A.
x

x
6.0 A
P

B = 0.040 T
x

x
Q

If the angle increases from 0o to 360o, which of the following is true about the
magnitude of the force which the field exerts on the wire?
A The force varies between 0 N to 0.012 N.
B The force varies between 0.012 N to 0.12 N.
C The force is constant at 0 N.
D The force is constant at 0.012 N.

13
27

A straight wire AB carrying current I is placed between the magnetic field as shown in
the diagram.
A

I
N

B
In which direction will the wire move?
A to the left
B to the right
C into the plane
D out of the plane

28

The photoelectric effect provides evidence that


A there exist discrete energy levels within atoms.
B the energy of a photon is independent of light intensity.
C light cannot behave as a wave.
D electrons can exhibit wave-like properties.

29

What is a reasonable estimate, to one significant figure, of the energy of a photon of


red light?
A 2 eV

30

B 3 eV

4 10 19 J

5 10 19 J

Which of the following statements is true?


A A beam of electrons directed at a vessel of cold gas can cause the formation of
either an absorption or emission line spectrum.
B A beam of white light directed at a vessel of cold gas can cause the formation of
either an absorption or emission line spectrum.
C A beam of electrons directed at a vessel of cold gas can only cause the formation
of an absorption line spectrum.
D A beam of electrons directed at a vessel of cold gas can only cause the formation
of an emission line spectrum.

End of paper

Name

Class

Index Number

PIONEER JUNIOR COLLEGE


JC2 PRELIMINARY EXAMINATION
PHYSICS
Higher 1
Paper 2 Structured Questions

8866/02
15 September 2008
2 hours

Candidates answer on the Question Paper.


No Additional Materials are required.

READ THESE INSTRUCTIONS FIRST


Write your name, class and index number on all the work you hand in.
Write in dark blue or black pen on both sides of the paper.
You may use a soft pencil for any diagrams, graphs or rough working.
Do not use staples, paper clips, highlighters, glue or correction fluid.
Section A
Answer all questions.
Section B
Answer any two questions.
At the end of the examination, fasten all your work securely together.
The number of marks is given in brackets [ ] at the end of each question or part question.
For Examiners Use
Section A
1
2
3
4
5
Section B
6
7
8
Total
This document consists of 23 printed pages.
[Turn over

2
Data
speed of light in free space,

c = 3.00 10 8 m s1

elementary charge,

e = 1.60 10 19 C

the Planck constant,

h = 6.63 10 34 J s

unified atomic mass constant,

u = 1.66 10 27 kg

rest mass of electron,

me = 9.11 10 31 kg

rest mass of proton,

m p = 1.67 10 27 kg

acceleration of free fall,

g = 9.81 m s2

Formulae

uniformly accelerated motion,

1 2
at
2
v 2 = u 2 + 2as
s = ut +

work done on/by a gas,

W = pV

hydrostatic pressure,

p = gh

electric potential,

V =

resistors in series,

R = R1 + R 2 + ...

resistors in parallel,

1/ R = 1/ R1 + 1/ R 2 + ...

Q
4 0 r

3
Section A

Answer all the questions in this section.


1

Two blocks X and Y, of masses 3.0 kg and 1.2 kg respectively, are connected by a
light inextensible cord passing over a light, free-running pulley. Block X starts from
rest and moves on a smooth ramp inclined at 30 to the horizontal. The initial
position of block X is 1.0 m from the edge of the ramp, as shown in Fig. 1.1.
1.0 m
X
30

1.5 m

ground
Fig. 1.1
(a)

(i)

Calculate the total change in potential energy of the system when block Y
moves up a distance of 0.50 m.

change in potential energy = J [3]


(ii)

Determine the total kinetic energy of the system when block Y moves up
a distance of 0.50 m.

kinetic energy = J [1]

[Turn over

4
(b)

The cord snaps at the instant when block Y just moved up a distance of
0.50 m.
(i)

Calculate the speed of block X at this instant.

speed = m s1
(ii)

[1]

Hence, calculate the speed at which block X leaves the ramp at the edge.

speed = m s1

[2]

5
2

A student throws a ball from point S to a friend at point F. The path of the ball is
shown in Fig. 2.1.
A
B

Fig. 2.1

The points S and F are on the same horizontal level. Air resistance is negligible. The
ball is thrown from point S with velocity v, represented by the vector arrow shown on
Fig. 2.1.
(a)

On Fig. 2.1,
(i)

(ii)

(b)

draw arrows from point S to represent the initial horizontal and vertical
components of the velocity v (label these components vH and vV
respectively).
[1]
draw arrows at A and at B to represent the horizontal and vertical
components of the velocity at these two points.
[2]

(i)

Sketch velocity-time graphs to represent the balls vertical component of


velocity, vV and horizontal component of velocity vH, between point S and
point F.
vV

vH

[2]
(ii)

Explain how the shape of the vV against t graph in (b)(i) will change when
the effect of air resistance cannot be ignored.
.
.
.
.

[3]

[Turn over

3 (a)

A rectangular block of cross-sectional area 600 m2 and height 6.0 m is made of


a new composite material and placed in a pool of water. It floats with 1.6 m of
its height above the water surface. The atmospheric pressure above the
surface of the water is 1.0 10 5 Pa. The water in the pool has a density of
1.0 10 3 kg m-3.

1.6 m

block
water

Fig. 3.1
(i)

On Fig. 3.1, draw two arrows to show the directions of the forces due to
the pressures on the top of the block and on the base of the block.
[1]

(ii)

Calculate the resultant of the two forces in (i).

resultant force = N [2]

7
(b) A uniform plank of mass 20.0 kg and length 3.00 m is hinged to a wall at point
A where it is allowed to rotate freely. The other end, B is tied to a light
inextensible string which is attached to the wall. The plank is held in
equilibrium.

light inextensible string

B
70o

plank

Fig. 3.2
(i)

On Fig. 3.2, draw the free body diagram of the forces acting on the plank.

(ii)

The string at B suddenly breaks. Calculate the moment exerted by the


weight of the plank about point A at this instant.

moment = Nm

[2]

[2]

(iii) The uncertainties in the measurements of length and mass of the plank
are 0.01 m and 0.1 kg respectively. Determine from b(ii), the absolute
uncertainty in the moment exerted by the weight of the plank about point
A (Assume there is no uncertainty in the measurement of angle). Express
the moment together with its uncertainty.

moment = () Nm [3]

Fig. 4.1 shows two coherent loudspeakers S1 and S2 placed 4.0 m apart. D is a
detector placed in the same horizontal plane as the loudspeakers, 12.0 m away
from S2. When the loudspeakers are switched on, sound of frequency 1780 Hz is
emitted from the two loudspeakers in antiphase. The lines S1S2 and S2D are
perpendicular to each other.

S1

4.0 m

S2

12.0 m

Fig. 4.1
(a) Given that the speed of sound in air is 330 ms-1, calculate the wavelength of the
sound emitted from S1 and S2.

= ........................................ m [1]
(b) Calculate the path difference, in terms of , between the sound waves reaching
D from S1 and S2. You may assume that the two loudspeakers and the detector
are point objects.

path difference =

[2]

(c)

Explain whether D would detect a minimum or maximum intensity.


....
....
....
.... [3]

[Turn over

10

Ultrasonic sound waves (ultrasound) have frequencies outside the audible range of
the human ear, that is, greater than about 20 kHz. As ultrasound passes through a
medium, wave energy is absorbed. The rate at which energy is absorbed by unit
mass of the medium is known as dose-rate. The dose-rate is measured in W kg-1.
The total energy absorbed by unit mass of the medium is known as the absorbed
dose. This is measured in J kg-1 or kJ kg-1.
Under certain circumstances, biological cells may be destroyed by ultrasound. The
effect on a group of cell is measured in terms of the survival fraction (SF),
SF =

number of cells surviving after exposure


number of cells before exposure

For any particular absorbed dose, it is found that the survival fraction changes as
the dose-rate increases. Fig. 5.1 shows the variation with dose-rate of the survival
fraction for samples of cells in a liquid. The absorbed dose for each sample cells
was 240 kJ kg-1.
0.080

0.070

0.060

survival fraction

0.050

0.040

0.030

0.020

0.010

0.000
0

50

100

150

200

250

300

350

dose rate / W kg-1

Fig. 5.1
(a)

(i)

Read off from Fig. 5.1 the survival fraction for a dose-rate of 200 W kg-1.
survival fraction = [1]

11
(ii)

Calculate the exposure time for an absorbed dose of 240 kJ kg-1 and at
a dose-rate of 200 W kg-1.

exposure time = s [2]


Survival fraction depends not only on dose-rate but also on absorbed dose.
Fig. 5.2 shows the variation with dose-rate of log10(SF) for different values of
absorbed dose.
dose-rate / W kg-1
0

50

100

150

200

250

300

350

0.00

-0.50

50 kJ kg-1
100 kJ kg-1

-1.00

160 kJ kg-1

-1.50

log10 (SF)

(b)

240 kJ kg-1
-2.00

340 kJ kg-1
-2.50

450 kJ kg-1

-3.00

-3.50

560 kJ kg-1
-4.00

Fig. 5.2

The bold line represents the data given in Fig. 5.1, but with survival fraction
plotted on a logarithmic scale.
(i)

Suggest a reason for plotting survival fraction on a logarithmic scale.

[1]

[Turn over

12

By reference to Fig. 5.2, complete the table of Fig. 5.3 for a dose-rate of
[2]
200 W kg-1.

(ii)

absorbed
dose / kJ kg-1
50

log 10 (SF)

100
160
240
340
450
560
Fig. 5.3
(iii) Using the relevant value of log10 (SF) from Fig. 5.3, calculate the
survival fraction for an absorbed dose of 160 kJ kg-1 at a dose-rate of
200 W kg-1.

survival fraction = [1]


Use your values in the table of Fig. 5.3 to plot, on the axes of Fig. 5.4, a
graph to show the variation with absorbed dose of log10 (SF) for the dose-rate
[2]
of 200 W kg-1.

(c)

dose / kJ kg
0

50

100

150

200

250

300

0.00
-0.50
-1.00

log 10 (SF)

-1.50
-2.00
-2.50
-3.00
-3.50
-4.00

Fig. 5.4

350

-1

400

450

500

550

600

650

13

Section B

Answer two questions from this section.


6 (a)

An object falling experiences a resistive force, F = kv where k is a constant and


v is the terminal speed.
(i)

Determine the SI base units of k.

SI base units of k = [2]


(ii)

A student defines speed as distance travelled per second. Explain clearly


why this statement is technically wrong. Hence, write down the correct
definition for speed.
....
....
....
.... [2]

(iii) The student attempts to determine the terminal speed v by measuring the
time t taken for the ball to fall a vertical height h.

Suggest two reasons why, although the value of t is measured precisely, it


may not be accurate.
1. ...
....
2. ...
.... [2]

(b) (i)

State Newtons second law of motion.


....
.... [1]

[Turn over

14
(ii)

Show that this definition leads to the equation F = ma .

[2]

(iii) State the principle of conservation of momentum.

....
.... [1]
(c)

An army marksman, together with his rifle and bullets, has a combined mass of
90 kg. He stands at rest on a pair of ice skating blades and fires 10 shots in
quick succession horizontally in the forward direction from his rifle. Each bullet
has a mass of 1.0 10 2 kg and leaves the muzzle with a speed of 750 m s1.
(i)

Calculate the total momentum of the 10 shots of bullets.

momentum = kg m s1 [1]
(ii)

If the marksman slides backwards, estimate his speed at the end of the 10
shots.

speed = m s1 [2]

15
(iii) If the shots were fired in 3 seconds, what was the average force exerted
on the marksman and his rifle?

average force = N [3]


(d) The shots from the rifle emit loud sounds of frequency 4.0 kHz when it is fired.
The sounds cause vibration of the diaphragm of the marksmans eardrum. The
speed of sound is 330 m s1.
(i)

Sound is a longitudinal wave. Explain what is meant by a longitudinal


wave.
....
.... [1]

(ii)

The distance between the source of the sound to the marksmans eardrum
is 0.103 m. Calculate the phase difference of the sound wave between the
source of the sound and the eardrum.

phase difference = rad [3]

[Turn over

16
7

(a)

Sketch the magnetic field lines due to a long straight current carrying wire.
Indicate clearly the direction of the magnetic field.
[2]
Top view
(Current flowing into the paper)

(b)

Two long straight parallel wires are separated by a distance d. Each carries
current I in the opposite directions. Explain the origin of the forces which exist
between the two wires and predict the directions of the forces.

[3]

17
(c)

In an experiment to determine the magnetic flux density due to a magnet, a


wire frame ABCD supported on two knife edges P and Q is placed horizontally
next to the magnet as shown in Fig. 7.1. Sides BC and AD are 5.0 cm and
sides AB and DC are 8.0 cm. P and Q are at the midpoints of AB and DC
respectively. When there is no current in the circuit, the frame is balanced
horizontally.
to battery
A

I
P

Q
I
I

to battery
side view
D

Fig. 7.1

(i)

When there is a current flowing as shown in Fig 7.1, state the directions of
the magnetic force, if any, on
1. side QC,
[1]
2. and side BC.
[1]

(ii)

A mass of 21.0 g has to be placed on side AD to balance the wire frame


when there is a current of 2.0 A. Find the magnetic field strength
experienced by side BC.

magnetic field strength = T [3]

[Turn over

18
(d) Use energy considerations to distinguish between electromotive force (e.m.f.) and
potential difference (p.d.).

.......................................................................................................................................
.......................................................................................................................................
......................................................................................................................................[2]
(e) A filament lamp (rated 12 V, 24 W) is connected to a battery of e.m.f. 12 V and
negligible internal resistance.
(i) Sketch the I-V characteristics of a filament lamp.

[1]

(ii) Calculate the resistance of the filament lamp when it is working normally.

resistance =

[2]

(iii) State and explain the effect on brightness of the filament lamp if another filament
lamp is connected in parallel to it.

[2]

19
(f) The lamp in (e) is connected to the circuit in Fig. 7.2. AB is a variable resistor with
movable contact C. The total resistance of AB is 16 . The cell has e.m.f. E and
negligible internal resistance. The variable resistor is used as a potential divider to
provide a p.d. to the bulb. When the resistance of AC is 4.0 , the lamp operates
normally. Calculate the value of E.

A
C

Fig. 7.2

E = V

[3]

[Turn over

20
8

(a)

Electrons are diffracted when they travel through a thin sample of graphite.
State what is demonstrated by the diffraction of electrons.
..
... [1]

(b) Material scientists use the diffraction of electrons to study the atomic structure
of materials. The atoms and spacing between the atoms diffract the electrons.

Calculate
(i)

the speed of an electron with a de Broglie wavelength of 3.4 x 10-11 m.

speed = m s-1 [3]


(ii)

the kinetic energy of the electron in electronvolt (eV).

kinetic energy = eV [2]

21
(c)

Explain what is meant by a photon.


...
... [1]

(d) A photoelectric emission experiment is shown in Fig. 8.1. Electromagnetic


radiation is illuminated at metal plate E. The emitted photoelectrons move
towards plate C and current I is detected in the ammeter. A potential
difference V is applied across the metal plates.

Plate C
variable d.c.
supply

Plate E

Fig. 8.1

[1]

(i)

On Fig. 8.1, indicate the direction of current I in the circuit.

(ii)

Ultraviolet radiation of wavelength 254 nm and of intensity 210 W m-2


was incident on plate E, so that an area of 12 mm2 was illuminated.
A saturation current of 4.8 x 10-10 A was detected.
Calculate
1. the energy of the ultraviolet radiation.

energy = J [1]
2. the rate of incidence of photons on plate E.

rate of incidence = ...s-1 [2]

[Turn over

22

3. the rate of emission of photoelectrons.

rate of emission = s-1 [2]

(iii)

The work function energy of metal plate E is 2.0 eV. Calculate the
maximum kinetic energy of the photoelectrons.

maximum kinetic energy = . J [2]

(iv)

Hence, calculate the stopping potential.

stopping potential = V [1]

23

(v)

Sketch a graph to represent the variation of photoelectric current with


potential difference for this photoelectric experiment.

[2]

photoelectric current / x10-10 A

potential difference / V

(vi)

When the experiment was repeated with visible light of wavelength


650 nm, no photoelectrons were emitted. Explain this observation.

End of Paper

[2]

1
Answers to PJC Prelim H1 Physics Paper 1
1
2
3
4
5

A
A
C
B
D

6
7
8
9
10

C
D
D
B
D

11
12
13
14
15

B
B
B
B
A

16
17
18
19
20

D
B
A
D
C

21
22
23
24
25

B
B
B
A
C

26
27
28
29
30

D
C
B
A
D

Suggested Solutions:
1

A
The error of reading is half the smallest division of the measuring instrument.

g=

F
m

Base units of g =

kg m s 2
kg

= m s 2
3

C
An accurate measurement is one where the readings taken are close to the true value,
while a precise measurement is one where the readings taken are very close to one
another.

B
The acceleration-time graph is obtained from the velocity-time graph. Point Q is the
turning point of the velocity-time graph and so is the maximum value.

Fraction =

(u sin )2
u2

C
Action-reaction pair do not act on the same body.

By Newtons second law of motion,

T
a
mg

T mg = ma
T = ma + mg
Hence, T > mg.

RESTRICTED

2
8

mg sin

mg cos
W

B
Taking moments about the centre of gravity of the plank, the clockwise moment must
be equal to the anti-clockwise moment.
L
3L
F1 = F2
4
8
F2 2
=
F1 3

10

D
loss in K.E. = gain in G.P.E. + work done against friction
K.E. at X 25 = 30 + 10
K.E. at X = 65 kJ

11

B
power = work done / time
= gain in K.E. / time

100 10 3
1
1500
3600
2

47.8 kW
12

12.1

B
loss in K.E. = gain in G.P.E.
2.80 K.E.f = 0.20 9.81 1.0 sin 30
K.E.f 1.82 J

RESTRICTED

3
13

intensity =

power
1
, I 2
area
r

P1
A1
P1 = I1A1
P2 = I 2 A2
P2 I 2 A2
=
P1 I1A1

I1 =

3=

I 2 ( 4 )(3.0)2
0.18( 4 )(1.5)2

I 2 = 0.135 W m2
14

B
We need to first assume that the wave is progressing from the right to the left. By
drawing the wave profile at the next instant, it can then be verified that the particles are
moving in the same directions as the corresponding particles shown in the diagram.
Hence the assumption is correct and it can then be deduced that energy is being
transferred from the right to the left.

15

A
The maximum velocity of the particle is at the equilibrium position. By drawing the
wave profile for the next instant, it can be seen that point A is moving upwards with the
maximum velocity.

16

D
Particles in adjacent loops move in anti-phase with one another, while particles within a
loop are all in phase.

17

Using x =

D
a

600 10 9 D
---------- (1)
a
D
=
---------- (2)
a

0.40 10 3 =
0.33 10 3

Solving,
0.33 10 3
=
600 10 9
0.40 10 3
= 495 nm

RESTRICTED

4
18

I A 2 (intensity proportional to square of amplitude)


When both S1 and S2 are opened, the amplitude at M is 2A. When only one source is
present, the amplitude at M is just A.

(2A)2 2I

4A2 = k(2I)
Hence, A2 = k(I/2)
A2
19

I
2

D
dA = 2dB
l
l
R=
=
A d 2

1
R 2
d
2
1
R A (d B )
=
=
2
4
R B (d A )

Using I AR A = I B RB ,
R
I A = B IB = 4IB
RA
4
Therefore, I A = of the total current.
5
20

Since R =
R=

R=

L
A

, for the same thickness d,

Ld

Hence, R is independent of the length, and so the ratio


21

RX
is 1.
RY

B
Energy per unit charge across R is the potential difference across R.

RESTRICTED

5
22

6V

When a voltmeter is connected across A, it gives a reading of 3 V. This concludes that


RXY = RYZ = R
RXY = R RV = 2R
When voltmeter is connected across resistor B, combined resistance across YZ,
R 2R
RYZ =
R + 2R
= 2R/3
2R
3
VYZ =
6 = 1.5 V
2R
+ 2R
3
23

B
Let the current through the whole circuit be I, then the current through P and Q is I/2
respectively.
The total power developed in the circuit is 12 W. Hence,
power across P + power across Q + power across R = 12
2

I
I
2
2R + 2R + I R = 12
2
2

I
2R = 3 W
2
24

I = I + I+
Nq
Nq
+

t t +

I =

8.16 10 3 = 2.58 1016 1.60 10 19 + (

N
3.2 10 19 )
t

N
= 1.26 10 16 s1
t
25

RESTRICTED

6
26

D
F = BIL sin
= angle between B and I.
= 90o regardless of angle .
F = BIL sin
= (0.040) (6.0) (0.050) sin 90
= 0.012 N

27

C
Using Flemings left hand rule, the wire will move into the plane of the paper.

28

B
Photoelectric effect demonstrates light can be treated as particles and energy of the
photon is independent of intensity. It does not prove that light cannot behave as a
wave.

29

A
Wavelength of red light 700 nm
hc (6.63 10 34 )(3.0 10 8 )
E=
=
3 10 19 J 2 eV (1 s.f.)
9

700 10

30

D
For absorption spectrum, there must be a whole spectrum of light (white light) passing
through the gas to provide photons of different wavelengths that are equivalent to the
differences in energy levels of the cold gas atoms. (A and C wrong)
When components of white light are absorbed by cold gas atoms and photons are reemitted by the gas atoms, they are emitted in all directions.

RESTRICTED

1
Answers to PJC Prelim H1 Physics Paper 2
Suggested Solutions:
1 (a)(i)

total change in g.p.e. by system

[1]for correct
change in g.p.e
formulae
[1]for correct
substitution
[1] for correct
answer

= m x gd m x gd sin 30
= (1.2 9.81 0.50 ) (3.0 9.81 0.50 sin 30)
= 1.4715 J
1.47 J
(ii)

(b)(i)

(ii)

Total gain in k.e. by system


= total loss in g.p.e. by system
= 1.47 J

[1] for correct


answer

Since total gain in k.e. by system = 1.4715 J,


1
(m X + mY )v 2 = 1.4715
2
1
(4.2)v 2 = 1.4715
2
v 0.837 m s1

[1] for correct


answer

Let the take-off speed at the ramp be v ' . Considering the next [1] for correct
substitution
0.5 m moved by block X, we have
2
2
v ' = v + 2as
[1] for correct
v ' 2 = 0.837 2 + (2 9.81sin 30 0.50 )
answer
v ' 2.37 m s1

2.(a)(i)
(ii)

B
vV

Magnitude of
vH should
remain
constant. vv at
B is shorter
than that at S.

vH

vH

vV

F
vH

(b)(i)

Fig. 2.2

Taking upwards as positive


vV

vH

[1] for correct


vv against t
graph. Area
under the
graph should
be same for
upwards and
downwards
motion.
[1] for correct
vH against t
graph

RESTRICTED

(b)(ii)

The gradient of the vv against t graph will increase at first and then
decrease. The graph will be a curve. Air resistance will increase
the deceleration of the ball when it is moving upwards, and
decrease the acceleration when it is moving downwards. The vv will
reach zero velocity at a shorter time and the area under graph before
vv = 0 will be smaller.

3(a)(i)

1.6 m

block
water

(a)(ii)

Resultant force = (Patm + P) A - PatmA


= PA
= hgA
= (6-1.6)(1x103)(9.81)(600)
= 2.59x107 N

(b)(i)

light inextensible string

Normal Reaction
Force

70o

Tension

B
plank

Weight

(b)(ii)

Taking moment about A,


Moment about A = mA x g x 1.50 sin 70o
= 276.55 Nm
= 277 Nm
RESTRICTED

[1] Direction of
both arrows.
Magnitudes
(lengths) of
arrows are not
tested here as
question only
asks for the
direction. If
magnitude is
considered,
the bottom
arrow should
be longer.
[1] Correct
formulae F=PA
[1] correct
substitution
and answer

3
(b)(iii)

moment A ma L A
=
+
moment A
ma
LA
moment A
0.1 0.01
=
+
= 0.011667 (g and sin 70o have no errors)
20.0 1.50
moment A
momentA = 0.011667 x 276.55 = 3.23 Nm
= 3 (1 sf)
Moment of weight of beam OA about A = (277 3) Nm

[1] correct
formuale
[1] calculate
the uncertainty
at A
[1] moment
together its
uncertainty

[1] for correct


answer

4(a)

4(b)

S1 D = 12 2 + 4 2

v
f
330
=
1780
= 0.185m

= 12.649 m
Path difference = 12.649 12

0.649

0.18539
= 3.5

[1]

[1]

Phase difference at D is 0 rad.[1] Since the two sources are in


antiphase, the waves meet at D exactly in phase and there is
constructive interference.[1] Hence a maximum intensity is
detected.[1]

4(c)

5 (a)(i)

(ii)

(b)(i)

From Fig.5.1
survival fraction = 0.027
exposure time =

absorbed dose 240000


=
s = 1200 s
dose rate
200

[1] for
correct
answer
[1] for
correct
answer

The survival fraction decreases to a very small value when dose rate [1]
increases. The change in survival fraction will not be apparent as it
fluctuates over a very small range. Plotting on a logarithmic scale is
essential for the trends to be observable.

RESTRICTED

4
(ii)

absorbed
dose / kJ kg-1
50

log 10 (SF)
-0.65

100
160

-1.20

240

-1.55

340

-2.15

450

-2.85

560
(iii)

-0.80

-3.75

From Fig 5.3,


lg10 (SF) = -1.2

[1]

SF = 0.063
(c)
dose / kJ kg
0

50

100

150

200

250

300

350

-1

400

-0.5
-1

log10 (SF)

-1.5
-2

-2.5
-3

-3.5
-4

RESTRICTED

450

500

550

600

65

6 (a)(i)

F = kv

Base units of k = kg m s1 (m s1) 1


= kg s1

[1] for correct


units for F and
k
[1] for correct
manipulation
and base
units

(ii)

Distance is a physical quantity while second is a unit. The physical [1] for
quantity speed should be defined in terms of quantities, and not a appreciating
mixture of a quantity and a unit.
the difference
between
quantities and
units
The correct definition for speed is the distance travelled per unit [1] for correct
time.
definition of
speed

(iii)

(b)(i)

(ii)

The timer may be calibrated incorrectly leading to a systematic [2] for any
error in the recording of time.
reasonable
answer
There may be time lag in the starting and ending of the timer due to
human reaction time.
Newtons second law of motion states that the rate of change of
momentum of a body is directly proportional to the net external
force on the body, and the change in momentum is in the direction
of the force.

[1] for
recognizing
the
relationship
between rate
of change of
momentum
and force, and
the direction

dp
dt
d (mv )
F=
dt
dm
dv
F =v
+m
dt
dt

F=

For constant mass,

dm
= 0 . Therefore,
dt

dv
dt
F = ma

F =m

(iii)

[2]

The principle of conservation of momentum states that the total [1]


momentum of a system remains constant provided that no external
forces act on the system.
RESTRICTED

6
(c)(i)

(ii)

Total momentum = Nmv


= 10 1.0 10 2 750
= 75 kg m s1

[1] for correct


answer

By principle of conservation of momentum,


MV = Nmv
90 10 1.0 10 2 V = 75
V 0.834 m s1

[1] for correct


substitution
[1] for correct
answer

By Newtons second law of motion,


dp
F=
dt
75 0
F=
3
F = 25 N

[2] for correct


substitution
and answer

(iii)

)]

[1] for using


Newtons third
law

Therefore, the force on the bullets is 25 N.


By Newtons third law of motion, the force on the marksman and his
rifle is 25 N.
(d)(i) A longitudinal wave is a wave whose particles vibrate in a direction [1]
parallel to the direction of wave travel.
(ii)

v = f
330 = 4.0 10 3
= 0.0825 m

[1]

Number of wavelengths between source and marksmans eardrum


0.103
=
0.0825
1.25

[1]

Phase difference

0.25

1.57 rad

RESTRICTED

[1]

[1] for correct


shape

7 (a)

[1] for correct


I direction
and Bdirection
x I

(b)

Current out

The current in wire 1 produces a circular magnetic field cutting


wire 2, as illustrated in the diagram.
according to Flemings left-hand rule, the resulting magnetic
force on wire 2 is away from wire 1.
By similar consideration, the magnetic force on wire 1 can be
shown to be the same magnitude and away from wire 2.
(c)(i)
(c)(ii)

1. no direction
2. downwards
to balance, torque on frame = 0
taking pivot at P and Q,
mg = BIL
(0.021)(9.81) = B(2.0)(0.05)
B = 2.1 T

RESTRICTED

[1]
[1]
[1]
[1]
[1]
[1]
[1]
[1]

(d)

The e.m.f. of a source is the electrical energy converted from


other forms of energy per unit charge moved round a complete
circuit. [1]
The p.d. between two points is the amount of electrical energy
converted into other forms of energy per unit charge moved
across the two points. [1]

(e)(i)

[2]

I/ A

V /V

(e)(ii)

V2
P
12 2
R=
[1]
24
R = 6.0 [1]

(e)(iii)

The brightness will remain the same. The potential difference [2] Award
across the lamp will remain the same when another lamp is marks only if
explanation is
connected in parallel to it.
correct
Let R be the effective resistance across CB and the bulb in
parallel.

(f)

R=

R=

12 6
= 4.0 [1]
12 + 6

Since resistance of AC is also 4.0 , p.d. across bulb equals half of


E. [1]
Since p.d. across bulb = 12 V (working normally)
E = 2 x 12 = 24 V [1]

8(a)
(b)(i)

The wave property of particles.


23
6.63 10 31
= 1.95 10
3.4 10 11
mv = 1.95 10 23

p=

v=

[1]
[1]
[1]

1.95 10 23
9.11 10 31

= 2.14 x107 m s-1

[1]

RESTRICTED

9
(b)(ii)

(c)

K.E = mv2
= (9.11x10-31)(2.1405x107)2
= 2.08698 x10-16 J
= 1304 eV

[1] correct
substitution
[1] correct
answer

A photon is a quantum of electromagnetic radiation of energy hf, [1]


where f is the frequency of the radiation.
[1] correct
direction

(d)(i)
I
Plate C
Variable d.c.
supply

Plate E

(d)ii)
1.
2.

3.

(d)(iii)

Energy =

hc

(6.63 10 34 )(3 10 8 )
= 7.83 x 10-19J
254 10 9

Power , P = IA = 210(12 10 6 ) = 2.52 10 3


N (hf )
=P
t
N
P
2.52 10 3
=
=
= 3.22 1015 s 1
19
t
hf 7.8307 10
ne
=I
t
n I 4.8 10 10
= =
= 3.0 10 9 s 1
19
t e 1.6 10
1
hf = + ( mv 2 ) max
2
1
( mv 2 ) max = 7.8307 10 19 2(1.6 10 19 )
2
= 4.63 x 10-19 J

(d)(iv)

[1]

[1]
[1]
[1]
[1] correct
substitution
[1] correct
answer

eVs =( mv2)max

1
( mv 2 ) max
4.6307 10 19
=
= 2.89V
Vs = 2
e
1.6 10 19

(dv)

[1]

Photoelectric current/ 10-10 A

RESTRICTED

[1]

10

[1] Correct
shape
[1] Correct
label of
values.

4.8

-2.9

(dvi)

Potential difference / V

The energy of the visible light photon of 650 nm is 1.91 eV, which
is less than the work function energy 2.0 eV. Hence, it is unable to [1]
remove electron from the metal surface and there is therefore no [1]
emission of electrons.

RESTRICTED

RAFFLES JUNIOR COLLEGE


2008 Preliminary Examination

PHYSICS
Higher 1

8866 / 01

Paper 1 Multiple Choice


19 September 2008
1 hour
Additional Materials:

OMR form
Soft clean eraser
Soft pencil (type B or HB is recommended)

READ THESE INSTRUCTIONS FIRST


Do not open this booklet until you are told to do so.
Fill in your particulars on the OMR form.
There are thirty questions on this paper. Answer all questions. For each question
there are four possible answers A, B, C and D. Choose the one you consider correct
and record your choice in soft pencil on the OMR form.
Read the instructions on the OMR form very carefully.
Each correct answer will score one mark. A mark will not be deducted for a wrong
answer.
Any rough working should be done in this booklet.

This booklet consists of 13 printed pages including the cover page.

Data

speed of light in free space,

c = 3.00 x 108 m s1

elementary charge,

e = 1.60 x 10-19 C

the Planck constant,

h = 6.63 x 1034 J s

unified atomic mass constant,

u = 1.66 x 1027 kg

rest mass of electron,

me = 9.11 x 1031 kg

rest mass of proton,

mp = 1.67 x 1027 kg

acceleration of free fall,

g = 9.81 m s2

Formulae

uniformly accelerated motion,

s = ut + at2
v2 = u2 + 2as

work done on/by a gas,

W = pV

hydrostatic pressure,

p = gh

resistors in series,

R = R1 + R2 +

resistors in parallel,

1/R = 1/R1 + 1/R2 +

A student measured the speed of sound as 328.24 m s -1. He estimated the percentage
uncertainty as 2.5%. Which one of the following gives his result to the appropriate
number of significant figures?
A

300 m s-1

330 m s-1

328 m s-1

328.2 m s-1

Four physical quantities W, X, Y and Z are related by the equation W = kX + YZ where k


is a dimensionless constant. Which of the following statement is true?
A

W, X, Y and Z are all vector quantities.

W, X, Y and Z all have the same units.

The product YZ is numerically equal to W - kX.

D
3

The product YZ has the same units as W and kX.

A golf ball is released from rest so that it falls vertically to the ground and bounces back
again. Taking velocity downwards as positive, which one of the following graphs best
represents the variation of velocity v versus time t?
A

Two particles X and Y are travelling along a straight path PQ of length 20 m. X leaves
P, heading for Q, from rest with a uniform acceleration of 2.0 m s-2 and at the same
time, Y leaves Q, in the direction of P, from rest with a uniform acceleration of 5.0 m s-2.
Determine how far is X from P when the two particles collide?
A

3.7 m

5.7 m

7.2 m

1.3 m

2.6 m

5.2 m

8.3 m

A helicopter of mass 5.0 x 103 kg rises vertically with a constant speed of 25 m s-1. What
is the upward resultant force acting on the helicopter?
A

Assuming a constant deceleration of 300 m s-2, how far will the front of a car collapse if
the car is travelling at a speed of 28.0 m s-1 just before impact and subsequently comes
to a complete stop?
A

2.4 m

0N

4.9 x 104 N

9.8 x 104 N

1.3 x 105 N

A man of mass M is standing on a weighing scale in an elevator. The elevator


undergoes several different motions as described below.
Elevators motion

Reading on weighing scale

Moving upward with a uniform


acceleration.

R1

Moving upward and coming to rest.

R2

Moving downward with uniform velocity.

R3

Which one of the following is correct?


A
B

R1 < R3 < R2

R2 = R3 < R1

R1 = R2 = R3

R2 < R3 < R1

A light spring has a mass of 0.30 kg suspended from its lower end. A second mass of
0.20 kg is suspended from the first by a thread. The arrangement is allowed to come
into static equilibrium and the thread is burned through. At this instant, what is the
upward acceleration of the 0.20 kg mass?
A

0.0 m s-2

6.5 m s-2

9.8 m s-2

16 m s-2

Forces of 3 N and 8 N act at a point. Which one of the following could not be the
magnitude of their resultant?
A

1N

5N

6N

11 N

10

A cup half-filled with water resting on a weighing scale registers a reading W. When a
boy dips his finger into the water without touching the base, the reading of the weighing
scale is W.

Which of the following statements is correct?


A
B

W < W because the water exerts an upthrust on the boys finger.

W > W because the water exerts an upthrust on the boys finger and as a result,
the boys finger exerts a force back on the water.

11

W = W because the water has not overflowed from the beaker.

W > W because the weight of the boys finger is added to that of the water in the
beaker.

A uniform horizontal beam of length L and weight 200 N is attached to the wall by
means of a cable as shown in the figure below. A boy weighing 500 N, stands
away from the wall. The tension in the cable is

cable
60o
beam

200 N

231 N

500 N

808 N

1
L
5

12

A block slides from rest down a frictionless ramp of height h. It reaches speed v at the
bottom. To reach a speed of 2v, the block would need to slide down a ramp of height.
A

13

2h

3h

5.00 s

10.0 s

15.0 s

20.0 s

Water waves travel across a ripple tank. The horizontal distance between a crest and
the neighbouring through is 25 mm and the vertical distance between a crest and a
trough is 5.0 mm. A crest travels 8.0 cm in 0.50 s. Which one of the following is correct?
Frequency of the water wave / Hz

Amplitude of the water wave / mm

3.2

2.5

3.2

5.0

6.4

2.5

D
15

4h

A car consumes 500 kJ of energy to accelerate uniformly from rest to a final speed of v.
The power output of the car's engine increases to a maximum value of 100 kW at speed
v. How long does it take the car to reach a speed of v?
A

14

1.41h

6.4

5.0

The figure below shows how the displacements of air molecules, in a longitudinal
progressive wave, from their mean positions vary with the distances of their mean
positions from the source O, at a particular instant. The displacement to the right is
taken to be positive. Which of the following statements is correct?

At this instant,
A

points marked P, Q, R, S, T, U and V are rarefactions.

points marked P, Q, R, S, T, U and V are compressions.

points marked Q and U are rarefactions and the point S is a compression.

points marked Q and U are compressions and the point S is a rarefaction.

16

Plane waves of wavelength in a ripple tank approach a straight barrier parallel to the
wave crests. There is a gap of width w in the middle of the barrier. Which of the
following and w will produce the largest diffraction?

/ cm
A

0.5

2.0

0.5

4.0

1.5

2.0

17

w / cm

1.5

4.0

A sound wave of amplitude 0.20 mm has an intensity of 3.0 W m2 . What will be the
intensity of a sound wave of the same frequency which has an amplitude of 0.40 mm?
A
B

6.0 W m2

9.0 W m2

18

4.2 W m2

12 W m2

Sound from a small loudspeaker L reaches a point P by two paths which differs in length
by 1.2 m. When the frequency of the sound is gradually increased, the result intensity at
P goes through a series of maxima and minima. A maximum occurs when the frequency
is 1000 Hz and the next maximum occurs at 1200 Hz. What is the speed of sound in the
medium between L and P?
A

200 m s1

240 m s1

480 m s1

1200 m s1

19

The diagram below shows a string with ends P and T fixed. The string is made to vibrate
transversely so that P, R and T are the only points on the string which are nodes.
L

L
x

The vibration of the two points Q and S, shown on the string, have
A
B

different amplitudes and are in phase.

different amplitudes and differ in phase by 180.

20

the same amplitude and are in phase.

the same amplitude and differ in phase by 180.

A student tries to use a wire to construct the letters RJC. His letter R consists of a
semi-circle and part of a right angled triangle as shown in the figure below.

l
X

Given that the wire has a resistance of r for every length l. What is the effective
resistance of the letter R between points X and Y?
A

1.16 r

2.12 r

1.73 r

4.69 r

21

Cells that have the same e.m.f. E but different internal resistances are used to power
two lamps, either connected in series or parallel, as shown in the figures below.

P1

S1
E

1
2

1
P2

S2
E

2
1

Which of the following combination is in order of decreasing brightness?


A

22

P2 S2 S1 P1

P1 S1 S2 P2

P2 P1 S2 S1

S1 S2 P1 P2

The diagram shows two wires OP and PQ of equal length, joined in series with a cell. A
voltmeter is connected between the end of PQ and a point X on the wires. The e.m.f.
across the cell is . Wire PQ has twice the area of cross-section and twice the resistivity
of wire OP.

Assuming that the cell has no internal resistance, what is the ratio

potential difference across OP


?
potential difference across PQ
A

0.25

0.50

1.00

2.00

10

23

A battery of e.m.f. is connected in series with a resistor R of 2.00 and a switch as


shown below. A voltmeter connected across the battery reads 10.0 V when the switch is
open and 6.00 V when it is closed.
V

What is the internal resistance of the battery?


A

24

0.75

0.80

1.33

4.00

A cell of e.m.f. is connected to two identical resistors X and Y of 6 each. The cell
has an e.m.f. of 12 V and internal resistance 3 .
X

Y
What is the current in Y?
A

0.5 A

1.0 A

2.0 A

4.0 A

11
25 The figure below shows a square coil WXYZ of sides 0.25 m, lying in a vertical plane
and carrying a current I of 2.0 A. The magnetic flux density B of 0.010 T is parallel to XY.

Y
B

Z
I

What is the magnitude of the torque and its direction when viewed from the top?
A

1.3 10-3 N m, anticlockwise

1.3 10-3 N m, clockwise

2.5 10-3 N m, anticlockwise

2.5 10-3 N m, clockwise

26 The diagram below shows three parallel wires X, Y and Z placed in a horizontal plane.
Wires X and Z carry current I in opposite directions. Wire Y carries a current of 3I in the
direction shown and is equidistant from the other wires.

X
d
d

3I

If the magnitude of the force per unit length acting between two parallel wires placed a
distance d apart, each carrying a current of I, is 2.0 10-6 N m-1, what is the direction
and magnitude of the net force per unit length acting on wire Z?
[The force per unit length is proportional to the product of the currents in the two wires
and inversely proportional to the separation between them.]
Direction

Magnitude

Towards Y

2.0 10-6 N m-1

Towards Y

5.0 10-6 N m-1

Away from Y

5.0 10-6 N m-1

Away from Y

7.0 10-6 N m-1

12

27

Two identical, flexible conducting ribbons l 1 and l 2 are connected to junctions X and Y,
and junctions P and Q, respectively. Direct currents I passes through l 1 and 2I passes
through l 2 in opposite directions.

l1

X
P

Y
Q

l2

Which of the following diagram best represents the shapes of the ribbons?
A

B
X

D
X

In a photoelectric experiment, electromagnetic radiation of wavelength 240 nm and


intensity 8.2 103 W m-2 is incident normally on a metal surface of area 2.0 104 m2.
What is the number of photons incident per second?
A

29

28

2.0 1015

2.0 1018

2.0 1021

2.0 1024

What is the de Broglie wavelength of a particle of mass, m and kinetic energy, E?


A

h
mE

h 2mE

2
mE

2mE

13

30

The figure below shows four energy levels E1, E2, E3 and E4, of the hydrogen atom.
E4 = -0.85 eV
E3 = -1.50 eV
E2 = -3.40 eV

E1 = -13.60 eV

An absorption line spectral corresponding to 488 nm could result from an electron


transition between levels
A

E2 to E3

E2 to E4

E3 to E2

** END OF PAPER **

E4 to E2

RAFFLES JUNIOR COLLEGE


2008 Preliminary Examination
Centre Number

Index Number

Name

Class

PHYSICS
Higher 1

8866 / 02

Paper 2

17 September 2008
2 hours

Candidates answer on the Question Paper.


No Additional Materials are required.
READ THESE INSTRUCTIONS FIRST
Write your Centre number, index number, name and class in the spaces provided at the top of this
page.
Write in dark blue or black pen.
You may use a soft pencil for any diagrams, graphs or rough working.
Do not use staples, paper clips, highlighters, glue or correction fluid.

Section A
Answer all questions.
Section B
Answer any two questions.

For Examiners Use


1
2

At the end of the examination, enter the numbers of the


Section B questions you have answered in the grid.
The number of marks is given in brackets [ ] at the end of
each question or part question.

/ 6
/ 6

Write your answers in the spaces provided in this


booklet.

/ 7

Section A

/ 5

/16
/ 20

Section B

/ 20
Total

This booklet consists of 19 printed pages including the cover page.

/ 80

2
Data
speed of light in free space,

c = 3.00 x 108 m s1

elementary charge,

e = 1.60 x 10-19 C

the Planck constant,

h = 6.63 x 1034 J s

unified atomic mass constant,

u = 1.66 x 1027 kg

rest mass of electron,

me = 9.11 x 1031 kg

rest mass of proton,

mp = 1.67 x 1027 kg

acceleration of free fall,

g = 9.81 m s2

Formulae
uniformly accelerated motion,

s = ut + at2
v2 = u2 + 2as

work done on/by a gas,

W = pV

hydrostatic pressure,

p = gh

Electric potential

V =

Q
4 o r

resistors in series,
resistors in parallel,

R = R1 + R2 +
1/R = 1/R1 + 1/R2 +

3
Section A
Answer all the questions in this section.
1

A student sets up the circuit shown in Fig 1 in order to determine the resistance
of a wire and hence the resistivity of the wire.

Fig 1
The following readings were obtained for the experiment:
Voltmeter reading Ammeter reading
Length of wire
Diameter of wire
2.40 0.01 V
0.82 0.01 A
64.5 0.2 cm
0.48 0.02 mm
(i)
Calculate the value of the resistance of the wire with its uncertainty.

[2]
(ii)

Calculate the value of the resistivity of the wire with its uncertainty.

[3]

4
2

(a)

One end of a spring is attached to a rigid support and a mass is hung at the other end.
The mass oscillates about the equilibrium position as shown in the Fig. 2a
A

Un-stretched
spring

A
B
equilibrium

Fig. 2a
Ignoring the effects of air resistance, describe the energy changes from position
A to B and from B to C

[3]

5
(b)

Fig. 2b shows two masses A and B connected by a light cord passing over a
light, frictionless pulley. When both masses are released, mass A starts from rest
and moves on a rough plane inclined at 30o to the horizontal. Assume that the
friction acting on mass A is 2.0 N.

2.0 kg
A
B

5.0 kg

Friction = 2.0 N
5.0 m

30o

Fig. 2b

Calculate the speed of mass B just before it hits the ground.

[4]

6
3

Fig. 3 shows a portable barbeque grill connected to a battery with a terminal potential
difference of 240 V with internal resistance r. The grill is made out of twelve identical coils of
cast iron rods of resistance R. The rods are connected to each other and the grill is
operated by using two switches which are ganged together. This means that when one
switch is off; both are off; if one is at A, both are at A; if one is at B, both are at B.
A
Off
B
A
Off
B
Left-side Grill

Right-side Grill

Battery
e.m.f. E

Fig. 3
(a)

Suggest one advantage of the grills switching arrangement.

[1]
(b)

Given that when both switches are at A, there is a power output of 190 W per rod,
calculate
(i) the resistance R of each rod.

R=

[2]

[3]

(ii) the e.m.f. E of the battery, given that r is 2.60 .

E=

7
4

(i) State what is meant by the photoelectric effect.

[2]
(ii) Write down Einsteins photoelectric equation, relating the frequency, f of
the photon, work function, and the maximum speed, vmax of the emitted
electrons, defining clearly any other quantities used.

[1]

(iii) Given that the work function of the metal lithium is 2.30 eV, calculate the
maximum speed of the photoelectrons if UV radiation of 300 nm is incident on
lithium.

vmax =

ms-1

[3]

8
5

A student performed an experiment to investigate the terminal velocity of a small, spherical


steel pellet in a viscous liquid. Terminal velocity is the uniform velocity attained when the
upward viscous force and the upthrust acting on the pellet are balanced by its weight.
Assume that the upthrust is negligible and the magnitude of the viscous force is F = k v where
v is the speed of the ball-bearing and k is a constant.
During the experiment, the student released the pellet from a short distance above the
surface of the liquid which was contained in a tall, wide measuring cylinder. By using a data
logger, the student recorded the downward displacement y, of the pellet, with time t after
entering the liquid as shown in the table below.
[ Take g = 9.81 m s-2 ]
t/s
y/m
(a)

0
0

0.05
0.032

0.10
0.055

0.15
0.070

0.20
0.083

0.25
0.095

0.30
0.105

0.35
0.115

0.40
0.125

0.45
0.135

In the grid provided below, plot the graph of downward displacement y / m against
time t / s.
y/m

0.20

0.15

0.10
0.05

t/s

0.10

0.20

0.30

0.40

0.50
[2]

(b)

Calculate the initial velocity of the pellet upon entry into the liquid.

Initial velocity =

m s-1

[3]

9
(c)

Calculate the height above the surface of the liquid at which the pellet was released
from rest. Assume no air resistance air resistance.

height =
(d)

[2]

Describe qualitatively the motion of the pellet from t = 0 to t = 0.45 s.

[2]
(e)

(i) Determine the magnitude of the terminal speed of the pellet.

terminal speed =

m s-1

[2]

(ii) Given that the mass of the pellet is 0.550 g, determine the value of constant k.

k=

kg s-1

[2]

10

(f)

State the effect on the terminal speed if the mass of the pellet is increased and
briefly explain why.

[2]
(g)

State how is it possible to cause the same pellet to attain terminal speed quicker.

[1]

11
Section B
Answer two questions from this section.
6

(a)

(i)

Define linear momentum.

[1]
(ii)

State the principle of conservation of momentum.

[2]
(iii)

Explain what is meant by an elastic collision between two bodies.

[2]
(b)

A stationary object of 1.0 kg contains some explosives and is at rest on a smooth


ground. When the explosives detonate, the object splits into two smaller pieces W
and X each of 0.15 kg and 0.85 kg respectively. After the explosion, they move
horizontally on the floor. W has a kinetic energy of 1.8 J.
(i) Determine the speed of W.

[2]

(ii) Determine the speed of X.

[2]

12
(iii) Determine the amount of energy that is stored in the explosives.

(c)

[2]

Body Z of mass m is directed, with an initial velocity v0, head-on towards a


stationary body Y of mass M. During the collision, both bodies are temporarily
compressed and remain in contact for a short time. It is known that M is larger
than m. The variations of their velocities during the collision are shown in Fig. 6.

Fig. 6
(i) Write down the expression for the momentum of Y at time t1. Explain how
you deduce this expression.

[3]

13

(ii) At t2, Z is closest to Y and the velocity of Z is v.


1. Describe the motion of the two bodies at this time.

[1]
2.

Find the velocity v.

[2]

(iii) Suggest, with reasons, whether the collision of Y and Z is elastic.

[3]

14

(a)

Different musical instruments do not sound the same even if the same note is played.
Fig. 7.1(a), (b) and (c) show the frequencies produced when the same note is played
by wind instruments A, B and C, respectively.

(i) State one similarity between the note produced by the three wind [1]
instruments.

(ii) For instruments A, B and C, state whether they are open or closed pipes.

[2]

(iii) Explain why the same note sounds different.

[2]

(iv) State the fundamental frequency produced by instrument C.

[1]

(v) Sketch the waveform of the first overtone produced by instrument C.

[1]

(vi) Hence calculate the effective length of instrument C given that speed of [3]
sound is 335 m s-1. [Ignore any end correction.]

15
(b)

Two coherent sources A and B, which are in phase with each other, emit microwave of
wavelength 40.0 mm. The amplitude of the wave from source B is twice that of source
A. A detector is placed at the point P where it is 1.00 m from A and 1.18 m from B as
shown in Fig. 7.2.

(i) Explain why the intensity detected at P is a minimum.

[2]

(ii) Determine the ratio of the intensity at P to the intensity at O.

[3]

(iii) As the detector moved from P to O, a series of minima and maxima were [4]
observed. Explain this phenomenon and state the total number of minimum
and maximum points that will be encountered along the way to O (inclusive
of P and O).

(iv) State the type of interference at P when source B is changed to be 180 [1]
out-of-phase with source A.

16
8

(a)

(i) Define the tesla.

[1]

(ii) Write down the expression for the force F acting on a long, straight [1]
conductor of length l that is carrying a current I, which makes at an angle
to a uniform magnetic field of flux density B.

(iii) Draw a labelled diagram to illustrate the direction of the force F relative to [2]
the directions of the current I and the magnetic field B. Indicate the angle
in your diagram.

17
(b)

A U-shaped magnet sits on a top pan balance as shown in Fig. 8. A wire is


placed horizontally between the poles of the magnet such that a length of
4.20 cm lies in the field of the magnet.
Wire (current pointing out of paper)

Fig. 8

Initially, no current flows in the wire and the balance reads 159.46 g. When a
steady current of 2.00 A (pointing out of the plane of the paper) is passed
through the wire, the balance reads 159.72 g.
(i) Sketch the magnetic field lines around the wire, as viewed from the front,
(1)

due to the current alone;

(2)

due to the combined effect of the current and the U-shaped magnet.

[2]

[2]

18
(ii) Explain why the reading on the balance increases when the steady current [2]
passes through the wire.

(iii) Calculate the force experienced by the wire and state its direction.

[3]

(iv) Hence, determine the magnitude of the magnetic flux density experienced [2]
by the wire.

19
(v) If the direction of the current is reversed, determine the new reading on the [2]
balance. State your answer to 2 decimal places in grams.

(c)

The magnetic flux density B at a perpendicular distance r from a long, straight


wire carrying current I is given by

B =

0 I
2 r

(i) With reference to information found in part (b), determine the distance from [2]
the wire where there is no resultant magnetic field.

(ii) Indicate this position in Fig. 8 with a cross (X).

End of Paper

[1]

2008 RJC Physics H1 P1 Preliminary Examination


Question
1
2
3
4
5
6
7
8
9
10
11
12
13
14
15

Answer
C
D
D
C
A
A
D
B
A
C
B
D
B
A
D

Question
16
17
18
19
20
21
22
23
24
25
26
27
28
29
30

Answer
C
D
B
D
B
A
C
C
B
B
B
B
B
D
B

Raffles Junior College

2008

H1 Physics JC2 Preliminary Examinations 2008


Paper 2 Suggested Solutions
Section A
1

(i)

Vertical components:

1 2 1
gt = 9.81 2.22
2
2
s = 23.7 m
s = ut +

(ii)

[2]

Vertical components:

v y = uy + at = 9.81 2.2
v y = 21.6 ms-1

(a)

(b)

[3]

A to B : Gravitational potential energy converted to Kinetic energy and Elastic potential


energy
B to C: Gravitational potential energy and Kinetic energy converted to Elastic potential
energy
(i) Work done against friction = 2 5 = 10 J
(ii)

KE1 + PE1 + work done by friction = KE2 + PE2

1
( 7 ) v 2 + 2 ( 9.81) 5 sin 30o 5 ( 9.81)( 5 )
2
v = 7.30 ms-1

0 + 0 10 =

(a)
(b)

The heating power can be varied with this arrangement


(i) the resistance R of each rod.

V2
PT =
RT

( 240 )
190 12 =

P=
2

or

3R

( 247.7 )

1
190 12 3
= 8.4211 = 8.42

R=

190 =

Vper rod2

RT
240

R
2

1
240
R =
190
6
= 8.4211 = 8.42
R=

(ii) the e.m.f. E of the battery, given that r is 2.60 .

[2]

190 12
= 9.50 A
240
= Ir + V
= (9.50)(2.60) + 240
= 264.7 V

total current drawn from cell =

E=
4

[3]

(i) Photoelectric effect is the emission of electrons from the surface of a metal when
it is exposed to electromagnetic radiation of sufficiently high frequency.
(ii)

1
2
hf = + mv max
2

where m is the mass of an electron


h is the Planck constant.

(iii)

E = hf = +
v max =

1
2
mv max
2

2 (E - )
m

E = hf
hc
=

( 6.63 10 )( 3.0 10 )
=
34

(M1)

300 10 9
= 6.63 10-19 J

v max =

2 6.63 10-19 2.30 1.6 10 19


9.11 10

31

)
(M1)

= 8.05 105 m s1
(A1)

3
5

(a)
y/m

0.20

0.15

0.10
0.05

t/s

0.10

0.20

0.30

0.40

0.50
[2]

(b)

Gradient (at t = 0 s) = [0.175 (0.045)]/[0.20 (0.05)] ------[1] formula +


substn
= 0.88 m s1 ------[1] Correct ans + units + 10% within range
(range: 0.79 to 0.97)

(c)

v2 = u2+ 2as
0.882 = 0 + (2)(9.8) s
s = 0.0395 = 0.040 m

------[1] formula & substn


------[1] Correct ans + units

(d)

From t = 0 to t = 0.25 s, the pellets velocity is increasing but at a decreasing rate.


The pellets velocity became constant from t = 0.25 s onwards i.e. terminal velocity
has been attained.
(e)

(i)
vT =

( 0.135 0.105 ) = 0.20 m s-1


( 0.45 0.30 )

(ii)

(mg - kvT )
=0
m
mg 0.550 103 9.81
k =
=
= 0.027 kg s-1
0.20
vT
a=

(f)

Terminal speed will be bigger.

The heavier pellet will accelerate for a longer duration of time due to a larger
weight resulting in a larger terminal speed.
[2]
(g)

State how is it possible to cause the same pellet to attain terminal speed quicker.
By choosing a liquid with higher k (for e.g. more viscous).
Drop from a higher point.

[1]

5
Section B
6

(a)

(i) Linear momentum is the product of mass and velocity


(ii) The total momentum of the system is constant provided there is no
external resultant force. / The total momentum of an isolated system is
constant.
(iii) Kinetic energy is constant before and after collision. / Kinetic energy is
conserved in the collision.
Relative speed of approach is equal to the relative speed of separation.

(b)

(i) Kinetic energy = mv2


(4 x 1.66 x 10-27) v2 = 8.0 x 10-13
v = 1.55 x 107 m s-1
(ii)

(c)

Using conservation of momentum


mRnvRn = mv
220u x vRn = 4u x 1.5523 x 107
vRn = 2.82 x 105 m s-1

(i)

0.5mv0
Since there is no external force on the system, total momentum must be
conserved.
Since the total momentum of the system is mv0, and the momentum of the particle at t1 is 0.5mv0, the momentum of the nucleus is also 0.5mv0.

(ii) 1.

Both are moving with same velocity.

2.

Let V be the velocity of the nucleus.


Using conservation of momentum,
mv0 = mv + MV
Since v = V,
mv0 = mv + Mv
m
v0
v=
m +M

(iii) Argument 1
The speed of the -particle and the gold nucleus are about 2% of v0 hence both
are nearly at rest and the assumption is valid.

Argument 2
The speed of the -particle and the gold nucleus is non-zero and the use of
conservation of energy by assuming that all the kinetic energy is converted to
electrical potential energy is not valid. The use of the conservation of energy
must be modified to take into account the kinetic energies of the particles during
this point.
(iv) Argument 1
When the -particle approaches the nucleus, kinetic energy of the -particle is
converted into electrical potential energy. When they separate, the potential
energy is converted back to kinetic energy of the -particle and the nucleus).

Therefore the kinetic energy of the system remains constant before and after
the interaction. Hence the interaction can be considered as an elastic collision.
Argument 2
When the -particle approaches the nucleus, it experiences deceleration and
emits electromagnetic radiation. Therefore the total energy and hence the
kinetic energy of the system must decrease after the interaction. Hence the
interaction is not an elastic collision.
[Solution such as interaction is not elastic because the speed is not the
same as the original value and hence kinetic energy is not conserved will not
be awarded any marks because the discussion is not complete.]

(a)

(i) Same fundamental frequency.


(ii) Instrument A and B are open-pipe while instrument C is closed-pipe.
(iii) The number and relative amplitude of the overtones are different.
(iv) 400 Hz
(v)

(vi)

Since v = f and L =
L=

(b)

3
,
4

3 v 3 335
=
= 0.209 m
4 f 4 1200

(i) Path difference


= 0.18 m = 4.5
The waves arrived at P 180 out-of-phase and destructive interference takes
placed.

(ii)

Intensity at P Amplitude at P 2 A A
=
= 0.111
=
Intensity at O Amplitude at O 2 A + A

(iii) The path difference varies from zero at O to 4.5 at P.


Maxima occurs when = n where n is 0, 1, 2, 3 or 4.

Minimum occurs when = ( n +

1
2

where n is 0, 1, 2, 3 or 4.

So total number of minimum and maximum points is 10.


(iv) Constructive interference.

(a)

(i) The tesla is defined as the magnetic flux density experienced if a wire of
length 1m and carrying a current of 1A has a force of 1N exerted on it in a

direction perpendicular to both the flux and the current.

(ii)

F = BIl sin

Diagram showing F, I, B, l and correctly orientated according to Flemings


Left Hand Rule.
Note: Deduct [1] for every mistake.

(b)

(i)

(ii) The current-carrying wire experiences an upward magnetic force acting on


it due to the presence of a magnetic field from the U-shaped magnet.
M1

By Newtons third law, the current-carrying conductor will also exert a


downward force on the U-shaped magnet.
A1
This causes the reading on the top-pan balance to increase.
(iii) By Newtons third law,
FB = force acting downwards on the U-shaped magnet
= (mfinal reading minitial reading)g
= (159.72 159.46) 10-3 kg (9.81 m s-2)
M1
= 2.551 10-3 N
A1

Magnetic force pointing upwards.


B1
(iv)

Since FB = BIl

B=

FB
2.551 103 N
=
I l (2.00 A)(4.20 10 -2 m)

C1
= 3.04x10-2 T
A1
(v)

New reading on the top-pan balance


= (159.46 g) (159.72 g 159.46g)
M1
= 159.20 g
A1
Alternative solution:
The net force acting on the top-pan balance by the magnet
is now,
F
= Weight of the magnet Magnetic force on the
wire
= mmagnetg - FB
= (159.46 10-3 kg) (9.81 m s-2) (2.551 10-3 N)
= 1.5618 N
M1

Reading on the balance

F 1.5618 103 N
=
g
9.81 m s -2

= 159.20 g
A1
(c)

(i)

Bwire = Bhorizontal

o I
= 3.04 x 10-2
2 r
[M1]
r = 1.32x10 m
-5

(ii)

Vertically above the wire

ST. ANDREWS JUNIOR COLLEGE


JC2 2008
Preliminary Examination
PHYSICS, Higher 1
Paper 1

8866/01

27th August 2008 (1300 Hrs 1400 Hrs)

1 hour

Additional Materials: Optical Mark Sheet (OMS)

READ THESE INSTRUCTIONS FIRST


Write in soft pencil.
Do not use staples, paper clips, highlighters, glue or correction fluid.
Write your name, Civic Group and index number on the Answer Sheet in the spaces
provided unless this has been done for you
There are thirty questions in this paper. Answer all the questions. For each question
there are four possible answers A, B, C, D
Choose the one you consider correct and record your choice in soft pencil on the
separate Optical Mark Sheet (OMS).
Read the instructions on the Answer Sheet very carefully.
Each correct answer will score one mark. A mark will not be deducted for a wrong
answer. Any rough working should be done in this booklet.

Instructions for using the Optical Mark Sheet (OMS)

Class no.
Write your
numbers here
Register no.

1.
2.
3.

Fill in your class number (e.g. 07S29 = "29", 07A01 = 01) in the first two
rows.
Fill in your class register number in the next two rows. (e.g. register number 1
is filled in as "01").
Write your class and register numbers into the column on the left.

This Question Paper consists of 14 printed pages

SAJC 2008

Prelim/8866/01

[Turn over

2
Data
speed of light in free space,

= 3.00 x 108 m s-1

elementary charge,

= -1.60 x 10-19 C

the Planck constant,

= 6.63 x 10-34 J s

unified atomic mass constant,

= 1.66 x 10-27 kg

rest mass of electron,

me

= 9.11 x 10-31 kg

rest mass of proton,

mp

= 1.67 x 10-27 kg

acceleration of free fall,

= 9.81 m s-2

= u t + a t2

v2

= u2 + 2 a s

work done on/by a gas,

= p V

hydrostatic pressure,

=gh

electric potential,

Q
= 4 r
o

resistors in series,

= R1 + R2 + ...

resistors in parallel,

1 / R = 1 / R1 + 1 / R2 + ...

Formulae
uniformly accelerated motion,

SAJC 2008

Prelim/8866/01

[Turn over

3
1

Which pair includes a vector quantity and a scalar quantity?


A
B
C
D

Which list of SI units contains only base units?


A
B
C
D

Weight; Power
Work; Potential Energy
Displacement; Acceleration
Kinetic Energy; Speed

kelvin, metre, second, ohm, mole


kelvin, metre, mole, ampere, kilogram
kilogram, Newton, metre, ampere. ohm
newton, kelvin, second, volt, mole

Four students each made a series of measurements of the acceleration


of free fall g. The table shows the results obtained. Which student
obtained a set of results that could be described as precise but not
accurate?
Results g / m s-2

student
A
B
C
D

4.

8.66
9.81
9.45
10.12

8.64
9.82
9.23
9.89

8.65
9.82
9.86
9.75

8.67
9.82
9.81
9.81

A motorcycle is travelling at a constant acceleration in a straight line.


When it passes by the first lamp-post, it is travelling at 10 m s-1. When
is passes by the next lamp-post which is 200 m from the first lamppost, the motorcycle is travelling at 16 m s-1. What is the acceleration of
the motorcycle?
A
B
C
D

SAJC 2008

0.17 ms-2
0.22 ms-2
0.33 ms-2
0.39 ms-2

Prelim/8866/01

[Turn over

4
5

James tries to throw a tennis ball from the balcony to his friend at the
caf as shown in Fig. 5.1 below. He shoots from a position 3 m above
the caf, and shoots at 5 ms-1, 30 below the horizontal. What is the
speed of the tennis ball when it reaches the floor of the caf?
30

3m
Fig. 5.1
6.80 m s-1
7.67 m s-1
9.16 m s-1
10.0 m s-1

A
B
C
D

A girl of mass 40.0 kg stands on a lift which is descending with a


constant velocity of 6.0 m s-1. What is the magnitude and direction of
the force acting on the girl by the floor?

A
B
C
D

SAJC 2008

Magnitude / N
240.0
240.0
392.4
392.4

Direction
Downwards
Upwards
Downwards
Upwards

Prelim/8866/01

[Turn over

5
7

A ball of weight W slides along a smooth horizontal surface until it falls


off the edge at time T, as shown in Fig. 7.1.

Fig. 7.1
Which graph represents how the resultant vertical force F, acting on
the ball, varies with time t as the ball moves from position X to position
Y?

A 77 kg archer stands on a smooth surface and fires a 0.35 kg arrow


horizontally at a velocity of 50 m s-1. Neglecting friction, find the velocity
of the archer after firing the arrow.
A
B
C
D

SAJC 2008

0.15 m s-1
0.18 m s-1
0.20 m s-1
0.23 m s-1

Prelim/8866/01

[Turn over

6
9

A trolley runs down a slope with a constant acceleration a. The mass of


the trolley is now doubled and the trolley is allowed to run down the
same slope. In both cases, effects of friction and air resistance are
negligible.
Which statement is correct for the second experiment?
A
B
C
D

10

The accelerating force is the same.


The acceleration is a.
The acceleration is a.
The acceleration is 2a.

Mass A of 1 kg is suspended as shown in Fig. 10.1 by two strings,


which pass over smooth pulleys, and are attached to objects B and C
which have the same mass. What is the mass of each of object B and
C?

40 40

C
A

A
B
C
D

SAJC 2008

0.50 kg
0.65 kg
1.00 kg
1.31 kg

Fig. 10.1

Prelim/8866/01

[Turn over

7
11

The minimum friction between the table and the 10 kg mass before it
moves is 50 N. Assuming the pulley is frictionless, what is the minimum
value of m to have the 10 kg mass move to the left?

2r

10 kg

m kg

A
B
C
D
12

Fig. 11.1

2.55
5.00
10.0
15.1

The figure below shows three tangential forces applied to a circular


object.

Fig. 12.1
Which of the following states correctly the resultant moment and
resultant force acting on the object?

A
B
C
D

13

Resultant moment
Zero
Zero
Non-zero
Non-zero

Resultant force
Non-zero
Zero
Non-zero
Zero

The engine of a train travelling at 40 m s-1 delivers a power of 2.0 MW.


What is the tractive force exerted by the engine?
A
B
C
D

SAJC 2008

5.0 kN
8.0 kN
50.0 kN
80.0 kN

Prelim/8866/01

[Turn over

8
14

A steel ball is falling at constant speed in oil. Which graph shows the
variation with time of the gravitational potential energy Ep and the
kinetic energy Ek of the ball?

15

A barrel of mass 50 kg is loaded onto the back of a lorry 1.6 m high by


pushing it up a smooth plank 3.4 m long as shown in Fig. 15.1 below.

Fig. 15.1
What is the minimum work done?
A

SAJC 2008

80 J

170 J

Prelim/8866/01

780 J

1700 J

[Turn over

9
16

A wire of uniform cross-sectional area has a resistance R. The wire is


stretched until its length increases by 10%. By assuming that the wire
so formed is uniform, the new resistance of the wire is
A

17

0.9 R

1.0 R

1.1 R

1.2 R

A number of identical resistors, each of resistance R, are arranged


differently. Which diagram shows the arrangement of resistors that
gives the smallest effective resistance?
R

R
R

18

A p.d. of 12 V is connected between P and Q as shown in Fig. 18.1.

Fig. 18.1
What is the p.d. between X and Y?
A

SAJC 2008

0V

4V

Prelim/8866/01

6V

8V

[Turn over

10
19

The figure below shows the displacement-time graphs for two


oscillations.

displacement

time

Fig. 19.1
What is the phase difference between the oscillations?
A
B
C
D

20

The wavelength of a progressive wave can be calculated if one knows


the
A
B
C
D

21

frequency
speed and amplitude
amplitude and frequency
frequency and speed

In a Youngs double slit experiment, a small detector measures an


intensity of illumination of I units at the centre of the fringe pattern.
What will be the measured intensity if one of the two (identical) slits is
now covered?
A

22

rad
rad
rad
rad

2I

A suspension bridge is to be built across a valley where it is known that


the wind can gust at 5 s intervals. It is estimated that the speed of
transverse waves along the span of the bridge would be 400 m s-1. The
danger of resonant motions in the bridge at its fundamental frequency
would be greatest if the span had a length of
A

SAJC 2008

80 m

400 m
Prelim/8866/01

1000 m

D 2000 m
[Turn over

11
23

Which of the following is necessary for interference patterns to be


observed?
A
B
C
D

24

The sources must be coherent.


The sources must generate waves of the same amplitude.
The sources must generate waves which are polarized parallel
to each other.
The sources must only be light waves.

Fig. 24.1a and Fig. 24.1b below show a negatively charged particle
moving in various situations and the direction of the magnetic force it
will experience as it moves. What is the direction of the magnetic field
acting on the particle in each situation?
F
F

v (in)

v (out)

Field 1

Field 2

Fig. 24.1a

Field 1
A
B
C
D

25

Upwards
Upwards
Downwards
Downwards

Fig. 24.1b

Field 2
Towards the right
Towards the left
Toward the right
Toward the left

Fig. 25.1 below shows two parallel wires P and Q in the plane of the
paper. P is fixed, Q is free to move.
I
I

P
Q

Fig. 25.1
When the same current I passes through each wire in the same
direction, Q moves
A

away from P in the plane of the paper.

towards P in the plane of the paper.

downwards into the paper.

upwards out of the paper.

SAJC 2008

Prelim/8866/01

[Turn over

12
26

Fig. 26.1 below shows the path taken by an electron as it enters a


region of magnetic field directed downwards into the plane of the
paper. Which of the indicated paths is the one taken by the electron in
the magnetic field?

Electron
entering
magnetic field

Fig. 26.1

27

Which of the following provides evidence for the wave nature of


particles?
A
B
C
D

SAJC 2008

Diffraction of light
Photoelectric effect
Cathode-ray tube
Electron diffraction

Prelim/8866/01

[Turn over

13
28

A beam of monochromatic light incident on a metal surface causes the


emission of photoelectrons. The length of time that the surface is
illuminated by this beam is varied, but the intensity of the beam is kept
constant. Which graph best represents the relationship between the
total number of photoelectrons emitted and the length of time of
illumination?

29

In a photoelectric emission experiment, photoelectrons are produced


when an electromagnetic radiation is incident on a metal surface. Both
the intensity and the wavelength of the electromagnetic radiation are
then reduced resulting in
A
B
C
D

SAJC 2008

a reduction in the maximum kinetic energy of the electrons but


an increase in their rate of emission.
a reduction in both the maximum kinetic energy of the electrons
and their rate of emission.
in increase in both the maximum kinetic energy of the
photoelectrons and their rate of emission.
an increase in the maximum kinetic energy of the electrons but a
reduction on their rate of emission.

Prelim/8866/01

[Turn over

14
30

Fig. 30.1 below shows the first four energy levels of an electron in a
certain atom.
E4
E3
E2

E1
Fig. 30.1
The transition from E3 to E1 produces a green line. Which transition
could give rise to a blue line?
A
B
C
D

SAJC 2008

E2 to E1
E4 to E2
E3 to E2
E4 to E1

Prelim/8866/01

[Turn over

1
Class

Index Number

Name

07
ST. ANDREWS JUNIOR COLLEGE
JC 2 2008
Preliminary Examinations
PHYSICS, Higher 1
Paper 2

8866/02

27th August 2008 (1430 Hrs 1630 Hrs)

2 hours

Candidates answer on the question paper.


No additional materials are required.
READ THESE INSTRUCTIONS FIRST
Write your name, index number and Civics Group on all the work you hand in.
Write in dark blue or black pen.
You may use a soft pencil for any diagrams, graphs or rough working.
Do not use paper clips, highlighters, glue or correction fluid.
Section A
Answer all questions.
Section B
Answer any two questions.
You are advised to spend about one hour on each section.
At the end of the examination, fasten all your work securely together.
The number of marks is given in brackets [ ] at the end of each question or part
question.

For Examiners Use


Sect A
Sect B

/ 80

Total

This question paper consists of 23 printed pages including this page.

SAJC 2008

Prelim/8866/02

[Turn Over

2
Data
speed of light in free space,

= 3.00 x 108 m s-1

elementary charge,

= -1.60 x 10-19 C

the Planck constant,

= 6.63 x 10-34 J s

unified atomic mass constant,

= 1.66 x 10-27 kg

rest mass of electron,

me

= 9.11 x 10-31 kg

rest mass of proton,

mp

= 1.67 x 10-27 kg

acceleration of free fall,

= 9.81 m s-2

= u t + a t2

v2

= u2 + 2 a s

work done on/by a gas,

= p V

hydrostatic pressure,

=gh

electric potential,

Q
= 4 r
o

resistors in series,

= R1 + R2 + ...

resistors in parallel,

1 / R = 1 / R1 + 1 / R2 + ..

Formulae
uniformly accelerated motion,

SAJC 2008

Prelim/8866/02

[Turn Over

3
Section A
Answer all the questions in the space provided.
1

(a)

What are meant by


(i)

systematic errors

.
. [ 1 ]
(ii)

random errors

.
. [ 1 ]
(b)

Suggest why velocity cannot be defined as displacement per second.


.
..[ 1 ]

(c)

The equation below shows how the dependent variable is related to the
independent variables , , .

4 2
3

The value of is found to be 1314.63 and the estimated percentage


uncertainties in the measurement of , , are 0.02 %, 0.5 % and 0.1 %
respectively.
Express with its associated uncertainty to the appropriate number of
significant figures.

= [ 3 ]

SAJC 2008

Prelim/8866/02

[Turn Over

4
2

Two similar blocks A and B, each of mass 6 kg, are resting on a compressed spring
along a smooth slope as shown in Fig. 2.1. The spring constant for the spring is
400 N m-1, and the angle of elevation of the slope is 40 above the horizontal.

B
Spring

40
Fig. 2.1
(a)

Compute the magnitude of the compressive force exerted on the spring due to
the blocks.

compressive force = N [ 2 ]
(b)

Hence, compute the compression of the spring.

compression = m [ 2 ]
(c)

Mass B is removed abruptly from the system. Compute the magnitude of the
initial acceleration of mass A.

acceleration = m s-2 [ 2 ]

SAJC 2008

Prelim/8866/02

[Turn Over

5
3

(a)

The energy stored in the elastic string of a bow just before the arrow is
released is 95 J. When the arrow of mass 170 g is fired, 90% of the energy
stored is transferred to the arrow. Calculate the speed of the arrow as it
leaves the bow.

speed = .m s-1 [ 2 ]
(b)

For a cyclist travelling at a constant speed, explain if the chemical energy


from the cyclist is transformed into kinetic energy. Hence, or otherwise, state
the energy transformation taking place.
.
..
..
[ 2 ]

SAJC 2008

Prelim/8866/02

[Turn Over

6
4

A wire frame ABCD is supported on two knife-edges P and Q so that the section
PBCQ of the frame lies within a solenoid as shown in Fig. 4.1 below.

B
C

to battery
Top view

Fig. 4.1
When there is no current in the solenoid and frame, the frame is horizontal.
(a)

When a current passes through the solenoid and frame (see the directions in
the figure above), what can you say about the direction of the force (due to
the magnetic field of the solenoid), if any, acting on
(i)

side BC
.. [ 1 ]

(ii)

side PB
. [ 1 ]

(b)

The solenoid has 700 turns m-1 and carries a current of 3.50 A. Given that the
magnetic flux density B on the axis of the solenoid is 3.00 mT, calculate the
force acting on BC if BC has a length of 5.00 cm.

force = N [ 2 ]

SAJC 2008

Prelim/8866/02

[Turn Over

7
(c)

A piece of paper of mass 0.100 g is placed on the side DA and positioned so


as to keep the frame horizontal. Given that QC is of length 15.0 cm, determine
the distance from the knife-edge where the paper should be positioned.

Distance = .. m [ 2 ]

SAJC 2008

Prelim/8866/02

[Turn Over

8
5

(a)

(i)

De Broglie found that when he passed fast-moving electrons through a


crystalline solid, diffraction images obtained were similar to those found
in diffraction experiments using X-rays through crystalline solids.
Explain what can be deduced from this experiment concerning the
nature of electrons.
..
. [ 1 ]

(ii)

Calculate the De Broglies wavelength associated with electrons


moving with a velocity of 7.0 x 104 m s-1.

wavelength = .. m [ 1 ]

SAJC 2008

Prelim/8866/02

[Turn Over

9
(b)

Fig. 5.1 is a simplified representation of the 5 lowest energy levels of the


outermost electron in the sodium atom.

Fig. 5.1
(i)

Sodium atoms emit a spectral line with a wavelength of 588.5 nm.


Express the energy carried by the corresponding photon in electron
volt.

energy of photon = .. eV [ 2 ]
(ii)

Which transition produces this spectral line?


. [ 1 ]

(iii)

A student wrote that the ionisation energy of the sodium atom is


6.02 x 10-19 J. Explain why it is not possible to deduce the value of the
ionisation energy from Fig. 5.1.
..
.. [ 1 ]

SAJC 2008

Prelim/8866/02

[Turn Over

10

The circuit shown in Fig. 6.1 is used to measure the internal resistance r of a battery
with an e.m.f. of 3.00 V. R is a variable resistor. The resistance of the ammeter is
negligible and the voltmeter has an infinite resistance.

Fig. 6.1
The resistance of R is varied. Fig. 6.2 shows the variation of the power dissipated
in R with the potential difference V across R.

Fig. 6.2

SAJC 2008

Prelim/8866/02

[Turn Over

11
(a)

Define the unit ohm.


..
.. [ 1 ]

(b)

Use Fig. 6.2 to determine


(i)

the maximum power dissipation in R,


maximum power = . W [ 1 ]

(ii)

the potential difference across R when the maximum power


is dissipated
potential difference = . V [ 1 ]

(c)

Using your answer in (b), determine the value of R when P is maximum.

R = . [ 2 ]
(d)

(i)

Using Fig. 6.2, determine the current through R when V is 1.12 V.

current = . A [ 1 ]
(ii)

Hence, calculate the internal resistance r of the battery.

r = . [ 2 ]

SAJC 2008

Prelim/8866/02

[Turn Over

12
(e)

With reference to Fig. 6.2, it can be seen that there are two values of potential
difference V for which the power dissipation is 1.05 W. Explain which value of
V will result in less power being dissipated in the internal resistance.
..
..
..
.. [ 3 ]

(f)

Show by completing the circuit in Fig. 6.3 how you would reconnect the circuit
if the resistance of the ammeter cannot be ignored.
[1]

Fig. 6.3

SAJC 2008

Prelim/8866/02

[Turn Over

13
Section B
Answer any two questions from this section
7

Legendary British snooker champion Jimmy White aims at the black ball using the
cue and the white cue ball, as shown in Fig. 7.1. This should be an easy shot as the
balls and the cue stick are all in line with one another. The average friction between
the surface of the table and the balls is 1 N and the masses of the black and white
cue balls are 0.050 kg and 0.047 kg respectively.

15 cm

cue

Fig. 7.1

(a)

20 cm

(i)

Show that the minimum initial speed required for the black ball at the
start to reach the pocket is 2.83 m s-1.
[2]

(ii)

In order to achieve the speed in (a)(i), the black ball undergoes an


elastic collision with the white cue ball. Calculate the velocity of the
white cue ball before and after the collision.

velocity before collision = .. m s-1


velocity after collision = .. m s-1 [ 4 ]

SAJC 2008

Prelim/8866/02

[Turn Over

14
(iii)

Hence, show that the minimum speed of the white cue ball just after
leaving the cue stick is 3.86 m s-1.
[2]

(iv)

Given that the time of impact between the cue stick and white cue ball
is 0.5 ms. Compute the minimum force required by Jimmy White to
exert on the white cue ball using the cue stick to get the black ball into
the pocket.

minimum force = .. N [ 2 ]
(v)

Give two reasons why, in practice, Jimmy White would need a greater
minimum force than the force calculated in (iv) to bring the black ball
into the pocket.
..
..
..
..... [ 2 ]

SAJC 2008

Prelim/8866/02

[Turn Over

15
(b)

An Olympic shooter is aiming at the target as shown in Fig. 7.2 at the same
height. He is standing such that the barrel of the gun is 100 m behind the
target. He tilted his gun slightly to aim 8 cm above the target as shown in
Fig. 7.3. The speed of the bullet as it leaves the barrel is 900 m s-1 and it is
assumed not to be affected by air resistance.
8 cm
Target

Target
100 m

100 m

Fig. 7.2

(i)

Fig. 7.3

Calcuate the time taken for the bullet to travel 100 m horizontally.

t = .. s [ 2 ]
(ii)

Show that the shooter is not able to hit the target.

[2]

(iii)

The shooter decides to change the bullet to a heavier one. He claims


that: With a heavier mass, there will be greater weight pulling the
bullet down. Hence the downward acceleration would be higher, and
the bullet could go lower and hit the target.
Comment on the validity of the shooters claim.
..
..
..
..... [ 2 ]

SAJC 2008

Prelim/8866/02

[Turn Over

16
(c)

Explain, using Newtons Law, why it is more tiring for a person to stand on the
ground compared to sitting on a chair.
..
..
..
...... [ 2 ]

SAJC 2008

Prelim/8866/02

[Turn Over

17
8

In Fig. 8.1 below, two radio transmitters separated by 300 m simultaneously transmit
identical signals of the same wavelength. A radio in a car travelling due north
receives the signals.

400 m
300 m

1000 m
Fig. 8.1
(a)

State the principle of superposition.


.....
.....
..... [ 2 ]

(b)

(i)

Given that the car is at the position of the second maxima (excluding
the maxima at the central) from the central, determine the wavelength
of the signals.

wavelength = .. m [ 3 ]
(ii)

Hence determine the frequency of the signals.

frequency = .. Hz [ 2 ]

SAJC 2008

Prelim/8866/02

[Turn Over

18
(iii)

Determine the distance the car must travel to encounter the next
minimum in reception.

distance = .. m [ 1 ]
(c)

The waves from one of the radio transmitters can reach a home receiver by
two different paths according to Fig. 8.2 below. One is a straight-line path
from the transmitter to the home, a distance of 30.0 km. The second path is
by reflection from a storm cloud. Assume that this reflection takes place at a
point midway between receiver and transmitter.

h
30.0 km

Fig. 8.2
(i)

State the condition, in terms of , for the path difference for destructive
interference for two in-phase sources.
path difference = .. [ 1 ]

(ii)

Determine the minimum height h of the storm cloud that will produce
destructive interference between the direct and reflected beams.
( Assume no phase changes upon reflection., and using the same
wavelength as in (b)(i) )

h = .. m [ 4 ]

SAJC 2008

Prelim/8866/02

[Turn Over

19
(iii)

Each radio transmitter has a power of 20 kW. Given that the minimum
intensity that can be detected by any home receiver is 1.3 10-6 W m-2,
determine the maximum distance that the home receiver can be placed
from the transmitter.

maximum distance = .. m [ 2 ]
(d)

The signals received are then transformed into sound waves of frequency
250 Hz through the radio. A resonance tube, which is closed at one end, is
used to measure the speed of sound in air. The first two strong resonances
are obtained at tube lengths of 0.300 m and 0.960 m respectively. Determine
the speed of sound in air and the end-correction of the tube.

speed of sound = .. m s-1


end correction = .. m [ 5 ]

SAJC 2008

Prelim/8866/02

[Turn Over

20
9

(a)

(i)

Explain what is meant by the photoelectric effect.


..
..
..... [ 2 ]

(ii)

Observations from the photoelectric experiment provided the first


evidence for the particulate nature of light. Describe two of these
observations and how much each deviates from predictions of the
classical wave theory to provide evidence for the particulate nature of
light.
..
.
..
..
..
..
..
.... [ 4 ]

(b)

Fig. 9.1 shows the experimental setup used to investigate the photoelectric
effect.

Fig. 9.1

SAJC 2008

Prelim/8866/02

[Turn Over

21
The frequency f of the incident radiation is first increased to obtain the graph
shown in Fig. 9.2 where I is the current obtained in the ammeter. The
frequency is then kept constant but the voltage V is varied to obtain the graph
in Fig. 9.3. The intensity of the radiation was kept constant throughout the
experiments.

Fig. 9.2

Fig. 9.3

Deduce from the graphs the following:


(i)

The number of photo-electrons emitted in a minute.

number of photo-electrons per min = .. [ 2 ]


(ii)

The workfunction of the metal.

workfunction = .. J

SAJC 2008

Prelim/8866/02

[2]

[Turn Over

22
(iii)

The wavelength of the incident radiation.

wavelength = .. m [ 3 ]
(iv)

A second experiment was performed to produce the line A in Fig. 9.4


below:

Fig. 9.4
What change(s) could have been made in this experiment to obtain
graph A?
..
..
.... [ 2 ]

SAJC 2008

Prelim/8866/02

[Turn Over

23
(v)

In Fig. 9.5 below, draw a graph of photocurrent vs voltage for a third


experiment where the intensity of the incident radiation is increased
and a metal with a greater workfunction is used. Label this line B. [ 2 ]

Fig. 9.5
(c)

Vega is the brightest star in the constellation Lyra and the fifth brightest star in
the sky. The graph in Fig. 9.6 shows part of the visible region of the spectrum
of Vega.

Fig. 9.6
The absorption lines are due to excited hydrogen atoms. Explain how
absorption lines are produced by the hydrogen atoms.
..
..
..
..
.. [ 3 ]
[End of Paper]

SAJC 2008

Prelim/8866/02

[Turn Over

SAJC H1 Physics Prelims 2008


Suggested Solutions
Paper 1 (MCQ)
Q1
A

Q2
B

Q3
A

Q4
D

Q5
C

Q6
D

Q7
B

Q8
D

Q9
C

Q10
B

Q11
A

Q12
A

Q13
C

Q14
B

Q15
C

Q16
D

Q17
A

Q18
B

Q19
A

Q20
D

Q21
A

Q22
C

Q23
A

Q24
B

Q25
B

Q26
C

Q27
D

Q28
B

Q29
D

Q30
D

Paper 2 Section A
1

(a)

Systematic errors are fixed errors that occur, causing readings to be


either consistently smaller or consistently larger than the true value.
[1]

(ii)
(b)

(i)

Random errors are those that occur, causing readings to have different
magnitudes and signs in repeated measurements.
[1]

Displacement is a physical quantity, whereas the second is a unit, thus velocity


should be defined as displacement per unit time.
[1]

(c)

[1]

[1]
[1]

(a)

Component of weights of blocks parallel to the slope,


W //

= 2 ( mg sin 40o )
= 2 ( 6 x 9.81 x sin 40o )

[1]

= 75.7 N
Therefore, the compressive force, F, on the spring is 75.7 N
(b)

F=kx
[1]

75.7 = 400 x

x = 0.189 m
(c)

[1]

[1]

Considering block A, and taking forces acting on A along the slope-direction,


Net Force = m a
[1]

F mg sin 40 = m a
75.7 (6 x 9.81 x sin 40) = 6 x a
a = 6.31 ms-2

[1]

(a)

0.90 x 95 = mv2
v = 31.7 m s-1

[1]
[1]

(b)

No, because the total KE is constant.

[1]

Chemical energy converted into energy to overcome resistive forces between


road and tyres, surroundings and cyclist, and to heat and sound. [ 1 ]
4

(a)

Force is into the paper

[1]

(ii)
(b)

(i)

No force exists

[1]

F =BIL

[1]
-3

= 3.00 10 3.50 5.00 10

-2

= 5.25 10-4 N

(c)

[1]

Taking moments about the knife-edge,


F dQC

-4

5.25 10 0.15
d

mgd

[1]
-3

0.100 10 9.81 d

0.0803 m

[1]

(a)

(i)

Electrons exhibit wave-like nature as diffraction is a wave property. [ 1 ]

(ii)

= h / mv
= (6.63 x 10-34) / (9.11 x 10-31 x 7.0 x 104)
= 1.03 x 10-8 m

(b)

(i)

Energy of photon

[1]
=hc/
= (6.63 x 10-34)(3.0 x 108) / (588.5 x 10-9)

[1]

= 3.38 x 10-19 J

[1]

= 2.11 eV

(ii)
(iii)

Transition from level 2 to level 1.

[1]

To get the ionisation energy, we need the value of energy at n = . Since


this is not obtainable from the data given (i.e. Fig. 5.1), it is not possible
to deduce the ionisation energy value.
[1]

(a)

The ohm is defined as the resistance between two points when a potential
difference of one volt between them produces a current of one ampere. [ 1 ]
OR
One volt per unit ampere.

(b)

(i)

1.12 W < Max power < 1.14 W

[1]

(ii)

1.48 V < V < 1.52 V

[1]

(c)

(d)

V2 1.502
R = P = 1.13
= 1.991
= 1.99 (acceptable range: 1.92 to 2.06 )
(i)

(ii)

(e)

[1]
[1]

P
1.05
I = V = 1.12
= 0.938 A

[1]

From V = E Ir
1.12 = 3.00 0.9375 r
r = 2.00

[1]
[1]

The larger value (1.90 V).


[1]
A larger p.d. across the variable resistor R would mean a smaller p.d. across
internal resistor r.
[1]
This will result in less power P being dissipated in the internal
V2
resistance (as P = r )
[1]

(f)

Connect voltmeter across variable R

[1]

Paper 2 Section B
7

(a)

(i)

average deceleration =

Frictional Force
1
=
= 20 m s-2
Mass
0.05

[1]

Assuming the minimum speed required to reach the pocket, the speed at
the pocket would have been zero.
Using v 2 = u 2 + 2as

0 = u 2 + 2( 20)(0.2)

[1]

u = 2.83 m s-1
Or : using the Energy conversion method,
Assuming all the kinetic energy lost is used to overcome the friction
against the 1 N force,
Work done against friction = Loss of KE
[1]

F d =

1 0.2 =

1 2
mv
2

1
0.05v 2
2

[1]

v = 2.83 m s-1
(ii)

By Principle of Conservation of Linear Momentum,

mW uW + mB u B = mW vW + mB v B
0.047uW + 0 = 0.047vW + 0.05(2.83) -------------(1)

[1]

Since it is elastic,
Relative velocity of approach = relative velocity of separation

uW u B = v B vW
uW 0 = 2.83 vW -----------------(2)

[1]

Solving (1) & (2),

vW = - 0.09 ms-1

[1]

uW = 2.92 ms-1

[1]

[ Deduct 1 mark if neither is negative ]


(iii)

deceleration of white cue ball =

1
ms-2
0.047

[1]

2
2
Using v = u + 2as

2.92 2 = u 2 + 2
0.15
0.047

[1]

u = 3.86 ms-1 (proved)


(iv)

From Newtons 2nd Law, F =

F=

mv 0.047(3.86 0 )
=
t
0.5 10 3

F = 363 N

(v)

mv
t
[1]
[1]

Any two of : [ 1 mark each ]


=>
Overcoming Air resistance. (Cannot accept friction as answer,
because we have considered friction between the ball and the
table in the question.)
=>

Non-elastic collision between cue and white ball, thus some


energy is lost as sound.

=>

The collision may not be head-on or linear; if there is a slight


deviation of the white cue ball or the cue stick, the linear
momentum of the ball will be less, and hence higher force is
required.

(b)

(i)

tan =

0.08
100

= 0.046

[1]

Using t = distance / horizontal speed,

t=

100
900 cos 0.046

t = 0.111 s
(ii)

From

sy = u yt +

[1]

1
ayt 2
2

(& taking upwards as positive),

s = (900 sin 0.046)0.111 +

1
( 9.81)0.1112
2

s = 1.94 cm
Since displacement is not equal to zero, it means that the
shooter did not hit the target.
(iii)

[1]
[1]

Although there is a greater weight with higher mass, it can be shown that
acceleration is independent of the mass of the object:
Fnet = ma
Weight of bullet, mg = ma
a=g

[1]

The acceleration is a constant value of g (9.81 ms-2), so the bullet would


not be able to go lower and hit the target.
[1]
(c)

When standing, the leg supports the entire upper body weight
When sitting down, leg supports much less as the upper body weight is
supported by the chair, due to Newtons Third Law.
{ or if the foot does not even touch the ground, the entire body weight is
supported by the chair. }

[1]
[1]

(a)

The principle of superposition states that when two or more waves travel past a
point simultaneously, the resultant displacement at that point
[1]
is equal to the vector sum of the individual displacements of the waves at that
point.
[1]

(b)

(i)

d1 = [ (400 150)2 + (1000)2 ] = 1030.78 m


d2 = [ (400 + 150)2 + (1000)2 ] = 1141.27 m

[1]

Path difference = d2 d1 = 110.49 m


Path difference = 2 =>
(ii)

110.49
= 55.2 m
2

v =f

[1]
[1]
[1]

f = (3.0 108) 60
= 5.43 106 Hz
(iii)

[1]

x = distance between a maximum and its adjacent maximum


= 2 distance between maximum and minimum
Distance to travel = 0.5 200
= 100 m

(c)

[1]

(i)

Path difference = (n + )

[1]

(ii)

Minimum path difference

[1]

= 55.2 = 27.6 m
{ (Distance of path taken by reflected beam) (30.0 103) }
= min path difference = 27.6m

[1]

Distance of path taken by reflected beam = 30027.6 m


Using Pythagoras Theorem,
2

30000
30027.6
2
+h
=

2
2

[1]

h = 643.6 m

(iii)

[1]

P = IA

[1]
-6

20 000 = 1.3 10 4 r
r = 35.0 103 m

[1]

1st resonance:

= 0.300 + end correction ---------- (1)

[1]

2nd resonance:

(d)

= 0.960 + end correction ---------- (2)

[1]

eqn (2) eqn (1):


= 0.660 m
= 1.32 m

[1]

v = f
= 250 1.32
= 330 m s-1

[1]

End correction = (1.32) 0.300


= 0.030 m

(a)

(i)

[1]

Photoelectric effect refers to the emission of electrons from a cold


metal surface
[1]
when electromagnetic radiation of sufficiently high frequency falls on it.
[1]

(ii)

Observations: [ 2 ] for any 2


1) Existence of a threshold frequency below which no photoelectric
emission occurs.
2) Maximum KE of the emitted photo-electrons is independent of the
intensity of the radiation.
3) Maximum KE of the emitted photo-electrons is dependent on the
frequency of the EM radiation.
4) Photoelectric emission takes place instantaneously.
Explanation of how each deviates from the classical wave theory:
[ 2 ] for any 2
1) According to the classical wave theory, the photoelectric effect should
occur for any frequency of light, provided the light is intense enough to
supply the energy to eject the photo-electrons.
2) Classical wave theory suggests that the KE of the photo-electrons
should increase as the light is made more intense.
3) According to the Classical wave theory, if the energy acquired by a
photo-electron is absorbed directly from the wave incident on the
metal, if the light is feeble enough, there should be a measurable
time-lag for the electron to absorb energy in order to escape.

(b)

(i)

Using Q = n e & Q = I t
number of electrons in 1 min, n = I t / e

[1]

n = (4.0 x 10-9)(60) / (1.6 x 10-19)


= 1.5 x 1012
(ii)

[1]

= h fo = (6.63 x 10-34)(5.0 x 1014)


= 3.32 x 10

-19

[1]

[1]

Using hc / = + e Vs

[1]

(6.63 x 10-34)(3.0 x 108) / = 3.32 x 10-19 + (1.6 x 10-19)(1.1)

(iii)

[1]

= 3.92 x 10-7 m
(iv)

[1]

Graph A has a smaller maximum photocurrent but the same stopping


potential as the original graph. Since the stopping potential is the same,
the frequency of the incident radiation must be the same as the original
setup/ experiment. The lower maximum photocurrent is due to the smaller
intensity of the incident radiation.
[2]

(v)

[ 1 ] for smaller stopping


potential
[ 1 ] for higher max
current

(c)

The energy levels in the hydrogen atom are quantized.


[1]
When electromagnetic radiation passes through the hydrogen gas, photons of
energy corresponding to the exact difference in energy between two energy
levels in the atom are absorbed.
[1]
This results in the lines missing from the spectrum after the radiation passes
through the gas.
[1]

[ End of Paper ]

SERANGOON JUNIOR COLLEGE


General Certificate of Education Advanced Level
Higher 1
NAME
CG

INDEX NO.

PHYSICS

8866/01
th

Preliminary Examination
Paper 1 Multiple Choice
Additional Materials:

26 August 2008
1 hour

Multiple Choice Answer Sheet

READ THESE INSTRUCTIONS FIRST


Write in soft pencil.
Do not use staples, paper clips, highlighters, glue or correction fluid.
Write your name, civics group and index number in the spaces at the top of this page and on the
Answer Sheet in the spaces provided.
In the table of Index Number on the Answer Sheet, write down the number 2, followed by the last 4
digits of your NRIC number and shade the corresponding boxes.
Eg. A student with NRIC number of S1234567A will write and shade 24567.
There are thirty questions on this paper. Answer all questions. For each question there are four
possible answers A, B, C and D.
Choose the one you consider correct and record your choice in soft pencil on the separate Answer
Sheet.
Read the instructions on the Answer Sheet very carefully.
Each correct answer will score one mark. A mark will not be deducted for a wrong answer.
Any rough working should be done in this booklet.

This document consists of 14 printed pages and 0 blank pages.


SRJC 2008

8866/01/PRELIM/2008

[Turn Over

2
DATA AND FORMULAE
Data
speed of light in free space,

= 3.00 x 108 ms1

elementary charge,

= 1.60 x 1019 C

the Planck constant,

= 6.63 x 1034 Js

unified atomic mass constant,

= 1.66 x 1027 kg

rest mass of electron,

me = 9.11 x 1031 kg

rest mass of proton,

mp = 1.67 x 1027 kg

acceleration of free fall,

= 9.81 ms2

uniformly accelerated motion,

s
v2

=
=

work done on/by a gas,

W =

pV

hydrostatic pressure,

gh

resistors in series,

R1 + R2 +

resistors in parallel,

1/R =

Formulae

SRJC 2008

ut + at2
u2 + 2as

1/R1 + 1/R2 +

8866/01/PRELIM/2008

3
Answer all questions
1

Certain derived units can be expressed in alternative forms. Which of the following
examples shows a corresponding alternative form of unit?
Unit

Alternative form

A s1

watt (W)

J m1

volt (V)

J C1

coulomb (C)

tesla (T)

N m1

With regards to precision and accuracy, which of the following statements is true?
A
B
C
D

The external diameter and the internal diameter of a metal tube are measured to be
50.0 0.5 mm and 48.0 0.5 mm respectively. What is the percentage uncertainty in
finding the thickness of the metal tube?
A
B
C
D

A set of results with good precision suggests that there are little random
errors in the experiment.
A set of results with poor accuracy could be improved by taking the average
of several readings.
A set of results with high precision can be achieved by taking the average of
several readings.
None of the above

5%
20%
25%
50%

A cannon at the top of a 30 m high hill fires a shell at an angle of 30.0 upwards from
the horizontal with a speed of 50.0 m s-1. Taking air resistance to be negligible, what is
the angle to the vertical at which the shell lands on level ground?
A
B
C
D

SRJC 2008

7.93
38.9
51.2
82.1

8866/01/PRELIM/2008

[Turn Over

What are the necessary assumptions for usage of the equations of motion?
I.
II.
III.
IV.
A
B
C
D

particle must move in a straight line


particle must move with constant velocity
particle must move with increasing velocity
particle must move with constant acceleration
III
I and IV
I, II and IV
I, III and IV

An elastic rubber ball is dropped from a height. It comes to a momentary stop when it
hits the ground and its compression is at a maximum. Which of the following
statements best explains its resultant force at this instant?
A
B
C
D

Its resultant force is zero because it is instantaneously at rest.


Its resultant force is zero because at maximum compression, the force from the
ground is exactly equals to its weight.
Its resultant force is upwards because it is experiencing an upward acceleration.
Its resultant force is downwards because the rubber ball is pushing the ground
with a force greater than its weight due to its compressed state.

Three crates of masses 3M, M and 5M are stacked on top of one another on the floor
of a lift as shown below.
3M

M
5M

When the lift is accelerating upwards with an acceleration a, the magnitude of the
force mass M exerts on the mass 5M is given by
A
B
C
D

SRJC 2008

4Mg 5Ma
4Mg 4Ma
4Mg + 4Ma
4Mg + 5Ma

8866/01/PRELIM/2008

A burglar of mass 70 kg jumped down to a concrete ground from a window ledge


0.50 m above the ground in the attempt to escape. In a fluster, he forgot to bend his
knees while landing. What was the average force that the jump had on his bone
structure if he took 15 ms to come to a complete stop after landing (assuming his
velocity at the window ledge is zero)?
A
B
C
D

A boy starts to jump vertically from a horizontal ground. Which of the following pairs
of forces form an action-reaction pair?
A
B
C
D

The frictional force and his weight.


The normal force and his weight.
The thrust he exerts on the ground and his weight.
The thrust he exerts on the ground and the normal force.

The graph below shows what happens to a material as an increasing amount of force
is applied to it. What does the shaded area represent?
Force / N

10

14.6 N
45.7 N
14.6 x 103 N
45.7 x 103 N

Extension/ m
A
B
C
D

SRJC 2008

Change in momentum
Rate of change in momentum
Spring constant
Elastic potential energy stored

8866/01/PRELIM/2008

[Turn Over

6
11

The following diagrams show forces acting on uniform rulers. Which of the following do
not show a couple only?
I

II
20 N

20 N

20 N

30 N

III

IV

20 N

10 N

10 N

20 N
A
B
C
D

12

I and III only


I, III and IV only
I, II and III only
All of the above

A very long glass tube opened at one end is filled with water to a depth of 1.00 m and
inclined at 40 to the horizontal. What is the pressure at the bottom of the tube?
(density of water = 1000 kg m-3, atmospheric pressure = 1.01 x 105 Pa)
A
B
C
D

SRJC 2008

6.27 x 103 Pa
6.31 x104 Pa
1.07 x 105 Pa
1.11x 105 Pa

8866/01/PRELIM/2008

7
The following graph shows the variation of the potential energy of a particle with its
displacement.
Potential Energy (J)

13

50

25

10

Displacement (m)

What is the force exerted on the particle at the point marked x?


A
B
C
D

14

In a bleak winter morning, Bobby finds a portable generator (normally used to generate
electricity) in the attic of his house. He reads the label stuck to it and finds that when a
test (to check the electrical energy produced) was last done on it, the generator was
80% efficient. The energy produced included, but was not limited to, 1800 J of
electrical energy, 100 J of heat energy and 100 J of sound energy. As he is feeling
very cold, he decides to turn on the generator for warmth. If the test results still hold
true, what is the efficiency of the generator for his situation?
A
B
C
D

15

5 N in the positive direction


5 N in the negative direction
62.5 N in the positive direction
62.5 N in the negative direction

4%
5%
68%
70%

An empty lift weighing 8000 N moves upwards at a constant velocity of 5.0 m s-1. A
constant frictional force of 1000 N retards its motion upwards. What power is the lift
motor delivering, assuming it is operating at 100 % efficiency?
A
B
C
D

SRJC 2008

5 kW
35 kW
40 kW
45 kW

8866/01/PRELIM/2008

[Turn Over

8
16

A loudspeaker emits a sound wave of amplitude A and intensity I. After some time, the
intensity decreased by 6.00 %. The corresponding change in amplitude is
A
B
C
D

17

2.96 %
3.05 %
94.0 %
97.0 %

The graph below shows the how the pressure P of a region varies with distance d
from a fixed point.
P/ Pa
Patm + Po
Normal atmospheric
pressure, Patm

d/ m
do

Patm - Po

Taking leftward as positive direction, which one of the following shows how the
displacement of particles, x, varies with d in the given region?
x/ m

x/ m
xo

xo
d/ m

0
do
- xo

do
- xo

B
x/ m

x/ m
xo

xo

do d/ m

- xo

d/ m

0
do
- xo
D

SRJC 2008

d/ m

8866/01/PRELIM/2008

9
18

What of the following situations would result in two-source interference fringes not
being observed?
I The sources having significantly different amplitudes
II The sources having different wavelengths
III The sources having the same frequency
IV Transverse waves polarized in the same plane
A
B
C
D

19

A microwave transmitter of frequency, f, is placed several metres away from a metal


plate deflector. A microwave receiver, connected to a microammeter, is used to detect
nodes and antinodes along the normal from the source to the plate. The microwave
receiver is moved from one node through 10 antinodes to another node, a distance of
L m. What is the speed of the microwave?
A
B
C
D

20

III only
I and II
III and IV
I, II and IV

0.16fL
0.18fL
0.20fL
0.25fL

The figure below shows plots of the current I through the cross section of a wire over 4
different time periods.
I

Rank the plots in ascending order according to the net charge that passes through the
wire.
A
B
C
D
SRJC 2008

W, Z, Y, X
X, Y, Z, W
W, Y, Z, X
Z, W, Y, X
8866/01/PRELIM/2008

[Turn Over

10
21

A lighting circuit consists two lamps of similar resistances connected to a constant d.c.
source. The variable resistors RX and RY control the relative intensities of the lamps.

RX

RY

If power output of lamp X is thrice that of lamp Y, the ratio of the currents in RX to that
in RY should be
A
B

1:

C
D

22

1:3

3 :1
3:1

A power supply is known to have an e.m.f. of 1000 V. However, when a voltmeter of


resistance 10 k is connected to the terminals of the supply, only a reading of 200 V
is obtained.
What is the internal resistance of the power supply?
A
B
C
D

SRJC 2008

40
50
40 k
50 k

8866/01/PRELIM/2008

11
23

A 230V power supply is providing power for identical lamps each rated 230V 45 W
connected in parallel.
6A fuse

230 V

The maximum number of lamps (operating normally) that can be connected to the
mains in parallel without activating the 6 A fuse is
A
B
C
D

24

6
7
30
31

Five resistors are connected as shown in the figure below.


2
Y
1

1
2

X
What is the equivalent resistance between X and Y?
A
B
C
D

SRJC 2008

0.47
0.68
1.45
1.84

8866/01/PRELIM/2008

[Turn Over

12
25

The figure below shows a 2 m long conductor carrying a current of 3 A into the plane
of the page.
50o
P
magnetic field

If the magnetic flux density is 0.5 mT, what is the magnitude and direction of the
resultant force acting on the conductor?

A
B
C
D

26

Direction
P
Q
P
Q

Magnitude
0.002 N
0.002 N
0.003 N
0.003 N

Three current-carrying conductors X, Y and Z are placed perpendicular to the plane of


the page as shown below.
X
60o

60o
Y

Z
In which of the following directions does the resultant force on Z act?

60o

60o

45o

45o

SRJC 2008

8866/01/PRELIM/2008

13
27

Which of the following pair correctly define the magnetic flux density and the tesla?
Tesla

Magnetic flux density


A

The number of magnetic flux lines


per unit area where a currentcarrying conductor is placed.

The magnetic flux density of a wire


of length one metre carrying a
current of one ampere has a force
of one newton in a direction at right
angles to both the flux and the
current.

The force per unit current per unit


length of the conductor when the
conductor is at right angles to the
field.

The force acting on a wire of length


one metre and carrying a current of
one ampere when it is placed at
right angles to the magnetic field.

The number of magnetic flux lines


per unit area where a currentcarrying conductor is placed

The force acting on a wire of length


one metre and carrying a current of
one ampere when it is placed at
right angles to the magnetic field.

The force per unit current per unit


length of the conductor when the
conductor is at right angles to the
field.

The magnetic flux density of a wire


of length one metre carrying a
current of one ampere has a force
of one newton in a direction at right
angles to both the flux and the
current.

SRJC 2008

8866/01/PRELIM/2008

[Turn Over

14
28

In an experiment on the photoelectric effect, monochromatic light is directed onto the


surface of a metal plate and the kinetic energy of the fastest moving emitted electrons
is measured. The graph shows how this kinetic energy changes as the frequency of
the light is changed.
kinetic energy
of electrons

frequency
Which one of the following statements about the experiment is true?
A
B
C
D

29

What will happen to electrons which have kinetic energy of a few hundreds of eV,
when they are incident onto carbon film?
A
B
C
D

30

The intercept on the frequency axis would change if light of a different intensity
were used.
The gradient of the graph would change if a different metal were used.
The intercept on the frequency axis would change if a different metal were used.
The gradient of the graph would change if light of a different intensity were
used.

They will not be absorbed by the atoms because the energies of the electrons
do not correspond to the difference in energy levels.
They will be absorbed by the carbon atoms and photons of the same energy
will be given out.
Most of them will pass straight through as there are a lot of empty space in the
carbon film.
They see the carbon lattice as a diffraction grating and get diffracted.

The photoelectric effect is a result of


A
B
C
D

photoelectrons hitting the surface of metal to release more photoelectrons.


the accumulation of light wave to release photoelectrons.
the accumulation of wave energy to release photons.
electrons absorbing photons of sufficient energy to release themselves.

-END OF PAPERSRJC 2008

8866/01/PRELIM/2008

SERANGOON JUNIOR COLLEGE


General Certificate of Education Advanced Level
Higher 1
NAME
CG

INDEX NO.

PHYSICS

8866/02
26th August 2008
2 hours

Preliminary Examination
Paper 2
Structured Questions
No Additional Materials are required.

Candidates answer on the Question Paper.

READ THIS INSTRUCTIONS FIRST


Write your name, civics group and index number in the spaces at the top of this page and on all the work
you hand in.
Write in dark blue or black pen.
You may use a soft pencil for any diagrams, graphs or rough working.
Do not use staples, paper clips, highlighters, glue or correction fluid.
Section A
Answer all questions.
Show all your workings clearly on the question paper.
Section B
Answer any two questions. Circle the questions chosen
in the table on the right bottom corner of this page.

For Examiners Use


Marks awarded
Q1
Q2
Q3

/ 9
/ 6
/ 10

Q6

/ 20

Q7

/ 20

Q8

Section B

/ 10

Q5

Section A

/ 5

Q4

At the end of the examination, fasten all your work


securely together.
The number of marks is given in bracket [ ] at the end of
each question or part question.

/ 20
/ 80

Total marks

This document consist of 22 printed pages and 1 blank page


SRJC 2008

8866/02/PRELIM/2008

[Turn Over

2
DATA AND FORMULAE
Data
speed of light in free space,

= 3.00 x 108 ms1

elementary charge,

= 1.60 x 1019 C

the Planck constant,

= 6.63 x 1034 Js

unified atomic mass constant,

= 1.66 x 1027 kg

rest mass of electron,

me = 9.11 x 1031 kg

rest mass of proton,

mp = 1.67 x 1027 kg

acceleration of free fall,

= 9.81 ms2

ut + at2

v2

u2 + 2as

Formulae
uniformly accelerated motion,

work done on/by a gas,

W =

pV

hydrostatic pressure,

gh

resistors in series,

R1 + R2 +

resistors in parallel,

1/R =

SRJC 2008

1/R1 + 1/R2 +

8866/02/PRELIM/2008

For
Examiners
Use

3
Section A
Answer all questions
The graph in Fig. 1 shows the velocity of an object travelling over a certain time
period. The displacement of the object is 0 m at t = 0 s.

v / m s-1
8.0
3.0

9.0

12.0

14.5 17.0

0
- 4.0

t/s

Fig. 1
(a)

Calculate the displacement of the object at t = 3.0 s, 9.0 s, 12.0 s, 14.5 s and 17.0 s.

At t = 3.0s, displacement = ..... m


At t = 9.0s, displacement = ..... m
At t = 12.0s, displacement = ..... m
At t = 14.5s, displacement = ..... m
At t = 17.0s, displacement = ..... m
[2]
SRJC 2008

8866/02/PRELIM/2008

[Turn Over

For
Examiners
Use

4
(b)

Sketch the displacement-time graph for the motion of the car in the space provided
below. Include in your sketch all appropriate labels, including the displacement of
the object at t = 3.0 s, 9.0 s, 12.0 s, 14.5 s and 17.0 s.
s/m

3.0

9.0

12.0 14.5 17.0

t/s

[3]

2(a)

Define linear momentum.


.
....[1]

(b)

State the principle of conservation of momentum. Discuss how this principle is


consistent with Newtons first law of motion using the motion of a single body.
.
.
.
.
.
....[3]

SRJC 2008

8866/02/PRELIM/2008

For
Examiners
Use

5
(c)

A stationary 238U nucleus decays (i.e split up) into a


as shown in the diagram below.

238

v1

Before decay

234

Th nucleus and an -particle

v2

234

Th

After decay

The mass of the 234Th nucleus is larger than that of the -particle.
With appropriate working, determine if the kinetic energy of the -particle is greater
than the kinetic energy of the 234Th nucleus.

[4]

(d)

Explain if the above situation is inelastic.

.
.
.
.
....[2]
SRJC 2008

8866/02/PRELIM/2008

[Turn Over

For
Examiners
Use

6
3 (a)

A simple current balance can be set up by inserting a light rectangular conducting


frame partially into a solenoid as shown in Fig. 3.1. The frame is pivoted off-centre,
with two-third of its length within the solenoid. The width of the frame which is
outside of the solenoid has a force meter attached to it.

Solenoid

3.0 cm
A

Light frame
D

3.0 A

Force
meter

to d.c. source
Fig. 3.1
(i) Sketch on Fig. 3.1 how a 6 V battery should be inserted in the dotted box above
to ensure that this current balance setup can operate normally.
[1]

(ii) If the resistance of the solenoid is 1.5 , calculate the current in the solenoid.

Current = ...... A [1]


(iii) If the strength of the magnetic field within the solenoid is 2.5 T, calculate the
magnitude of the resultant force acting on side AB of the frame?

Force = .......... N [2]


SRJC 2008

8866/02/PRELIM/2008

For
Examiners
Use

7
(iv) Calculate the reading on the force meter when the wire frame is balanced?

Reading = .......... N [2]

(b)

Charged particles are given an initial velocity within uniform magnetic field strength,
B, are shown in the diagrams below. Sketch the subsequent paths taken by these
particles.
x

(i)

x
_

(ii)

+
x

4(a)

x
x
x
magnetic field

x
magnetic field

[3]

Sketch and explain the I-V characteristics of a thermistor.

.
.
.
.
.
....[3]
SRJC 2008

8866/02/PRELIM/2008

[Turn Over

8
(b)

A cable consists of a steel core coated with aluminium as shown below. The inner
radius (steel core) is 5 mm and the total radius (steel and aluminium) is 10 mm.

Steel core

Aluminium surround
Calculate the resistance of a 100 m length of the cable when a current flows
through the length of the cable.
(Given that the resistivity of steel is 9.0 x 10-8 m and that of the aluminium is
2.5 x 10-8 m)

Resistance of cable = .......... [3]

SRJC 2008

8866/02/PRELIM/2008

For
Examiners
Use

For
Examiners
Use

9
Table 5.1 below shows data on typical ball speeds and striker speeds for different
sports.

______________________________________________________________________________
Ball
Ball mass/ kg Ball velocity/ m s-1
Striker velocity/ m s-1
Impact time / ms
before
after
before
after
______________________________________________________________________________
Cricket ball
0.16
0
39
31
27
1.4
(hit from rest)
Football (free
kick)

0.42

28

18

12

8.0

Golf ball
(drive)

0.046

69

45

32

1.3

Hand ball
(serve)

0.061

23

19

14

1.4

Squash ball
(serve)

0.032

49

44

34

3.0

Tennis ball
(serve)

0.058

51

38

33

4.0

Rally ball
0.20
0
30
35
28
1.5
(serve)
____________________________________________________________________________
Table 5.1

Using data from the table above, answer the following:


(a)

(i) Calculate how much energy is wasted when the 0.7 kg golf club hits the golf ball
to move it horizontally only.

Energy wasted = .......... J [3]


(ii) Explain if the principle of conservation of energy is violated.
.
....[1]
SRJC 2008

8866/02/PRELIM/2008

[Turn Over

10
(iii) Calculate the efficiency in driving the golf ball for this particular case.

Efficiency = .......... % [1]


(iv) Suggest the effect of a longer impact time between the golf ball and golf club,
assuming the average force applied to the golf ball throughout the time of
impact has not changed.
.
.
.
....[2]

(b)

The new game, Rally Ball, involves the usage of a gas piston as striker as shown in
Fig. 5.1. The ball is placed on top of the piston, and when the gas expands, the ball
fires out. If the work done by the gas piston is entirely converted to the kinetic
energy of the ball, and the gas has a constant pressure of 1.0 x 105 Pa, calculate
the change in volume of the gas. Assume negligible friction and air resistance.

gas

Fig. 5.1

Change in volume = .......... m-3 [3]


SRJC 2008

8866/02/PRELIM/2008

For
Examiners
Use

11

For
Examiners
Use

Section B
Answer 2 out of 3 questions in this section.

6(a)

An aeroplane of mass 1.5 x 105 kg moves horizontally with constant velocity. The
forces exerted on the aeroplane are as shown in the Fig. 6.1 below.
Lift

Drag
Thrust (0.60 x 106 N)

Weight

Fig. 6.1
(i) Calculate the values of the lift and the drag.

Lift = .......... N
Drag = .......... N
[2]
(ii) State the conditions for the aircraft to be in equilibrium.
.
.
....[2]
(iii) The horizontal separation of the lines of action of lift and weight is 0.8 m. What
is the vertical separation of the lines of action of thrust and drag?

Vertical separation = .......... m [2]


SRJC 2008

8866/02/PRELIM/2008

[Turn Over

For
Examiners
Use

12
(b) In a training session, both engines on one wing are shut down. The situation at the
instant of shutdown is then as shown in Fig 6.2.

C.G.
5

1.5 x 10 N

C.G.
10 m

1.5 x 105 N

20 m

1.5 x 10 N

10 m

1.5 x 105 N

Fig. 6.2

20
20 m
P

Fig 6.3

One way to counteract the turning moment produced by the two engines is to adjust
the rudder at the tail of the aircraft at an angle as shown in Fig. 6.3. A perpendicular
push P on the rudder by the resultant air current is created.
(i) If the rudder is tilted by 20o, and P is acting at a point approximately 30 m
measured along the axis (shown as dotted line in Fig. 6.3) from center of gravity,
calculate the value for P that can prevent the aircraft from turning.

P = .......... N [3]
(ii) The value you found in (b)(i) is rather large and the rudder may not be able to
withstand it. Suggest a way the pilot has of reducing the value of P?
.
....[1]

SRJC 2008

8866/02/PRELIM/2008

For
Examiners
Use

13
(c)

In a rural village, a group of villagers attempt to transport a heavy load of 2000 N


across a stream of width 2x. A rope is attached to the 2000 N load and is slung over
the branch of a tree which is above the edge of the right bank. A second rope
attached to the load is thrown across the stream to some villagers on the left bank.
Fig. 6.4 shows the situation when the load is halfway across the stream.

branch

x
T2
T1
right bank

left bank
Fig. 6.4

(i) Calculate the tensions T1 and T2 in the left and right rope respectively when the
load is in equilibrium.

T1 = .......... N
T2 = .......... N
[4]
SRJC 2008

8866/02/PRELIM/2008

[Turn Over

For
Examiners
Use

14
(ii) Calculate the tensions T1 and T2 when the load has reached the left bank but
not been lowered down?

T1 = .......... N
T2 = .......... N
[3]
(iii) If a typical villager can exert a maximum horizontal force of 300 N and has a
weight of 800 N, estimate with appropriate workings, the minimum number of
villagers needed to move the load across the stream this way.

Minimum number = .......... [3]


SRJC 2008

8866/02/PRELIM/2008

For
Examiners
Use

15
7(a)

An open metal pipe can be extended by pulling both ends of the pipe at a steady
speed of 5.0 cm s-1, as shown in Fig. 7.1. A sound wave of frequency 1250 Hz
passes through the pipe.
5.0 cm s-1

5.0 cm s-1

Fig 7.1
(i) The effective length L of the pipe at time t is expressed as
L = L0 + 2(5.0 102)t
where L0 is the unextended length
If the time between consecutive loud sounds being heard is 2.7 s, calculate the
wavelength of sound.

Wavelength = ............... m [3]


(ii) Hence calculate the speed of sound.

Speed = ........... m s-1 [2]


(iii) Sketch on Fig. 7.2 the stationary wave pattern of the highest possible frequency,
if each segment of the pipe has a length of 50 cm.

Fig. 7.2
[2]
SRJC 2008

8866/02/PRELIM/2008

[Turn Over

For
Examiners
Use

16
(b)

Fig. 7.3 below shows two speakers S1 and S2 placed in an open field on a windless
day. Their separation is 3.81 m. D is a microphone placed in the same horizontal
plane as the speakers, and at a distance of 10.0 m from S1, and
10.7 m from S2. The lines S1S2 and S1D are perpendicular to each other. When the
speakers are switched on, sound of wavelength 0.20 m is emitted in phase.
S2

10.7 m

3.81 m

S1

10.0 m
Fig 7.3

(i) State how coherence could have been achieved in this case.
.
....[1]

(ii) Calculate the phase difference between the sound waves reaching D from S1
and S2.

Phase difference = ............... [2]


(iii) Hence explain whether a minimum intensity or a maximum intensity would be
detected by D.
.
.
....[2]

SRJC 2008

8866/02/PRELIM/2008

17
(iv) When the wavelength of the sound was slowly varied to a value 1, the
microphone D detected one cycle of change in intensity. Calculate two possible
values of 1.

Possible values of 1 = ................ m


................ m
[4]

SRJC 2008

8866/02/PRELIM/2008

[Turn Over

For
Examiners
Use

For
Examiners
Use

18
(c)

Parallel water waves strike a straight wall with an opening in the centre, as shown
in Fig. 7.4 and Fig. 7.5.
For each of the figure below, draw to scale, the pattern of the wavefronts after
passing through the opening.

wavefronts

wall

Fig 7.4

wavefronts

wall

Fig 7.5

SRJC 2008

[4]
8866/02/PRELIM/2008

19
8(a)

A parallel beam of violet light of wavelength 4.5 x 10-7 m and intensity 700 W m-2 is
incident normally on a surface.
(i) Explain what is meant by a photon?
.
....[1]
(ii) Calculate the momentum of a photon of the violet light.

Momentum = .............. kg m s-1 [2]

(b)

In the photoelectric experiment, the apparatus were arranged such that the intensity
and frequency of the radiation may be varied. If each of these was increased in turn
whilst the other was kept constant, explain the effect on the
(i) number of electrons emitted per second, and
.
.
.
.
.
....[3]
(ii) maximum speed of the photoelectrons.
.
.
.
.
.
....[3]

SRJC 2008

8866/02/PRELIM/2008

[Turn Over

For
Examiners
Use

For
Examiners
Use

20
(c)

The energy levels of three isolated atoms of different elements are shown in
Fig. 8.1. The lowest level shown in each diagram corresponds to the ground state.
-0.381
-0.54 eV
-0.85 eV

-0.049
-0.18 eV

-1.5 eV

-2.5 eV

-0.61 eV

-3.4 eV

-5.7 eV

-1.7 eV

-14 eV
Hydrogen

-0.67 eV
-0.94 eV
-1.4 eV

-8.7 eV

-3.8 eV

Lithium

Sodium

Fig. 8.1
(i) The three atoms from cooled gas samples were bombarded with electrons of
energy 5 eV. Draw on Fig. 8.1 all possible transitions due to the bombardment
of electrons.
[3]

(ii) The source of the electrons was replaced by a light source with energy also
equal to 5 eV. Suggest and explain if there would be any changes in the
observed outcome.
.
.
.
.
.
....[3]

SRJC 2008

8866/02/PRELIM/2008

21

(iii)

For
Examiners
Use

Some emission lines as shown in Fig. 8.2 were observed using an accurate
and sophisticated detector from a very dim unknown star. State with reasons,
which of the 3 elements is/are possibly present in that star?

92 nm 94 nm

99 nm

117 nm

Fig. 8.2
.
.
.
....[3]
(iv) The actual values of the wavelengths are slightly shorter at 83 nm, 85 nm,
90 nm and 107 nm respectively. Suggest a possible cause for the difference.
.
.
.
....[2]

~END OF PAPER~

SRJC 2008

8866/02/PRELIM/2008

[Turn Over

22

BLANK PAGE

SRJC 2008

8866/02/PRELIM/2008

SERANGOON JUNIOR COLLEGE


General Certificate of Education Advanced Level
Higher 1
NAME
CG

INDEX NO.

PHYSICS

8866/01
th

Preliminary Examination
Paper 1 Multiple Choice
Additional Materials:

26 August 2008
1 hour

Multiple Choice Answer Sheet

READ THESE INSTRUCTIONS FIRST


Write in soft pencil.
Do not use staples, paper clips, highlighters, glue or correction fluid.
Write your name, civics group and index number in the spaces at the top of this page and on the
Answer Sheet in the spaces provided.
In the table of Index Number on the Answer Sheet, write down the number 2, followed by the last 4
digits of your NRIC number and shade the corresponding boxes.
Eg. A student with NRIC number of S1234567A will write and shade 24567.
There are thirty questions on this paper. Answer all questions. For each question there are four
possible answers A, B, C and D.
Choose the one you consider correct and record your choice in soft pencil on the separate Answer
Sheet.
Read the instructions on the Answer Sheet very carefully.
Each correct answer will score one mark. A mark will not be deducted for a wrong answer.
Any rough working should be done in this booklet.

This document consists of 14 printed pages and 0 blank pages.


SRJC 2008

8866/01/PRELIM/2008

[Turn Over

2
DATA AND FORMULAE
Data
speed of light in free space,

= 3.00 x 108 ms1

elementary charge,

= 1.60 x 1019 C

the Planck constant,

= 6.63 x 1034 Js

unified atomic mass constant,

= 1.66 x 1027 kg

rest mass of electron,

me = 9.11 x 1031 kg

rest mass of proton,

mp = 1.67 x 1027 kg

acceleration of free fall,

= 9.81 ms2

uniformly accelerated motion,

s
v2

=
=

work done on/by a gas,

W =

pV

hydrostatic pressure,

gh

resistors in series,

R1 + R2 +

resistors in parallel,

1/R =

Formulae

SRJC 2008

ut + at2
u2 + 2as

1/R1 + 1/R2 +

8866/01/PRELIM/2008

3
Answer all questions
1

Certain derived units can be expressed in alternative forms. Which of the following
examples shows a corresponding alternative form of unit?
Unit

Alternative form

coulomb (C)

A s1

watt (W)

J m1

volt (V)

J C1

tesla (T)

N m1

Ans: C
Potential difference (V) is the energy (E) dissipated per unit charge (Q).

With regards to precision and accuracy, which of the following statements is true?
A
B
C
D

A set of results with good precision suggests that there are little random
errors in the experiment.
A set of results with poor accuracy could be improved by taking the average of
several readings.
A set of results with high precision can be achieved by taking the average of
several readings.
None of the above

Ans: A
Results with good precision is the same as a set of results with a small spread,
suggesting that there is little random error. Note, however, that the systematic
error could still be high if the mean of these values deviate to a large extent with
respect to the true value.

The external diameter and the internal diameter of a metal tube are measured to be
50.0 0.5 mm and 48.0 0.5 mm respectively. What is the percentage uncertainty in
finding the thickness of the metal tube?
A
B
C
D

5%
20%
25%
50%

Ans: D
t = (D d) / 2 = D/2 d/2
SRJC 2008

8866/01/PRELIM/2008

[Turn Over

4
t = D + d
= (0.5) + (0.5)
= 0.5
t = (50.0 48.0)/2
= 1.0
%t/t = 0.5/1.0 100% = 50%

A cannon at the top of a 30 m high hill fires a shell at an angle of 30.0 upwards from
the horizontal with a speed of 50.0 m s-1. Taking air resistance to be negligible, what is
the angle to the vertical at which the shell lands on level ground?
A
B
C
D

7.93
38.9
51.2
82.1

Ans: C
vertically,
vy2 = uy2 + 2 aysy
vy2 = (-50.0 sin 30.0)2 + 2 (9.81) (30)
vy= 34.8 ms-1
horizontally,
vx = (50.0 cos 30.0)
= 43.3 ms-1
tan = 43.3/ 34.8
= 51.2

What are the necessary assumptions for usage of the equations of motion?
I.
II.
III.
IV.

particle must move in a straight line


particle must move with constant velocity
particle must move with increasing velocity
particle must move with constant acceleration

A
B
C
D

III
I and IV
I, II and IV
I, III and IV

Ans: B

SRJC 2008

8866/01/PRELIM/2008

5
II and III are wrong because the particle may have an increasing velocity but still
have a constant acceleration (so II is wrong) or have a constant velocity with a
constant acceleration (so III is wrong)
6

An elastic rubber ball is dropped from a height. It comes to a momentary stop when it
hits the ground and its compression is at a maximum. Which of the following
statements best explains its resultant force at this instant?
A
B
C
D

Its resultant force is zero because it is instantaneously at rest.


Its resultant force is zero because at maximum compression, the force from the
ground is exactly equals to its weight.
Its resultant force is upwards because it is experiencing an upward acceleration.
Its resultant force is downwards because the rubber ball is pushing the ground
with a force greater than its weight due to its compressed state.

Ans: C
The ground is exerting an upward force on the ball during impact

Three crates of masses 3M, M and 5M are stacked on top of one another on the floor
of a lift as shown below.
3M

M
5M

When the lift is accelerating upwards with an acceleration a, the magnitude of the
force mass M exerts on the mass 5M is given by
A
B
C
D

4Mg 5Ma
4Mg 4Ma
4Mg + 4Ma
4Mg + 5Ma

Ans: C
Considering forces acting on the combined 3M and M crates,
Fby 5M on M
4M

Fby 5M on M 4Mg = 4Ma


Fby 5M on M = 4Mg + 4Ma
By Newtons 3rd Law, Fby 5M on M = Fby M on 5M

4 Mg
SRJC 2008

8866/01/PRELIM/2008

[Turn Over

A burglar of mass 70 kg jumped down to a concrete ground from a window ledge


0.50 m above the ground in the attempt to escape. In a fluster, he forgot to bend his
knees while landing. What was the average force that the jump had on his bone
structure if he took 15 ms to come to a complete stop after landing (assuming his
velocity at the window ledge is zero)?
A
B
C
D

14.6 N
45.7 N
14.6 x 103 N
45.7 x 103 N

Ans: C
Using v2 = u2 +2as
v = 3.13 ms-1
Using the impulse momentum theorem,
average force = change in momentum / time = (70 x 3.13) / 0.015 = 14.6 x 103 N

A boy starts to jump vertically from a horizontal ground. Which of the following pairs
of forces form an action-reaction pair?
A
B
C
D

The frictional force and his weight.


The normal force and his weight.
The thrust he exerts on the ground and his weight.
The thrust he exerts on the ground and the normal force.

Ans: D

The graph below shows what happens to a material as an increasing amount of force
is applied to it. What does the shaded area represent?
Force / N

10

Extension/ m
A
Change in momentum
SRJC 2008
8866/01/PRELIM/2008

11

B
Rate of change in momentum
C
Spring constant
D
Elastic potential energy stored
Ans: D
The following diagrams show forces acting on uniform rulers. Which of the following do
not show a couple only?
I

II
20 N

20 N

20 N

30 N

III

IV

20 N

10 N

10 N

20 N
A
B
C
D

I and III only


I, III and IV only
I, II and III only
All of the above

Ans: C

12

A very long glass tube opened at one end is filled with water to a depth of 1.00 m and
inclined at 40 to the horizontal. What is the pressure at the bottom of the tube?
(density of water = 1000 kg m-3, atmospheric pressure = 1.01 x 105 Pa)
A
B
C
D

6.27 x 103 Pa
6.31 x104 Pa
1.07 x 105 Pa
1.11x 105 Pa

Ans: C
P = (1.00 sin 40) water g + 1.01 x 105
= (1.00 sin 40) x 1000 x 9.81 + 1.01 x 105
= 1.07 x 105 Pa
SRJC 2008
8866/01/PRELIM/2008

[Turn Over

The following graph shows the variation of the potential energy of a particle with its
displacement.
Potential Energy (J)

13

50

25

10

Displacement (m)

What is the force exerted on the particle at the point marked x?


A
B
C
D

5 N in the positive direction


5 N in the negative direction
62.5 N in the positive direction
62.5 N in the negative direction

Ans: B
F = -dU/dx
= -(25/5)
= -5 N
= 5 N in the negative direction
(some may try to take area under graph and end up with 62.5 N instead)

14

In a bleak winter morning, Bobby finds a portable generator (normally used to generate
electricity) in the attic of his house. He reads the label stuck to it and finds that when a
test (to check the electrical energy produced) was last done on it, the generator was
80% efficient. The energy produced included, but was not limited to, 1800 J of
electrical energy, 100 J of heat energy and 100 J of sound energy. As he is feeling
very cold, he decides to turn on the generator for warmth. If the test results still hold
true, what is the efficiency of the generator for his situation?
A
B
C
D

4%
5%
68%
70%

Ans: A
Input energy = 1800 x 100/80 = 2250 J
SRJC 2008
8866/01/PRELIM/2008

9
Heat is the useful energy produced in this case.

15

Efficiency = useful energy output/ total energy input x 100%


= 100 / 2250 x 100% = 4 %
An empty lift weighing 8000 N moves upwards at a constant velocity of 5.0 m s-1. A
constant frictional force of 1000 N retards its motion upwards. What power is the lift
motor delivering, assuming it is operating at 100 % efficiency?
A
B
C
D

5 kW
35 kW
40 kW
45 kW

Ans: D
P = Fv = (8000 + 1000) (5.0)
= 45 kW

16

A loudspeaker emits a sound wave of amplitude A and intensity I. After some time, the
intensity decreased by 6.00 %. The corresponding change in amplitude is
A
B
C
D

2.96 %
3.05 %
94.0 %
97.0 %

Ans: B
I A2 A I
A2
=
A1

0.94I 0.9695
=
I
1

% change =

A2 A1
100%
A1

0.9695 1
100%
1
= 3.05%
=

SRJC 2008

8866/01/PRELIM/2008

[Turn Over

10

17

The graph below shows the how the pressure P of a region varies with distance d
from a fixed point.
P/ Pa
Patm + Po
Normal atmospheric
pressure, Patm

d/ m
do

Patm - Po

Taking leftward as positive direction, which one of the following shows how the
displacement of particles, x, varies with d in the given region?
x/ m

x/ m
xo

xo
d/ m

0
do
- xo

do
- xo

B
x/ m

x/ m
xo

xo

do d/ m

- xo

d/ m

0
do
- xo
D

C
Ans: C

SRJC 2008

d/ m

8866/01/PRELIM/2008

11

18

What of the following situations would result in two-source interference fringes not
being observed?
I The sources having significantly different amplitudes
II The sources having different wavelengths
III The sources having the same frequency
IV Transverse waves polarized in the same plane
A
B
C
D

III only
I and II
III and IV
I, II and IV

Ans: B

19

A microwave transmitter of frequency, f, is placed several metres away from a metal


plate deflector. A microwave receiver, connected to a microammeter, is used to detect
nodes and antinodes along the normal from the source to the plate. The microwave
receiver is moved from one node through 10 antinodes to another node, a distance of
L m. What is the speed of the microwave?
A
B
C
D

0.16fL
0.18fL
0.20fL
0.25fL

Ans: C
Microphone moves through 5 wavelengths, time taken = 5T = 5 / f
Speed = L / (5 / f) = 0.20 fL

SRJC 2008

8866/01/PRELIM/2008

[Turn Over

12

20

The figure below shows plots of the current I through the cross section of a wire over 4
different time periods.
I

Rank the plots in ascending order according to the net charge that passes through the
wire.
A
B
C
D

W, Z, Y, X
X, Y, Z, W
W, Y, Z, X
Z, W, Y, X

Ans: D
Area under graph = charge.
21

A lighting circuit consists two lamps of similar resistances connected to a constant d.c.
source. The variable resistors RX and RY control the relative intensities of the lamps.

RX

RY

If power output of lamp X is thrice that of lamp Y, the ratio of the currents in RX to that
in RY should be
A

1:3

1:

C
D

3 :1
3:1

Ans: C
SRJC 2008

8866/01/PRELIM/2008

13

22

Use P = I2R
Since R is constant for both lamps, P is proportional to square of I.
A power supply is known to have an e.m.f. of 1000 V. However, when a voltmeter of
resistance 10 k is connected to the terminals of the supply, only a reading of 200 V
is obtained.
What is the internal resistance of the power supply?
A
B
C
D

40
50
40 k
50 k

Ans: C
Current in the voltmeter = V / R = 200 / 10000 = 0.02A
Voltage across the internal resistor = 1000 200 = 800 V
Internal resistance = 800 / 0.02 = 40 K (definitely unrealistic)

23

A 230V power supply is providing power for identical lamps each rated 230V 45 W
connected in parallel.
6A fuse

230 V

The maximum number of lamps (operating normally) that can be connected to the
mains in parallel without activating the 6 A fuse is
A
B
C
D

6
7
30
31

Ans: C
P=IV
45 = I (230)
Current in each lamp = 0.20 A
For current to exceed 6 A
N (0.20) = 6.0
N = 30

SRJC 2008

I = 0.20 A

8866/01/PRELIM/2008

[Turn Over

14

24

Five resistors are connected as shown in the figure below.


2
Y
1

1
2

X
What is the equivalent resistance between X and Y?
A
B
C
D

0.47
0.68
1.45
1.84

Ans: D
The two 1 are in parallel with each other. resistance = 0.5
It is then in series with the 2 on top, hence resistance = 2 + 0.5 = 2.5
The above calculated resistance is in parallel with the 7 (5 and 2 resistors
which are in series).
Hence effective resistance = 1 / (1/7 +1/2.5) = 1.84

25

The figure below shows a 2 m long conductor carrying a current of 3 A into the plane
of the page.
50o
P
magnetic field

If the magnetic flux density is 0.5 mT, what is the magnitude and direction of the
resultant force acting on the conductor?

A
B
C
D

Magnitude
0.002 N
0.002 N
0.003 N
0.003 N

Ans: D
SRJC 2008

Direction
P
Q
P
Q

8866/01/PRELIM/2008

15

26

Three current-carrying conductors X, Y and Z are placed perpendicular to the plane of


the page as shown below.
X
60o

60o
Y

Z
In which of the following directions does the resultant force on Z act?

60o

60o

45o

45o

Ans: B

SRJC 2008

8866/01/PRELIM/2008

[Turn Over

16

27

Which of the following pair correctly define the magnetic flux density and the tesla?
Tesla

Magnetic flux density


A

The number of magnetic flux lines


per unit area where a currentcarrying conductor is placed.

The magnetic flux density of a wire


of length one metre carrying a
current of one ampere has a force
of one newton in a direction at right
angles to both the flux and the
current.

The force per unit current per unit


length of the conductor when the
conductor is at right angles to the
field.

The force acting on a wire of length


one metre and carrying a current of
one ampere when it is placed at
right angles to the magnetic field.

The number of magnetic flux lines


per unit area where a currentcarrying conductor is placed

The force acting on a wire of length


one metre and carrying a current of
one ampere when it is placed at
right angles to the magnetic field.

The force per unit current per unit


length of the conductor when the
conductor is at right angles to the
field.

The magnetic flux density of a wire


of length one metre carrying a
current of one ampere has a force
of one newton in a direction at right
angles to both the flux and the
current.

Ans: D

SRJC 2008

8866/01/PRELIM/2008

17

28

In an experiment on the photoelectric effect, monochromatic light is directed onto the


surface of a metal plate and the kinetic energy of the fastest moving emitted electrons
is measured. The graph shows how this kinetic energy changes as the frequency of
the light is changed.
kinetic energy
of electrons

frequency
Which one of the following statements about the experiment is true?
A
B
C
D

The intercept on the frequency axis would change if light of a different intensity
were used.
The gradient of the graph would change if a different metal were used.
The intercept on the frequency axis would change if a different metal were used.
The gradient of the graph would change if light of a different intensity were
used.

Ans: C
hf = hfo + K
K = hf - hfo
Gradient will never change as h is always a constant.
Change metal used will change the intercept.

29

What will happen to electrons which have kinetic energy of a few hundreds of eV,
when they are incident onto carbon film?
A
B
C
D

They will not be absorbed by the atoms because the energies of the electrons
do not correspond to the difference in energy levels.
They will be absorbed by the carbon atoms and photons of the same energy
will be given out.
Most of them will pass straight through as there are a lot of empty space in the
carbon film.
They see the carbon lattice as a diffraction grating and get diffracted.

Ans: D
Electrons will not be absorbed by the atoms.
SRJC 2008

8866/01/PRELIM/2008

[Turn Over

18
It will pass through, but most of them will not pass through straight. They will
get diffracted as the wavelength is about that of the lattice spacing.
30

The photoelectric effect is a result of


A
B
C
D

photoelectrons hitting the surface of metal to release more photoelectrons.


the accumulation of light wave to release photoelectrons.
the accumulation of wave energy to release photons.
electrons absorbing photons of sufficient energy to release themselves.

Ans: D

-END OF PAPER-

SRJC 2008

8866/01/PRELIM/2008

SERANGOON JUNIOR COLLEGE


General Certificate of Education Advanced Level
Higher 1
NAME
CG

INDEX NO.

PHYSICS

8866/02
26th August 2008
2 hours

Preliminary Examination
Paper 2
Structured Questions
No Additional Materials are required.

Candidates answer on the Question Paper.

READ THIS INSTRUCTIONS FIRST


Write your name, civics group and index number in the spaces at the top of this page and on all the work
you hand in.
Write in dark blue or black pen.
You may use a soft pencil for any diagrams, graphs or rough working.
Do not use staples, paper clips, highlighters, glue or correction fluid.
Section A
Answer all questions.
Show all your workings clearly on the question paper.
Section B
Answer any two questions. Circle the questions chosen
in the table on the right bottom corner of this page.

For Examiners Use


Marks awarded
Q1
Q2
Q3

/ 9
/ 6
/ 10

Q6

/ 20

Q7

/ 20

Q8

Section B

/ 10

Q5

Section A

/ 5

Q4

At the end of the examination, fasten all your work


securely together.
The number of marks is given in bracket [ ] at the end of
each question or part question.

/ 20
/ 80

Total marks

This document consist of 22 printed pages and 1 blank page


SRJC 2008

8866/02/PRELIM/2008

[Turn Over

2
DATA AND FORMULAE
Data
speed of light in free space,

= 3.00 x 108 ms1

elementary charge,

= 1.60 x 1019 C

the Planck constant,

= 6.63 x 1034 Js

unified atomic mass constant,

= 1.66 x 1027 kg

rest mass of electron,

me = 9.11 x 1031 kg

rest mass of proton,

mp = 1.67 x 1027 kg

acceleration of free fall,

= 9.81 ms2

ut + at2

v2

u2 + 2as

Formulae
uniformly accelerated motion,

work done on/by a gas,

W =

pV

hydrostatic pressure,

gh

resistors in series,

R1 + R2 +

resistors in parallel,

1/R =

SRJC 2008

1/R1 + 1/R2 +

8866/02/PRELIM/2008

For
Examiners
Use

3
Section A
Answer all questions
1

The graph in Fig. 1 shows the velocity of an object travelling over a certain time
period. The displacement of the object is 0 m at t = 0 s.
v / m s-1
8.0
3.0

9.0

12.0

14.5 17.0

0
- 4.0

t/s

Fig. 1
(a)

Calculate the displacement of the object at t = 3.0 s, 9.0 s, 12.0 s, 14.5 s and 17.0 s.
When t = 3.0s, displacement = (8.0 x 3.0) =24 m
When t = 9.0s, displacement = 24 + (0.5 x 6.0 x 8) = 48 m
When t = 12.0s, displacement = 48 - (0.5 x 3 x 4) = 42 m
When t = 14.5s, displacement = 42 (0.5 x 2.5 x 4 =37 m
When t = 17.0s, displacement =37 m

SRJC 2008

8866/02/PRELIM/2008

[Turn Over

For
Examiners
Use

4
(b)

Sketch the displacement-time graph for the motion of the car in the space provided
below. Include in your sketch all appropriate labels, including the displacement of
the object at t = 3.0 s, 9.0 s, 12.0 s, 14.5 s and 17.0 s.
s/m
48
42
37
24

3.0

9.0

12.0 14.5 17.0

t/s

Appropriate between labeling on the vertical axis.


No kinks seen.
Correct shape/ Overall trend is correct for each portion.

2(a)

Define linear momentum.


Linear momentum is defined as the product of mass and velocity.

(b)

State the principle of conservation of momentum. Discuss how this principle is


consistent with Newtons first law of motion using the motion of a single body.
The principle of conservation of momentum states that the total momentum
of a system is constant unless an external force acts on the system.
Newtons Law of Motion states that a body will continue in its state of rest or
uniform motion in a straight line unless an external force acts on it.
Hence in the absence of external force, the speed of the body remains
constant and the momentum of the body will also remain constant.

SRJC 2008

8866/02/PRELIM/2008

For
Examiners
Use

5
(c)

A stationary 238U nucleus decays (i.e split up) into a


as shown in the diagram below.

v1

238

234

Th nucleus and an -particle

v2

234

Before decay

Th

After decay

The mass of the 234Th nucleus is larger than that of the -particle.
With appropriate working, determine if the kinetic energy of the -particle is greater
than the kinetic energy of the 234Th nucleus.
By conservation of momentum,
Total initial momentum = total final momentum
0
= m1(-v1) + m2v2
v2
= m1v1 / m2
Kinetic of particle

= m2 v22
= m2 (m1v1 / m2)2
= m1 v12(m1 / m2)

Since the mass of 234Th is larger than that of , the KE of is greater than that
of Th.

(d)

Explain if the above situation is inelastic.

For an inelastic collision, the kinetic energy of the system is not conserved.
Since the total initial KE (0 J) is not equal to the total final KE for the above
situation, it is similar to that of an inelastic collision.

SRJC 2008

8866/02/PRELIM/2008

[Turn Over

For
Examiners
Use

6
3 (a)

A simple current balance can be set up by inserting a light rectangular conducting


frame partially into a solenoid as shown in Fig. 3.1. The frame is pivoted off-centre,
with two-third of its length within the solenoid. The width of the frame which is
outside of the solenoid has a force meter attached to it.

Solenoid

3.0 cm
A

Light frame
D

3.0 A

Force
meter

to d.c. source
Fig. 3.1
(i) Sketch on Fig. 3.1 how a 6 V battery should be inserted in the dotted box above
to ensure that this current balance setup can operate normally.

(ii) If the resistance of the solenoid is 1.5 , calculate the current in the solenoid.
Current
(iii) If the strength of the magnetic field within the solenoid is 2.5 T, calculate the
magnitude of the resultant force acting on side AB of the frame?
Magnitude of force

SRJC 2008

8866/02/PRELIM/2008

For
Examiners
Use

7
(iv) Calculate the reading on the force meter when the wire frame is balanced?
Sum of clockwise moments = sum of anticlockwise moments
Hence,
(b)

Charged particles are given an initial velocity within uniform magnetic field strength,
B, are shown in the diagrams below. Sketch the subsequent paths taken by these
particles.
x

(ii) x

(i)

x
_

x
+

4(a)

x
x
x
magnetic field

x
magnetic field

Sketch and explain the I-V characteristics of a thermistor.


I

R is ing

V
As larger p.d is applied, the vibration of the atoms become larger and
resistance increases due to more collisions. At the same time, the atoms
release more mobile electrons which form part of the current.
The effect on resistance due to the large increase in the number of mobile
electrons (charge carriers) is much more significant than that of the increase
in atomic vibration. Hence the overall resistance decreases.

SRJC 2008

8866/02/PRELIM/2008

[Turn Over

8
(b)

A cable consists of a steel core coated with aluminium as shown below. The inner
radius (steel core) is 5 mm and the total radius (steel and aluminium) is 10 mm.

Steel core

Aluminium surround
Calculate the resistance of a 100 m length of the cable when a current flows
through the length of the cable.
(Given that the resistivity of steel is 9.0 x 10-8 m and that of the aluminium is
2.5 x 10-8 m)

Rsteel =

R Al =

l 9.0 10 8 100
=
= 0.115
r 2
(5 10 3 ) 2

l
2.5 10 8 100
=
= 0.0106
r 2 (10 10 3 ) 2 (5 10 3 ) 2

The conductors are essentially in parallel, hence the cable resistance


1
1
3
Rtotal = 1
+
= 9.71 10
0.115 0.0106

SRJC 2008

8866/02/PRELIM/2008

For
Examiners
Use

9
5

For
Examiners
Use

Table 5.1 below shows data on typical ball speeds and striker speeds for different
sports.

______________________________________________________________________________
Ball
Ball mass/ kg Ball velocity/ m s-1
Striker velocity/ m s-1
Impact time / ms
before
after
before
after
______________________________________________________________________________
Cricket ball
0.16
0
39
31
27
1.4
(hit from rest)
Football (free
kick)

0.42

28

18

12

8.0

Golf ball
(drive)

0.046

69

45

32

1.3

Hand ball
(serve)

0.061

23

19

14

1.4

Squash ball
(serve)

0.032

49

44

34

3.0

Tennis ball
(serve)

0.058

51

38

33

4.0

Rally ball
0.20
0
30
35
28
1.5
(serve)
____________________________________________________________________________
Table 5.1

Using data from the table above, answer the following:


(a)

(i) Calculate how much energy is wasted when the 0.7 kg golf club hits the golf ball
to move it horizontally only.
Loss in kinetic energy of the golf club = (0.7) (452 322) = 350 J
Gain in kinetic energy of the golf ball = (0.046) (692) = 110 J
Energy wasted = 350 110 = 240 J
(ii) Explain if the principle of conservation of energy is violated.
The principle of conservation of energy is not violated. Some of the kinetic
energy lost by the golf club was transferred to the golf ball, the remaining
kinetic energy lost by the golf club was wasted as sound energy and heat
energy.

SRJC 2008

8866/02/PRELIM/2008

[Turn Over

10
(iii) Calculate the efficiency in driving the golf ball for this particular case.
Efficiency = (110 / 350) x 100%
= 31 %
(iv) Suggest the effect of a longer impact time between the golf ball and golf club,
assuming the average force applied to the golf ball throughout the time of
impact has not changed.
Having a longer impact time (or contact time), whilst keeping the average
force the same, would result in a larger change in momentum, which
would result in a final larger final velocity of the ball.
(b)

The new game, Rally Ball, involves the usage of a gas piston as striker as shown in
Fig. 5.1. The ball is placed on top of the piston, and when the gas expands, the ball
fires out. If the work done by the gas piston is entirely converted to the kinetic
energy of the ball, and the gas has a constant pressure of 1.0 x 105 Pa, calculate
the change in volume of the gas. Assume negligible friction and air resistance.

gas
Fig. 5.1
Kinetic energy of the rally ball after it is being launched = m v2
= (0.20) (30)2
= 90 J
= work done by gas
Work done by gas (piston) = p (V)
90 = 105 (V)
V = 9.0 x 10-4 m3

SRJC 2008

8866/02/PRELIM/2008

For
Examiners
Use

11
For
Examiners
Use

Section B
Answer 2 out of 3 questions in this section.

6(a)

An aeroplane of mass 1.5 x 105 kg moves horizontally with constant velocity. The
forces exerted on the aeroplane are as shown in the Fig. 6.1 below.
Lift

Drag
Thrust (0.60 x 106 N)

Weight

Fig. 6.1
(i) Calculate the values of the lift and the drag.
Fy = may
Fx = max

L 1.5 x 105(9.81) = 0
0.60 x 106 drag = 0

L = 14.7 x 105 N
drag = 0.60 x 106 N

(ii) State the conditions for the aircraft to be in equilibrium.


Vector sum of forces must be zero.
Vector sum of torque must be zero.
(iii) The horizontal separation of the lines of action of lift and weight is 0.8 m. What
is the vertical separation of the lines of action of thrust and drag?
Sum of clockwise torque = Sum of anti-clockwise torque
=
14.7 x 105(0.8)
0.60 x 106 d
Vertical separation, d =
1.96 m

SRJC 2008

8866/02/PRELIM/2008

[Turn Over

For
Examiners
Use

12
(b) In a training session, both engines on one wing are shut down. The situation at the
instant of shutdown is then as shown in Fig 6.2.

C.G.
5

1.5 x 10 N

C.G.
10 m

1.5 x 105 N

20 m

1.5 x 10 N

10 m

1.5 x 105 N

Fig. 6.2

20
20 m
P

Fig 6.3

One way to counteract the turning moment produced by the two engines is to adjust
the rudder at the tail of the aircraft at an angle as shown in Fig. 6.3. A perpendicular
push P on the rudder by the resultant air current is created.
(i) If the rudder is tilted by 20o, and P is acting at a point approximately 30 m
measured along the axis (shown as dotted line in Fig. 6.3) from center of gravity,
calculate the value for P that can prevent the aircraft from turning.
150 000 x 10 + 150 000 x 20 = 30 x P cos 20o
P = 150 000 x 30 / (27.0 x cos 20o) = 1.73 x 105 N
(ii) The value you found in (b)(i) is rather large and the rudder may not be able to
withstand it. Suggest a way the pilot has of reducing the value of P?
Shut down the outer engine or reduce the thrust of the working engines.

SRJC 2008

8866/02/PRELIM/2008

For
Examiners
Use

13
(c)

In a rural village, a group of villagers attempt to transport a heavy load of 2000 N


across a stream of width 2x. A rope is attached to the 2000 N load and is slung over
the branch of a tree which is above the edge of the right bank. A second rope
attached to the load is thrown across the stream to some villagers on the left bank.
Fig. 6.4 shows the situation when the load is halfway across the stream.

branch

x
T2
T1
right bank

left bank
Fig. 6.4

(i) Calculate the tensions T1 and T2 in the left and right rope respectively when the
load is in equilibrium.
x
x

= tan-1 ( ) = 450
Fy = 0
T2 sin 450 W = 0
W
1000
T2 =
= 2828 N
=
0
sin 45
sin 45 0
FX = 0
T2 cos 450 -T1 = 0
T1 = T2 cos 450 = 2828 cos 450 = 2000 N
OR

T2

using a vector diagram to solve for T1 and T2


W
2000
W
since tan =
T1 =
=
= 2000 N
T1
tan tan 450
sin 450 =

SRJC 2008

W
T2

T2 =

W
2000
=
= 2828 N
sin sin 450

8866/02/PRELIM/2008

T1

=450

[Turn Over

14
(ii) Calculate the tensions T1 and T2 when the load has reached the left bank but
not been lowered down?
When the load is halfway across the stream, its horizontal distance from
the edge of the right bank is also x. This is evident from the 450 angle
subtended by the right rope with the horizontal.
When the load reached the left bank, its horizontal distance from the edge
of the right bank is 2x.
x
Angle subtended by the right rope when the

load reached the left bank


2x
= tan-1 () = 26.70
Fy = 0
T2
T2 sin 26.70 W = 0
2000
26.70
T2 =
= 4451 N
T1
0
sin 26.7
FX = 0 i.e.
T2 cos 26.70 -T1 = 0
T1 = T2 cos 26.70 = 2236 cos 450 = 3976 N
W
(iii) If a typical villager can exert a maximum horizontal force of 300 N and has a
weight of 800 N, estimate with appropriate workings, the minimum number of
villagers needed to move the load across the stream this way.
N1: no. of students needed on the left bank
N2: no. of students needed on the right bank
On the right bank, the total weight of the students 4451 N
N2 (800) 4451
N2 5.6 therefore minimum N2 = 6
On the left bank, the total force exerted by the students 3976 N
N1 (300) 3976
N1 13.3 therefore minimum N1 = 14
Minimum no. of villagers = 14 + 6 = 20

SRJC 2008

8866/02/PRELIM/2008

For
Examiners
Use

For
Examiners
Use

15
7(a)

An open metal pipe can be extended by pulling both ends of the pipe at a steady
speed of 5.0 cm s-1, as shown in Fig. 7.1. A sound wave of frequency 1250 Hz
passes through the pipe.
5.0 cm s-1

5.0 cm s-1

Fig 7.1
(i) The effective length L of the pipe at time t is expressed as
L = L0 + 2(5.0 102)t
where L0 is the unextended length
If the time between consecutive loud sounds being heard is 2.7 s, calculate the
wavelength of sound.
L + 2(5.0 102)t1 = n
eqn (1)
L + 2(5.0 102)t2 = (n+1) eqn (2)
(2) (1)
2(5.0 102)(t2 t1) =
= 2(5.0 102) 2.7 = 0.27 m
(ii) Hence calculate the speed of sound.
v = f = 1250 0.27 = 338 m s1
(iii) Sketch on Fig. 7.2 the stationary wave pattern of the highest possible frequency,
if each segment of the pipe has a length of 50 cm.

Fig. 7.2

SRJC 2008

8866/02/PRELIM/2008

[Turn Over

16
(b)

For
Examiners
Use

Fig. 7.3 below shows two speakers S1 and S2 placed in an open field on a windless
day. Their separation is 3.81 m. D is a microphone placed in the same horizontal
plane as the speakers, and at a distance of 10.0 m from S1, and
10.7 m from S2. The lines S1S2 and S1D are perpendicular to each other. When the
speakers are switched on, sound of wavelength 0.20 m is emitted in phase.
S2

10.7 m

3.81 m

S1

10.0 m

Fig 7.3
(i) State how coherence could have been achieved in this case.
By connecting the two loudspeakers to the same signal generator.
(ii) Calculate the phase difference between the sound waves reaching D from S1
and S2.
=

10.7 10.0
2
0.200
= 7
=

[1]
[1]

(iii) Hence explain whether a minimum intensity or a maximum intensity would be


detected by D.
The waves meet exactly out of phase [1], thus there is destructive
interference, so an intensity minimum is formed [1].

SRJC 2008

8866/02/PRELIM/2008

17
(iv) When the wavelength of the sound was slowly varied to a value 1, the
microphone D detected one cycle of change in intensity. Calculate two possible
values of 1.
For 1 cycles of intensity, the phase difference corresponds to (7 + 2 =
9) or (7 2 = 5).

x
2

0.7
2
=
9
= 0.16 m

SRJC 2008

x
2

0.7
=
2
5
= 0.28 m

[1]
[1]

[1]
[1]

8866/02/PRELIM/2008

[Turn Over

For
Examiners
Use

18
(c)

Parallel water waves strike a straight wall with an opening in the centre, as shown
in Fig. 7.4 and Fig. 7.5.
For each of the figure below, draw to scale, the pattern of the wavefronts after
passing through the opening.

Fig 7.4

Fig 7.5

SRJC 2008

8866/02/PRELIM/2008

For
Examiners
Use

19
8(a)

A parallel beam of violet light of wavelength 4.5 x 10-7 m and intensity 700 W m-2 is
incident normally on a surface.
(i) Explain what is meant by a photon?
Photon is a quantum pack of energy.
(ii) Calculate the momentum of a photon of the violet light.
P=h/
= 1.5 x 10-27 kg ms-1

(b)

In the photoelectric experiment, the apparatus were arranged such that the intensity
and frequency of the radiation may be varied. If each of these was increased in turn
whilst the other was kept constant, explain the effect on the
(i) number of electrons emitted per second, and
Increasing the intensity implies an increase in the number of photon per
second. Since there is an increase in this number per second, there will be an
increase in the number of photoelectron liberated per second. Increasing the
frequency will not change the number of electrons emitted per second.
(ii) maximum speed of the photoelectrons.
Increasing the frequency will increase the energy of the photon. With the
work function remaining the same , therefore the kinetic energy of the
photoelectron will increase, that is, the maximum speed will increase.
Increasing the frequency will not affect the maximum speed of the
photoelectron .

SRJC 2008

8866/02/PRELIM/2008

[Turn Over

For
Examiners
Use

For
Examiners
Use

20
(c)

The energy levels of three isolated atoms of different elements are shown in
Fig. 8.1. The lowest level shown in each diagram corresponds to the ground state.
-0.381
-0.54 eV
-0.85 eV

-0.049
-0.18 eV

-1.5 eV

-2.5 eV

-0.61 eV

-3.4 eV

-5.7 eV

-1.7 eV

-14 eV
Hydrogen

-0.67 eV
-0.94 eV
-1.4 eV

-8.7 eV

-3.8 eV

Lithium

Sodium

Fig. 8.1
(i) The three atoms from cooled gas samples were bombarded with electrons of
energy 5 eV. Draw on Fig. 8.1 all possible transitions due to the bombardment
of electrons.

(ii) The source of the electrons was replaced by a light source with energy also
equal to 5 eV. Suggest and explain if there would be any changes in the
observed outcome.
There will not be any spectra lines being observed. This is because if
photon was used, it must be wholly absorbed. Since this is a 5 eV photon,
and there is no corresponding energy difference that is equal to 5 eV, then
there will not be absorption at all.

SRJC 2008

8866/02/PRELIM/2008

For
Examiners
Use

21
(iii)

Some emission lines as shown in Fig. 8.2 were observed using an accurate
and sophisticated detector from a very dim unknown star. State with reasons,
which of the 3 elements is/are possibly present in that star?

92 nm 94 nm

99 nm

117 nm

Fig. 8.2

E = hc/ = hc/(117 nm)


E = 1.70 x 10-18 J = 10.6 eV
Looking at even the largest wavelength, that is, the smallest energy, the
only possible element found is the unknown star is hydrogen.
(iv) The actual values of the wavelengths are slightly shorter at 83 nm, 85 nm,
90 nm and 107 nm respectively. Suggest a possible cause for the difference.
The star may be on another galaxy which is moving towards ours, thus
making the wavelength slightly shorter. This is known as the red-shift.

~END OF PAPER~

SRJC 2008

8866/02/PRELIM/2008

[Turn Over

22

BLANK PAGE

SRJC 2008

8866/02/PRELIM/2008

TEMASEK JUNIOR COLLEGE


2008 Preliminary Examination
Higher 1

PHYSICS

8866/01

Paper 1 Multiple Choice


19 September 2008
1 hour
Additional Materials:

Multiple Choice Answer Sheet

READ THESE INSTRUCTIONS FIRST


Write in soft pencil.
Do not use staples, paper clips, highlighters, glue or correction fluid
Write your name, Centre number, Index number and C.G. on the Answer Sheet in the
spaces provided.
There are thirty questions in this paper. Answer all questions. For each question there are
four possible answers, A, B, C and D.
Choose the one you consider correct and record your choice in soft pencil on the separate
Answer Sheet.
Read the instructions on the Answer Sheet very carefully.
Each correct answer will score one mark. A mark will not be deducted for a wrong answer.
Any rough working should be done in this booklet.

This document consists of 12 printed pages.


[Turn over

Data
speed of light in free space,

3.00 x 108 m s-1

elementary charge,

1.60 x 10-19 C

the Planck constant,

6.63 x 10-34 J s

unified atomic mass constant,

1.66 x 10-27 kg

rest mass of electron,

me

9.11 x 10-31 kg

rest mass of proton,

mp

1.67 x 10-27 kg

ut + at2

v2

u2 + 2as

work done on/by a gas,

pV

hydrostatic pressure,

gh

resistors in series,

R1 + R2 +

resistors in parallel,

1/R

1/R1 + 1/R2 +

Formulae
uniformly accelerated motion,

The drag force F is related to the velocity v by the expression


F = KAv2
where K is a constant, is the density of air, A is the cross-sectional area of the
raindrop and v is the velocity of the raindrop.
What is the unit of K?
A

m s-1

C
2

dimensionless
kg m s-1

m-2 s-4

The distance between adjacent atoms in a solid is of the same order of magnitude as
A

infra-red radiation.

C
3

radio waves.
visible light.

X-rays.

Given that the quantities a, b and c are related by a = bc4.


What is the percentage uncertainty in b if the maximum percentage uncertainties in a
and c are 1 % and 2 % respectively?
A

1%

3%

7%

9%

Which of the following experimental techniques does not reduce random error of the
quantity being investigated?
A
B

Timing a large number of oscillations to find the period.

Repeating the experiment with a new set of apparatus.

D
5

Plotting data on a graph to get a best fit line.

Repeating the experiment and calculating mean value for the readings.

Ball A is dropped from the top of a building. One second later, ball B is dropped from
the same building. Neglecting air resistance, as time progresses, the difference in their
speeds
A
B

increases.

decreases.

D
6

remains constant.

increases and then decreases.

A car decelerates uniformly from a speed of 30 m s-1 to rest in 20 s. The distance


travelled by the car in the first 10 s is
A

150 m

225 m

300 m

375 m

The gondola of a cable-car system lifts skiers to the top of a mountain. Normally, the
gondola hangs vertically, but on a windy day, the gondola is blown through an angle by
the force of the wind as shown in the figure.

cable
support
force of the wind

gondola

The weight of the gondola is 1900 N and the tension in the support is 2100 N when the
gondola is in equilibrium.
The magnitude of the horizontal force acting on the gondola is
A

1900 N

C
8

890 N
1000 N

2800 N

A railway trolley starts from rest and is driven along a straight horizontal track by a
motor which exerts a constant force. The effects of friction and air resistance can be
neglected. Which of the graphs below best represents the variation of the momentum p
with distance travelled s?
A

B
p

A spring obeying Hookes law has an unstretched length of 60 mm and a spring


constant of 500 N m-1. What is the tension in the spring when its overall length is 90
mm?
A

10

8N

15 N

30 N

45 N

A mass hangs by a string from the ceiling of a carriage in a train and is just above a
certain mark on the floor when the train is at rest. When the train is moving forward with
constant velocity, the mass
A
B

is behind the mark in a position in which the horizontal force exerted by the train on
the mass is balanced by the horizontal component of the tension in the string.

remains over the mark because the force due to the motion of the train is balanced
by the reaction of the string on the support.

11

is behind the mark, so that the string is along the resultant of the forces due to the
motion of the train and gravity.

remains over the mark because the motion of the train produces no additional force
on the mass.

Two blocks, one made of wood and the other made of copper, are arranged at rest on
the ground as shown in combinations X and Y below.

wood

copper

copper

wood

ground

ground

Which one of the following statements is correct?


A

The force by the ground on the copper block in X is greater than the force by the
ground on the wooden block in Y because the copper block, being denser than the
wooden block, exerts more force on the ground.

The force exerted by the wooden block on the copper block in X is the same as
that by the copper block on the wooden block in Y by virtue of Newtons third law.

The force exerted by the copper block on the wooden block in X is greater than that
by the wooden block on the copper block in Y.

The force by the wooden block on the copper block is equal to the weight of the
wooden block in X while the force by the copper block on the wooden block is
equal to the weight of the copper block in Y.

12

A neutron moving with an initial velocity u has a head-on elastic collision with a
stationary proton. After the collision, the velocity of the neutron is v and that of the
proton is w. Taking the masses of the neutron and proton to be equal, which one of the
following statements is wrong?
A
B

Conservation of energy shows that u2 = v2 + w2.

The fact that the collision is elastic implies that the neutron and proton move off in
opposite directions with equal speeds.

13

Conservation of momentum shows that u = v + w.

The momentum and energy equations taken together imply that the speed of the
proton after the collision is the same as that of the neutron before the collision.

The diagram below shows two pucks on the surface of an ice rink just prior to their
head-on collision.

1.0 m s-1

3.0 m s-1

0.10 kg

0.20 kg

Immediately after the collision the puck of mass 0.10 kg has a velocity of 2.0 m s-1 to
the left. The velocity of the puck of mass 0.20 kg immediately after the collision is
A

1.5 m s-1 to the right

C
14

zero
1.5 m s-1 to the left

2.5 m s-1 to the right

A golfer misjudges a putt and leaves her ball way short, the ball going only one-third of
the way to the hole. If the speed of the ball leaving the putter in the first case is vo and
the force of resistance due to the grass remains the same, what speed should she have
given to the ball to make the original putt?
A

15

2 vo

3 vo

6 vo

3 vo

The velocity-time graph of an object is as shown below.


v/ m s-1
10.0

6.0
t/ s
10.0
0
If the mass of the object is 2.0 kg, find the work done by the external force on the
object.
A

-64 J

-32 J

32 J

64 J

7
16

A space vehicle of mass m re-enters the Earths atmosphere at an angle of to the


horizontal. Because of air resistance, the vehicle travels at a constant speed v. The
heat shield of the vehicle dissipates heat at a rate P, so that the mean temperature of
the vehicle remains constant.
Taking g as the relevant value of the acceleration of free fall, which expression is equal
to P?
A

17

mgv

mgv sin

mv2

mv2 sin2

A small container which is open to the atmosphere contains a layer of liquid L, floating
on liquid M. Liquid M has a density which is twice as great as that of liquid L.
x

Liquid L
Liquid M
Which graph shows how the pressure, p, at a point varies with its height, x above the
base of the container?
A

x
18

A body of mass 4.0 kg falls vertically through air. When the magnitude of air resistance
is 30 N, what is the acceleration of the body?
A

0.4 m s-2

2.3 m s-2

6.5 m s-2

17.3 m s-2

19

In which part of the electromagnetic spectrum does a photon having an energy of 7.10 x
10-19 J occurs ?
A
B

X-ray

infra-red

D
20

ultraviolet

visible

An ideal organ pipe resonates at frequencies 50 Hz, 150 Hz, 250 Hz, .
The pipe is
A
B

open at both ends and is of length 3.4 m.

closed at one end, open at the other and of length 1.7 m.

D
21

open at both ends and is of length 1.7 m.

closed at one end, open at the other and of length 3.4 m.

The figure shows a sound wave traveling to the right in air. Air particles P and Q are at
the centres of a rarefaction and a compression respectively. Which of the following
gives correctly the directions of motion of P and Q at the moment shown?
Direction of wave
propagation

Particle P
A

to the right

to the left

to the right

at rest

to the left

to the right

D
22

Particle Q

to the left

at rest

Youngs fringes are viewed using light of wavelength 600 nm and the fringe separation
is found to be 1.0 mm. When the distance between the double slits and the plane in
which the fringes are viewed is increased by 2.0 m, the fringe separation is found to be
3.0 mm. What is the separation, in mm, of the double slits producing the fringes ?
A

0.20

0.40

0.60

0.90

9
23

A mains circuit contains six similar bulbs connected in series. One of the bulbs has a
broken filament. Voltmeters X and Y of infinite resistance are placed in the circuit shown
below.

Which of the following voltmeter readings are correct?


Voltmeter X
A

240

240

240

D
24

Voltmeter Y

240

In the circuit below, R1 and R2 are fixed resistors and R is a variable resistor.

I2

I1

R2
R1
R

As R decreases,
A

I1 increases, I2 decreases.

I1 increases, I2 increases.

I1 remains unchanged, I2 increases.

I1 remains unchanged, I2 decreases.

10
25

A d.c.circuit is set up below.

12
8

The potential at point X on the above circuit is


A
26

3V

5V

11 V

13 V

Two parallel, straight conductors, placed at right angles to a ruler, carry currents of 2I and
I flowing in opposite directions.
2I

0
|

10
|

20
|

40
|

50
|

70
|

80
|

90
|

100
|

At which point on the ruler is the resultant magnetic field zero? You may assume that the
Earths magnetic field is negligible.
(Magnetic flux density B at a distance a from a long, straight conductor is given by
o I
B=
where o is the permeability of free space and I is the current in the
2 a
conductor.)
A

0 cm mark

30 cm mark

50 cm mark

90 cm mark

11
27

A long conductor carrying a current is placed in a rectangular region of magnetic field B of


flux density 0.65 T.
0.56 m
B = 0.65 T
0.35 m

conductor

0.22 m
What is the magnitude and direction of the current that produces a 1.6 N force on the
wire directed out of the plane of the paper?
Magnitude of current
A

4.4 A

Right

4.4 A

Left

11 A

Right

D
28

Direction of current

11 A

Left

In an experiment on photoelectric effect, monochromatic light is directed onto the


surface of a metal plate and the kinetic energy of the fastest moving emitted electrons is
measured. The graph shows how this kinetic energy changes as the frequency of the
light is changed.
kinetic energy
of electrons

frequency

Which one of the following statements about the experiment is true?


A

The intercept on the frequency axis would change if a different metal were used.

The intercept on the frequency axis would change if more intense light were used.

The gradient of the graph would increase if more intense light is used.

The gradient of the graph would change if a different metal were used.

12
29

When a parallel beam of white light passes through a cool gas, dark lines appear in the
spectrum of the emergent light. This is because energy is absorbed by the gaseous
atoms and
A
B

is re-radiated uniformly in all directions.

is re-radiated at different frequencies.

D
30

is not re-radiated at all.

converted to heat.

A beam of electrons is incident on a crystal lattice. The regularly spaced parallel planes
of ions in the lattice can serve as a diffraction grating. The spacing between each plane
is 1 x 10-8 m. In order for significant diffraction to occur, the kinetic energy of the each
electron should be of the order
A

10-2 eV

10-21 eV

10-26 eV

10-34 eV

TEMASEK JUNIOR COLLEGE


2008 Preliminary Examination
Higher 1
NAME

CG

CENTRE
NUMBER

INDEX
NUMBER

PHYSICS

8866/02

Paper 2

12 September 2008

Candidates answer on the Question Paper.


No Additional Materials are required.

2 hours

READ THESE INSTRUCTIONS FIRST


Write your name and C.G. on all the work you hand in.
Write in dark blue or black pen on both sides of the paper.
You may use a soft pencil for any diagrams, graphs or rough working.
Do not use staples, paper clips, highlighters, glue or correction fluid.
Section A
Answer all questions.
Section B
Answer any two questions.
You are advised to spend about one hour on each section.

For Examiners Use


1

At the end of the examination, fasten all your work securely


together.

2
Section A
3

The number of marks is given in brackets [ ] at the end of each


question or part question.

4
5

Section B
Total
This document consists of 20 printed pages.
[Turn over

Data

speed of light in free space,

3.00 x 108 m s-1

elementary charge,

1.60 x 10-19 C

the Planck constant,

6.63 x 10-34 J s

unified atomic mass constant,

1.66 x 10-27 kg

rest mass of electron,

me

9.11 x 10-31 kg

rest mass of proton,

mp

1.67 x 10-27 kg

ut + at2

v2

u2 + 2as

work done on/by a gas,

pV

hydrostatic pressure,

gh

resistors in series,

R1 + R2 +

resistors in parallel,

1/R

1/R1 + 1/R2 +

Formulae
uniformly accelerated motion,

Section A
Answer all questions in the spaces provided.
It is recommended that you spend about one hour on this section.
1

A park ranger has discovered that the bridge over Ulu gorge was swept away by floods. The
ranger is setting up a road block 100 m from the gorge when a mountain biker rides past
toward the gorge. The biker does not hear the rangers warning and continues toward the
gorge at a constant speed. The ranger, who is originally at rest, immediately pursues the
biker in her jeep with an acceleration of 4.0 m s-2.
(a)

What maximum speed can the biker have so that the ranger can catch up to her
before she falls into the gorge?

bikers speed =
(b)

[3]

m s-1

[2]

What is the rangers speed when she catches up to the biker?

rangers speed =
(c)

m s-1

Using the same axes, draw graphs to show how the displacement of the mountain
biker and the park ranger vary as a function of time.

[2]

(a)

It is often stated that many forms of transport transform chemical energy into kinetic
energy. Explain why a cyclist traveling at constant speed is not making this
transformation. Explain what transformations of energy are taking place.

[3]

(b)

By reference to the movement of charge carriers, explain why power is required to


maintain an electric current in a metallic conductor.

[4]

Fig. 3.1 shows two dippers, P and Q, mounted on the same vibrating beam. The dippers
touch the surface of the shallow water in a ripple tank. When the beam vibrates, waves
travel outwards in all directions on the surface of the water from each dipper.
beam
Fig. 3.1
vertical
oscillations
dipper Q
dipper P
(a)

Explain why a stationary wave will be formed on the surface of the water along the line
joining P and Q.

[2]
(b)

When the beam vibrates at a certain frequency, the distance between two adjacent
nodes along the line joining P and Q is 12 mm. When the frequency of vibration is
increased by 2.0 Hz, the distance between two adjacent nodes decreases to 10 mm.
(i)

Explain what is meant by a node.

[1]
(ii)

Calculate
1.

the frequency at which the beam vibrated originally,

frequency =

Hz

[3]

2.

the speed at which the waves traveled on the surface of the water.

m s-1

speed =
4

[1]

Two long vertical parallel wires X and Y placed 6.0 cm apart, carry steady currents 4.0 A
and 2.0 A respectively as shown in Fig. 4.1. The current in X is flowing out of the paper and
the current in Y is flowing into the paper.
X

Z
10.0 cm

6.0 cm

Fig. 4.1
(a)

A student standing at a point Z, 10.0 cm from Y, is concerned about the strength of


the magnetic field he experiences. Calculate the magnitude of the resultant
magnetic flux density at Z due to currents in X and Y.
(The magnetic flux density B at a distance a from a wire is given by

B=

0 I
where o is the permeability of free space and I is the current in the wire.)
2a

Take o = 4 x 10-7 H m-1.

resultant magnetic flux density at Z =


(b)

[2]

If the Earths magnetic flux density is 4.0 x 10-5 T, should the student be concerned
about the strength of the magnetic field he is experiencing at Z. Explain.

[1]

7
(c)

Determine the magnitudes of the force per unit length, F1 and F2 acting on wires X
and Y respectively. Indicate, by using arrows, the directions of F1 and F2 in Fig. 4.1.

F1 =
F2 =
5

N m-1
N m-1

[4]

Nowadays, manufacturers are giving considerable thought to making vehicles more energy
efficient. One way of doing this is to fit a flywheel on a vehicle. The flywheel is a solid
cylinder which can store kinetic energy when it is rotating. When the vehicle slows down its
kinetic energy is used to increase the rotational speed of the flywheel. This stored energy is
used later to accelerate the vehicle. A similar arrangement is used in some toy cars to make
them travel further after being pushed.
In order to investigate the practicalities of flywheel energy storage, it is necessary to use
some equations from the physics of rotation.
Fig. 5.1 shows a cylindrical flywheel of length l, radius r and mass m spinning about its
central axis with frequency of rotation f.

Fig. 5.1

The kinetic energy of this flywheel is given by the equation


kinetic energy = 2 2 f2 I
where I is called the moment of inertia and is given by
I = m r2
Some details of three flywheels are given in Fig. 5.2. Assume that all the flywheels are
made of steel of density 7800 kg m-3.

8
bus flywheel

car flywheel

toycar flywheel

Length l/m

0.40

0.20

0.008

Radius r/m

0.20

0.10

0.015

Volume V/m3

0.050

0.0063

5.7 x 10-6

Mass m/kg

390

Maximum frequency of
rotation f/Hz
Moment of Inertia I

200

Maximum kinetic energy


stored/J

0.044
200

7.8

5.0 x 10-6

6.2 x 106

0.50
Fig. 5.2

(a)

Deduce the base units for moment of inertia I.

base units of I =
(b)

[2]

Calculate the missing data in Fig. 5.2 and complete the table. Show your workings
below.

[6]

9
(c)

Explain
(i)

why a flywheel system is likely to be more commercially successful for a town


bus than for a car,

[2]
(ii)

why is it advantageous to enclose the flywheel in a strong, evacuated container.

[2]

10

Section B
Answer two questions from this section. Each question carries 20 marks.
It is recommended that you spend about one hour on this section.

(a)

State the conditions for the equilibrium of a body which is acted upon by a
number of forces.

[2]
(b)

An aerial mast is hinged at its base as shown in the Fig. 1.1.


A

Fig. 1.1
15.0 m
700 N
5.0 m

40
B
hinge

(i)

The aerial A is a wire attached horizontally to the mast at a height of 15.0


m above the ground. Another wire B, making an angle of 40 with the
vertical, is attached to the mast at a height of 5.0 m. The tension in B, TB,
is 700 N. Show that the tension in A, TA, is 150 N.

[2]

11
(ii)

The weight of the mast is 2000 N. The hinge exerts a resultant force F on the
mast. Find the magnitude and direction of F.

F=
direction =
(c)

[4]
[2]

State the Principle of conservation of momentum.

[2]

12
(d)

Fig 1.2 shows all the forces acting on a 0.80 kg trolley placed on top of a smooth
ramp.

smooth ramp

Wooden block
70 cm

Smooth
surface

30
weight
Fig 1.2
When released from a height of 70 cm, the trolley accelerates down the ramp. It
then travels at a constant velocity along a smooth horizontal surface before
colliding with a wooden block of mass 0.40 kg. After the collision, the velocity of
the trolley decreases to 40% of its initial value and the wooden block moves
along the horizontal surface with a constant velocity.
(i) Find the speed of the trolley at the bottom of the ramp.

speed of trolley =
(ii)

m s-1

[2]

Find the speed of the wooden block after the trolley has collided with it.

speed of wooden block =

m s-1

[3]

13
(iii) Calculate the impulse on the trolley.

Impulse on trolley =
2

(a)

Ns

[3]

A car battery has an internal resistance of 0.060 . It is re-charged using a battery


charger having an e.m.f. of 14 V and an internal resistance of 0.10 as shown in Fig.
2.1.

Fig. 2.1

(i)

At the beginning of the re-charging process, the current in the circuit is 42 A


and the e.m.f. of the battery is E (measured in volts).
For the circuit of Fig. 2.1, state
1.

the magnitude of the total resistance,


resistance =

2.

the total e.m.f. in the circuit. Give your answer in terms of E.


e.m.f. =

(ii)

[2]

Use your answer to (i) and data from the question to determine the e.m.f. of the
car battery at the beginning of the re-charging process.

e.m.f. =

[2]

14

(b)

For the majority of the charging time of the car battery, the e.m.f. of the car battery is 12
V and the charging current is 12.5 A. The battery is charged at this current for 4.0 hours.
Calculate, for this charging time,
(i)

the charge that passes through the battery,

(ii)

[2]

the energy supplied from the battery charger,

energy =
(iii)

[2]

charge =

the total energy dissipated in the internal resistance of the battery charger and the
car battery.

energy =

[2]

(c) Use your answer in (b) to indicate the percentage efficiency of transfer of energy from
the battery charger to stored energy in the car battery.

efficiency =
(d)

[2]

A heating element in an electric heater has a power rating of 960 W and is


designed to be used with a 240 V supply.
(i)

Calculate the resistance of the element.

resistance=

[2]

15
(ii)

Explain why the cables to the element do not become hot, yet the element itself,
with the same current, does become hot.

[2]
(iii) In the manufacture of the wire for the element in the electric heater, a material of
resistivity 1.10 x 10-6 m is used. What length of wire, of diameter 0.46 mm, is
required for the element?

length=

[2]

(iv) Suggest an advantage of using a long wire of this diameter rather than a shorter
but thinner wire of the same resistance.

[2]

16

(a) The work function of a metal X can be found using the set-up as shown in Fig. 3.1.
Light of different frequencies and intensities can be shone onto the photocathode C
which is made of metal X.

Fig. 3.1
(i)

Draw, using the same axes, an I-V graph to show how the photocurrent I detected
by the microammeter will vary with the potential difference V across the
photocathode and anode when the light is of (1) high intensity and (2) low
intensity.

[2]
(ii) Explain the meaning of the term work function.

[1]

17
(iii) Explain how the set-up in Fig. 3.1 can be used to measure the work function of
metal X accurately.

[3]
(iv) Hence, or otherwise, explain one experimental evidence to show that light is a
particle.

[1]
(b) The graph in Fig. 3.2 indicates there is a threshold frequency below which no photoelectrons are emitted.

Fig. 3.2

18

(i)

Determine the threshold frequency.

threshold frequency =

Hz [1]

(ii) Calculate the kinetic energy of the fastest moving electrons emitted by light of
frequency 5.5 x 1014 Hz.

kinetic energy =

[2]

(iii) Hence, or otherwise, obtain values for the Planck's constant and work function.

Planck's constant =
work function =

Js
J

[3]

19
(c) Fig. 3.3 shows some of the energy levels of the outermost electron in a mercury atom.
Level 1 represents the lowest possible energy level.

Level

Energy
0.0 eV

-1.6 eV

-3.7 eV

-5.5 eV

-10.4 eV

Fig. 3.3
Fig. 3.3

(i) Explain clearly why a line spectrum results from an atom with such energy levels.

[2]
(ii) State the possible level(s) which the mercury atom could be excited to if a moving
electron of energy 7.0 eV collides with the atom in its ground state?

[1]
(iii) How much kinetic energy, in eV, is retained by an incident electron after a collision
leading to each possible transition in (c)(ii)?

[2]

20

(iv) Instead of an electron, a photon of energy 7.0 eV is incident on the mercury atom
in the ground state. Which transitions, if any, would be made by the electron in the
ground state? Explain your answer.

[2]

TAMPINES JUNIOR COLLEGE


Preliminary Examination 2008

8866/01

PHYSICS
Higher 1
PAPER 1

Multiple Choice

THURSDAY 25 AUGUST 2008


1 hour

Time 1130 1230 hr


Additional Materials: Multiple Choice Answer Sheet

READ THESE INSTRUCTIONS FIRST


Write in soft pencil.
Do not use staples, paper clips, highlighters, glue or correction fluid.
Write your name and class on the Answer Sheet in the spaces provided.
There are forty questions on this paper. Answer all questions.
For each question there are four possible answers A, B, C and D. Choose the one you
consider correct and record your choice in soft pencil on the separate Answer Sheet.
Read the instructions on the Answer Sheet very carefully.
Each correct answer will score one mark. A mark will not be deducted for a wrong answer.
Any rough working should be done in this booklet.

This document consists of 12 printed pages and 0 blank pages.

Data

speed of light in free space,

3.00 108 m s1

elementary charge,

1.60 1019 C

the Planck constant,

6.63 1034 J s

unified atomic mass constant,

1.66 1027 kg

rest mass of electron,

me

9.11 1031 kg

rest mass of proton,

mp

1.67 1027 kg

acceleration of free fall,

9.81 m s2

Formulae
s

ut + at2

v2

u2 + 2as

work done on/by a gas,

p V

hydrostatic pressure

gh

resistors in series,

R 1 + R 2 + ...

resistors in parallel,

1/R

1/R 1 + 1/R 2 + ...

uniformly accelerated motion,

Errors in measurement may be either systematic or random in nature. Which of the


following involves random error?
A
B

Not subtracting background count rate when determining the count rate from a
radioactive source.

Stopping a stopwatch at the end of a race.

Not accounting for zero error on a micrometer screw gauge.

Giving g a value of 10 N kg1 when calculating weight from mass.

The radiancy, R T , is defined as the total energy emitted per unit time per unit area
from a blackbody at thermodynamic temperature T. It was first stated empirically in
1879 in the form given by

RT T 4
where is the Stefan-Boltzmann constant. Which of the following is a unit for ?
A

W m-2 K-4

J s m-2 K-4

C kg s-3 K4

J s-1 m-2 K4

In various experiments to determine the value of the acceleration of free fall (g=9.81
m s-2), the following results and their uncertainties were obtained as follows:
A

9.7 0.1 m s-2

9.24 0.02 m s-2

9.555 0.001 m s-2

9.734 0.005 m s-2

Which of the above result is the most accurate but not very precise?

A girl leaves the classroom and walks 10 m north to a drinking fountain. Then she
turns and walks 30 m west to a physics laboratory. What is the girls total
displacement from the classroom to the physics laboratory?
A

20 m West

40 m North

C 32 m 71.6o
west of north

32 m 18.4o north
of east

The velocity-time graph of an object moving linearly past a point Q at time t = 0 is


given below. Which of the following statements about the motion of the object is
correct?
v

t3

t1

t2

t
t4

Figure 5
A
B

At time t 2 , the object is furthest from Q.

At time t 3 , the object is momentarily at rest.

At time t 1 , the object has maximum acceleration.

At time t 4 , the object is back at Q.

An object was projected from the ground with a velocity v at an angle to the
horizontal. It traces a parabolic path as shown below, reaching a maximum height H at
a horizontal distance of H from the point of launch and after a time T.

Figure 6
If g is the acceleration due to gravity, which of the following is a correct equation
relating some of these quantities?
A

v2 sin2 = 2gH

v cos = - gT

vTcos = 2H

v2T2 = 2H2

An object held at rest inside a viscous fluid is dropped and allowed to fall vertically. If
the velocity and acceleration of the object is v and a respectively, which one of the
following graphs correctly represents the variation of v or a with time?
A

C
v

0
B

D
a

A non-uniform rod of mass 4.0 kg is suspended by a string attached at B and is


hinged at A so that it is horizontal. The centre of gravity of the rod is 60 cm from A
and 40 cm from B. The string makes an angle 30o with the vertical as shown.
Assuming the pulley is frictionless, find the mass M.

B
Figure 8

2.8 kg

4.8 kg

31 kg

47 kg

30o

6
9

A helicopter of mass 3.0 x 103 kg rises vertically with a constant speed of 25 m s-1.
Taking the acceleration of free fall to be 10 m s-2, what is the lift force acting on the
helicopter?
A
B

3.0 x 104 N

7.5 x 104 N

10

0N

10.5 x 104 N

An object of mass 1.4 kg is moving with a constant velocity of 2.0 m s-1 along a line
PQ as shown below.
S

30O

R
Figure 10

If the object is to be made to move along the path RS which is at an angle of 30O to
PQ, but the component of velocity along PQ remains unchanged, which of the
following has to be applied at R?
A
B

1.6 Ns in a direction perpendicular to PQ.

2.3 Ns in the direction along RS.

11

1.2 Ns in a direction perpendicular to PQ.

3.2 Ns in the direction along RS.

A mass of 1.3 kg attached to a spring causes it to extend by 6.7 cm at equilibrium.


What is the potential energy stored in the spring?
A

12

43 J

0.044 J

0.43 J

An object initially moving with constant velocity


eastwards is acted on by 3 forces as shown in the
vector diagram on the right:

0.85 J

North
5.0 N

What is the work done on the object if it moves a


distance of 5.0 m after the forces are applied?

0 J

15 J

30 J

60 J

3.0 N
Figure 12

4.0 N

7
13

Two objects P and Q are originally held together at rest with a compressed spring
between them. P has a larger mass compared to Q. Which of the following statements
is correct about the motion of P and Q upon release and the spring is allowed to
extend? Assume there is no friction.
P

Q
Figure 13

A
B

P has a larger acceleration compared to Q.

The magnitudes of the velocity of P and Q are equal.

14

P has a smaller momentum compared to Q.

The kinetic energy of P is less than that of Q.

An electric motor is required to haul a lift of mass 400 kg up a mine shaft through a
vertical height of 1200 m in 2.0 minutes. What will be the electrical power required if
the overall efficiency is 80%? [Take g = 10 m s-2]
A
B

32 kW

5.0 kW

15

50 kW

3.2 kW

Transverse sinusoidal waves of wavelength are progressing along a horizontal rope.


P and Q are points on the rope 1.25 apart and the waves are travelling from Q to P.
Which one of the following correctly describes Q at an instant when P is displaced
downwards but moving upwards?
Displacement of Q

Movement of Q

Upwards

Downwards

Upwards

Upwards

Downwards

Upwards

Downwards

Downwards

16

Figure 16a shows the displacement-position graph of a progressive wave at a


particular instant of time and Figure 16b represents the displacement-time graph of a
particle on the wave.

Figure 16b

Figure 16a
The speed of the wave is
A

17

0.30 m s-1

0.40 m s-1

300 m s-1

400 m s-1

The least distance between two points of a progressive transverse wave which have a
phase difference of

rad is 0.100 m. If the frequency of the wave is 600 Hz, what is

the speed of the wave?


A

18

100 m s-1

180 m s-1

360 m s-1

720 m s-1

In figure 18 below, Q is a microwave transmitter and R a detecting probe. As R is


moved towards the metal reflecting plate, a series of maxima are found separated by
20 mm

Figure 18
What is the frequency of the waves?
A

7.5 x109 Hz

0.9 x10 4 Hz

0.9 x10 10 Hz

7.5 x1 0 3 Hz

19

A student sets up a typical Young's double slit experiment in the laboratory using light
of frequency f and a double slit with slit-separation a. It is found that the screen is too
small to fit an adequate number of fringes. If the screen cannot be replaced or
moved, which of the following sets of changes can be made to solve this problem?
Frequency of light used
A

1.2f

0.8a

1.2f

1.2a

0.8f

0.8a

20

Slit- separation

0.8f

1.2a

Over a period of 5 s, a current is reduced uniformly from 150 mA to 30 mA. What is


the charge that flows during this time?
A

21

150 mC

450 mC

750 mC

900 mC

A battery of e.m.f E and internal resistance r delivers a current I through a resistance


R as in Figure 21.
E

Figure 21

R is set at two different values and the corresponding currents I are measured using
an ammeter of negligible resistance.
R/

I/A
3.0
2.0

1.0
2.0
What is the value of the e.m.f. E ?
A

3.0 V

3.5 V

4.0 V

6.0 V

10

22

Two wires X and Y, each of the same length and the same material, are connected in
parallel to a battery. The diameter of X is two times that of Y.
What fraction of the total current passes through Y?
A
B

0.25

0.33

23

0.20

0.50

Two electrical conductors of 200 k and 50 k respectively, form a potential divider.


Junction Y is maintained at +30 V and junction P is earth as shown
P

200 k

50 k

Figure 23
What is the potential at junction X?
A

24

120 V

120 V

150 V

150 V

The diagram shows a battery, 2 ammeters X and Y of negligible resistance and a


'black box' which consists of a combination of identical resistors.

Figure 24
Which of the combination of identical resistors would result in both ammeters
showing the same reading?

11

25

Two long straight current carrying wires, X and Y are placed perpendicular to each
other as shown in the figure 25 below.
Wire X
Wire Y

Figure 25
Current flows into the page in wire X and from left to right in wire Y. What is the
direction of the force acting on wire Y at point P due to the magnetic field produced by
wire X?
A

towards wire X

26

out of the page


into the page

away from wire X

Figure 26 shows how the stopping potential varies with the frequency of radiation
used in a photoelectric experiment for two different metals.
Stopping
potential
Metal A
Metal B

Frequency

Figure 26
Which of the following statements is true?
A

A higher frequency is needed to produce photoelectric effect with metal A than


with metal B.

Metal A has a higher work function than metal B.

Metal A has a smaller threshold frequency than metal B.

The fact that the two lines are parallel indicates that the work function is a
constant.

12

27

In a photoelectric experiment the wavelength of the light incident on the target material
is increased. What effect will this change of wavelength have on the kinetic energy of
the electrons emitted?
A

The maximum kinetic energy of the electrons is decreased.

The maximum kinetic energy of the electrons is increased.

The initial kinetic energies of the electrons remain unchanged.

The average kinetic energy of the electrons is increased.

28 Light quanta of energy 3.50 x 10-19 J fall onto the cathode of a photocell. The current
through the cell is just reduced to zero by applying a stopping potential of 0.25 V.
What is the work function energy of the cathode?
A
B

3.1 x 10-19 J

3.5 x 10-19 J

29

2.9 x 10-19 J

3.9 x 10-19 J

A surface is bombarded normally by photons of frequency . On average, n photons


strike a unit area of the surface each second. Assuming that the photons are
absorbed by the surface, what is the pressure exerted on the surface? (h is the Planck
constant, c is the speed of light)
A

30

2nhv

2nhv
c

nhv
c

nv

Figure 30 below shows three discrete energy levels of an atom of a gas.


n=3

- 2.4 eV

n=2

- 5.2 eV

n=1
(ground state)

- 25.3 eV
Figure 30

The gas is subjected to electrical excitation and subsequently emits photons of


different energies. Considering only transitions between the 3 levels shown, what is
the longest wavelength of photons emitted?
A

518 nm

444 nm

62 nm

54 nm

Candidate Name __________________________________ Civics Class ___________

TAMPINES JUNIOR COLLEGE


Preliminary Examination 2008

8866/02

PHYSICS
Higher 1
PAPER 2

FRIDAY 22 AUGUST 2008


2 hours

Time

0800 - 1000 hr

Candidates answer on the Question Paper.


No additional materials are required.

READ THESE INSTRUCTIONS FIRST


Write your name and class on all the work you hand in.
Write in dark blue or black pen in the spaces provided on the Question Paper.
You may use a soft pencil for any diagrams, graphs or rough
working.
For Examiner's Use
Do not use paper clips, highlighters, glue or correction fluid.
1
5

Section A
Answer all questions.

10

Section B

14

20

20

20

Sig. Fig.

Total

80

Answer any two questions.


You are advised to spend about one hour on each section.
At the end of the examination, fasten all your work securely
together.
The number of marks is given in brackets [ ] at the end of each
question or part question.

This document consists of 17 printed pages and 1 blank page.

Data
speed of light in free space,

3.00 108 m s1

elementary charge,

1.60 1019 C

the Planck constant,

6.63 1034 J s

unified atomic mass constant,

1.66 1027 kg

rest mass of electron,

me

9.11 1031 kg

rest mass of proton,

mp

1.67 1027 kg

acceleration of free fall,

9.81 m s2

Formulae
uniformly accelerated motion,

ut + at2

v2 =

u2 + 2as

work done on/by a gas,

W =

p V

hydrostatic pressure

gh

resistors in series,

R =

R 1 + R 2 + ...

resistors in parallel,

1/R =

1/R 1 + 1/R 2 + ...

3
Section A
Answer all the questions in the spaces provided.
It is recommended that you spend about one hour on this section.
1. An object falls off a cliff which is at a height, h, from the ground. The object took 13.2 seconds
to hit the ground, and it is estimated that there is a percentage uncertainty of 2 % in measuring
this time interval. The height of the cliff is determined to be (860 10) m.
(a)

Calculate the acceleration of free fall of the object (to three significant figures).

acceleration = .. [2]
(b)

Determine the absolute uncertainty of the value in (a) and hence give the value of g, with
its uncertainty, to an appropriate number of significant figures.

g = ...... [3]

4
2. (a)

State the conditions for a body to remain in static equilibrium.


..................................................................................................................................
..................................................................................................................................
............................................................................................................................. [2]

(b)

A horizontal force F is applied on a crate which remains stationary, as shown below.


G is the centre of mass of the object.

F
G

Figure 2.1

On the diagram, draw the following forces acting on the crate:


(i)
(ii)

(c)

weight W,
resultant force R that the ground exerts on the crate.

Draw a vector diagram of these forces which shows why the crate is stationary.

[3]

[1]

5
(c)

T1

14.4o

30o T 2

0.75L

0.25L

Figure 2.2
400 N
A uniform rod of length L and weight 120 N is supported by two ropes as shown above.
A 400 N weight is suspended one quarter way from the left end.
(i)

Calculate the tension T 2 .

T 2 = ...... [2]

(ii)

Calculate the tension T 1 .

T 1 = ...... [2]

6
3. (a)

The velocity-time graph of an object is shown below. Sketch an acceleration-time graph


for the object, including numerical values.
[3]

6.0

Figure 3.1

2.0

6.0

9.0

t /s

t /s

(b)

A bomber, flying horizontally at a speed of 150 m s-1 and at an altitude of 3.0 km above
the sea level, spotted an enemy vessel moving at constant velocity towards it. When the
vessel was at a horizontal distance of 4.0 km away from the bomber, a bomb was
released to hit the vessel.

3.0 km
Figure 3.2
4.0 km
Calculate the time taken for the bomb to hit the vessel.

time taken = ......

[2]

7
4. (a)

Define the tesla.

[2]

(b) Three current-carrying wires A, B and C are placed as shown in the diagram below.
B
2.0 A
3.0 m
A
2.0 A

6.0 A

x
C

Figure 4.1

3.0 m
(i) On the diagram given, indicate the
1. direction of the magnetic field which the current A causes at C.

[1]

2. direction of the magnetic field which the current B causes at C.

[1]

(ii) Using your answers in (b)(i), determine the direction of the resultant magnetic force
acting on wire C. (The magnetic flux density of (b)(i)(1) and (b)(i)(2) are equal in
magnitude)

Direction (in bearing) = [2]

8
5. A wire-wound resistor is manufactured by winding resistance wire on an insulating former. A
commonly used material for wire is an alloy of nickel and chromium called nichrome. The wire
is produced by pulling the nichrome through a suitably sized hole. Nichrome is sufficiently
ductile to be drawn into a wire without danger of it cracking or breaking after winding. It
resists corrosion and has a fairly high resistivity. The wire itself must be uniform and thin, and
is covered with an insulating material. A manufacturer of resistors of this type supplies
information concerning them in the form of a family of lines shown in the graph of Figure 5.1.
Resistors of different resistance R 1 , R 2 R 5 are shown by the separate lines.

Figure 5.1
(a)

By choosing some values of potential difference and current from Figure 5.1, complete
the table showing the resistances R 1 , R 2 R 5 .
[2]
R1 =
R2 =
R 3 = 1000
R4 =
R

R5 =
R

Table 5.1

(b)

(c)

Draw two additional lines on Figure 5.1:


(i)
one line for a resistance of 2000
(ii) one line for a resistance of 47

[4]

This particular set of resistors is manufactured so that the resistors can safely be used
with power dissipation up to 1 W. Complete the following table to show the maximum
safe current in the resistors for the potential differences given.
[2]
Potential difference

Maximum current

/V

/A

1000
100
10
1

Table 5.2

(d)

Plot the points in (c) on the graph of Figure 5.1. On the graph, indicate the region of safe
use for all these resistors.
[4]

(e)

The lines on Figure 5.1 represent ideal behaviour. Suggest, with a reason, how the
line for a real resistor might differ from the ideal.

[2]

10
Section B
Answer two questions for this section. Each question carries 20 marks.
It is recommended that you spend about one hour on this section.
6. (a)

Two identical objects A and B slide down at the same time from an initial height H from
the ground along two frictionless slopes as shown below,.
B

H/2

Figure 6.1
(i)

Write down an expression for the acceleration of object A as it slides down the
slope, explaining your symbols.

acceleration = .

[1]

(ii)

Sketch the velocity time graphs of A and B. Label your graphs clearly.

[3]

(iii)

Explain whether the two objects will have the same speed at the bottom of the
slope.

[2]

11
(iv)

Explain whether the two objects will reach the bottom of the slope at the same time.

[3]

(b) An object P of mass 1.50 kg slides on a frictionless ground with a constant velocity. It
collides head-on and elastically with another object Q of mass 2.35 kg that is at rest. The
variation of the force that P exerts on Q with time is shown below. The area under the
graph is 9.96 Ns.

uP
Figure 6.2

F/N

t/s

Figure 6.3

(i)

Explain what is meant by an elastic collision.

[1]

12
(ii)

Sketch on Figure 6.3, a graph that shows the variation of the force that Q exerts on
P with time. Explain your answer.

[2]
(iii)

Explain what the area under the graph you drew represents.

[2]
(iv)

Calculate the velocity of Q after the collision.

velocity = [2]
(v)

Calculate the velocities of P before and after the collision.

velocity of P before collision = [2]


velocity of P after collision = [2]

13
7. (a)

In a Youngs double-slit experiment, the fringe separation observed using yellow light was
found to be 0.275 mm. The yellow lamp, giving a wavelength of 5.50 x 10-7 m is replaced
by a purple one giving wavelengths of 4.00 x 10-7 m in the violet and 6.00 x 10-7 m in the
red. The remainder of the apparatus is undisturbed. Calculate
(i)

the distance between the fringes formed by the violet light.

distance = [2]
(ii)

the distance between the fringes formed by the red light

distance = [2]
(iii)

the distance from the purple fringe on the axis to the next purple fringe observed.

distance = [3]
(iv)

Hence, draw a diagram of the appearance of the new fringe system, indicating the
colours and extending as far as 1mm from the axis.
[3]

14
(b)

(i)

Distinguish between longitudinal and transverse waves.

[2]

(ii)

Give an example of a longitudinal and transverse wave.

[2]

(c)

Figure 7.1 shows a stationary wave on a string stretched between two points A and F
which are a distance L apart.

B
A

C
L
Figure 7.1

Compare the oscillations at points B, C, D and E in terms of their amplitudes and


relative phases of oscillation.
(1)

Amplitude:

(2)

Relative phase of oscillation:

15

[6]

8. (a)

A fluorescent tube is filled with mercury vapour at low pressure. In order to emit light, the
mercury atoms must first be excited. Explain what is meant by an excited mercury atom.

[1]

(b)

The diagram below shows some of the energy levels of the mercury atom.
Level 1 represents the lowest possible energy level. (The diagram is not drawn to scale)
Energy / eV

Level n

0
-0.70

-1.54

-2.69

-3.71

-5.72

-10.38

1
Figure 8.1

16
(i)

Explain how Figure 8.1 can be used to account for the emission spectrum.

[4]

(ii)

In a discharge tube, cool mercury vapour is bombarded with a stream of electrons


that have been accelerated from rest through a potential difference of 7.3 V.
1. State and explain how many different frequencies of electromagnetic radiation
will be emitted by the mercury vapour.

[2]

2. Calculate the longest wavelength of the electromagnetic radiation emitted.

wavelength= . [2]

(iii)

State the amount of energy, in electronvolts, required to ionise a mercury atom.

ionisation energy = . [1]

17
(c)

(i)

What is the photoelectric effect?

[1]
(ii)

State one observation from the photoelectric experiment, and explain how it shows
the particulate nature of light.

[3]
(d)

When a piece of metal is irradiated with monochromatic light of wavelength 404.7 nm


from a mercury discharge lamp, the stopping potential was found to be 1.12 V.
(i)

Determine the work function of the metal and the threshold frequency.

work function = . [2]


threshold frequency = . ..[2]

(ii)

If the power from the lamp incident on the metal is 0.120 W, find the rate of
incidence of photons on the metal.

rate of incidence = . [2]


END of PAPER

Tampines Junior College


Preliminary Examination 2008
H1 Physics Paper 1
1.

Answer C

2.

[ ] = [R T /T4] = J s-1 m-2/K4 = W m-2 K-4

3.

Answer D

30 m

4.

(Answer A)

resultant s = (102 + 302)1/2 = 32 m

10 m

(Answer C)

5.

Answer B

6.

: v2 = u2 + 2as
0 = (v sin)2 2gH
v2 sin2 = 2gH
(Answer A)

7.

Answer D

8.

Pulley frictionless tension in string on either side of pulley same.


Tension in string, T = Mg
Taking moment about A, Mg cos 30o x 1.0 = 4g x 0.6
M = 2.8 kg (Answer A)

9.

Constant speed

10.

P PQ = m(v tan ) = 1.42 (2.0 tan 30o) = 1.6 N s

11.

U = F e/2 = (1.3 x 9.81)(6.7 x 10-2) = 0.43 J (Answer C)

12.

Work done = Fd = (3.0 + 3.0)(5.0) = 30 J (Answer C)

13.

Conservation of momentum: 0 = m p v p + m Q v Q
V p = - (m Q /m p ) v Q
KE p < KE Q as m p > m Q
(Answer D)

14.

Power of motor = (WD in hauling lift /time taken)/overall efficiency of motor


= [(400 x 10 x 1200)/ 2 x 60] 0.8 = 50 kW (Answer A)

15.

Answer B

16.

v= /T = 0.80/ [(4/3.0 X 10-3) x 2.0] = 300 m s-1

17.

x/ = /2
0.100/ = (/3)/ 2 = 0.600 m.
(Answer C)

F net = 0 = T mg

T = mg (Answer B)
(Answer B)

(Answer C)

v = f = (600)(0.600) = 360 m s-1

18.

distance from maxima to the next maxima = /2 = 20 x 10-3 m


v=f
3.0 x 108 = f ( 2 x 20 x 10-3)
f = 7.5 x 109 Hz

(Answer A)

19.

x = D/a = vD/fa

(Answer B)

20.

I average = charge flowed/time taken


(30 + 150)/2 = Q/5
Q = 450 mC
(Answer B)

21.

E = 3.0 (1.0 + r)
--- (1)
E = 2.0 (2.0 + r)
--- (2)
Solving,
3.0 (1.0 + r) = 2.0 (2.0 + r)
r = 1.0
E = 6.0 V
(Answer D)

22.

R = l/A = l/(d2/4)

2
R x /R y = (d y /d x )
R x = R y /4
I total = I x + I y
IR
I y / I total = 0.20 (Answer A)

23.

Current flows from Y to P to X.


V x < V P (= 0 V)
V XP /V YP = R XP /R YP
V XP /30 = 200/50
V XP = 120 V
Potential at junction X = -120 V
(Answer B)

24.

(Answer B)

25.

(Answer A)

26.

hf = + eV s

27.

E = hc/ and hc/ = + K max

28.

hf = + eV s
3.5 x 10-19 = + 1.6 x 10-19 x 0.25
= 3.1 x 10-19 J
(Answer B)

29.

p = h/ = h/c
Force, F = p / t = n (h/c 0) = n h/c

30.

hf = hf o + eV s

R 1/d2

(Answer C)
(Answer A)

(Answer C)

= hc/E = (6.63 x 10-34) (3.0 x 108)/(5.2 2.4)(1.6 x 10-19) = 444 nm (Answer B)

Qn
1
6
11
16
21
26

Answer
C
A
C
C
D
C

Qn
2
7
12
17
22
27

Answer
A
D
C
C
A
A

Qn
3
8
13
18
23
28

Answer
D
A
D
A
B
B

Qn
4
9
14
19
24
29

Answer
C
B
A
B
B
C

Qn
5
10
15
20
25
30

Answer
B
B
B
B
A
B

Candidate Name __________________________________ Civics Class ___________

TAMPINES JUNIOR COLLEGE


Preliminary Examination 2008

8866/02

PHYSICS
Higher 1
PAPER 2

FRIDAY 22 AUGUST 2008


2 hours

Time

0800 - 1000 hr

Candidates answer on the Question Paper.


No additional materials are required.

READ THESE INSTRUCTIONS FIRST


Write your name and class on all the work you hand in.
Write in dark blue or black pen in the spaces provided on the Question Paper.
You may use a soft pencil for any diagrams, graphs or rough
working.
For Examiner's Use
Do not use paper clips, highlighters, glue or correction fluid.
1
5

Section A
Answer all questions.

10

Section B

14

20

20

20

Sig. Fig.

Total

80

Answer any two questions.


You are advised to spend about one hour on each section.
At the end of the examination, fasten all your work securely
together.
The number of marks is given in brackets [ ] at the end of each
question or part question.

This document consists of 17 printed pages and 1 blank page.

Data
speed of light in free space,

3.00 108 m s1

elementary charge,

1.60 1019 C

the Planck constant,

6.63 1034 J s

unified atomic mass constant,

1.66 1027 kg

rest mass of electron,

me

9.11 1031 kg

rest mass of proton,

mp

1.67 1027 kg

acceleration of free fall,

9.81 m s2

Formulae
uniformly accelerated motion,

ut + at2

v2 =

u2 + 2as

work done on/by a gas,

W =

p V

hydrostatic pressure

gh

resistors in series,

R =

R 1 + R 2 + ...

resistors in parallel,

1/R =

1/R 1 + 1/R 2 + ...

3
Section A
Answer all the questions in the spaces provided.
It is recommended that you spend about one hour on this section.
1. An object falls off a cliff which is at a height, h, from the ground. The object took 13.2 seconds
to hit the ground, and it is estimated that there is a percentage uncertainty of 2 % in measuring
this time interval. The height of the cliff is determined to be (860 10) m.
(a)

Calculate the acceleration of free fall of the object (to three significant figures).
h = at2/2 as initial speed = 0

2h
t2

[C1]

= 2 x 860/(13.2)2

= 9.87 m s-1

[B1]
acceleration = . [2]

(b)

Determine the absolute uncertainty of the value in (a) and hence give the value of g, with
its uncertainty, to an appropriate number of significant figures.

a h
t

2
a
h
t
a (

[C1]

h 2t

)a
h
t

10
2
2(
)) x9.87
100
860

= 0.5 m s-2
g = (9.9 0.5) m s-2

[A1]
[A1]

g = ............

[3]

4
2. (a)

State the conditions for a body to remain in static equilibrium.

Conditions for a body to remain in static equilibrium:


Net force acting on the body [B1] and the net torque acting on the body is zero [B1].
............................................................................................................................. [2]
(b)

A horizontal force F is applied on a crate which remains stationary, as shown below.


G is the centre of mass of the object.

Figure 2.1

On the diagram, draw the following forces acting on the crate:


(i)

weight W,

(ii)

resultant force R that the ground exerts on the crate.

[B1] weight
[B1] force due to ground,R
[B1] R passes through intersection

[3]

R
F
G

W
(c)

Draw a vector diagram of these forces which shows why the crate is stationary.

[B1] closed right angle triangle

W
F

[1]

5
(c)

T1

14.4o

30o T 2

0.75L

0.25L

Figure 2.2
400 N
A uniform rod of length L and weight 120 N is supported by two ropes as shown above.
A 400 N weight is suspended one quarter way from the left end.
(i)

Calculate the tension T 2 .

(i) By considering moments about the left end of the rod:


0.25L x 400 + 0.50L x 120 = T 2 cos 300 x L
T 2 = 185 N

[M1]

[A1]

T 2 = ...... [2]

(ii)

Calculate the tension T 1 .

(ii) horizontally: 185 sin 300 = T 1 sin 14.40 [M1]


T 1 = 372 N [A1]
OR

T 2 cos 300 + T 1 cos 14.40 = 400 + 120 [M1]


T 1 = 371 N [A1]

T 1 = ...... [2]

6
3. (a)

The velocity-time graph of an object is shown below. Sketch an acceleration-time graph


for the object, including numerical values.
[3]

6.0

Figure 3.1

6.0

2.0

9.0

a /m s-2
3.0
0
-2.0

(b)

t /s

shape [1]
value [1]

6.0

2.0

9.0 t /s t /s

A bomber, flying horizontally at a speed of 150 m s-1 and at an altitude of 3.0 km above
the sea level, spotted an enemy vessel moving at constant velocity towards it. When the
vessel was at a horizontal distance of 4.0 km away from the bomber, a bomb was
released to hit the vessel.

3.0 km
Figure 3.2
4.0 km
Calculate the time taken for the bomb to hit the vessel.

Vertically:
Using s = ut + gt2

[u = 0 as bomb is released]

3000 = (9.81) t2 [M1]


t = 24.7 s

[A1]

time taken = ......

[2]

4. (a)

Define the tesla.


One tesla is the magnetic flux density in which a conductor one metre long and carrying a
current of one ampere placed at right angle to the field experiences a force of one
newton.
[ 0 or B2]
[2]

(b) Three current-carrying wires A, B and C are placed as shown in the diagram below.
B
2.0 A

BA

3.0 m

1 mark for each


arrow

2.0 A

C
3.0 m

BB

Figure 4.1

FR

(i) On the diagram given, indicate the


1. direction of the magnetic field which the current A causes at C.

[1]

2. direction of the magnetic field which the current B causes at C.

[1]

(ii) Using your answers in (b)(i), determine the direction of the resultant magnetic force
acting on wire C. (The magnetic flux density of (b)(i)(1) and (b)(i)(2) are equal in
magnitude)
tan = B A / B B
= 1 (since given B A = B B )
= 45o
[M1]
therefore required bearing of resultant force = 90o + 45o
= 135o [A1]

Direction (in bearing) = [2]

8
5. A wire-wound resistor is manufactured by winding resistance wire on an insulating former. A
commonly used material for wire is an alloy of nickel and chromium called nichrome. The wire
is produced by pulling the nichrome through a suitably sized hole. Nichrome is sufficiently
ductile to be drawn into a wire without danger of it cracking or breaking after winding. It
resists corrosion and has a fairly high resistivity. The wire itself must be uniform and thin, and
is covered with an insulating material. A manufacturer of resistors of this type supplies
information concerning them in the form of a family of lines shown in the graph of Figure 5.1.
Resistors of different resistance R 1 , R 2 R 5 are shown by the separate lines.

Figure 5.1
(a)

By choosing some values of potential difference and current from Figure 5.1, complete
the table showing the resistances R 1 , R 2 R 5 .
[2]

minus 1 mark for each


wrong answer.
Maximum -2 marks

Using R x = V x /I x
(no working needed,

R 1 = 10
= 3/0.3
R 2 = 100
= 3/0.03
R 3 = 1000
R 4 = 10000
= 30/0.003
R 5 = 100000
= 300/0.003

Table 5.1

just answers)
(b)

Draw two additional lines on Figure 5.1:


(i)
one line for a resistance of 2000
(ii) one line for a resistance of 47
[4]

Possible points to be plotted


[(2/0.001), (20/0.01), (200/0.1)]
[(4.7/0.1), (47/1.0), (8.4/0.2), (14.1/0.3)]

Plot a few points that gives V/I = 2000 and join the points to form a straight line.
Repeat for (ii). 1 mark each for correct line. 1 mark each for plotting the points.

(c)

This particular set of resistors is manufactured so that the resistors can safely be used
with power dissipation up to 1 W. Complete the following table to show the maximum
[2]
safe current in the resistors for the potential differences given.
Potential difference

Maximum current

/V

/A

1000

0.001

100

0.01

10

0.1

P = V2/R = I2R
If P = 1 W,
1 = V2/R,
1 = I2R
Combining,
I = (1/V)

Minus 1 mark for each wrong answer. Maximum marks deducted - 2 marks
(d)

Plot the points in (c) on the graph of Figure 5.1. On the graph, indicate the region of safe
use for all these resistors.
[4]
Plot the points from (c), join the points to obtain a straight line, the region of safe use will be
the set that IV < 1 W.

(e)

1 mark for each point plot correctly. (Total points plotted 4 )

The lines on Figure 5.1 represent ideal behaviour. Suggest, with a reason, how the
line for a real resistor might differ from the ideal.

- a curve and not a straight line might be obtained for a real resistor, especially at higher
current. [B1]
- because at higher current temperature this will be different due to the possibility that
resistance might not be constant. [B1]

[2]

10
Section B
Answer two questions for this section. Each question carries 20 marks.
It is recommended that you spend about one hour on this section.
6. (a)

Two identical objects A and B slide down at the same time from an initial height H from
the ground along two frictionless slopes as shown below,.
B

H/2

Figure 6.1
(i)

Write down an expression for the acceleration of object A as it slides down the
slope, explaining your symbols.
g sin [B1]

where g free fall aceleration


acceleration = .

(ii)

[1]

Sketch the velocity time graphs of A and B. Label your graphs clearly.

[3]

Correct shape for object A


and B [B1]
Object B
Object A

Check that gradient for 2nd


half of A and B same [B1]
Having same final velocity
[B1]
t

(iii)

Explain whether the two objects will have the same speed at the bottom of the
slope.
concept of conservation of energy : potential energy is converted to kinetic
energy,
[M1]
OR
Loss in PE = gain in KE
i.e. mgH = mv2 - 0 (u = 0 m s-1)
Equal speed at the bottom.

[A1]

[2]

11
(iv)

Explain whether the two objects will reach the bottom of the slope at the same time.
For B, the initial slope is steeper, hence acceleration is larger
[M1].
It will take a shorter time to reach H/2
[A1].
From H/2 to the ground both objects take the same time to reach the ground.[B1]
Object B will reach bottom of slope first.
[3]

(b)

An object P of mass 1.50 kg slides on a frictionless ground with a constant velocity. It


collides head-on and elastically with another object Q of mass 2.35 kg that is at rest. The
variation of the force that P exerts on Q with time is shown below. The area under the
graph is 9.96 Ns.
P

Q
uP
Figure 6.2

F/N

t/s

0
Figure 6.3
(i)

Explain what is meant by an elastic collision.


In an elastic collision, the total kinetic energy of the bodies before and after
collision is conserved.
[B1] [1]

(ii)

Sketch on Figure 6.3, a graph that shows the variation of the force that Q exerts on
P with time. Explain your answer.

F/N

t/s

Shape [B1]
Newtons 3rd law: F PQ = F QP , equal in magnitude but forces are opposite in direction. [B1]
[2]

12
(iii)

Explain what the area under the graph you drew represents.
Area under the graph is the change in momentum of P or impulse on P.
Change in momentum or impulse [B1]
On P, not Q
[B1]
(note: since the given graph is the force that Q exerts on P, it affects P, not Q).
[2]

(iv)

Calculate the velocity of Q after the collision.


change in momentum of Q = mv Q 0 (as it is at rest initially) = area under graph
2.35 x v Q = 9.96

[M1]

v Q = 4.24 m s-1

[A1]

velocity = [2]
(v)

Calculate the velocities of P before and after the collision.

Relative speeds of approach = relative speeds of separation


uP 0 = vQ - vP

[C1] (as Q is at rest initially)

u p = (4.24 v p )

uP
1.50u P = 2.35v Q + 1.50v p
1.50(4.24 v p ) = 2.35 x 4.24 + 1.50v P
v p = - 1.20 m s-1

Before impact

[C1]

vP
[A1]

After impact
vQ

u P = 4.24 (- 1.20) = 5.44 m s-1 [A1]

velocity of P before collision = [2]


velocity of P after collision = [2]

13
7. (a)

In a Youngs double-slit experiment, the fringe separation observed using yellow light was
found to be 0.275 mm. The yellow lamp, giving a wavelength of 5.50 x 10-7 m is replaced
by a purple one giving wavelengths of 4.00 x 10-7 m in the violet and 6.00 x 10-7 m in the
red. The remainder of the apparatus is undisturbed. Calculate
(i)

the distance between the fringes formed by the violet light.


y = D/d
y (D, d same)

y violet = ( violet / yellow ) x y yellow

= (4.00 x 10-7/5.50 x 10-7) x 0.275 x 10-3


= 2.00 x 10-4 m

[M1]
[A1]
distance = [2]

(ii)

the distance between the fringes formed by the red light and

y red = ( red / yellow ) x y yellow

= (6.00 x 10-7/5.50 x 10-7) x 0.275 x 10-3


= 3.00 x 10-4 m

[M1]
[A1]
distance = [2]

(iii)

the distance from the purple fringe on the axis to the next purple fringe observed.
Red mixes with violet to give purple
[A1]
nd
rd
From (i) and (ii), the 2 order of red fringe will coincide with 3 order of violet.
[A1]
The next purple fringe will be at 6.00 x 10-4 m from the axis.
[A1]
distance = [3]

(iv)

Hence, draw a diagram of the appearance of the new fringe system, indicating the
colours and extending as far as 1mm from the axis.
[3]

axis

purple
3.0 x 10 -4 m

2.0 x 10-4 m

Violet, 1st order

Red 1st order


3.0 x 10 -4 m

2.0 x 10-4 m
Red 2nd order

The 2nd order Red coincides with the 3rd order Violet to give purple
Order of fringe correct
Distance of fringes correct

Violet, 3rd order

[B1]
[B1]
[B1]

14
(b)

(i)

Distinguish between longitudinal and transverse waves.

Longitudinal Waves
o
Particles vibrate along the direction of propagation of the wave.
[A1]
Transverse Waves
o
Particles vibrate perpendicular to the direction of propagation of the wave.
[A1]
(0 or 2m)

[2]

(ii)

Give an example of a longitudinal and transverse wave.

Example of Longitudinal Waves: sound waves, waves produced in a vertical


oscillating spring under tension
[B1]
Example of Transverse Waves: rope waves, water waves, electromagnetic waves.
[B1]
[2]
(c)

Figure 7.1 shows a stationary wave on a string stretched between two points A and F
which are a distance L apart.

B
A

L
Figure 7.1
Compare the oscillations at points B, C, D and E in terms of their amplitudes and
relative phases of oscillation.
(1)

Amplitude:

Point B is a node i.e. it does not oscillate at all, its amplitude is zero.

[B1]

Point C and Point E are antinodes i.e. they vibrate with the largest amplitude
[B1, B1]
Point D vibrates with smaller amplitude as compared to C
(2)

[B1]

Relative phase of oscillation:

Point D vibrates in phase with point C.


Point E is vibrating 1800 out of phase as compared to both points C and
D.[B1]

[B1]

15

8. (a)

A fluorescent tube is filled with mercury vapour at low pressure. In order to emit light, the
mercury atoms must first be excited. Explain what is meant by an excited mercury atom.
An excited mercury atom is one in which an electron in the atom has jumped to a
higher energy level (leaving an unoccupied level below it).
OR
An excited mercury atom is one with energy greater than that of the ground state
(lowest energy state of the atom).
[A1]
[1]

(b)

The diagram below shows some of the energy levels of the mercury atom.
Level 1 represents the lowest possible energy level. (The diagram is not drawn to scale)
Energy / eV

Level n

0
-0.70

-1.54

-2.69

-3.71

-5.72

-10.38

1
Figure 8.1

(i)

Explain how Figure 8.1 can be used to account for the emission spectrum.

The diagram shows that atoms have discrete energy levels. [B1]
When an atom in a particular excited state falls to a lower energy state, it emits a photon
of energy that is exactly equal to the difference in energy between the two states.
[B1]
Hence only photons of specific energies that are equal to the difference between two
energy levels of the atom will be emitted.
[B1]

The wavelengths (or frequencies) corresponding to these photon energies give rise to
an emission line spectrum.
[B1]
[4]

16
(ii)

In a discharge tube, cool mercury vapour is bombarded with a stream of electrons


that have been accelerated from rest through a potential difference of 7.3 V.
1. State and explain how many different frequencies of electromagnetic radiation
will be emitted by the mercury vapour.
Energy of electrons = 7.3 eV
Possible transitions are:
-5.72 eV to -10.38 eV
-3.71 eV to -10.38 eV
-3.71 eV to -5.72 eV

[M1]

Therefore, the number of possible transitions (or frequencies) is 3.

[A1]
[2]

2. Calculate the longest wavelength of the electromagnetic radiation emitted.


The longest wavelength is produced from the smallest energy transition
-3.71 eV to -5.72 eV.
Identifying the correct transition
[M1]
Applying E = hc / ,
(5.72 3.71) (1.6010-19) = (6.63x10-34) (3.00x108) /
= 618x10-9 m (or 618 nm) [A1]

[M1]

wavelength= . [2]

(iii)

State the amount of energy, in electronvolts, required to ionise a mercury atom.


Ionisation energy = 10.38 eV

(c)

(i)

[A1]
ionisation energy = . [1]

What is the photoelectric effect?

Photoelectric effect is the emission of electrons from a metal surface


when electromagnetic radiation (or light) of sufficient energy is incident
upon it.
[B1, B1]
[2]
(ii)

State one observation from the photoelectric experiment, and explain how it
shows the particulate nature of light.

Observation: Radiation below a threshold frequency does not cause emission of electrons
regardless of the intensity or duration of irradiation.
Explanation: According to wave theory, photoelectric effect should occur for any frequency
of radiation provided that the light is intense enough to supply the energy
needed to eject the photoelectrons

17
Observation: KE max is independent of intensity but dependent on frequency.
Explanation: According to wave theory, the kinetic energy should increase as the light is
made more intense.
Observation: Electrons are emitted immediately when the light hits the metal surface.
Explanation: According to wave theory, the energy of the radiation is uniformly distributed
over the wavefront. The effective area for an electron in the metal is limited
to a circle of diameter roughly equal to that of the size of an atom. Thus, if
the intensity is low, there should be a measurable time lag between the
incidence of radiation on the surface and the ejection of the photoelectron.
[A1] mark for observation
[A1] mark for explanation
[3]
(d)

When a piece of metal is irradiated with monochromatic light of wavelength 404.7 nm


from a mercury discharge lamp, the stopping potential was found to be 1.12 V.
(i)

Determine the work function of the metal and the threshold frequency.

From Einsteins equation for photoelectric effect: hf = + e V s


3 10 8
we have = (6.636 x 10-34 x
) - (1.6 x10-19 x 1.12) [M1]
9
404.7 10
= (4.92 1.79) x 10-19
=

3.13 x 10-19 J (or 1.96 eV)

[A1]

Work Function, = hf 0 = 3.13 x 10-19 J (or 1.96 eV)

[M1]

Threshold frequency, f 0 = / h = 4.72 x 1014 Hz

[A1]

work function = . [2]


threshold frequency = . ..[2]
(ii)

If the power from the lamp incident on the metal is 0.120 W, find the rate of
incidence of photons on the metal.
P

N hc
t

0.120 =

N
t

hc
N
(
)
t 404.7 10 - 9
2.44 x 1017 s-1

[M1]

[A1]
rate of incidence = . [2]
END of PAPER

Name: ____________________________ CTG:

___________ (

YISHUN JUNIOR COLLEGE


JC 2 Preliminary Examinations 2008
PHYSICS 8866
Higher 1
Paper 1

26 August 2008
Tuesday
1 hour

Candidates answer on the Optical Answer Sheet.


No Additional Materials are required.
YISHUN JUNIOR COLLEGE YISHUN JUNIOR COLLEGE YISHUN JUNIOR COLLEGE YISHUN JUNIOR COLLEGE YISHUN JUNIOR COLLEGE
YISHUN JUNIOR COLLEGE YISHUN JUNIOR COLLEGE YISHUN JUNIOR COLLEGE YISHUN JUNIOR COLLEGE YISHUN JUNIOR COLLEGE
YISHUN JUNIOR COLLEGE YISHUN JUNIOR COLLEGE YISHUN JUNIOR COLLEGE YISHUN JUNIOR COLLEGE YISHUN JUNIOR COLLEGE
YISHUN JUNIOR COLLEGE YISHUN JUNIOR COLLEGE YISHUN JUNIOR COLLEGE YISHUN JUNIOR COLLEGE YISHUN JUNIOR COLLEGE
YISHUN JUNIOR COLLEGE YISHUN JUNIOR COLLEGE YISHUN JUNIOR COLLEGE YISHUN JUNIOR COLLEGE YISHUN JUNIOR COLLEGE
YISHUN JUNIOR COLLEGE YISHUN JUNIOR COLLEGE YISHUN JUNIOR COLLEGE YISHUN JUNIOR COLLEGE YISHUN JUNIOR COLLEGE
YISHUN JUNIOR COLLEGE YISHUN JUNIOR COLLEGE YISHUN JUNIOR COLLEGE YISHUN JUNIOR COLLEGE YISHUN JUNIOR COLLEGE
YISHUN JUNIOR COLLEGE YISHUN JUNIOR COLLEGE YISHUN JUNIOR COLLEGE YISHUN JUNIOR COLLEGE YISHUN JUNIOR COLLEGE
YISHUN JUNIOR COLLEGE YISHUN JUNIOR COLLEGE YISHUN JUNIOR COLLEGE YISHUN JUNIOR COLLEGE YISHUN JUNIOR COLLEGE
YISHUN JUNIOR COLLEGE YISHUN JUNIOR COLLEGE YISHUN JUNIOR COLLEGE YISHUN JUNIOR COLLEGE YISHUN JUNIOR COLLEGE
YISHUN JUNIOR COLLEGE YISHUN JUNIOR COLLEGE YISHUN JUNIOR COLLEGE YISHUN JUNIOR COLLEGE YISHUN JUNIOR COLLEGE
YISHUN JUNIOR COLLEGE YISHUN JUNIOR COLLEGE YISHUN JUNIOR COLLEGE YISHUN JUNIOR COLLEGE YISHUN JUNIOR COLLEGE
YISHUN JUNIOR COLLEGE YISHUN JUNIOR COLLEGE YISHUN JUNIOR COLLEGE YISHUN JUNIOR COLLEGE YISHUN JUNIOR COLLEGE

READ THESE INSTRUCTIONS FIRST


Write in soft pencil.
Do not use staples, paper clips, highlighters, glue or correction fluid.
Write your name, CTG and class number on the MCQ Answer Sheet in the spaces provided
unless this has been done for you.
Section A
There are thirty questions in this section. Answer all questions. For each question there are
four possible answers A, B, C and D.
Choose the one you consider correct and record your choice in soft pencil on the separate
MCQ Answer Sheet.
Each correct answer will score one mark. A mark will not be deducted for a wrong answer.
Read the instructions on the Answer Sheet very carefully.
Any rough working should be done in this booklet.

This question paper consists of 14 printed pages.

2
Data
speed of light in free space,

c = 3.00 x 108 m s1

elementary charge,

e = 1.60 x 1019 C

the Planck constant,

h = 6.63 x 1034 J s

unified atomic mass constant

u = 1.66 x 1027 kg

rest mass of electron,

me = 9.11 x 1031 kg

rest mass of proton,

mp = 1.67 x 1027 kg

acceleration of free fall,

g = 9.81 m s2

Formulae
uniformly accelerated motion,

s =

ut +

1
2

at2

v2 = u2 + 2as
work done on/by a gas,

W = pV

hydrostatic pressure,

P = gh
V =

Q
4 o r

resistors in series,

R = R1 + R2+.

resistors in parallel,

1
R

JC2Prelim/8866/1/YJC2008

1
1
+
+ ........
R1 R 2

If p is the momentum of an object of mass m, the expression

p2
has identical base
m

units to

A
B
C
D

energy.
force.
power.
velocity.

When comparing systematic and random errors, the following pairs of characteristics of
errors in an experimental measurement may be contrasted:

X1:
X2:
Y1:
Y2:
Z1:
Z2:

error can possibly be eliminated


error cannot possibly be eliminated
error is of constant sign and magnitude
error is of varying sign and magnitude
error will be reduced by averaging repeated measurements
error will not be reduced by averaging repeated measurements

Which properties apply to systematic errors?

A
B
C
D

X1, Y1, Z2
X1, Y2, Z2
X2, Y2, Z1
X2, Y1, Z1

The initial velocity of a projectile is 10 m s1 parallel to the ground. Its final velocity
before hitting the ground is 15 m s1 at an angle of 20o from the ground.
What is the change in velocity of the projectile?

A
B
C
D

3.6 m s1
5.1 m s1
6.6 m s1
14.9 m s1

JC2Prelim/8866/1/YJC2008

4
4

A car is travelling along a straight road. The graph shows the variation with time t of its
acceleration a during part of the journey.

a
W
X

t
Y

At which points on the graph does the car have its greatest velocity and greatest
displacement?

greatest velocity
W
W
X
X

A
B
C
D

greatest displacement
X
Z
Y
Z

A stone is dropped from the top of a tower of height 40 m. The stone falls from rest and
air resistance is negligible.
How long does it take for the stone to fall the last 10 m to the ground? (g = 10 m s2)

A
B
C
D

0.38 s
1.4 s
2.5 s
2.8 s

JC2Prelim/8866/1/YJC2008

5
Which graph best shows the variation with time of the momentum of a body
accelerated by a constant force?

momentum

time

momentum

time

momentum

time

momentum

time

A trolley runs down a slope with a constant acceleration a. The mass of the trolley is
now doubled and the trolley is allowed to run down the same slope. In both cases
effects of friction and air resistance are negligible.
Which statement is correct for the second experiment

A The accelerating force is the same.


B The acceleration is

1a.
2

C The acceleration is a.
D The acceleration is 2a.

JC2Prelim/8866/1/YJC2008

6
8

Two similar spheres, each of mass m and travelling with speed v, move towards each
other before becoming involved in a head-on elastic collision.
v

Which statement is correct?

A
B
C
D

The spheres stick on impact.


The total kinetic energy after impact is mv2.
The total kinetic energy before impact is zero.
The total momentum before impact is 2mv.

A negative charge is in a uniform electric field as shown.

A
D

electric field

negative charge

In which of the directions shown must the charge be moving such that it loses kinetic
energy?

10

The diagram below shows the directions of three co-planar forces applied to a particle
at P, with magnitudes 20 N, 40 N and 50 N respectively.

North (bearing 0o)


40 N
West
(bearing 270o)

50 N

20 N
P

East (bearing
90o)

What is the approximate bearing of the additional force required to maintain


equilibrium?

A
B
C
D

37o
127o
143o
217o

JC2Prelim/8866/1/YJC2008

7
11

A uniform plank of weight 60 N is 2000 mm long and rests on a support that is 600 mm
from end E.

600 mm

2000 mm

At what distance from E must a 160 N weight be placed in order to balance the plank?

A
B
C
D

12

150 mm
225 mm
375 mm
450 mm

An object resting on a horizontal frictionless surface is accelerated from rest by a


constant force from a motor.
Which of the following graphs shows the variation of the motor power P with time t?

JC2Prelim/8866/1/YJC2008

8
13

A motor operates a machine which conveys 200 balls, each of mass 10.0 g, up a
vertical height of 0.50 m every minute and discharges each at a speed of 2.0 m s1.
If 40 % of the work done by the motor is lost, what is the power developed by the
motor?

A
B
C
D

14

0.0019 W
0.38 W
0.57 W
23.0 W

The graph below shows the relationship between the energy of electromagnetic
radiation and the wavelength of the waves.

energy

wavelength
Which of the following has the lowest energy?

A
B
C
D

15

Infra-red
Microwaves
Ultra-violet
X-rays

A pulse is shown in the graph below in which the displacement of the particles of a
spring are plotted against position along the spring at time t = 0 s. The speed of the
pulse is 2.0 m s1 to the right.

0.2
0.1
0
-0.1

Which particle is moving with the greatest speed?

JC2Prelim/8866/1/YJC2008

9
16

Three Polaroid sheets P, Q and R are placed along a straight line with a lamp and a
detector as shown. Initially the directions of polarization of P and Q are parallel and are
both normal to that of R.

lamp
Detector

What happens to the intensity I recorded by the detector when R is being rotated
slowly through 90 degrees until its direction of polarization is parallel to that of Q?

A I remains unchanged.
B I increases throughout.
C I decreases throughout.
D I increases and then decreases.

17

The water level in a vertical glass tube 1.0 m in height can be adjusted to any position
in the tube. A tuning fork vibrating at 660 Hz is held just over the open top end of the
tube.
If the speed of sound in air is 330 m s1, at which positions of the water level will there
be resonance?

A
B
C
D

18

1/2 m
1/3 and 2/3 m
1/4, 1/2 and 3/4 m
1/8, 3/8, 5/8 and 7/8 m

Sound from a small loudspeaker L reaches a point P by two paths which differ in length
by 1.2 m. When the frequency of the sound is gradually increased, the resultant
intensity at P goes through a series of maxima and minima. A maximum occurs when
the frequency is 1000 Hz and the next maximum occurs at 1200 Hz.
What is the speed of sound in the medium between L and P?

A
B
C
D

200 m s1
240 m s1
1200 m s1
1440 m s1

JC2Prelim/8866/1/YJC2008

10
19

In the circuit below, the battery converts an amount E of chemical energy into electrical
energy when charge Q passes through the resistor in time t.

Which expressions give the e.m.f. of the battery and the current in the resistor?

e.m.f.
EQ
EQ
E/Q
E/Q

A
B
C
D

20

current
Q/t
Qt
Q/t
Qt

Aluminium and copper rods are designed to have the same length and the same
resistance. The resistivity of copper is half that of aluminium, but its density is three
times that of aluminium.
What is the ratio of the mass of the aluminium rod to the mass of the copper rod?

A
B
C
D

21

1:6
2:3
3:2
6:1

The filament of a 240 V, 100 W electric lamp heats up from room temperature to its
operating temperature. As it heats up, its resistance increases by a factor of 16.
What is the resistance of this lamp at room temperature?

A
B
C
D

36
580
1.5 k
9.2 k

JC2Prelim/8866/1/YJC2008

11
22

A cell of e.m.f. 6.0 V with internal resistance 1.0


shown in the diagram below.

What is the power supplied to the 2.0

A
B
C
D

23

is connected to three resistors as

resistor in the diagram?

0.7 W
2.3 W
4.5 W
9.0 W

Which of the following statements is a correct description about the circuit as shown?
(Assume that the batteries and light bulbs are of the same type and the internal
resistance of the batteries is small but not negligible.)

Bulb 2

Bulb 1
A
B
C
D

Both bulbs are equally bright.


Bulb 2 is slightly less bright than bulb 1.
Bulb 2 is slightly brighter than bulb 1.
Bulb 2 is twice as bright as bulb 1.

JC2Prelim/8866/1/YJC2008

12
24

A potential divider is used to give outputs of 2 V and 3 V from a 5 V source, as shown.


+5 V

R1
+3 V

R2
+2 V

R3
0V

What are the possible values for the resistances R1, R2, and R3?

A
B
C
D

25

R1 / k
2
3
4
4

R2 / k
1
2
2
6

R3 / k
5
2
4
10

The diagram shows a flat surface with lines OX and OY at right angles to each other.
Y
P

Q
O

Which current in a straight conductor will produce a magnetic field at O in the direction
OY?

A
B
C
D

at P into the plane of the diagram


at P out of the plane of the diagram
at Q into the plane of the diagram
at Q out of the plane of the diagram

JC2Prelim/8866/1/YJC2008

13
26

An electron enters a solenoid carrying a current as shown.


Plan View; not-to-scale
initial velocity

solenoid

End View
solenoid

current in

axis of solenoid
electron
electron
current out

Which of the following is a correct statement about the electromagnetic force acting on
the electron?

A
B
C
D

27

The force acts radially inwards.


The force acts radially outwards.
The force acts in the direction of motion.
No force acts.

The diagram shows two insulated wires P and Q perpendicular to each other. P is
fixed, Q is free to move. P lies just above Q and the wires are not touching.

I
P

When the same current I passes through each wire in the directions shown, Q

A
B
C
D

moves towards the left.


moves towards the right.
rotates anticlockwise until it is parallel to P.
rotates clockwise until it is parallel to P.

JC2Prelim/8866/1/YJC2008

14
28

Which of the following has the largest energy content?

A
B
C
D
29

105 photons of wavelength 2 nm (X-rays)


106 photons of wavelength 5 m (infra-red radiation)
107 photons of wavelength 300 nm (ultraviolet radiation)
108 photons of wavelength 600 nm (yellow light)

Which one of the following statements, referring to photoelectric emission, is always


true?

A No emission of electrons occur for very low intensity illumination.


B For a given metal there is a minimum frequency of radiation below which no
emission occurs.
C The number of electrons emitted per second is independent of the intensity of the
incident radiation.
D The number of electrons emitted per second is proportional to the frequency of the
incident radiation.
30

The diagram shows part of a typical line emission spectrum. This spectrum extends
through the visible region of the electromagnetic spectrum into the ultraviolet region.

Which statement is true for emission line X of the spectrum?

A
B
C
D

It has the highest frequency and is at the red end of the spectrum.
It has the longest wavelength and is at the red end of the spectrum.
It has the lowest frequency and is at the ultraviolet end of the spectrum.
It has the shortest wavelength and is at the ultraviolet end of the spectrum.

* * * End of Paper 1 * * *

JC2Prelim/8866/1/YJC2008

Name: ____________________________ CTG:

___________ (

YISHUN JUNIOR COLLEGE


JC 2 Preliminary Examination 2008
PHYSICS 8866
Higher 1
Paper 2

21 August 2008
Thursday
2 hours

Candidates answer on the question paper.


No additional materials are required.
YISHUN JUNIOR COLLEGE YISHUN JUNIOR COLLEGE YISHUN JUNIOR COLLEGE YISHUN JUNIOR COLLEGE YISHUN JUNIOR COLLEGE
YISHUN JUNIOR COLLEGE YISHUN JUNIOR COLLEGE YISHUN JUNIOR COLLEGE YISHUN JUNIOR COLLEGE YISHUN JUNIOR COLLEGE
YISHUN JUNIOR COLLEGE YISHUN JUNIOR COLLEGE YISHUN JUNIOR COLLEGE YISHUN JUNIOR COLLEGE YISHUN JUNIOR COLLEGE
YISHUN JUNIOR COLLEGE YISHUN JUNIOR COLLEGE YISHUN JUNIOR COLLEGE YISHUN JUNIOR COLLEGE YISHUN JUNIOR COLLEGE
YISHUN JUNIOR COLLEGE YISHUN JUNIOR COLLEGE YISHUN JUNIOR COLLEGE YISHUN JUNIOR COLLEGE YISHUN JUNIOR COLLEGE
YISHUN JUNIOR COLLEGE YISHUN JUNIOR COLLEGE YISHUN JUNIOR COLLEGE YISHUN JUNIOR COLLEGE YISHUN JUNIOR COLLEGE
YISHUN JUNIOR COLLEGE YISHUN JUNIOR COLLEGE YISHUN JUNIOR COLLEGE YISHUN JUNIOR COLLEGE YISHUN JUNIOR COLLEGE
YISHUN JUNIOR COLLEGE YISHUN JUNIOR COLLEGE YISHUN JUNIOR COLLEGE YISHUN JUNIOR COLLEGE YISHUN JUNIOR COLLEGE
YISHUN JUNIOR COLLEGE YISHUN JUNIOR COLLEGE YISHUN JUNIOR COLLEGE YISHUN JUNIOR COLLEGE YISHUN JUNIOR COLLEGE
YISHUN JUNIOR COLLEGE YISHUN JUNIOR COLLEGE YISHUN JUNIOR COLLEGE YISHUN JUNIOR COLLEGE YISHUN JUNIOR COLLEGE
YISHUN JUNIOR COLLEGE YISHUN JUNIOR COLLEGE YISHUN JUNIOR COLLEGE YISHUN JUNIOR COLLEGE YISHUN JUNIOR COLLEGE
YISHUN JUNIOR COLLEGE YISHUN JUNIOR COLLEGE YISHUN JUNIOR COLLEGE YISHUN JUNIOR COLLEGE YISHUN JUNIOR COLLEGE
YISHUN JUNIOR COLLEGE YISHUN JUNIOR COLLEGE YISHUN JUNIOR COLLEGE YISHUN JUNIOR COLLEGE YISHUN JUNIOR COLLEGE

READ THESE INSTRUCTIONS FIRST


Write your name, class and index number on all the work you hand in.
Write in dark blue or black pen.
You may use a soft pencil for any diagrams, graphs or rough working.
Do not use staples, paper clips, highlighters, glue or
correction fluid.
For Examiners Use
Section A
Answer all questions.

Section A
/7

/7

/ 12

Section B
Answer any two questions in the spaces provided.

/ 14

Sub-total
Additional information

/ 40

Section B
5

Read the instructions on the Answer Sheet very carefully.


Any rough working should be done in this booklet.

/ 20

/ 20

/ 20

Sub-total

/ 40

Total

/ 80

This question paper consists of 19 printed pages.

2
Data
speed of light in free space,

c = 3.00 x 108 m s1

elementary charge,

e = 1.60 x 1019 C

the Planck constant,

h = 6.63 x 1034 J s

unified atomic mass constant

u = 1.66 x 1027 kg

rest mass of electron,

me = 9.11 x 1031 kg

rest mass of proton,

mp = 1.67 x 1027 kg

acceleration of free fall,

g = 9.81 m s2

Formulae
uniformly accelerated motion,

s =

ut +

1
2

at2

v2 = u2 + 2as
work done on/by a gas,

W = pV

hydrostatic pressure,

P = gh
V =

Q
4 o r

resistors in series,

R = R1 + R2+.

resistors in parallel,

1
R

JC2Prelim/8866/2/YJC2008

1
1
+
+ ........
R1 R 2

3
Section A
Answer all questions in this section.
1

(a) Explain the meaning of the term derived unit.


......
........ [1]
(b) Give an SI unit and an estimate of the magnitude of each of the following physical
quantities. (Marks will be awarded for the correct order of magnitude of each
estimate, and not for its accuracy.)
[2]
Physical Quantity

SI unit

magnitude

resistance of a domestic filament lamp


Earths magnetic field strength
(c) The Youngs modulus E of a material is given by E =
where

F
L
A
e

=
=
=
=

FL
Ae

force exerted on the material,


original length of the material,
cross-sectional area of the material over which the force acts,
extension of the material,

A student performs an experiment to find the Youngs modulus of an unknown


material which is cylindrical in shape. The student records the readings as follows:
Force exerted on material,
Original length of material,
Diameter of material,
Extension of material,

F = (20.0 0.1) N
L = (400 1) mm
d = (0.40 0.01) mm
e = (0.5 0.01) mm

Find the value of the Youngs modulus and express the answer together with its
associated uncertainty and units.

Youngs modulus = ....................., unit = ..... [2]


uncertainty = ..................... [2]

JC2Prelim/8866/2/YJC2008

4
2

(a) (i)

Define magnetic flux density.


...............................................................................................................................
...............................................................................................................................
...................................................................................................................... [1]

(ii)

Define the tesla.


...............................................................................................................................
...............................................................................................................................
...................................................................................................................... [1]

(b) Sketch the magnetic flux pattern of a current-carrying circular coil.

[2]

(c) The simplest motor in the world is called the homopolar motor, which was first
invented by Michael Faraday. In contrast to most DC motors where commutators are
used to reverse the direction of current flow in order to maintain continuous rotation,
a homopolar motor is able to produce continuous rotation without such reversal in
current.

Fig. 2.1

JC2Prelim/8866/2/YJC2008

5
(i)

Draw the force due to the magnetic field acting on the disc on Fig. 2.1.

(ii)

Explain why the force will result in the rotation of the disc.

[1]

...............................................................................................................................
...............................................................................................................................
...................................................................................................................... [2]

(a) (i)

Describe the photoelectric effect.


...............................................................................................................................
...............................................................................................................................
...................................................................................................................... [1]

(ii)

State two aspects of the photoelectric effect that cannot be explained by the
wave model of light. Describe how the particle model provides an explanation
for them.
...........................................................................................................................
...........................................................................................................................
...........................................................................................................................
...........................................................................................................................
...........................................................................................................................
...........................................................................................................................
...........................................................................................................................
...........................................................................................................................
...........................................................................................................................
...........................................................................................................................
...........................................................................................................................
...........................................................................................................................
...........................................................................................................................
...........................................................................................................................
.................................................................................................................... [6]

JC2Prelim/8866/2/YJC2008

6
(b) A student wanted to investigate how the stopping potential is dependent on the
frequency of the incident electromagnetic radiation.
(i)

Complete the circuit diagram, Fig. 3.1 below, to show the correct position of the
voltmeter, ammeter and variable e.m.f. source in order to determine the
stopping potential. Label the positive terminal of the e.m.f. source clearly on
your circuit diagram.
[2]

incident light

metal surface

vacuum

Fig. 3.1
(ii)

Explain why stopping potential increases with higher frequency and why it is not
affected by the intensity of the incident light.
...........................................................................................................................
...........................................................................................................................
...........................................................................................................................
...........................................................................................................................
...........................................................................................................................
.................................................................................................................... [3]

JC2Prelim/8866/2/YJC2008

7
4

Multi-bladed low-speed wind turbines (windmills) similar to the one shown below in Fig.
4.1 have been used since the 18th century, particularly for pumping water onto farms.

Fig 4.1

The turbine blades cover almost the whole surface of the wheel and a tail vane behind
the windmill keeps the wheel facing the wind. The diameters of the wheels of windmills of
this type vary from about 2.0 m to a practical maximum of about 12.0 m. Because of this
size limitation, they are not suited to large power outputs. They will start freely with wind
speeds as low as 2.0 m s1 and at these low speeds, can produce large torques.

JC2Prelim/8866/2/YJC2008

8
Fig. 4.2 shows how P, the output power of these windmills, varies with the overall
diameter of the wheel for different wind speeds, v.

4000

3000

2000

1000

6
Fig. 4.2

JC2Prelim/8866/2/YJC2008

10
diameter d/ m

9
(a) Use the data provided in Fig. 4.2 to tabulate corresponding values of power output
and wind speed for a particular multi-blade low-speed windmill with a wheel of
diameter 6.0 m.

wind speed v / m s1

output power P / W

[2]

(b) (i)

When the wind speed is 8.0 m s1, what volume of air reaches the 6.0 m
diameter wheel of the windmill in one second?

Volume of air = m3 s1 [2]

(ii)

The density of the air is 1.3 kg m3. What is the kinetic energy of the volume of
moving air calculated in (b)(i)?

Kinetic energy = J s1 [2]

JC2Prelim/8866/2/YJC2008

10
(c) (i)

Using the answer in part (b)(ii) and data provided in Fig. 4.2, find the efficiency
of the windmill with 6.0 m diameter wheel at a wind speed of 8.0 m s1.

Efficiency = % [2]

(ii)

Predict what happens to the efficiency of the windmill when the diameter
increases. Explain your answer.
...........................................................................................................................
...........................................................................................................................
.................................................................................................................... [2]

(d) State two factors, other than wind speed and diameter of wheel, that are likely to
influence the efficiency. Explain how the efficiency is likely to be affected.
.......................................................................................................................................
.......................................................................................................................................
.......................................................................................................................................
.......................................................................................................................................
........................................................................................................................... [3]

(e) In practice, it has been found that it is difficult to scale up a windmill such as this, say
to a wheel of 30 m in diameter, to achieve power outputs of the order of megawatts.
Suggest a reason for this.
.......................................................................................................................................
.......................................................................................................................................
........................................................................................................................... [1]

JC2Prelim/8866/2/YJC2008

11
Section B
Answer any two questions in the spaces provided.
5

(a) (i)

Define acceleration. Using your definition, or an example, explain how it is


possible for the velocity of a body to be in a different direction from its
acceleration.
....
....
....
.......... [2]

(ii)

Discuss whether the resultant force on a body may or may not be in the same
direction as its acceleration.
....
....
....
.......... [2]

(iii) A body moves in a horizontal circle with uniform speed as shown in Fig. 5.1.
The velocity v at point A and the velocity v an instant later is shown.
body

v
v'

Fig. 5.1

1.

Deduce that there must be a force acting on the body.


..
..
..
........... [2]

JC2Prelim/8866/2/YJC2008

12
2.

With the aid of Fig. 5.1 and (a)(i), deduce that the direction of this force is
towards the centre of the circle.

..
..
........ [3]

(b) The moving head of an electronic printer has a mass of 0.20 kg and moves along the
line of print in a jerky motion. After printing each character, the head accelerates
sideways under a force of 10 N and then immediately decelerates to rest under a
force of 30 N ready to print the next character. The characters are 2.5 mm apart.
(i)

Sketch a graph to show how the velocity of the head of the printer varies with
time.
[2]

(ii)

Show that the maximum velocity of the printer head is 50to, where to is the time
at which this speed is attained.
....
....
....
.......... [2]

(iii) Show that the printer head starts and comes to rest in a time of

4
to .
3

....
....
....
.......... [2]

JC2Prelim/8866/2/YJC2008

13
Hence deduce the time taken for the printer head to move between characters.

time = .................... s [2]

(iv) The velocity-time you have sketched in (b)(i) is not realistic. Suggest why this is
so and sketch a more realistic velocity-time graph on your diagram in (b)(i).
....
....
....
.......... [3]

JC2Prelim/8866/2/YJC2008

14
6

(a) What is meant by the amplitude of a wave? Intensity is defined as the rate of transfer
of energy per unit area normal to the direction of propagation of the wave.
Given that intensity I is related to amplitude A of a sound wave by I = cA 2 , write
down the SI units for constant c.
......
......
......
........ [3]

(b) A wave of amplitude A and intensity I is coincident with a second wave of amplitude
3A. Both waves have the same frequency. Write down, in terms of A and c, the
resultant amplitude and intensity when the phase difference is
(i)

zero,
....
.......... [2]

(ii)

radians.
....
.......... [2]

(c) A radar transmitter produces pulses of microwaves each with a mean power P which
are emitted uniformly in all directions. A small spherical target of effective area S is
placed at a distance d from the transmitter. The target reflects a fraction k of the
energy incident on it uniformly in all directions as shown in Fig. 6.1.

transmitter

target

Fig. 6.1

JC2Prelim/8866/2/YJC2008

15
The mean intensity Ir of the reflected pulse when it is received back at the transmitter
is given by the word equation,

Ir =

power received by target fraction reflected


area of sphere with target at centre and transmitter at surface

Using the word equation, show that I r =

PkS

16 2 d 4

[3]

[The surface area of a sphere of radius r is 4r 2 .]

If the mean power P is 2 MW and the pulse duration is 3 s, calculate

(i)

the energy in each emitted pulse,

energy = .................... J [2]

(ii)

the mean intensity of the reflected pulse when received back at the transmitter
if the range is 50 km and the product kS = 1 m2.

mean intensity = .................... W m2 [1]


Briefly discuss the effect on (c)(ii) if the pulses were emitted in an almost parallel
beam from transmitter to target.
......
......
........ [2]

JC2Prelim/8866/2/YJC2008

16
(d) Two radio transmitters emit vertically polarised electromagnetic waves of frequency
9 10 7 Hz, such that standing waves are set up along the line joining the transmitters.
(i)

Calculate the internodal distance along the line joining the transmitters.

Internodal distance = .................... m [2]

(ii)

A mobile receiver moves along the straight line joining the transmitters at a
speed of 6 102 m s1. Calculate the rate at which nodes in this standing wave
are passed by the moving receiver.

rate of passing nodes = .................... s1 [3]

JC2Prelim/8866/2/YJC2008

17
7

(a) Distinguish between the terms potential difference and electromotive force.
......
......
......
........ [2]
There are systems in which current is not directly proportional to applied potential
difference. Give a sketch graph showing the form of the I-V relationship of such a
system and explain the form of the graph.

......
........ [3]

The current I through a certain device is related to the p.d. V across it by the
equation

I = a e bV 1 ,
where a = 104 A and b = 40 V1. When the device is connected to a battery of e.m.f.
1.5 V, a current of 0.1 A flows.

(i) Show that V =

1 I

ln + 1 , where ln denotes the natural logarithm.


b a

[2]

(ii) Hence, or otherwise, calculate the internal resistance of the battery.

internal resistance = .................... [2]

JC2Prelim/8866/2/YJC2008

18
(b) A set of 12 identical lamps is designed for use with a 240-V supply. The lamps are
arranged in series as shown in Fig. 7.1. The total power supplied is 60 W.

240-V supply

Fig. 7.1
For a single lamp in the set connected in series, calculate

(i)

the current,

current = .................... A [1]

(ii)

the potential difference,

potential difference = .................... V [1]

(iii) the resistance.

resistance = .................... [2]

(c) The lamps do not light up when the set is plugged in so a voltmeter is used to test the
circuit. For each of the following observations, identify the fault.
(i)

The potential difference between A and M is zero.


....
.......... [1]

(ii)

The potential difference is zero across every lamp except EF, across which the
potential difference is 240 V.
....
.......... [1]

JC2Prelim/8866/2/YJC2008

19
(iii) The potential difference between A and M is 240 V but the potential difference
is zero across every single lamp.
....
....
.......... [1]

(iv) Some lamps are designed so that when the filament fails the resistance of the
lamp drops to zero. If this happens to one of the lamps in the set, calculate the
fractional increase in the power dissipated in each of the remaining lamps,
assuming that the resistance of these lamps does not change.

fractional increase = .................... [4]


* * * End of Paper 2 * * *

JC2Prelim/8866/2/YJC2008

Solutions
Name: ____________________________ CTG:

___________ (

YISHUN JUNIOR COLLEGE


JC 2 Preliminary Examinations 2008
PHYSICS 8866
Higher 1
Paper 1

26 August 2008
Tuesday
1 hour

Candidates answer Section A on the Optical Answer Sheet.


No Additional Materials are required.

YISHUN JUNIOR COLLEGE YISHUN JUNIOR COLLEGE YISHUN JUNIOR COLLEGE YISHUN JUNIOR COLLEGE YISHUN JUNIOR COLLEGE
YISHUN JUNIOR COLLEGE YISHUN JUNIOR COLLEGE YISHUN JUNIOR COLLEGE YISHUN JUNIOR COLLEGE YISHUN JUNIOR COLLEGE
YISHUN JUNIOR COLLEGE YISHUN JUNIOR COLLEGE YISHUN JUNIOR COLLEGE YISHUN JUNIOR COLLEGE YISHUN JUNIOR COLLEGE
YISHUN JUNIOR COLLEGE YISHUN JUNIOR COLLEGE YISHUN JUNIOR COLLEGE YISHUN JUNIOR COLLEGE YISHUN JUNIOR COLLEGE
YISHUN JUNIOR COLLEGE YISHUN JUNIOR COLLEGE YISHUN JUNIOR COLLEGE YISHUN JUNIOR COLLEGE YISHUN JUNIOR COLLEGE
YISHUN JUNIOR COLLEGE YISHUN JUNIOR COLLEGE YISHUN JUNIOR COLLEGE YISHUN JUNIOR COLLEGE YISHUN JUNIOR COLLEGE
YISHUN JUNIOR COLLEGE YISHUN JUNIOR COLLEGE YISHUN JUNIOR COLLEGE YISHUN JUNIOR COLLEGE YISHUN JUNIOR COLLEGE
YISHUN JUNIOR COLLEGE YISHUN JUNIOR COLLEGE YISHUN JUNIOR COLLEGE YISHUN JUNIOR COLLEGE YISHUN JUNIOR COLLEGE
YISHUN JUNIOR COLLEGE YISHUN JUNIOR COLLEGE YISHUN JUNIOR COLLEGE YISHUN JUNIOR COLLEGE YISHUN JUNIOR COLLEGE
YISHUN JUNIOR COLLEGE YISHUN JUNIOR COLLEGE YISHUN JUNIOR COLLEGE YISHUN JUNIOR COLLEGE YISHUN JUNIOR COLLEGE
YISHUN JUNIOR COLLEGE YISHUN JUNIOR COLLEGE YISHUN JUNIOR COLLEGE YISHUN JUNIOR COLLEGE YISHUN JUNIOR COLLEGE
YISHUN JUNIOR COLLEGE YISHUN JUNIOR COLLEGE YISHUN JUNIOR COLLEGE YISHUN JUNIOR COLLEGE YISHUN JUNIOR COLLEGE
YISHUN JUNIOR COLLEGE YISHUN JUNIOR COLLEGE YISHUN JUNIOR COLLEGE YISHUN JUNIOR COLLEGE YISHUN JUNIOR COLLEGE

READ THESE INSTRUCTIONS FIRST


Write in soft pencil.
Do not use staples, paper clips, highlighters, glue or correction fluid.
Write your name, CTG and class number on the MCQ Answer Sheet in the spaces provided
unless this has been done for you.
Section A
There are thirty questions in this section. Answer all questions. For each question there are
four possible answers A, B, C and D.
Choose the one you consider correct and record your choice in soft pencil on the separate
MCQ Answer Sheet.
Each correct answer will score one mark. A mark will not be deducted for a wrong answer.
Read the instructions on the Answer Sheet very carefully.
Any rough working should be done in this booklet.

This question paper consists of 18 printed pages.

2
Data
speed of light in free space,

c = 3.00 x 108 m s1

elementary charge,

e = 1.60 x 1019 C

the Planck constant,

h = 6.63 x 1034 J s

unified atomic mass constant

u = 1.66 x 1027 kg

rest mass of electron,

me = 9.11 x 1031 kg

rest mass of proton,

mp = 1.67 x 1027 kg

acceleration of free fall,

g = 9.81 m s2

Formulae
uniformly accelerated motion,

s =

ut +

1
2

at2

v2 = u2 + 2as
work done on/by a gas,

W = pV

hydrostatic pressure,

P = gh
V =

Q
4 o r

resistors in series,

R = R1 + R2+.

resistors in parallel,

1
R

JC2Prelim/8866/1/YJC2008

1
1
+
+ ........
R1 R 2

If p is the momentum of an object of mass m, the expression

p2
has identical base
m

units to

A
B
C
D

energy.
force.
power.
velocity.

p2/2m is the kinetic energy of a particle of mass m and momentum p. Thus the unit of p2/m
has identical base units to energy.
[A]
2

When comparing systematic and random errors, the following pairs of characteristics of
errors in an experimental measurement may be contrasted:

X1:
X2:
Y1:
Y2:
Z1:
Z2:

error can possibly be eliminated


error cannot possibly be eliminated
error is of constant sign and magnitude
error is of varying sign and magnitude
error will be reduced by averaging repeated measurements
error will not be reduced by averaging repeated measurements

Which properties apply to systematic errors?

A
B
C
D

X1, Y1, Z2
X1, Y2, Z2
X2, Y2, Z1
X2, Y1, Z1

Systematic errors may be eliminated, are of constant sign and magnitude, but cannot be
reduced by averaging repeated measurements.
[A]
3

The initial velocity of a projectile is 10 m s1 parallel to the ground. Its final velocity
before hitting the ground is 15 m s1 at an angle of 20o from the ground.
What is the change in velocity of the projectile?

A
B
C
D

3.6 m s1
5.1 m s1
6.6 m s1
14.9 m s1

Resolve final velocity vf into two perpendicular components:


vertical:
15.0 sin 20 = 5.13 m s1, horizontal: 15.0 cos 20 = 14.1 m s1
Horizontal component of change in velocity = 14.1 10 = 4.1 m s1
r
Magnitude of v = 4.12 + 5.13 2 = 6.6 m s1

JC2Prelim/8866/1/YJC2008

[C]

4
4

A car is travelling along a straight road. The graph shows the variation with time t of its
acceleration a during part of the journey.

a
W
X

t
Y

At which points on the graph does the car have its greatest velocity and greatest
displacement?

greatest velocity
W
W
X
X

A
B
C
D

greatest displacement
X
Z
Y
Z

Greatest positive area under at graph. = greatest velocity (X)


In general, greatest displacement occurs when velocity = 0. Total area up to point Z gives
a positive velocity, which implies displacement is still increasing.
[D]
5

A stone is dropped from the top of a tower of height 40 m. The stone falls from rest and
air resistance is negligible.
How long does it take for the stone to fall the last 10 m to the ground? (g = 10 m s2)

A
B
C
D

0.38 s
1.4 s
2.5 s
2.8 s

Time to fall 30 m =

2 30
= 6 s , time to fall 40 m =
10

Thus time to fall last 10 m = 8 6 = 0.38 s

JC2Prelim/8866/1/YJC2008

2 40
= 8s
10

[A]

momentum

Which graph best shows the variation with time of the momentum of a body
accelerated by a constant force?

time

momentum

time

momentum

time

momentum

time

By Newtons 2nd law, force is the rate of change of linear momentum. Since force is
constant, therefore rate of change of momentum is constant straight line graph with
positive slope.
[A]

JC2Prelim/8866/1/YJC2008

6
7

A trolley runs down a slope with a constant acceleration a. The mass of the trolley is
now doubled and the trolley is allowed to run down the same slope. In both cases
effects of friction and air resistance are negligible.
Which statement is correct for the second experiment

A The accelerating force is the same.


B The acceleration is

1a.
2

C The acceleration is a.
D The acceleration is 2a.

A free-body diagram will show the accelerating force to be mg sin , where is the angle
of inclination. The acceleration a = mg sin / m = g sin , which is independent of the
mass of the object.
[C]
8

Two similar spheres, each of mass m and travelling with speed v, move towards each
other before becoming involved in a head-on elastic collision.
v

Which statement is correct?

A
B
C
D

The spheres stick on impact.


The total kinetic energy after impact is mv2.
The total kinetic energy before impact is zero.
The total momentum before impact is 2mv.

Total momentum before impact = mv mv = 0. Total kinetic energy before impact = mv2
[B]
+ mv2 = mv2. Since collision is elastic, the spheres cannot stick on impact.

JC2Prelim/8866/1/YJC2008

7
9

A negative charge is in a uniform electric field as shown.

A
D

electric field

negative charge

In which of the directions shown must the charge be moving such that it loses kinetic
energy?

To lose kinetic energy, the charge must be moving against the electric force. Since the
force is directed to the left, the charge must be moving to the right.
[B]
10

The diagram below shows the directions of three co-planar forces applied to a particle
at P, with magnitudes 20 N, 40 N and 50 N respectively.

North (bearing 0o)


40 N
West
(bearing 270o)

50 N

20 N
P

East (bearing
90o)

What is the approximate bearing of the additional force required to maintain


equilibrium?

A
B
C
D

37o
127o
143o
217o

The resultant force is in the northeast direction Hence to maintain equilibrium, the
additional force should act in the southwest.
[D]

JC2Prelim/8866/1/YJC2008

8
11

A uniform plank of weight 60 N is 2000 mm long and rests on a support that is 600 mm
from end E.

600 mm

2000 mm

At what distance from E must a 160 N weight be placed in order to balance the plank?

A
B
C
D

150 mm
225 mm
375 mm
450 mm

By principle of moments, 160d = 60 400. Thus required distance = 450 mm


12

[D]

An object resting on a horizontal frictionless surface is accelerated from rest by a


constant force from a motor.
Which of the following graphs shows the variation of the motor power P with time t?

P = Fv. By kinematics equation, v = at = Ft/m. Thus P = (F2/m)t , implying P is directly


proportional to time.

JC2Prelim/8866/1/YJC2008

[B]

9
13

A motor operates a machine which conveys 200 balls, each of mass 10.0 g, up a
vertical height of 0.50 m every minute and discharges each at a speed of 2.0 m s1.
If 40 % of the work done by the motor is lost, what is the power developed by the
motor?

A
B
C
D

0.0019 W
0.38 W
0.57 W
23.0 W

Total energy output = 200 [mgh + mv2] = 13.81 J


Power output = 13.81/60 = 0.230 W
60 % 0.230 W, Thus 100%
0.383 W
14

[B]

The graph below shows the relationship between the energy of electromagnetic
radiation and the wavelength of the waves.

energy

wavelength
Which of the following has the lowest energy?

A
B
C
D

Infra-red
Microwaves
Ultra-violet
X-rays

E = hc/. Since infra-red has the longest wavelength, its energy is the lowest.

JC2Prelim/8866/1/YJC2008

[A]

10
15

A pulse is shown in the graph below in which the displacement of the particles of a
spring are plotted against position along the spring at time t = 0 s. The speed of the
pulse is 2.0 m s1 to the right.

0.2
0.1
0
-0.1

Which particle is moving with the greatest speed?

Velocity = rate of change of displacement. Thus greatest speed occurs at B.


16

[B]

Three Polaroid sheets P, Q and R are placed along a straight line with a lamp and a
detector as shown. Initially the directions of polarization of P and Q are parallel and are
both normal to that of R.

lamp
Detector

What happens to the intensity I recorded by the detector when R is being rotated
slowly through 90 degrees until its direction of polarization is parallel to that of Q?

A I remains unchanged.
B I increases throughout.
C I decreases throughout.
D I increases and then decreases.

Increases to a maximum, since when it is oriented perpendicular to Q, no light passes


through R.
[B]

JC2Prelim/8866/1/YJC2008

11
17

The water level in a vertical glass tube 1.0 m in height can be adjusted to any position
in the tube. A tuning fork vibrating at 660 Hz is held just over the open top end of the
tube.
If the speed of sound in air is 330 m s1, at which positions of the water level will there
be resonance?

A
B
C
D

1/2 m
1/3 and 2/3 m
1/4, 1/2 and 3/4 m
1/8, 3/8, 5/8 and 7/8 m

= v/f = 0.5 m. One end is closed, the other is opened.

L1 = /4 = 1/8 m
18

L2 = 3/4 = 3/8 m

[D]

L3 = 5/4 = 5/8 m

Sound from a small loudspeaker L reaches a point P by two paths which differ in length
by 1.2 m. When the frequency of the sound is gradually increased, the resultant
intensity at P goes through a series of maxima and minima. A maximum occurs when
the frequency is 1000 Hz and the next maximum occurs at 1200 Hz.
What is the speed of sound in the medium between L and P?

A
B
C
D

200 m s1
240 m s1
1200 m s1
1440 m s1

Let n be number of wavelengths corresponding to path difference.


(n)v/1000 = 1.2 m ---- (1), (n+1)v/1200 = 1.2 m ---- (2)
(1) (2) : n = 5. Therefore v = 240 m s1

JC2Prelim/8866/1/YJC2008

[B]

12
19

In the circuit below, the battery converts an amount E of chemical energy into electrical
energy when charge Q passes through the resistor in time t.

Which expressions give the e.m.f. of the battery and the current in the resistor?

e.m.f.
EQ
EQ
E/Q
E/Q

A
B
C
D

current
Q/t
Qt
Q/t
Qt

Definition emf = E/Q, current = Q/t.


20

[C]

Aluminium and copper rods are designed to have the same length and the same
resistance. The resistivity of copper is half that of aluminium, but its density is three
times that of aluminium.
What is the ratio of the mass of the aluminium rod to the mass of the copper rod?

A
B
C
D

1:6
2:3
3:2
6:1

m Al
V Al
1 A
= Al
= Al
m Cu
Cu VCu 3 ACu
Thus, m Al = 1 2 = 2
3
3
m Cu

, and R =

L
A

A
R
. Al = Cu Al = Al = 2
L
AAl
ACu
ACu
Cu

[B]

JC2Prelim/8866/1/YJC2008

13
21

The filament of a 240 V, 100 W electric lamp heats up from room temperature to its
operating temperature. As it heats up, its resistance increases by a factor of 16.
What is the resistance of this lamp at room temperature?

A
B
C
D

36
580
1.5 k
9.2 k

P=

V2
240 2
R=
= 576
R
100

(resistance at normal operating temperature.

Thus, at room temperature, resistance = 576 / 16 = 36


22

A cell of e.m.f. 6.0 V with internal resistance 1.0


shown in the diagram below.

What is the power supplied to the 2.0

A
B
C
D

[A]

is connected to three resistors as

resistor in the diagram?

0.7 W
2.3 W
4.5 W
9.0 W
1

Effective resistance = 1 + 1 + 1 = 1

4 2 4

(all connected in paralle)

P.d across the effective resistance = 3 V, Current through 2


Hence the power supplied to 2 = (1.5)2 2 = 4.5 W

JC2Prelim/8866/1/YJC2008

= 3/2 =1.5 A
[C]

14
23

Which of the following statements is a correct description about the circuit as shown?
(Assume that the batteries and light bulbs are of the same type and the internal
resistance of the batteries is small but not negligible.)

Bulb 2

Bulb 1
A
B
C
D

Both bulbs are equally bright.


Bulb 2 is slightly less bright than bulb 1.
Bulb 2 is slightly brighter than bulb 1.
Bulb 2 is twice as bright as bulb 1.

With two batteries, each battery delivers half the current to the bulb (compared to circuit
1). Thus less power is dissipated in the internal resistances. This makes the effective p.d.
across bulb 2 higher. Hence it is slightly brighter.
[C]

JC2Prelim/8866/1/YJC2008

15
24

A potential divider is used to give outputs of 2 V and 3 V from a 5 V source, as shown.


+5 V

R1
+3 V

R2
+2 V

R3
0V

What are the possible values for the resistances R1, R2, and R3?

A
B
C
D

R1 / k
2
3
4
4

R2 / k
1
2
2
6

R3 / k
5
2
4
10

P.d. across the resistors is in the ratio 2 : 1 : 2. Thus the resistors must also be in the
same ratio.
[C]
25

The diagram shows a flat surface with lines OX and OY at right angles to each other.
Y
P

Q
O

Which current in a straight conductor will produce a magnetic field at O in the direction
OY?

A
B
C
D

at P into the plane of the diagram


at P out of the plane of the diagram
at Q into the plane of the diagram
at Q out of the plane of the diagram

Draw a circle centred at Q and touching O. Then apply right-hand grip rule at Q.

JC2Prelim/8866/1/YJC2008

[C]

16
26

An electron enters a solenoid carrying a current as shown.


Plan View; not-to-scale
initial velocity

solenoid

End View
current in

solenoid

axis of solenoid
electron
electron
current out

Which of the following is a correct statement about the electromagnetic force acting on
the electron?

A
B
C
D

The force acts radially inwards.


The force acts radially outwards.
The force acts in the direction of motion.
No force acts.

The initial velocity can be resolved into two components one parallel to the magnetic
field and the other perpendicular to the magnetic field. No electromagnetic force arises as
a result of the parallel component. However, an electromagnetic force arises due to the
perpendicular component and this causes a circular motion, with the force always
directed radially towards the centre of the circle.
[A]

JC2Prelim/8866/1/YJC2008

17
27

The diagram shows two insulated wires P and Q perpendicular to each other. P is
fixed, Q is free to move. P lies just above Q and the wires are not touching.

I
P

When the same current I passes through each wire in the directions shown, Q

A
B
C
D

moves towards the left.


moves towards the right.
rotates anticlockwise until it is parallel to P.
rotates clockwise until it is parallel to P.

Current through P provides external magnetic field for Q. Applying right-hand grip rule,
magnetic field above P is pointing out of the paper (and vice versa). Applying Flemings
left hand rule, force on Q above P is directed to the right (and vice versa). Thus Q rotates
clockwise.
[D]
28

Which of the following has the largest energy content?

A
B
C
D

105 photons of wavelength 2 nm (X-rays)


106 photons of wavelength 5 m (infra-red radiation)
107 photons of wavelength 300 nm (ultraviolet radiation)
108 photons of wavelength 600 nm (yellow light)

Total energy content = 9.95 x 1012 J (A), 3.98 x 1014 J (B), 6.63 x 1012 J (C) and
3.32 x 1011 J (D).

JC2Prelim/8866/1/YJC2008

[D]

18
29

Which one of the following statements, referring to photoelectric emission, is always


true?

A No emission of electrons occur for very low intensity illumination.


B For a given metal there is a minimum frequency of radiation below which no
emission occurs.
C The number of electrons emitted per second is independent of the intensity of the
incident radiation.
D The number of electrons emitted per second is proportional to the frequency of the
incident radiation.

Fact a threshold (minimum) frequency exists for a given metal for photoelectric mission.
Below this value, no emission of electrons occurs.
[B]
30

The diagram shows part of a typical line emission spectrum. This spectrum extends
through the visible region of the electromagnetic spectrum into the ultraviolet region.

Which statement is true for emission line X of the spectrum?

A
B
C
D

It has the highest frequency and is at the red end of the spectrum.
It has the longest wavelength and is at the red end of the spectrum.
It has the lowest frequency and is at the ultraviolet end of the spectrum.
It has the shortest wavelength and is at the ultraviolet end of the spectrum.

Fact spectral lines belonging to a particular series converges as frequency of


electromagnetic radiation increases. High frequency corresponds to short wavelength. [D]
* * * End of Paper 1 * * *

JC2Prelim/8866/1/YJC2008

Mark Scheme
Name: ____________________________ CTG:

___________ (

YISHUN JUNIOR COLLEGE


JC 2 Preliminary Examination 2008
PHYSICS 8866
Higher 1
Paper 2

21 August 2008
Thursday
2 hours

Candidates answer on the question paper.


No additional materials are required.
YISHUN JUNIOR COLLEGE YISHUN JUNIOR COLLEGE YISHUN JUNIOR COLLEGE YISHUN JUNIOR COLLEGE YISHUN JUNIOR COLLEGE
YISHUN JUNIOR COLLEGE YISHUN JUNIOR COLLEGE YISHUN JUNIOR COLLEGE YISHUN JUNIOR COLLEGE YISHUN JUNIOR COLLEGE
YISHUN JUNIOR COLLEGE YISHUN JUNIOR COLLEGE YISHUN JUNIOR COLLEGE YISHUN JUNIOR COLLEGE YISHUN JUNIOR COLLEGE
YISHUN JUNIOR COLLEGE YISHUN JUNIOR COLLEGE YISHUN JUNIOR COLLEGE YISHUN JUNIOR COLLEGE YISHUN JUNIOR COLLEGE
YISHUN JUNIOR COLLEGE YISHUN JUNIOR COLLEGE YISHUN JUNIOR COLLEGE YISHUN JUNIOR COLLEGE YISHUN JUNIOR COLLEGE
YISHUN JUNIOR COLLEGE YISHUN JUNIOR COLLEGE YISHUN JUNIOR COLLEGE YISHUN JUNIOR COLLEGE YISHUN JUNIOR COLLEGE
YISHUN JUNIOR COLLEGE YISHUN JUNIOR COLLEGE YISHUN JUNIOR COLLEGE YISHUN JUNIOR COLLEGE YISHUN JUNIOR COLLEGE
YISHUN JUNIOR COLLEGE YISHUN JUNIOR COLLEGE YISHUN JUNIOR COLLEGE YISHUN JUNIOR COLLEGE YISHUN JUNIOR COLLEGE
YISHUN JUNIOR COLLEGE YISHUN JUNIOR COLLEGE YISHUN JUNIOR COLLEGE YISHUN JUNIOR COLLEGE YISHUN JUNIOR COLLEGE
YISHUN JUNIOR COLLEGE YISHUN JUNIOR COLLEGE YISHUN JUNIOR COLLEGE YISHUN JUNIOR COLLEGE YISHUN JUNIOR COLLEGE
YISHUN JUNIOR COLLEGE YISHUN JUNIOR COLLEGE YISHUN JUNIOR COLLEGE YISHUN JUNIOR COLLEGE YISHUN JUNIOR COLLEGE
YISHUN JUNIOR COLLEGE YISHUN JUNIOR COLLEGE YISHUN JUNIOR COLLEGE YISHUN JUNIOR COLLEGE YISHUN JUNIOR COLLEGE
YISHUN JUNIOR COLLEGE YISHUN JUNIOR COLLEGE YISHUN JUNIOR COLLEGE YISHUN JUNIOR COLLEGE YISHUN JUNIOR COLLEGE

READ THESE INSTRUCTIONS FIRST


Write your name, class and index number on all the work you hand in.
Write in dark blue or black pen.
You may use a soft pencil for any diagrams, graphs or rough working.
Do not use staples, paper clips, highlighters, glue or
correction fluid.
For Examiners Use
Section A
Answer all questions.

Section A
/7

/7

/ 12

Section B
Answer any two questions in the spaces provided.

/ 14

Sub-total
Additional information
Read the instructions on the Answer Sheet very carefully.
Any rough working should be done in this booklet.

/ 40

Section B
5

/ 20

/ 20

/ 20

Sub-total

/ 40

Total

/ 80

This question paper consists of 19 printed pages.

Mark Scheme

Data
speed of light in free space,

c = 3.00 x 108 m s1

elementary charge,

e = 1.60 x 1019 C

the Planck constant,

h = 6.63 x 1034 J s

unified atomic mass constant

u = 1.66 x 1027 kg

rest mass of electron,

me = 9.11 x 1031 kg

rest mass of proton,

mp = 1.67 x 1027 kg

acceleration of free fall,

g = 9.81 m s2

Formulae
uniformly accelerated motion,

s =

ut +

1
2

at2

v2 = u2 + 2as
work done on/by a gas,

W = pV

hydrostatic pressure,

P = gh
V =

Q
4 o r

resistors in series,

R = R1 + R2+.

resistors in parallel,

1
R

JC2Prelim/8866/2/YJC2008

1
1
+
+ ........
R1 R 2

Mark Scheme

Section A
Answer all questions in this section.
1

(a) Explain the meaning of the term derived unit.

A derived unit is expressed in terms as a product and/or quotient of the base units. [B1]
(b) Give an SI unit and an estimate of the magnitude of each of the following physical
quantities. (Marks will be awarded for the correct order of magnitude of each
estimate, and not for its accuracy.)
Physical Quantity

SI unit

magnitude

103

resistance of a domestic filament lamp

(c) The Youngs modulus E of a material is given by E =


where

F
L
A
e

=
=
=
=

105
Per correct row [B1]

Earths magnetic field strength

FL
Ae

force exerted on the material,


original length of the material,
cross-sectional area of the material over which the force acts,
extension of the material,

A student performs an experiment to find the Youngs modulus of an unknown


material which is cylindrical in shape. The student records the readings as follows:
Force exerted on material,
Original length of material,
Diameter of material,
Extension of material,

F = (20.0 0.1) N
L = (400 1) mm
d = (0.40 0.01) mm
e = (0.50 0.01) mm

Find the value of the Youngs modulus and express the answer together with its
associated uncertainty and units.

20.0 400 10 3

E=

3 2

= 12.7 1010 kgm 1s 2

0.40 10
0.50 10 3

E F L
d e
E
0.1
1
0.01 0.01
=
+
+2
+

=
+
+2
+
E
F
L
d
e
0.40 0.50
1.27 1011 20.0 400

E = 1 1010 (1 s.f.)

[C1]
[A1]

Therefore, Youngs modulus = (13 1) 1010 kgm 1s 2

JC2Prelim/8866/2/YJC2008

[A1, B1 units]

(a) (i)

Mark Scheme

Define magnetic flux density.

Magnetic flux density is defined as the force acting on a conductor of unit length
carrying unit current placed perpendicular to the magnetic field.
[B1]
(ii)

Define the tesla.

Tesla is defined as the force of 1 N acting on a conductor of 1 m carrying 1 A of


current placed perpendicular to the magnetic field.
[B1]
(b) Sketch the magnetic flux pattern of a current-carrying circular coil.

Direction of field lines correspond to right-hand grip rule.


At least 3 lines drawn, (2 of which must be looping around wire)

[B1]
[B1]

(c) The simplest motor in the world is called the homopolar motor, which was first
invented by Michael Faraday. In contrast to most DC motors where commutators are
used to reverse the direction of current flow in order to maintain continuous rotation,
a homopolar motor is able to produce continuous rotation without such reversal in
current.

Force directed
into page
Fig. 2.1
(i)

Draw the force due to the magnetic field acting on the disc on Fig. 2.1.

Force directed into page.


(ii)

[B1]

Explain why the force will result in the rotation of the disc.

The force due to the magnetic field does not pass through the central axis of the
disc, hence it will result in anticlockwise rotation of the disc when viewed from
above.
[B1 for each underlined phrase]

JC2Prelim/8866/2/YJC2008

5
3

(a) (i)

Mark Scheme

Describe the photoelectric effect.

Photoelectric effect is the emission of electrons from the surface of a (cold) metal
when electromagnetic radiation of suitable frequencies are incident on the metal.
[B1]
(ii)

State two aspects of the photoelectric effect that cannot be explained by the
wave model of light. Describe how the particle model provides an explanation
for them.

Regardless of the intensity of the em radiation, there is no photoemission when


the frequency of the em radiation is below a certain value.
[B1]
Explanation: The energy of each photon is dependent only on the frequency of the
em radiation. An electron receives energy to escape by absorbing a photon. [B1]
Since the electron needs a minimum amount of energy to escape from the metal,
hence the photon it absorbs must have a minimum frequency.
[B1]
OR
Photoemission is instantaneous even for very low intensities.

[B1]

Explanation: The energy of a photon is transferred to the electron in a single


collision event.
[B1]
If the photon energy is large enough, the electron will be liberated immediately.
[B1]
OR
The maximum kinetic energy of the ejected electron is only dependent on
frequency and not intensity.
[B1]
Explanation: KEmax = Energy of photon (hf) workfunction of metal ()

[B1]

Since energy of photon is dependent on frequency, thus maximum kinetic energy


is dependent on frequency but not intensity.
[B1]

Candidates need only to write on any two of the above.

JC2Prelim/8866/2/YJC2008

Mark Scheme

(b) A student wanted to investigate how the stopping potential is dependent on the
frequency of the incident electromagnetic radiation.
(i)

Complete the circuit diagram, Fig. 3.1 below, showing the correct position of the
voltmeter, ammeter and variable e.m.f. source in order to determine the
stopping potential. Label the positive terminal of the e.m.f. source clearly on
your circuit diagram.

incident light

metal surface
+
V
vacuum

Fig. 3.1

Positive terminal of emf source labelled correctly.


Voltmeter, ammeter, and variable emf source shown at correct positions.
(ii)

[B1]
[B1]

Explain why stopping potential increases with higher frequency and why it is not
affected by the intensity of the incident light.

Stopping potential is a measure of the maximum kinetic energy of the emitted


electrons.
[B1]
By Einsteins photoelectric equation, maximum kinetic energy is equal to the
difference between photon energy and work function of the metal. Hence stopping
potential increases with frequency of EM radiation.
[B1]
Varying the intensity of the incident light only affects the number of photons
incident on the metal, but not the energy of each photon.
[B1]

JC2Prelim/8866/2/YJC2008

7
4

Mark Scheme

Multi-bladed low-speed wind turbines (windmills) similar to the one shown below in Fig.
4.1 have been used since the 18th century, particularly for pumping water onto farms.

Fig 4.1

The turbine blades cover almost the whole surface of the wheel and a tail vane behind
the windmill keeps the wheel facing the wind. The diameters of the wheels of windmills of
this type vary from about 2.0 m to a practical maximum of about 12.0 m. Because of this
size limitation, they are not suited to large power outputs. They will start freely with wind
speeds as low as 2.0 m s1 and at these low speeds, can produce large torques.

JC2Prelim/8866/2/YJC2008

Mark Scheme

Fig. 4.2 shows how P, the output power of these windmills, varies with the overall
diameter of the wheel for different wind speeds, v.

4000

3000

2000

1000

6
Fig. 4.2

JC2Prelim/8866/2/YJC2008

10
diameter d/ m

Mark Scheme

(a) Use the data provided in Fig. 4.2 to tabulate corresponding values of power output
and wind speed for a particular multi-blade low-speed windmill with a wheel of
diameter 6.0 m.

[6 8 rows correct B2, 3 5 rows correct B1, 0 2 rows correct no marks]


(b) (i)

When the wind speed is 8.0 m s1, what volume of air reaches the 6.0 m
diameter wheel of the windmill in one second?

volume per second = (3)2 8


= 226 (or 230) m3 s1
(ii)

The density of the air is 1.3 kg m3. What is the kinetic energy of the volume of
moving air calculated in (b)(i)?

KE = mv2 = (1.3)(226)(8)2
= 9400 (or 9409) J s1
(c) (i)

[C1]
[A1]

Using the answers in part (b)(ii) and data provided in Fig. 4.2, find the efficiency
of the windmill with 6.0 m diameter wheel at a wind speed of 8.0 m s1.

From graph,
at 6.0 m diameter, wind speed of 8.0 m s-1, the output power is 2800 W
Efficiency = output / input x 100% = 2800 / 9409 x 100% = 29.8 %
(ii)

[C1]
[A1]

[B1]
[B1]

Predict what happens to the efficiency of the windmill when the diameter
increases. Explain your answer.

For the same wind speed, efficiency increases with diameter.


Since efficiency increases, output power also increases with diameter.

JC2Prelim/8866/2/YJC2008

[B1]
[B1]

10

Mark Scheme

(d) State two factors, other than wind speed and diameter of wheel, that are likely to
influence the efficiency. Explain how the efficiency is likely to be affected.

Friction between the wheel and the axis of rotation will result in greater power loss and
hence lower the efficiency.
OR
The height of the tower [affects] the windmill immensely. The higher a windmill is, the
higher the wind speed, and hence its efficiency.
OR
The angle of tilt (pitch) of blades is important as it may capture more wind at a particular
direction and thus more torque will be produced which subsequently increases
efficiency.
[Any two of the above B2]
Attempts to explain/deduce how efficiency will be affected in each case.

[B1]

(e) In practice, it has been found that it is difficult to scale up a windmill such as this, say
to a wheel of 30 m in diameter, to achieve power outputs of the order of megawatts.
Suggest a reason for this.

A 30-m diameter wheel will require a taller tower. The higher centre of gravity and the
larger weight of the wheel contribute to instability.
[B1]

JC2Prelim/8866/2/YJC2008

11

Mark Scheme

Section B
Answer any two questions in the spaces provided.
5

(a) (i)

Define acceleration. Using your definition, or an example, explain how it is


possible for the velocity of a body to be in a different direction from its
acceleration.
Acceleration is the rate of change of velocity of an object (with respect to time).
[B1]
Either:

OR:

OR:

(ii)

If the change of velocity is not in the same direction as the objects


velocity, then it is possible for the acceleration to be in a different
direction from its velocity.
[B1]
When an object slows down, the change in velocity (final velocity initial
velocity) is opposite in direction to the (initial) velocity. (Thus the
acceleration is also opposite to the velocity.)
[B1]
Suitably labeled vector diagram showing initial and final velocities,
change in velocity and acceleration.
[B1]

Discuss whether the resultant force on a body may or may not be in the same
direction as its acceleration.
By Newtons second law, F = ma.
[B1]
Thus resultant force and acceleration must be in the same direction, since m is a
positive scalar quantity.
[B1]

(iii) A body moves in a horizontal circle with uniform speed as shown in Fig. 5.1. The
velocity v at point A and the velocity v an instant later is shown.
body

v
v'

Fig. 5.1

1.

Deduce that there must be a force acting on the body.


Since the direction of the velocity changes,
[B1]
Thus by Newtons first law, there must be an external force acting on the body.
[B1]

JC2Prelim/8866/2/YJC2008

12
2.

Mark Scheme

With the aid of Fig. 5.1 and (a)(i), deduce that the direction of this force is
towards the centre of the circle.
Vector diagram showing change in velocity (v), final and initial velocities (v,

v).
[B1]
Deduce direction of v towards centre of circle (OR perpendicular to v). [B1]
By (a)(i), F is towards centre of circle since it is parallel to a which in turn is
parallel to v.
[B1]

(b) The moving head of an electronic printer has a mass of 0.20 kg and moves along the line
of print in a jerky motion. After printing each character, the head accelerates sideways
under a force of 10 N and then immediately decelerates to rest under a force of 30 N
ready to print the next character. The characters are 2.5 mm apart.
(i)

Sketch a graph to show how the velocity of the head of the printer varies with time.

velocity

time
to
First segment - straight line with positive slope passing through origin,
[B1]
Second segment straight line with steeper negative slope starting at highest
point of first segment and ending on the time axis.
[B1]
(ii)

Show that the maximum velocity of the printer head is 50to, where to is the time at
which this speed is attained.
Acceleration during first segment = 10 / 0.20 = 50 m s2.
By v = u + at, v = 50to.

(iii)

Show that the printer head starts and comes to rest in a time of

[B1]
[B1]
4
to .
3

Deceleration during second segment = 30 / 0.20 = 150 m s2.


Thus time taken to decelerate = (1/3)to. Total time is thus (4/3)to.
(iv)

[B1]
[B1]

Hence deduce the time taken for the printer head to move between characters.
Area under graph = distance moved by printer head
1 4
Thus t o 50t o = 2.5 10 3 t o = 8.7 10 3 s
2 3

JC2Prelim/8866/2/YJC2008

[C1, A1]

13
(v)

Mark Scheme

The velocity-time you have sketched in (b)(i) is not realistic. Suggest why this is so
and sketch a more realistic velocity-time graph using the same axes.

velocity

to

time

Force would not change instantaneously in magnitude and direction.


Thus maximum velocity would occur for a slightly longer duration.
Graph indicates maximum velocity for a short duration of time.
OR: Any other acceptable reason with corresponding changes to v-t graph.

JC2Prelim/8866/2/YJC2008

[B1]
[B1]
[B1]

14
6

Mark Scheme

(a) What is meant by the amplitude of a wave? Intensity is defined as the rate of transfer of
energy per unit area normal to the direction of propagation of the wave.
Given that intensity I is related to amplitude A of a sound wave by I = cA 2 , write down
the SI units for constant c.
Amplitude of a wave is the maximum displacement (of particles in a medium)
Units of I and A = W m2 and m respectively.
Thus units of c = (W m2) (m2) = W m4

[B1]
[C1]
[A1]

(b) A wave of amplitude A and intensity I is coincident with a second wave of amplitude 3A.
Both waves have the same frequency. Write down, in terms of A and c, the resultant
amplitude and intensity when the phase difference is
(i)

zero,
[B1]
[B1]

Resultant amplitude = 3A + A = 4A
Resultant intensity = c(4A)2 = 16cA2
(ii)

radians.
[B1]
[B1]

Resultant amplitude = 3A A = 2A
Resultant intensity = c(2A)2 = 4cA2

(c) A radar transmitter produces pulses of microwaves each with a mean power P which are
emitted uniformly in all directions. A small spherical target of effective area S is placed
at a distance d from the transmitter. The target reflects a fraction k of the energy
incident on it uniformly in all directions as shown in Fig. 6.1.

transmitter

target

Fig. 6.1

JC2Prelim/8866/2/YJC2008

15

Mark Scheme

The mean intensity Ir of the reflected pulse when it is received back at the transmitter is
given by the word equation,
Ir =

power received by target fraction reflected


area of sphere with target at centre and transmitter at surface

Using the word equation, show that I r =

PkS

16 2 d 4

[The surface area of a sphere of radius r is 4r 2 .]

Intensity of incident wave at target =


Power received by target =

P
4d 2

P
4d 2

[B1]
[B1]

S k

2
PkS

Thus, I r = 4d
Ir =
4d 2
16 2 d 4

[B1]

If the mean power P is 2 MW and the pulse duration is 3 s, calculate


(i)

the energy in each emitted pulse,


Energy per pulse = 2 106 3 106 = 6 J

(ii)

[C1, A1]

the mean intensity of the reflected pulse when received back at the transmitter if
the range is 50 km and the product kS = 1 m2.
Mean intensity =

2 10 6 1
2

16 50 10

3 4

= 2.0 1015 W m2

[A1]

Briefly discuss the effect on (c)(ii) if the pulses were emitted in an almost parallel beam
from transmitter to target.
The mean intensity would be much higher,
[B1]
since the energy of the waves is now spread over a much smaller area (compared to a
spherical area which gets increasingly larger).
[B1]

JC2Prelim/8866/2/YJC2008

16

Mark Scheme

(d) Two radio transmitters emit vertically polarised electromagnetic waves of frequency
9 10 7 Hz, such that standing waves are setup along the line joining the transmitters.
(i)

Calculate the internodal distance along the line joining the transmitters.

Internodal distance =

(ii)

3 10 8
9 10 7 2

= 1.7 m

[C1, A1]

A mobile receiver moves along the straight line joining the transmitters at a speed
of 6 102 m s1. Calculate the rate at which nodes in this standing wave are
passed by the moving receiver.
Time to pass between nodes = 1.7 / (6 102) = 2.83 ms
1
Rate of passing =
= 353 s 1
-3
2.83 10

JC2Prelim/8866/2/YJC2008

[B1]
[C1, A1]

17
7

Mark Scheme

(a) Distinguish between the terms potential difference and electromotive force.
Potential difference measures the energy dissipated per unit charge when the charge
flows around a circuit.
[B1]
Electromotive force measures the energy supplied per unit charge to drive the charge
around a circuit.
[B1]
There are systems in which current is not directly proportional to applied potential
difference. Give a sketch graph showing the form of the I-V relationship of such a
system and explain the form of the graph.
current

potential difference

Labelled graph (axes, name of electrical component must be stated) with correct I-V
characteristic.
[M1, A1]
As the p.d. increases, the power dissipated by the lamp increases and this causes the
resistance of the lamp to increase.
[B1]
The current I through a certain device is related to the p.d. V across it by the equation

I = a e bV 1 ,

where a = 104 A and b = 40 V1. When the device is connected to a battery of e.m.f. 1.5 V,
a current of 0.1 A flows.
(i) Show that V =

1 I

ln + 1 , where ln denotes the natural logarithm.


b a

I
Step 1: e bV = + 1
a
1 I
I

Step 2: bV = ln + 1 V = ln + 1
b a
a

[B1]
[B1]

(ii) Hence, or otherwise, calculate the internal resistance of the battery.


P.d. across device, V = 0.173 V
Using = V + Ir r = (1.5 0.173) / 0.1 = 13.3

JC2Prelim/8866/2/YJC2008

[B1]
[B1]

18

Mark Scheme

(b) A set of 12 identical lamps is designed for use with a 240-V supply. The lamps are
arranged in series as shown in Fig. 7.1. The total power supplied is 60 W.
A

240-V supply

Fig. 7.1
For a single lamp in the set connected in series, calculate
(i)

the current,
Current = power / p.d. = 60 / 240 = 0.25 A

(ii)

the potential difference,


[B1]

P.d. = 240 / 12 = 20 V
(iii)

[B1]

the resistance.
resistance = 20 / 0.25 = 80

[C1, A1]

(c) The lamps do not light up when the set is plugged in so a voltmeter is used to test the
circuit. For each of the following observations, identify the fault.
(i)

The potential difference between A and M is zero.


Switch is not turned on, OR connecting wire between supply and lamps is
broken.
[B1]

(ii)

The potential difference is zero across every lamp except EF, across which the
potential difference is 240 V.
All lamps are short-circuited, except between points E and F.

[B1]

OR
There is an open circuit across EF.
(iii)

[B1]

The potential difference between A and M is 240 V but the potential difference is
zero across every single lamp.
One (or more) lamp filament is broken.

JC2Prelim/8866/2/YJC2008

[B1]

19
(iv)

Mark Scheme

Some lamps are designed so that when the filament fails the resistance of the
lamp drops to zero. If this happens to one of the lamps in the set, calculate the
fractional increase in the power dissipated in each of the remaining lamps,
assuming that the resistance of these lamps does not change.
Initial power per lamp = 60 / 12 = 5.0 W

[B1]

( 240 / 11)
= 5.95 W
80
5.95 5.0
Fractional increase in power =
= 0.19
5.0
(A1 mark not awarded if student leaves answer in as a fraction)
Final power per lamp =

* * * End of Paper 2 * * *

JC2Prelim/8866/2/YJC2008

[B1]
[C1, A1]

You might also like